Location via proxy:   [ UP ]  
[Report a bug]   [Manage cookies]                
Download as pdf or txt
Download as pdf or txt
You are on page 1of 328

BUSINESS MATHS & STATISTICS (TC3)

TECHNICIAN DIPLOMA IN ACCOUNTING

THE INSTITUTE OF
T

MALAW
I
CHARTERED ACCOUNTANTS
BUSINESS MATHS & STATISTICS (TC3) IN MALAWI



‘January2014
BUSINESSMATHS&
STATISTICS(TC3)
TECHNICIAN DIPLOMA IN
ACCOUNTING

INSTITUTEOFCHARTEREDACCOUNTANTS
3
INMALAWI(ICAM)
BUSINESS MATHS & STATISTICS (TC3)
Copyright © The Institute of Chartered Accountants in Malawi – 2014

The Institute of Chartered Accountants in Malawi


P.O. Box 1 Blantyre
E-mail: icam@icam.mw
www.icam.mw

ISBN: 978-99908-0-414-0

All rights reserved. No part of this book may be reproduced or transmitted in any form or by any means-
graphic, electronic or mechanical including photocopying, recording, taping or information storage and
retrieval systems-without the written permission of the copyright holder.

Design
PRISM Consultants
prismmw@gmail.com

BUSINESS MATHS & STATISTICS (TC3)


PREFACE

INTRODUCTION

The Institute noted a number of difficulties faced by students when preparing for the Institute’s
examinations. One of the difficulties has been the unavailability of study manuals specifically
written for the Institute’s examinations. In the past students have relied on text books which were
not tailor-made for the Institute’s examinations and the Malawian environment.

AIM OF THE MAN8ALS

The manual has been developed in order to provide resources that will help the
Institute’s students attain the needed skills. It is therefore recommended that each student
should have their own copy.

HOW TO USE THE MANUAL

Students are being advised to read chapter by chapter since subsequent work often builds on
topics covered earlier.

Students should also attempt questions at the end of the chapter to test their understanding. The
manual will also be supported with a number of resources which students should keep checking
on the ICAM website.

BUSINESS MATHS & STATISTICS (TC3)


Contents
CHAPTER 1 PRELIMINARIES .................................................................................................................. 10
CHAPTER 2 FRACTIONS, DECIMALS AND PERCENTAGES ........................................................... 14
CHAPTER 3 RATIOS AND PROPORTIONS .......................................................................................... 21
CHAPTER 4 FUNCTIONS ........................................................................................................................ 27
CHAPTER 5 SEQUENCES AND SERIES ................................................................................................ 50
CHAPTER 6INEQUALITIES ................................................................................................................... 58
CHAPTER 7 LINEAR PROGRAMMING ................................................................................................. 65
CHAPTER 8 MATRICES ........................................................................................................................... 71
CHAPTER 9 DATA AND DATA COLLECTION .................................................................................... 97
CHAPTER 10 SAMPLING ..................................................................................................................... 106
CHAPTER 11 DATA PRESENTATION ............................................................................................... 116
CHAPTER 12 STATISTICAL MEASURES ......................................................................................... 152
CHAPTER 13 COMBINATORIAL ARITHMETIC .............................................................................. 180
CHAPTER 14 PROBABILITY THEORY.............................................................................................. 185
CHAPTER 15 REGRESSION AND CORRELATION ANALYSIS ..................................................... 201
CHAPTER 16 TIME SERIES ................................................................................................................. 229
CHAPTER 17 INDEX NUMBERS ........................................................................................................ 256
CHAPTER 18 FINANCIAL MATHEMATICS ..................................................................................... 272
CHAPTER 19 CALCULUS .................................................................................................................... 298

BUSINESS MATHS & STATISTICS (TC3)


TC3: BUSINESS MATHEMATICS & STATISTICS

AIM OF THE COURSE


To enable the student to understand mathematical and statistical principles and their applications in
business.

OBJECTIVES
On completion of this module, the candidate will be able to:
x Solve business equations.
x Sketch graphs of business functions
x Solve business problems using techniques of sequences and series
x Use inequalities, where appropriate, to solve simple commercial situations
x Apply the concept of matrices in business
x Describe data collection techniques and sources of data.
x Present data graphically and use data summarization techniques.
x Calculate measures of central tendency and dispersion
x Interpret measures of central tendency and dispersion
x Apply the concept of probability in solving business problems.
x Forecast using business data.
x Determine the degree of relationship between two variables.
x Interpret index numbers
x Perform investment appraisal.
x Apply calculus on revenue, cost and profit functions with the aim of finding optimum points.

FORMAT AND STANDARD OF THE EXAMINATION PAPER

The Business Mathematics and Statistics module will be assessed using a traditional 3 hour paper-based
examination. The examination paper will consist of two sections; section A and section B. Section A will be
compulsory and it will carry 60 marks. Section B will have 3 questions each carrying 20 marks. Candidates
will be required to answer any 2 questions from section B.

BUSINESS MATHS & STATISTICS (TC3)


SPECIFICATION GRID

This grid shows the relative weightings of topics within this course and should provide guidance regarding
study time to be spent on each.

Syllabus Area Weighting %


Functions, Equations and Graphs 10-15
Sequences and series 5-10
Inequalities and Linear Programming 5-10
Matrices 10-15
Sampling Data Presentation 5-10
Data presentation 10-15
Statistical Measures 10-15
Introduction to Probability 5—10
Correlation and Regression 10
Time Series Analysis 5-10
Index Numbers 10-15
Financial Mathematics 15-20
Introduction to Calculus 5-10

Learning Outcomes
1. Functions, equations and graphs
The candidate will be able to formulate and solve equations from real life situations.

In the assessment, the candidate may be required to:


i. formulate equations.
ii. solve linear equations.
iii. solve quadratic equations using graph, factorisation and formula.
iv. solve exponential and logarithmic equations.
v. solve compound interest problems using logarithms.

2. Sequences and series


The candidate will be able to identify patterns in a given set of data and use the appropriate method
to solve real-life problems.

In the assessment, the candidate may be required to:


i. Define an Arithmetic Progression
ii. Identify an Arithmetic Progression
iii. Identify a geometric progression
iv. Determine the ݊௧௛ ‫ ݏ݉ݎ݁ݐ‬of arithmetic and geometric progressions
v. Find the sum of a given AP
vi. Find the sum of a given GP

3. Inequalities and Linear Programming


The candidate will be able to use linear programming techniques to make managerial decisions.

In the assessment, the candidate may be required to:


i. formulate simple linear inequalities
ii. solve linear inequalities

BUSINESS MATHS & STATISTICS (TC3)


iii. apply inequalities to simple commercial situations
iv. find the objective function for a given problem.
v. come up with constraints from a given word problem.
vi. plot inequalities and determine a feasible region for a given word problem.
vii. Use graphical method to find the optimal solution to a linear programming problem.

4. Matrices
The candidate will be able to model relationships between financial or economic variables using a
set of linear equations, represent them using matrices and solve such models.

In the assessment, the candidate may be required to:


i. Represent data with matrices
ii. Add, subtract and multiply matrices
iii. Apply the concept of matrices in manipulating commercial data
iv. Find the determinant of matrices up to 3 by 3.
v. Find the inverse of ʹ ൈ ʹand ͵ ൈ ͵ matrices.
vi. Solve systems of linear equations (up to 3 variables) using the inverse method and Cramer’s
rule.

5. Sampling and data collection


The candidate will be able to choose among the various methods employed in choosing the subjects
for an investigation and differentiate the different ways of collecting data.

In the assessment, the candidate may be required to:


i. Distinguish between data and information
ii. Classify data
iii. Distinguish between data collection methods and select a suitable method
iv. State stages in statistical investigation

6. Data Presentation
The candidate will be able to present data using various data presentation techniques.

In the assessment, the candidate may be required to:


i. Present data using various techniques: pie chart, pictogram, bar chart, frequency tables,
histogram, Ogive, Lorenz curve and Z chart.
ii. Select an appropriate data presentation technique for specific data basing on type of data at
hand, and advantages and disadvantages of the technique.

7. Statistical Measures
The candidate will be able to calculate and interpret measures of central tendency and dispersion.

In the assessment, the candidate may be required to:


i. Calculate measures of central tendency from simple data: mean, mode, median, geometric
mean for grouped and un-grouped data.
ii. Interpret measures of central tendency
iii. Calculate measures of dispersion: range, mean deviation, variance and standard deviation,
quartile deviation, coefficient of variation and skewness for grouped and un-grouped data.
iv. Interpret measures of dispersions
v. Compare distributions using summary measures
7

BUSINESS MATHS & STATISTICS (TC3)


vi. Determine the skewness of a distribution: Pearson’s measure of skewness
vii. Interpret the skewness of a distribution

8. Probability
The candidate will be able to calculate and interpret various types of probability.

In the assessment, the candidate may be required to:


i. Describe the role of probability in decision making
ii. Describe the classical, empirical, and subjective approaches to probability
iii. Distinguish experiment, event and outcome
iv. Apply the rules of probability: addition and multiplication rules
v. Calculate marginal and conditional probabilities
vi. Use a tree diagram to organize and compute probabilities

9. Correlation and regression


The candidate will be able to determine the relationship between two numeric variables compute the
strength of the relationship.

In the assessment, the candidate may be required to:


i. explain the meaning of regression analysis
ii. identify practical examples where regression analysis can be used
iii. plot scatter diagrams
iv. construct a simple linear regression model
v. prepare estimates of the unknown variable using the regression model
vi. compute and interpret the Pearson product moment correlation coefficient
vii. compute and interpret the Pearson product correlation coefficient
viii. compute and interpret the coefficient of determination

10. Times Series Analysis


The candidate will be able to use time series data to forecast events or activities.

In the assessment, the candidate may be required to:


i. Plot time series data
ii. Describe times series models
iii. Distinguish components of a time series: Trend, Seasonal, Random and cyclical variation.
iv. Decompose a time series into its components: Trend and Seasonal variations using moving
averages.
v. Apply time series to make forecasts
vi. Compute and interpret de-seasonalised data.

11. Index Numbers


The candidate will be able to manipulate different published index series and construct new index
series.

In the assessment, the candidate may be required to:


i. Explain what an index number is.
ii. Distinguish between base year and current year
iii. Construct single item indices (price and quantity).
iv. Differentiate between weighted and un-weighted indices.
8

BUSINESS MATHS & STATISTICS (TC3)


v. Change the base of an index number
vi. Calculate the Laspeyres and Paasche Indices and explain the difference
vii. Measure changes in economic data using indices
viii. Adjust nominal money values into real terms (taking inflation into account)

12. Financial Mathematics


The candidate will be able to solve problems involving depreciation, interest calculations and
investment appraisal.

In the assessment, the candidate may be required to:


i. Calculate interest, principal or period in given various combinations of parameters
ii. Describe the different techniques of depreciation.
iii. Depreciate an asset using the various depreciation techniques
iv. Appraise an investment using payback period, Net Present Values and Internal Rate of
Return
v. Compare the various techniques of investment appraisal
vi. Calculate the maturity value of an annuity
vii. Calculate periodic payments for a sinking fund
viii. Calculate the fixed equal payment of annuity given the maturity value
ix. Describe amortisation as a method of debt repayment

13. Introduction to Calculus


The candidate will be able to apply the principles of differentiation and integration and apply these
to rates of change of functions e.g profit function and interpret the results to determine when
functions are at their minimum or maximum.

In the assessment, the candidate may be required to:


i. Differentiate functions up to the second derivative.
ii. Evaluate indefinite and definite integrals.
iii. Find minimum and maximum values of a given function.
iv. Apply calculus on revenue, cost and profit functions with the aim of finding optimum points.

REFERENCES

a. ICAM Business Mathematics and Statistics Manual


b. Clarke, G.M., and Cook, D., (1998) A Basic Course in Statistics, 4th Ed, Arnold
c. Francis, A., (2004) Business Mathematics and Statistics, 6th ed, Thomson Learning
d. Lucey, T., (2002) Quantitative Techniques, 6th E. Thomson Learning: London.
e. Rosser, M., (2003) Basic Mathematics for Economists, London: Routledge

BUSINESS MATHS & STATISTICS (TC3)


CHAPTER 1 PRELIMINARIES

LEARNING OBJECTIVES
By the end of this chapter thestudent should be able to:

i. Use the common symbols used in calculations


ii. Use the rules governing arithmetic operations
iii. Solve problems involving powers

1.0 INTRODUCTION

Mathematics and statistics use a lot of symbols to represent values, concepts, processes and results
management of which would be cumbersome if not impossible, if prose was used. Further calculations
involved are governed by fundamental rules. This chapter introduces the common symbols and basic rules
required for meaningful calculations.

1.1 MATHEMATICAL NOTATION


The term mathematical notation refers to a system of symbolic representations of mathematical objects and
ideas. Mathematical notations are used in mathematics, the physical sciences, engineering, and economics
and accounting.

In its broad sense the term mathematical notations include relatively simple symbolic representations, such
as numbers 1 and 2, function symbols such asǨ and൅; conceptual symbols, such as dy/dx, equations and
variables.

This chapter will introduce various symbols and explain a few functional notations.

1.2 MATHEMATICAL AND STATISTICAL SYMBOLS

The following table shows mathematical and statistical symbols commonly used in Accounting and
commerce.

10

BUSINESS MATHS & STATISTICS (TC3)


Figure 1.1 Mathematical and Statistical symbols

Symbol Meaning
+ Addition Sign. Often referred to as the 'plus' sign.
- Subtraction Sign. Often referred to as the 'minus' sign.
ൈ Multiplication Sign. Often referred to as the 'times' sign.
÷ Division Sign.
= Equal Sign.
™ Summation sign
|| Absolute Value
 Not Equal to.
() Brackets (round).
[] Brackets (square)
% Percent Sign - Out of 100.
ξ Square Root Sign.
< Inequality sign. Less Than.
> Inequality sign. Greater Than.
Ǩ Factorial sign
ൎ Approximately.
œ indefinite integral of or the anti-derivative of a function
1.3
IMPORTANT CONCEPTS AND RULES IN ARITHMETIC OPERATIONS

1.3.1 Number Systems

Certain sets of numbers are so frequently referred to that they are given special symbolic names.

Natural Numbers: This is the set of counting numbers 1,2,3,…. It is usually denoted as ࡺ. So
ࡺ ={1, 2, 3, . . .}. Elements of this set can be multiplied and added with the result being another natural
number. However, if we consider ͳ െ ʹ the result is െͳ which is not a natural number. We will say that a
set of numbers is closed under a given operation if combining two elements under the operation does not
take us out of the given set of numbers. We can therefore say that ࡺis closed under addition and
multiplication but not addition and division.

Integers: This is the set ࢆ = { . . . ,Ѹ2,Ѹ1, 0, 1, 2, . . .} containing natural numbers, zero and negative whole
numbers. ࢆis closed under addition, subtraction and multiplication but not division.

11

BUSINESS MATHS & STATISTICS (TC3)


Rational Numbers: The setof rational numbersdenoted byࡽcontainsall numbers that can be written as

quotient of two integers. We can write ࡽ ൌ  ሼ ǣ ܽǡ ܾ ‫Ͳ ് ܾ݀݊ܽࢆ א‬ሽ. It is worth noting that the set of

integers is a subset of the rationals. In our definition we have said that an element belonging to ࡽ can be
written as a quotient of two integers and as such ߨ is not a rational number. It is not correct to say that ߨ can
ଶଶ ଶଶ ଶଶ
be written as because the value of ߨ is not equal to , however we are allowed to take as an
଻ ଻ ଻
approximation to ߨ. The set ࡽis closed under all the four basic operations of addition, subtraction,
multiplication and division.

Irrational Numbers: These are numbers that cannot be written as a fraction of two integers. We may
denote irrational numbers as ࡵ. They are nonrepeating, nonterminating decimals. ߨ, e and ξʹ are irrational
numbers. ݁has an approximate value of 2.718.

Real Numbers: This set is denoted by ࡾand is the union of the rational and irrational numbers. That is, ࡾ ൌ
ࡽ ‫ࡵ ׫‬. Observe thatࡺ ‫ࡾ ك ࡽ ك ࢆ ك‬.
The set of real numbers is usually pictured as the set of all points on a line, as shown overleaf. The number 0
corresponds to a middle point, called the origin. A unit of distance is marked off, and each point to the right
of the origin corresponds to a positive real number found by computing its distance from the origin. Each
point to the left of the origin corresponds to a negative real number, which is found by computing its
distance from the origin and putting a minus sign in front of the resulting number. The set of real numbers
can be divided into three parts: the set of positive real numbers, the set of negative real numbers, and the
number 0. Note that 0 is neither positive nor negative.

Figure 1.2

1.3.2 Basic arithmetic operations.

Basic arithmetic operations are:


(a) Addition (+)
(b) Subtraction (-),
(c) Division (÷),
(d) Multiplication (x)

1.3.3 BODMAS
An acronym derived from “Brackets Of Division, Multiplication, Addition and Subtraction”. In
mathematics BODMAS is an order of operationsor a rule used to unambiguously clarify which procedures
should be performed first in a given mathematical expression.

Following the letters, the rule is that any items in Brackets should be combined first, division comes next
then multiplication. Addition and subtraction should be performed last.

12

BUSINESS MATHS & STATISTICS (TC3)


Example 3:

1) 3-5 ൈ 4 ÷ 8 + 2 = 3 -5 ൈ 0.5 +2 = 3-2.5 + 2 = 2.5

2) (3 – 5) ൈ 4 ÷ 8 + 2 = -2 ൈ4 ÷ 8 + 2 = -2 x 0.5 +2 = 1

In 1 above, brackets may be used to avoid confusion, thus the expression may also be interpreted as :

3 – (5 x 4 ÷ 8) + 2 = 3 – 2.5 + 2 = 2.5

CHAPTER SUMMARY

In this chapter we have looked at the following:

x Mathematical notation.
x Mathematical symbols
x BODMAS

END OF CHAPTER EXERCISES

ͳǤ State the meanings of the following symbols:Ǩǡ σǡ ൑ǡ േ.


ʹǤ Evaluate
a) െͷ ൈ Ͷ ൊ ʹ ൅ ͺ
b) Ȃ ሺͷ ൈ Ͷሻ  ൊ ʹ ൅ ͺ

3. Evaluate
଻ିଵ
a) ͹ൈͶ൅
ଵସ

଻ିଵ
b) ͹ ൈ ቀͶ ൅ ቁ
ଵସ

13

BUSINESS MATHS & STATISTICS (TC3)


CHAPTER 2 FRACTIONS, DECIMALS AND PERCENTAGES

LEARNING OBJECTIVES
At the end of this chapter the student should be able to.

i. Define a fraction and decimal


ii. Classify fractions
iii. Perform arithmetic operations on fractions and decimals
iv. Use them in a practical situation

2.0 INTRODUCTION
The term “fraction” refers to the concept of “part of a whole”. In practice measurements do not only concern
whole entities. They may involve parts or “fractions” of the entity. A decimal is a fraction expressed as a
part of a ten or power of ten. Thus fractions or decimals are an important aspect of measurement.

2.1 GENERAL EXPRESSION OF A FRACTION



A fraction is a part of an entity and generally a fraction is expressed as where:

x x is referred to as the numerator
x y is referred to as the denominatorǤ ‫ ݕ‬cannot be zero.

Example 1 The following are some fractions:

ଵଷ଻ ͹ ͳ
, , ,െ ,ͳ
ଶସ଺ ͸ ʹ

2.2 TYPES OF FRACTIONS

It is possible to distinguish between 3 types of fractions. These are:

2.2.1 Proper fractions


For these fractions ‫ ݔ‬൏ ‫ ݕ‬where ‫ ݔ‬is the numerator and ‫ ݕ‬denominator.

Example 2 ½, 17/19, -3/4, 7/10, are all proper fractions.

2.2.2 Improper fractions



Generally they take the form with ‫ ݔ‬൐ ‫ݕ‬.

଻ ଺ ଵ଴
Example 3 , , are improper fractions
ଶ ସ ଻

2.2.3 Mixed Numbers

This is where there is a whole number and a fraction.


14

BUSINESS MATHS & STATISTICS (TC3)


ଵ ଵ
Example 4 ͵ ,and ʹ are mixed numbers
ଶ ଷ

2.3 OPERATIONS INVOLVING FRACTIONS

Arithmetic operations on fractions include addition, subtraction, multiplication and addition.

Example 5 Basic arithmetic operations:

a) Addition: ͵ ͳ ͵ൈͳ ͵
ͳ ͵ ʹ൅͵ ͷ ͳ ൈ ൌ ൌ
൅ ൌ ൌ ൌͳ Ͷ ʹ Ͷൈʹ ͺ
ʹ Ͷ Ͷ Ͷ Ͷ

b) Subtraction: ଷ ଵ ଷ ଶ
͵ ͳ ͵െʹ ͳ d) Division: ൊ ൌ ൈ
ସ ଶ ସ ଵ
െ ൌ ൌ
Ͷ ʹ Ͷ Ͷ ͵ൈʹ ͸ ͵ ͳ
ൌ ൌ ൌ ൌͳ
c) Multiplication: Ͷൈͳ Ͷ ʹ ʹ

2.4 DECIMALS

2.4.1 Definition

A decimal is a fraction whose denominator is ten or a power of ten. The following examples show
simple decimals:

Example 6 Write down any 4 simple decimals

Solution 0.5, 0.75, 0.003, 0.8

2.4.2 Representation of Decimals


A decimal (the fraction part) is preceded by a dot “.”.

From Example 6 we have 0.5, 0.75, etc.

It is possible to have a larger number which has a whole number and a fraction part like the case of a mixed
number. The whole number willbe shown followed by a dot and then the fraction part.
For example, 24.75. (This is equivalent to 24¾ in terms of mixed numbers in normal fractions.

2.4.3 Converting fractions to decimal


A normal fraction or a mixed number can be converted to a fraction by dividing the numerator by the
denominator. If the number to be converted if a mixed number, it must be turned into an improper fraction
before conversion.

Example7 Convert a) ͳൗ b) ͷൗ c) ͵ ͳൗͶ d) ͵ ͳൗ͹


ʹ ͺ

15

BUSINESS MATHS & STATISTICS (TC3)



Solution a) ൌ ͳ ൊ ʹ ൌ ͲǤͷ


b)  ൌ ͷ ൊ ͺ ൌ ͲǤ͸ʹͷ

c) ͵ ͳൗͶ  ൌ ͳ͵ȀͶ ൌ ͳ͵ ൊ Ͷ ൌ ͵Ǥʹͷ
d) ͵ ͳൗ͹  ൌ  ʹʹൗ͹ ൌ ʹʹ ൊ ͹ ൌ ͵ǤͳͶ͵

Note that when the decimal part goes on and on as in part d) above, rounding up to a required number of
decimal places is used. See section 2.4.5 below.

2.4.4 Arithmetic operations on decimals


Like the case of fractions there are four basic arithmetic operations as shown below. While the calculations
can be done using a calculator we also encourage you to know how to handle these calculations by hand.

Example 8:

a) Addition: 2.492 +34.124


c) Multiplication:5.2 x 2.31
2.492 231
34.124 ൈ 52
36.616 462
11550
12.012
b) Subtraction:
34.124 Final number of decimals places = total decimals
-9.127 places in multiplicand and places in multiplier
24.997
d) Division: 24.66÷ 1.2
ʹͶǤ͸͸
ൌ ʹʹǤͷͷ
ͳǤʹ
2.4.5 Rounding
Often times figures may have too many digits when a few (shorter string of numbers) are enough to give
a good picture of the value measured. Rounding is that technique used to reduce the number of digits
used to a desired few.

Common forms of rounding are:

(a) to the nearest whole number


(b) to a number of decimal places
(c) to a number of significant figures

Rules for Rounding

(a) If the number you are rounding is followed by 5, 6, 7, 8, or 9, round the number up.
(b) If the number you are rounding is followed by 0, 1, 2, 3, or 4, round the number down.

Significant Figures

16

BUSINESS MATHS & STATISTICS (TC3)


The number of significant digits in an answer to a calculation will depend on the number of significant
digitsin the given data. Non-zero digits are always significant. Thus, 26 has two significant digits, and
27.9 has three significant digits.

With zeroes, the situation is more complicated:


a. Zeroes placed before other digits are not significant; 0.078 has two significant digits.
b. Zeroes placed between other digits are always significant; 609 has three significant digits.
c. Zeroes placed after other digits but behind a decimal point are significant; 8.10 has three significant
digits.
d. Zeroes at the end of a number are significant only if they are behind a decimal point as in (c).
Otherwise, it is impossible to tellif they are significant. For example, in the number 8200, it is not clear if
the zeroes are significant or not. The number ofsignificant digits in 8200 is at least two, but could be
three or four.

We are now ready to look at how one can round a number to a given number of significant figures.
Generally, rounding a number to n significant figures works the same way as rounding. If the first non-
significant figure is a 5,6,7,8,or 9 round up the last significant figure else round down.

Example 9

(a) Round the following to the nearest whole number:


(i) 47.25 =47
(ii) 47.52 = 48
(iii) 100.789 = 101
(iv) 79.49 = 79
(v) 1999.51 = 2000

(b) Round the following to the number of decimal places indicated:


(i) 47.257 to 2 decimal places = 47.26
(ii) 47.529 to 2 decimal places = 47.53
(iii) 100.98 to 1 decimal place =101.0
(iv) 1999.59 to 1 decimal place =1999.6

(c) Rounding to a number of significant figures


(i) 47.257 to 3 significant figures =47.3
(ii) 47.529 to 2 significant figures = 48
(iii) 100.998 to 1 significant figure = 100
(iv) 1999.56 to 3 significant figures = 2000

2.5 PERCENTAGES

A percentage is a way of expressing a number as a fraction of 100. The word per cent means per hundred.
The denominator “100” is not shown but it is denoted by the percent sign “%”

17

BUSINESS MATHS & STATISTICS (TC3)


Example 10

a) The fraction ½ can be expressed as 50/100 which is 50%

b) 4/10 can be expressed as 40/100 which is 40%

2.5.1 Converting Fractions and Decimals to percentages

Converting Fractions to percentage

The process is simply to multiply the fraction with 100 and reduce to lowest terms.

Example 11 Convert the following fractions to percentages


ଷ ଷହ ଷ଴
i) ii) iii)
ସ ଼଴ ଶହ

Solution
ଷ ଷ
i) ൌ ൈ ͳͲͲ ൌ ͹ͷΨ
ସ ସ

ଷହ ଷହ
ii) ൌ ൈ ͳͲͲ ൌ Ͷ͵Ǥ͹ͷΨ
଼଴ ଼଴

ଷ଴ ଷ଴
iii) ൌ ൈ ͳͲͲ ൌ ͳʹͲΨ
ଶହ ଶହ

Converting decimals to Percentages

Just like in the case of fractions, to convert a decimal to a percentage the decimal is multiplied by 100.

Example 12

Convert the following to percentages

i) 0.34 ii) 0.78 iii) 0.8

Solution

i) 0.34 to percentage: ͲǤ͵Ͷ ൈ ͳͲͲ ൌ ͵ͶΨ

ii) 0.78 to percentage: ͲǤ͹ͺ ൈ ͳͲͲ ൌ ͹ͺΨ

iii) 0.8 to percentage: ͲǤͺ ൈ ͳͲͲ ൌ ͺͲΨ


18

BUSINESS MATHS & STATISTICS (TC3)


2.5.3 Arithmetic operations on percentages

Addition and subtraction

As far as simple arithmetic operations are concerned percentages can be added or subtracted in the same
way as integers or normal decimals are added or subtracted.

Example 13

i) ͵ͲΨ ൅ ͶͷΨ ൌ ͹ͷΨ

ii) ͷ͹Ǥ͵Ψ ൅ ʹͷǤʹΨ ൌ ͺʹǤͷΨ

iii) ͺͷΨ െ ʹʹǤͶΨ ൌ ͸ʹǤ͸Ψ

Multiplication and division

Percentages can be multiplied or divided just like fractions and decimals can be multiplied or divided.
However rules governing the multiplication and division of fractions and decimals apply.

Example 14

i) ʹͲΨ ൈ ͷͲΨ (which is reallyʹͲΨ of ͷͲΨ) = ͲǤʹ ൈ ͲǤͷ ൌ ͲǤͳ ൌ ͳͲΨ

ii) ʹͲΨ ൊ ͷͲΨ ൌ ͲǤʹ ൊ ͲǤͷ ൌ ͲǤͶ ൌ ͶͲΨ

OR
ଶ଴ ହ଴
ʹͲΨ ൌ  and ͷͲΨ ൌ 
ଵ଴଴ ଵ଴଴

ଶ଴ ହ଴ ଶ଴ ଵ଴଴ ଶ
Therefore ʹͲΨ ൊ ͷͲΨ ൌ ൊ ൌ ൈ ൌ ൌ ͲǤͶ ൌ ͶͲΨ
ଵ଴଴ ଵ଴଴ ଵ଴଴ ହ଴ ହ

CHAPTER SUMMARY

In this chapter we have looked at the following:


x The difference between proper and improper fractions.
x Addition, subtraction, multiplication and division of fractions.
x Addition, subtraction, multiplication and division of decimals.
x Rounding off a number to a given number of decimal places.
x Rounding off a number to a given number of significant figures.
x Converting fractions and decimals to percentages.
x Arithmetic operations on percentages.

19

BUSINESS MATHS & STATISTICS (TC3)


END OF CHAPTEREXERCISES
1 Arrange the following in ascending order: 2/7, 3/8, 4/10 and 5/12

2 Evaluate: ½ + 1/3 + 2/5

3 Find a) 2/5 - 4/7 b) 2/5 ÷ 4/7

4 What is 20% of 20½

5 Simplify

a) ర
ଷା
మశሺమΤయሻ

ଵ଻ൗ ା൫ଶଵൗ ൊଷൗ ൯


ଷ ସ ସ
b) య
భర
6. A man uses 1/8 of his salary after tax on house rent and a 3/5 on transport, food and other
household items. He then reserves 5/8 of the remainder on leisure and incidentals while saving the
rest. If he saves K37,135 every month.
a) How much is his Salary after tax?
b) How much is the rent per month?
c) What is the percentage of the salary after tax spent on leisure and incidentals?

7. If the inlet pipe for a tank is opened, the tank will be filled in 6 hours. If the outlet pipe for a tank
is opened when the tank is full, the tank will be emptied in 8 hours. How long will it take to fill
the whole tank if both the inlet and outlet pipes are open?

20

BUSINESS MATHS & STATISTICS (TC3)


CHAPTER 3 RATIOS AND PROPORTIONS

LEARNING OBJECTIVES

By the end of this chapter the student should be able to:

i) define a ratio and a proportion


ii) to formulate and apply ratios and proportions
iii) understand the concept of proportional parts and solve associated problems

3.0 INTRODUCTION
Both ratios and proportions refer to a comparison between quantities. Ratios and proportions are used in
everyday life because in most dealings one has to make comparisons between entities.

3.1 RATIOS

3.1.1 Definition
A ratio is a comparison of two quantities. The items that they relate to may or may not be related.

Example 1 A mathematics class has 12 female and 15 male students. What is the ratio of female to
male students.

Solution The ratio of female to male students is 12 to 15 written as 12:15.

3.1.2 Expression of ratios


As indicated above a ratio is expressed as two numbers separated by colon.

Example 2 There are 10 goats and 25 cattle in a feeding pen. Express the number of goats and cattle
as a ratio.

Solution The ratio in this case of goats to cattle is 10:25

Note that a ratio can be expressed as a fraction.

Example 3 Express the ratios in examples 1 and 2 above as fractions

Solution The ratio in example 1 is 12:15 This can be written as 12/15

The ratio in example 2 is 10:25 This is 10/25

Further it is common to express ratios to their lowest terms

Example 4 Express the ratio 12:15 in its lowest terms

Solution Divide both by 3-(the highest common factor).

21

BUSINESS MATHS & STATISTICS (TC3)


The result is 12:15 = 4:5

Example 5 The ratio 10 = 2


15 3

Example 6 A concentrated chemical used to treat termites on a building site has to be diluted as
follows: 1½ litres of the chemical to 30 litres of water. Express the relative quantities of the chemical
and water as a ratio.

Solution The ratio is 1½ to 30 = 3/2 to 30


= 6 : 60
= 1:10 to the lowest terms

3.1.3 Application examples

Example 7 A Toyota Corolla goes 15KM on one litre of petrol. How much petrol will it need to get to
Kasungu from Lilongwe, Kasungu being 120KM away.

Solution The ratio of distance to fuel here is 15:1

Let the fuel required be ‫ݔ‬

Therefore ͳͷ ‫ ͳ ׷‬must equal ͳʹͲǣ‫ݔ‬

15 has increase by a factor 120/15

1 must increase by the same factor: Fuel required is 120 x 1 = 8 lt


15

Example 8 Two people share proceeds of a business venture in the ratio 2:3 If the proceeds came to
MK2,350,000 how much does each get?

Solution Let A and B represent the persons sharing the proceeds

Total parts from the ratio: 2+3=5

A therefore will receive 2 x 2,350,000 = MK94,000


5

B will receive 3 x 2,350,000 = MK1,410,000


5

Example 9 KAWIYA Tea Estate produces a high quality tea branded Kawiya by blending three types
of tea coded A, B, and C in the ration 1½ : 5 : 1. Originally Type A tea costs MK1,600 type B costs
MK800 and type C costs MK1,700 per Kg to produce. Kawiya Tea Estate packs Kawiya tea in packets of
825g each. Blending and packing costs are 40 per Kg.

22

BUSINESS MATHS & STATISTICS (TC3)


Required:
Determine the production cost for a packet of Kawiya tea.

Solution Ratio of the teas in the blend A:B:C = 1½ : 5 : 1


= 3 : 10: 2
Total parts = 3 + 10 + 2 = 15

Amount of tea A in the packet = 825 x 3 = 165g


15

Tea B: 825 x 10 = 550g


15
Tea C: 825 x 2 = 110g
15
Contribution in costs: A 165 x K1.60 = 264
B 550 x K0.80 = 440
C 110 x K1.70 = 187
Total cost of the teas in the packet 891
Blending and packaging 40

Total production cost per packet MK931

Example 10 Two brothers, Mayeso and Khoza earn K80,000 and K150,000 respectively as salaries per
month. The brothers come from a saving culture and Mayeso saves K25,000 while Khoza saves K30,000
per month. Who between the two brothers has a greater savings to salary ratio?

Solution Let “e” and “s” represent earnings and savings respectively

For Mayeso: s : e = 25000 : 80000 = 5 :16


= 5/16
For Khoza s:e = 30000 : 150000 = 1:5
= 1/5
Expressing the ratios to a common denominator:

5/16 = 25/80 and 1/5 = 16/80

Example 11 Mayeso has a greater savings to salary ratio. (note that percentages could have been used)
The rent for the house in which Mr. Chikumbu lives has been increased in the ratio 4:3. If the old rent
was K60,000 per month. Find the new rent.

Solution Let X = new rent.

4:3 = X : 50,000 or 4/3 = X/60000


X = 60,000 x 4/3
= K80,000

23

BUSINESS MATHS & STATISTICS (TC3)


Example 12 Akwawo, Bvuto and Chatha form a business.They agree to share profits in the ration 4:3:1
respectively and at the end of the year they make total sells of K457,820. If there total cost was
K250,000 how much does each receive?

Solution Profit = Revenue-Cost207,820 total profit= 457,820-250 000


= 207,820

Ratio for sharing the profit: 4:3:1 Total parts =4+3+1=8

Akwawo gets 4/8 x 207820 =K103,910.00

Bvuto gets 3/8 x 207820 =K77,932.50.

Chatha gets 1/8x 207820 =K25,977.50

3.2 PROPORTIONS

3.2.1 Definition
A proportion is a relationship between 2 parts of an entity. Emphasis here is on the fact that the parts
would be related. For example car which is supposed to travel a distance of 400 km only does 300 km.
The part it travelled can be expressed as a proportion of the entire distance.

Proportions are based on the concept of ratios (direct proportionality):

Example 13 The Business Mathematics class at your College has 20 female students and 30 male
students. Find

a) The ratio of female to male students.


b) The proportion of female students in the class.

Solution a) Female : Male 20:30 = 2:3

b) Female students/total class = 20/(20 +30)

=20/50 = 2/5 or 40%

Example 14 Chikhosi has just started work and just clocked 12 days to the end of the first month. As
per regulation his pay is to be prorated on days. His monthly salary is MK96,000 and assuming a 30-day
month, calculate his salary for the first month.

Solution Proportion of days on the job:12/30

Pay for the month 96,000 x 12/30 = 3200 x 12 = K38,400

Example 15 James can travel a distance of 10 km in 2hours how long would he take to cover 16 km.

24

BUSINESS MATHS & STATISTICS (TC3)


ଵ଺
Solution ൈ ʹ ൌ ͵Š”•ͳʹ‹ (1 hr =60 min so .2 hrs =12min).
ଵ଴

3.2.2 Proportional Parts


The concept of proportional parts is used to link two or more relationships (ratios) in order to relate the
elements in the different sets of ratios.

Example 16 Given a:b = 3:2 and b:c = 3:5 Find a:b:c

Solution: Step 1 Determine the common element. This is b


Step 2 Make the common element have the same value in each ratio. As a guide use the
lowest common multiple to calculate this value.

In this case we have 2 x 3 = 6

The ratios can be: a:b = ? : 6 and b:c : 6: ?

Step 3 In each ratio the other elements are multiplied or divided by the factor by which
the common element b has been changed.

In a:b, b has been multiplied by 3 therefore element a is multiplied by 3


3 x 3 = 9. Thus a:b 9:6
For b:c, b was multiplied by 2 to get to 6, therefore c should be multiplied by 2.

5 x 2 = 10 Thus b:c 6:10

a:b = 9:6 b:c = 6:10 a:b:c 9:6:10

Example 17 The ages of George and his sister Helen are in the ratio 3:2. The ratio of Georges age to
that of his friend Peter is 7:8. If George’s sister is 21 years old, how old is Peter?

Solution: The ratios are G:H = 3:2, G:P = 7:9

We need: G:H:P

Common element = G
The lowest common multiple is 3 x 7 = 21

Therefore G:H = 3:2 = 21:14, (3:2 multiplied by 7)


G:P = 7:9 = 21:27 (7:9 multiplied by 3)

G:H:P = 21:14:27 Helen is 28years old

Therefore G:H:P = 21:14:27 = ? : 21 : ?

21/14 = 1.5 All other elements should be multiplied by 1.5

Peter’s age = 27 x 1.5 = 40.5 year


25

BUSINESS MATHS & STATISTICS (TC3)


CHAPTER SUMMARY

In this chapter you have learned the following:

i. Definition of ratio and proportion.


ii. Formulated problems involving proportions
iii. Worked out practical examples on ratios and proportions.
iv. Applied the concept of proportional parts and solved associated problems.

END OF CHAPTER EXERCISES


1 Two types of tea costing K650 per kilogram and K720 per kilogram respectively are blended in
the ratio 7:3. Calculate the cost of 1kilogram of the blended tea.

2 A prudent farmer mixes fertiliser 25Kg of fertiliser A and 40Kg of fertiliser B per half acre of a
particular crop. Find the ratio of the two fertilisers in the mixture.

3 Three club employees agree to share their Christmas bonus in the ratios of their ages which are 45
years, 48 years and 51 years. The total bonus is K67,200. How much does each employee
receive?

4 Three persons contribute 200000, 300000 and 150000 to start a business . At the end of the year
their sales amount to K2,450,000 with costs of K505,000. They discover they do not qualify for
corporation tax but under the regime they are supposed to pay a tax calculated as 10% of revenue.
If they are to share the net profit according to their contributions, how much does each get.

5 If a:b = 3:2 and b:c = 3:4 Find a :b :c ;

6 Consider the example of Akwawo, Bvuto and Chatha, if Akwawo is to receive 11/2 as much as
Bvuto gets and Chatha gets ½ as much as Bvuto calculate what each receives if no taxes are
parables.

7 Titha Construction company is pulling down a building to pave way for a new one and it has two
teams that do the job. Team A can pull the building down and clear the site in 3 weeks. Team X
can do the job in 2 days. How long will it take them if they work together?

26

BUSINESS MATHS & STATISTICS (TC3)


CHAPTER 4 FUNCTIONS

OBJECTIVES

By the end of this chapter students should be able to:

i) Define a function.
ii) Distinguish the various types of functions.
iii) Formulate equations.
iv) Solve linear equations.
v) Solve quadratic equations by graph, factorisation and formula.
vi) Solve exponential and logarithmic equations.
vii) Solve compound interest problems using logarithms.

4.0 INTRODUCTION
This chapter seeks to introduce you to the very important concept of a function. The notion of a function
is important in that it gives a formula which relates seemingly (in some cases) unrelated objects. With a
formula in hand, one may be able to determine how a change in one variable affects the other variable.
The chapter will also introduce you to some business applications of functions.

4.1 FUNCTIONS IN GENERAL

4.1.1 Definition of a function.


In a mathematical context, a function is a relationship between variables. It relates an input or a set of
input to a unique output. For example, if we take a function which takes an input and squares it to give
the output, then 4 is the output if we input 2. We can think of a function as a machine which takes the
input, processes it and gives an output. In an algebraic function, one variable depends on or is influenced
by other variables.

4.1.2 Notation of a function


If ‫ ݔ‬is input into a function then it’s output will be denoted ݂ሺ‫ݔ‬ሻ. ݂ሺ‫ݔ‬ሻ is read as ‘ef of x’. We will also
write ‫ ݕ‬ൌ ݂ሺ‫ݔ‬ሻ if ‫ ݕ‬depends on ‫ ݔ‬or ‫ ݕ‬is a function of ‫ݔ‬. We call ݂ሺ‫ݔ‬ሻ the image of ‫ ݔ‬under ݂. Note that
we will also write a function ݂ሺ‫ݔ‬ሻ as ݂.

Example 1 ݂ሺ‫ݔ‬ሻ ൌ ͵‫ ݔ‬൅ ͳͲ is a function. It takes the input ‫ ݔ‬multiplies it by 3 and then adds 10.

Example 2 ‫ ܩ‬ൌ ͵݇ ൅ Ͷ, is a functionwhich can be written as ‫ ܩ‬ൌ ݂ሺ݇ሻ.

Example 3 Given ‫ ܭ‬ൌ  ‫ ݔ‬ଶ  ൅ ͺ‫ݕ‬, K is a function of x and y.

Example 4 ‫ ܯ‬ൌ ‫ ݔ‬ଵΤଶ is not a function since if ‫ ݔ‬ൌ Ͷ then ‫ ܯ‬ൌ േʹ and we know that each input has
to correspond to a unique output.

4.1.3 Domain and Range


A function's input is called the domain, and its output is called the range.

27

BUSINESS MATHS & STATISTICS (TC3)


Example 5 Consider the diagram below:

Figure 4.1 domain of a function

It is clear from the diagram that the domain of the function depicted isሼെͳǡͲǡʹǡͶሽand the range is the set
ሼͳǡͶǡͻሽ.

The domain of a function can be any set of numbers which have images. This could be a few numbers or
the whole set of real numbers. In cases where some elements have no images, such elements must be
excluded from the domain.

Example 6 State the values of‫ݔ‬that must be excluded from the domain of the following functions

a. ݂ሺ‫ݔ‬ሻ ൌ


b. ݃ሺ‫ݔ‬ሻ ൌ ͵ െ
௫ାଷ

Solution:

a. Every real number will have an image under this function except‫ ݔ‬ൌ Ͳ because the function is
undefined when ‫ ݔ‬ൌ Ͳ. So the domain of this functionis the setሼƒŽŽ”‡ƒŽ—„‡”•‫ݔ‬ǣ ‫Ͳ ് ݔ‬ሽ.

b. We begin by writing the expression with a common denominator.

ଷሺ௫ାଷሻିଵ ଷ௫ା଼
Doing this gives ݃ሺ‫ݔ‬ሻ ൌ ൌ . It is not difficult to see that the function is undefined if
௫ାଷ ௫ାଷ

we input ‫ ݔ‬ൌ െ͵(look at the denominator!).

As such the domain of this function is the set of all real numbers except ‫ ݔ‬ൌ െ͵.

Example 7 State the range of ݂ሺ‫ݔ‬ሻ ൌ ‫ ݔ‬ଶ .

Solution: Here the domain is the set of all real numbers. Now, if we input any real number
whether negative or positive we see that the output will be positive as such the range is the set of all
positive real numbers.

28

BUSINESS MATHS & STATISTICS (TC3)


4.2 EVALUATING FUNCTIONS

In this section we will look at how to evaluate functions. Let us do some examples:

Example 8 If ݂ሺ‫ݔ‬ሻ ൌ ‫ ݔ‬ଶ ൅ ʹ‫ ݔ‬െ ͳ, find ݂ሺͳሻǤ

Solution: All we need to do here is put 1 where ever there is ‫ݔ‬. Doing that gives
݂ሺͳሻ ൌ ͳଶ ൅ ʹሺͳሻ െ ͳ ൌ ʹ

Note: To evaluate ‫ ݕ‬ൌ ݂ሺ‫ݔ‬ሻǡeverywhere we see an x on the right side we will substitute whatever is in
the parenthesis on the left side.

Example 9 Given ݂ሺ‫ݔ‬ሻ ൌ െ‫ ݔ‬ଶ ൅ ͸‫ ݔ‬െ ͳͳ, find each of the following.
a) ݂ሺʹሻ
b) ݂ሺ‫ݐ‬ሻ
c) ݂ሺ‫ ݔ‬െ ͵ሻ

Solution
a) ݂ሺʹሻ ൌ െʹଶ ൅ ͸ሺʹሻ െ ͳͳ ൌ െͶ ൅ ͳʹ െ ͳͳ ൌ െ͵.

b) Remember that we substitute for the x’s WHATEVER is in the parenthesis on the left. Often this
will be something other than a number. So, in this case we put t’s in for all the x’s on the left.

This gives݂ሺ‫ݐ‬ሻ ൌ െ‫ ݐ‬ଶ ൅ ͸‫ ݐ‬െ ͳͳ.

c) To find ݂ሺ‫ ݔ‬െ ͵ሻ we evaluate the given function at ‫ ݔ‬െ ͵,


that is we take the input as ‫ ݔ‬െ ͵Ǥ
݂ሺ‫ ݔ‬െ ͵ሻ ൌ െሺ‫ ݔ‬െ ͵ሻଶ ൅ ͸ሺ‫ ݔ‬െ ͵ሻ െ ͳͳ ൌ െሺ‫ ݔ‬ଶ െ ͸‫ ݔ‬൅ ͻሻ ൅ ͸‫ ݔ‬െ ͳͺ െ ͳͳ ൌ െ‫ ݔ‬ଶ ൅ ͳʹ‫ ݔ‬െ ͵ͺǤ

4.3 COMBINING FUNCTIONS

Functions may be added, multiplied, subtracted anddivided to form new functions.


In general we will use the following:
Let f (x) and g(x) be functions then;

1. ሺ݂ േ ݃ሻሺ‫ݔ‬ሻ  ൌ ݂ሺ‫ݔ‬ሻ േ ݃ሺ‫ݔ‬ሻ,


2. ሺ݂݃ሻሺ‫ݔ‬ሻ  ൌ ݂ሺ‫ݔ‬ሻ݃ሺ‫ݔ‬ሻ is the product betweenf (x) and g(x) and
௙ ௙ሺ௫ሻ
3. ሺ‫ݔ‬ሻ ൌ , provided ݃ሺ‫ݔ‬ሻ ് Ͳ.
௚ ௚ሺ௫ሻ

Example 10 Given ݂ሺ‫ݔ‬ሻ ൌ ʹ ൅ ͵‫ ݔ‬െ ‫ ݔ‬ଶ and ݃ሺ‫ݔ‬ሻ ൌ ʹ‫ ݔ‬െ ͳ, evaluate each of the following
a) ሺ݂ ൅ ݃ሻሺͶሻ
b) ሺ݃ െ ݂ሻሺ‫ݔ‬ሻ
c) ሺ݂݃ሻሺ‫ݔ‬ሻ

d) ሺͲሻ.

29

BUSINESS MATHS & STATISTICS (TC3)


Solution: We will do a) and d) and leave b) and c) to the reader.
ሺ݂ ൅ ݃ሻሺͶሻ ൌ ݂ሺͶሻ ൅ ݃ሺͶሻ ൌ ሺʹ ൅ ͵ሺͶሻ െ Ͷଶ ሻ ൅ ሺʹሺͶሻ െ ͳሻ ൌ ͷ.
௙ ௙ሺ଴ሻ ଶ
ሺͲሻ ൌ ൌ ൌ െʹǤ
௚ ௚ሺ଴ሻ ିଵ

4.4 COMPOSITE FUNCTIONS


Consider the function ݂ሺ‫ݔ‬ሻ ൌ ʹ‫ ݔ‬൅ Ͷ. This function maps 4 onto 12, that is, ݂ሺͶሻ ൌ ͳʹ.
Let ݃ሺ‫ݔ‬ሻ be another function such that ݃ሺ‫ݔ‬ሻ ൌ ‫ ݔ‬൅ ͳ. We now apply ݃ to ݂ሺͶሻ to obtain ݃൫݂ሺͶሻ൯ ൌ
݃ሺͳʹሻ ൌ ͳ͵.
So Ͷ has been mapped onto 13 by ݂ followed by ݃. The question is, can we find a single function
݄which combines ݂ሺ‫ݔ‬ሻ and ݃ሺ‫ݔ‬ሻ?

In the example above, ‫ݔ‬is mapped onto ʹ‫ ݔ‬൅ Ͷ by ݂ and this is what we plug into g. So g maps ʹ‫ ݔ‬൅ Ͷ
onto ሺʹ‫ ݔ‬൅ Ͷሻ ൅ ͳ ൌ ʹ‫ ݔ‬൅ ͷ.

Hence ݄ሺ‫ݔ‬ሻ ൌ ʹ‫ ݔ‬൅ ͷ. If ‫ ݔ‬ൌ Ͷ the final result is ݄ሺ‫ݔ‬ሻ ൌ ʹሺͶሻ ൅ ͷ ൌ ͳ͵.
݄ is called the composite function ݃ሺ݂ሺ‫ݔ‬ሻሻ which we write as ݄ ൌ ݃൫݂ሺ‫ݔ‬ሻ൯ ൌ ݃ ‫݂ ל‬ሺ‫ݔ‬ሻ.
Note that in ݃ ‫݂ ל‬ሺ‫ݔ‬ሻ we apply ݂ሺ‫ݔ‬ሻ first and g second.

It is up to reader to confirm that generally


݃ ‫݂ ל‬ሺ‫ݔ‬ሻ ് ݂ ‫݃ ל‬ሺ‫ݔ‬ሻ using ݂ሺ‫ݔ‬ሻ ൌ ʹ‫ ݔ‬൅ Ͷƒ†݃ሺ‫ݔ‬ሻ ൌ ‫ ݔ‬൅ ͳ.
Hence as far as the composition of functions is concerned order is very important.

Example 11 If ݂ሺ‫ݔ‬ሻ ൌ ‫ ݔ‬ଶ and ݃ሺ‫ݔ‬ሻ ൌ ‫ ݔ‬െ ͳǡfind ݂݃and ݂݃.

Solution: ݂݃ ൌ ݂൫݃ሺ‫ݔ‬ሻ൯ ൌ ݂ሺ‫ ݔ‬െ ͳሻ ൌ ሺ‫ ݔ‬െ ͳሻଶ and ݂݃ ൌ ݃൫݂ሺ‫ݔ‬ሻ൯ ൌ ݃ሺ‫ ݔ‬ଶ ሻ ൌ ‫ ݔ‬ଶ െ ͳǤ

4.5 TYPES OF FUNCTIONS


There are many types of functions but of interest to us are linear, quadratic, logarithmic and exponential
functions.

4.5.1 LINEAR FUNCTIONS


a) General form of Linear Functions

In general linear functions are functions which have variables with highest power equal to one. A general
linear function takes the form ݂ሺ‫ݔ‬ሻ ൌ ݉‫ ݔ‬൅ ܿ where m and c are constants and m is nonzero.

Example 12 Which of the following are linear functions?

a. ‫ ݕ‬ൌ ͳͷ‫ ݔ‬൅ ͳͲ


b. ‫ݕ‬ൌ‫ݔ‬
c. ݇ ൌ ʹ‫ ݎ‬൅ ͵

d. ‫ݕ‬ൌ

e. ʹ‫ ݔ‬ൌ ͵ െ ͷ‫ݕ‬
f. ‫ ݕ‬ൌ ‫ ݔ‬ଶ ൅ ͵‫ ݔ‬െ Ͷ

30

BUSINESS MATHS & STATISTICS (TC3)


Solution: a), b), c), and e) are linear equations. d) and f) are not linear equations.

b) Linear Equations

A linear equation is derived from a linear function. The general format for a linear equation is therefore
yൌ ݉‫ ݔ‬൅ ܿ where ݉ ് Ͳand ܿ are constants.

c) Equations from some scenario


Sometimes equations can be presented in a form ready to be solved. At other times one needs to
formulate the equation from a context given.

Example 14
A manufacturer pays K25,000, on material and other variable inputs to produce one unit of a product
DS20. If rent and other fixed cost amount to K120,000 per month. Find an expression of the total cost of
making any quantity of the product.

Solution.

Step 1 Determine items that will vary (the variables). These are quantity and costs and let these
be x and y

Where x = quantity produced


y = Total cost

Step 2 Note and write out the way in which variables change

From the text, variable cost and quantity produced are linearly related
For every unit of x produced the variables cost is 25,000
Therefore if x are produced the total variable cost is ʹͷͲͲͲ‫ݔ‬

Fixed costs do not change.

Step 3 Write out the expression.

Total cost is an addition of the variable and fixed elements.

Therefore total cost = Fixed cost + variable cost


= ͳʹͲͲͲͲ ൅ ʹͷͲͲ‫ݔ‬
This is a linear equation of the form ‫ ݕ‬ൌ ݉‫ ݔ‬൅ ܿ.

d) The Graph of a linear function

The graph of a linear function ‫ ݕ‬ൌ ݂ሺ‫ݔ‬ሻ ൌ ݉‫ ݔ‬൅ ܿ is straight line. There are several ways of drawing the
graph of a linear function:

31

BUSINESS MATHS & STATISTICS (TC3)


i. Using a table of values: In this methodwe create a table to hold x and y values (coordinates) from
the equation. We choose ‫ ݔ‬െvalues according to an appropriate scale, plug the x- values into the given
function to obtain corresponding values of ‫ݕ‬. The ‫ ݔ‬and ‫ ݕ‬values are plotted on a graph and then a line is
drawn to pass through the plotted points.

ii. Plotting the ‫ ݔ‬െ and‫ ݕ‬െintercepts:Here we focus on points where we expect the line to cut the ‫ݔ‬
and ‫ݕ‬axes called the ‫ ݔ‬െ and ‫ ݕ‬െintercepts respectively. The ‫ ݔ‬െ intercept is found by setting ‫ ݕ‬ൌ Ͳ
and solving the equation for ‫ݔ‬. To find the ‫ ݕ‬െ intercept we set ‫ ݔ‬ൌ Ͳ and solve the equation for ‫ݕ‬. The
graph is found by drawing a line through the ‫ ݔ‬and ‫ ݕ‬intercepts.

Example 15: Draw the graph of y = 2x+3.

Solution

First determine the range of x values according to need. For this example x values from -4 to +6 are
adequate. For simplicity the x values should be integers. Substitute the values into the equation to come
up with the table as shown below. This technique is also useful when graphing non-linear equations.

Figure 4.2 Straight line graph

Coordinates:
X Ͳ3 Ͳ2 Ͳ1 0 1 2 3 4 5
Y Ͳ3 Ͳ1 1 3 5 7 9 11 13

Y
16
14
12
10 y=2x+3
8
6
4
2
0
Ͳ4 Ͳ2 2 4 6 X
Ͳ4

Note that the graph is a straight line (all points are on the line).

32

BUSINESS MATHS & STATISTICS (TC3)


Example 16: Plot the graph of ͸‫ ݔ‬൅ ‫ ݕ‬ൌ ͳʹ.

Solution: We will sketch the graph using the ‫ ݔ‬െ and ‫ ݕ‬െ intercepts.

Figure 4.3 Graph from two coordinates


Coordinates X 0 2 (justtwocoordinates(0,12)and(2,0)
Y 12 0 theseareplottedasthedotsbelow

20

10 6x+y=12

Ͳ2 Ͳ1 0 1 2 3 4 5
y

Ͳ10

Note the direction of the line. It is going downwards from left to right. This is a negative slope. The first
was a positive gradient.

e) The slope of a straight line graph

The slope of aline, also called the gradient,is a measure of the steepness of the line. Generally, if the
slope is positive the line moves upward as it moves to the right and if the slope is negative the line moves
downward as it moves to the right. This gives us a way of knowing whether the slope of a line is positive
or negative if we are given its graph. The slope of a line can be deduced from the general form of the
given equation. If we write a linear function in the form ‫ ݕ‬ൌ ݉‫ ݔ‬൅ ܿ, then ݉ is the slope of the line and
ܿis the ‫ ݕ‬െ intercept.

Alternatively the slope can be calculated from any two coordinates on a graph. The slope is the ratio of a
change in the ‫ ݕ‬coordinate to the corresponding change in the ‫ ݔ‬coordinate. This can be written as
ୡ୦ୟ୬୥ୣ୧୬௬ ο௬
•Ž‘’‡ ൌ or, equivalently, as .
ୡ୦ୟ୬୥ୣ୧୬୶ ο௫

୷మ ି୷భ ୷భ ି୷మ
Given the coordinates as ሺ‫ݔ‬ଵ ǡ ‫ݕ‬ଵ ሻ and ሺ‫ݔ‬ଶ ǡ ‫ݕ‬ଶ ሻ then the slope is given by Ž‘’‡ ൌ ൌ .
୶మ ି୶భ ୶భ ି୶మ

Example 17: Given the following linear graph, calculate the slope

33

BUSINESS MATHS & STATISTICS (TC3)


Figure 4.4 Slope of a graph
Y

20

10

Ͳ4 Ͳ2 0 2 4 6 X
Ͳ4
Ͳ6

Solution: Pick any two coordinates as indicated on the graph

Figure 4.5 Slope of a straight line graph


Y

20

10 (6,10)

(2,6)

Ͳ4 Ͳ2 0 2 4 6 X
Ͳ4

We have the two points ሺʹǡ͸ሻ and ሺ͸ǡͳͲሻ, therefore ሺ‫ݔ‬ଵ ǡ ‫ݕ‬ଵ ሻ ൌ ሺʹǡ͸ሻ which means ‫ݔ‬ଵ ൌ ʹ and ‫ݕ‬ଵ ൌ ʹ.
Also, ሺ‫ݔ‬ଶ ǡ ‫ݕ‬ଶ ሻ ൌ ሺ͸ǡͳͲሻ which means ‫ݔ‬ଶ ൌ ͸ and ‫ݕ‬ଶ ൌ ͳͲ. So we have;
୷మ ି୷భ ଵ଴ି଺ ସ
Ž‘’‡ ൌ ൌ = ൌ ͳǤ
୶మ ି୶భ ଺ିଶ ସ

f) Finding the equation of a line

To find the equation of a line we need to know either a point through which the line passes and its
gradient or two points on it.

g) Equation of a line given a point and gradient


௬ି௬భ
The equation of a line through ሺ‫ݔ‬ଵ ǡ ‫ݕ‬ଵ ሻ and having gradient ݉ is given by ൌ ݉. This is equivalent
௫ି௫భ
to ‫ ݕ‬െ ‫ݕ‬ଵ ൌ ݉ሺ‫ ݔ‬െ ‫ݔ‬ଵ ሻ called the point-slope equation of the line.

34

BUSINESS MATHS & STATISTICS (TC3)


h) Equation of a line through two given points
௬ି௬భ ௬మ ି௬భ
The equation of a line through ሺ‫ݔ‬ଵ ǡ ‫ݕ‬ଵ ሻ and ሺ‫ݔ‬ଶ ǡ ‫ݕ‬ଶ ሻ is given by ൌ . You will notice that in this
௫ି௫భ ௫మ ି௫భ
௬మ ି௬భ
equation is an expression for the gradient of the line. This tells us that even if we are given two
௫మ ି௫భ
points through which a line passes we can use the point-slope formula to find its gradient. We will
demonstrate this in the example below.

Example 18Given that a linear equation graph passes through the points (-3,0) and (2,7) find its equation.

Solution
Method 1 The equation takes the form ‫ ݕ‬െ ‫ݕ‬ଵ ൌ ݉ሺ‫ ݔ‬െ ‫ݔ‬ଵ ሻ where slope =݉.
͹െͲ ͹ ଻
Let us calculate݉ǡ ݉ ൌ
ʹെሺെ͵ሻ
ൌ ͷ.So the equation is ‫ ݕ‬െ Ͳ ൌ ହ ሺ‫ ݔ‬െ ሺെ͵ሻሻ which is ͷ‫ ݕ‬ൌ ͹‫ ݔ‬൅ ʹͳ

Method 2Remember that the equation of a line through ሺ‫ݔ‬ଵ ǡ ‫ݕ‬ଵ ሻ and ሺ‫ݔ‬ଶ ǡ ‫ݕ‬ଶ ሻ is given by
௬ି௬భ ௬మ ି௬భ
ൌ .
௫ି௫భ ௫మ ି௫భ
௬ି଴ ଻ି଴ ଻
Here we have the two points ሺെ͵ǡͲሻ and ሺʹǡ͹ሻ so the equation is ൌ ൌ .
௫ିሺିଷሻ ଶିሺିଷሻ ହ
௬ ଻
That is, ൌ .
௫ାଷ ହ
Simplifying this we get ͷ‫ ݕ‬ൌ ͹‫ ݔ‬൅ ʹͳ.

4.5. 2 LINEAR SIMULTANEOUS EQUATIONS


a) Definition

Simultaneous equations are equations which must be satisfied with one set of solutions. Such equations
have two or more variables in the equations

The following are examples of linear simultaneous equations (also referred to as systems of linear
equations).
a. ‫ ݕ‬ൌ Ͷ‫ ݔ‬൅ ʹƒ†‫ ݕ‬ൌ ͵ െ ʹ‫ݔ‬
b. ʹ‫ ݔ‬൅ ͵‫ ݕ‬ൌ ͳͳƒ†š െ › ൌ െʹ
c. ʹ‫ ݔ‬൅ ͵‫ ݕ‬െ ‫ ݖ‬ൌ ͷǡ ‫ ݔ‬െ ‫ ݕ‬൅ ʹ‫ ݖ‬ൌ ͷƒ†͵š ൅ ʹ› ൅ œ ൌ ͳͲ

b) Solving simultaneous equations

Methods of solving simultaneous equations


i. Graphical technique
ii. Elimination method
iii. Substitution method

c) The Graphical technique

For purposes of the level addressed by this text the graphical method is suitable for systems of equations
with two variables and two equations. To determine the solution:
35

BUSINESS MATHS & STATISTICS (TC3)


i. Draw the linear equations on the same axes
ii. Note the values of ‫ ݔ‬and ‫ ݕ‬where the graphs intersect. These values are the solution.

Example 19
Solve the following system of equations using the graphical technique:‫ ݕ‬ൌ ʹ‫ ݔ‬൅ ͳand ‫ ݕ‬ൌ ͻ െ ʹ‫ݔ‬.

Solution: We will use ‫ ݔ‬െ and ‫ ݕ‬െ intercepts to plot the graphs.
For ‫ ݕ‬ൌ ʹ‫ ݔ‬൅ ͳ, when ‫ ݔ‬ൌ Ͳ, ‫ ݕ‬ൌ ͳ and when ‫ ݕ‬ൌ Ͳ, ‫ ݔ‬ൌ െ ͳൗʹ. Therefore the line passes through
the points ሺͲǡͳሻ and ሺെ ͳൗʹ ǡ Ͳሻ.

For ‫ ݕ‬ൌ ͻ െ ʹ‫ݔ‬, when ‫ ݔ‬ൌ Ͳ, ‫ ݕ‬ൌ ͻ and when ‫ ݕ‬ൌ Ͳ, ‫ ݔ‬ൌ ͻൗʹ. Therefore the line passes through the
points ሺͲǡͻሻ and ሺͻൗʹ ǡ Ͳሻ.

Plotting the coordinates we have the following graph

Figure 4.6 Graphical solution of simultaneous equations

10
9 y=2x+1
8
7
Sol: (2,5) 6
5
4
3 y=9Ͳ2x
2
1
Ͳ5 Ͳ4 Ͳ3 Ͳ2 Ͳ1 0 1 2 3 4

From the graph x = 2 and y = 5 (point of intersection)

d) Using the elimination method

In this method we continuously eliminate one variable by making suitable manipulations so that one
equation in one unknown remains.

Example 20 Given y = 2x + 1
y = -2x + 9

Solution

36

BUSINESS MATHS & STATISTICS (TC3)


Number the equations as follows:

y = 2x + 1 equation (1)
y = -2x + 9 equation (2)

Note that the two equations have the coefficients on y and x similar. The variable x can be eliminated by
adding (it has opposite signs).

Thus equation (1) + (2): 2y = 10 therefore y = 10/2 = 5

using equation (1) y = 2x + 1 5 = 2x + 1


2x = 4
x = 4/2 = 2
x = 2 and y = 5

Example 21 Find x and y using the elimination method given


2x + 3y = 11
x – y = -2

Solution 2x + 3y = 11 (1)
x – y = -2 (2)

Since the coefficients on x and y are not the same, make one of them the same:

Multiply equation (2) by 3 and add to eliminate y:

2x + 3y = 11
3x – 3y = -6
The sum 5x + 0 = 5 therefore x = 5/5 = 1

Substitute in (2) (any of the equations can do but this is simpler):

1 - y = -2 y = 3

Therefore x = 1 and y = 3

e) Using the substitution method

Here we solve one equation for one variable in terms of the other. We then substitute this expression into
the other equations to determine the value of the first variable (if possible). Then substitute this value to
determine the value of the other variable.

Example 22 Consider the same problem as above and solve for x and using the using substitution:

Solution 2x + 3y = 11 (1)
x – y = -2 (2)
37

BUSINESS MATHS & STATISTICS (TC3)


From equation (2) x=y–2 (3)
x into equation (1): 2(y- 2) + 3y = 11
2y – 4 + 3y = 11
5y = 15
y = 3

Using (3) again x=y–2


= 3- 2
x = 1

Example 23 The price of the particular product is set at 12 kwacha per item for ten (10) items and falls
to K8.50 per item when 30 are ordered.Derive the Demand function. Assuming it is linear

Solution: Let p = price and q = quantity

As a linier function demand should be

p = a + bq

The data provided is


q p
10 K12
30 K8.50

These can be used to form two equations of the form

12 = a + 10b equation 1
8.5 = a + 30b equation 2

From equation 1 we have

a = 12 – 10b equation 3

Equation 3 into 2

8.5 = 12 – 10b + 30b


-3.5 = 20b
-3.5/20 = b

-0.175 = b

Substitute b into equation 3

a = 12 – 10 x (-0.175)
= 13.75

The demand function: p = 13. 75 – 0.175q


38

BUSINESS MATHS & STATISTICS (TC3)


f) System of equations with 3 unknowns and 3 equations

A system of more than two equations can be solved by using either the substitution method or the
elimination method.

Example 24

Solve the following for x, y and z


4 x  y  2 z 12
3x  2 y  2 z 5
5x  y  z 0

Solution: Let the equations be


4x + y + 2z = 12 (i)
3x – 2y + 2z = 5 (ii)
5x – y – z = 0 (iii)

Using equation (iii) make y the subject.

-y = z – 5x
y = 5x – z (iv)

Substitute (iv) into (i) 4x + (5x – z) + 2z = 12


4x + 5x – z + 2z = 12
9x + z = 12 (v)

Substitute (iv) into (ii) 3x – 2 (5x – 2) + 2z = 5


3x – 10x + 2z + 2z = 5
-7x + 4z = 5 (vi)

We can now solve (v) and (vi) for x and z having eliminated y

9x + z = 12 (v)
-7x + 4z = 5 (vi)

Making z the subject in (v): z = 12 – 9x (v)

Substitute into (vi) -7x + 4(12 - 9x) = 5


-7x + 48 – 36x = 5
-43x = 5 – 48 = -43
 43 x  43
 43 =  43

x = 1

Therefore x = 1
39

BUSINESS MATHS & STATISTICS (TC3)


x into equation (vii) z = 12 – 9 = 3 therefore Z = 3

Using equation (iv) y = 5(1) – 3 = 2

Therefore x, y, z = 1, 2, 3

Example 25 Kupanga ltd makes 3 types of shoes code named X241, Youthquake and Zachangu. Each
of the products pass through three machines in their manufacture. The time in hours each unit takes in a
particular machine is given below:

Product TimespentinMachine
A B C
X241 3 3 1
Youthquake 3 2 3
Zachangu 2 0 1

There 130 hours, 85 hours, and 60 hours of machines A, B, and C respectively, available in a week. If the
aim is to use up all the machine time available, find quantities of x, y and z that can be produced.

Solution Let x represent the units of X241 y, units of Youthquake and z units of Zachangu to be
made,

Total time all products use in machine A 3x + 3y + 2z =130


For machine B: 3x + 2y = 85
For machine C: x + 3y + z = 60

Numbering the equations we have: 3y+3x+2z=130 (i)


3x+2y=85 (ii)
x+3y+z=60 (iii)

In equation (iii) z = 60-x-3y (iv)

z into eq 1 3x+3y+2(60-x-3y)=130
x-3y=10 (v)

Rearranged (v) gives x=10+3y (vi)

(vi) into (ii) 3(10+3y)+2y= 85


30+9y+2y = 85
11y = 55
y = 5
y into (vi) x=10+3y=10+15=25, x=25

Using (iv) z=60-x-3y=60-25-15 z=20

Therefore x = 25, y = 5, z = 20.


40

BUSINESS MATHS & STATISTICS (TC3)


4.5.3 BUSINESS APPLICATIONS
a) Demand Function

Let q be the demand (quantity) of a commodity and p the price of that commodity. The demand function
is defined as q = f(p) where p and q are positive. Generally, p and q are inversely related.

b) Supply Function

Let x denote the amount of a particular commodity that sellers offer in the market at various price p, then
the supply function is given by x = f(p) where x and p are positive.

c) Cost Function

Normally total cost consists of two parts: (i) Variable cost and (ii) fixed cost.
Variable cost is a single -valued function of output, but fixed cost is independent of the level
of output.
Let f (x ) be the variable cost and k be the fixed cost when the output is x units. The total cost function is
defined as ‫ܥ‬ሺ‫ݔ‬ሻ = f(x) + k, where x is positive.
Note that f(x) does not contain a constant term.

Revenue Function
Let x units be sold at p Kwacha per unit. Then the total revenue ܴሺ‫ݔ‬ሻ is defined as R(x) = px, where p and
x are positive.

Profit Function
The profit function ܲሺ‫ݔ‬ሻ is defined as the difference between the total revenue and the total cost.
i.e. ܲሺ‫ݔ‬ሻ  ൌ ܴሺ‫ݔ‬ሻ  െ ‫ܥ‬ሺ‫ݔ‬ሻ.

Example 26: Ifܴሺ‫ݔ‬ሻ ൌ ʹͶͲ ൅ ͳͶ‫ ݔ‬and ‫ܥ‬ሺ‫ݔ‬ሻ ൌ ͳͺ‫ݔ‬, find ܲሺ‫ݔ‬ሻ.

Solution: ܲሺ‫ݔ‬ሻ ൌ ܴሺ‫ݔ‬ሻ െ ‫ܥ‬ሺ‫ݔ‬ሻ so ܲሺ‫ݔ‬ሻ ൌ ʹͶͲ ൅ ͳͶ‫ ݔ‬െ ͳͺ‫ ݔ‬ൌ ʹͶͲ െ Ͷ‫ݔ‬.

Example 27: A rental company purchases a truck for K 1,7000,000. The truck requires an average of
K1250 per day in maintenance.
a. Find the linear function that expresses the total cost C of owning the truck after t days.
b. The truck rents for K 5500 a day. Find the linear function that expresses the revenue R when the
truck has been rented for t days.
c. Find the profit function ܲሺ‫ݐ‬ሻ.
d. Use ܲሺ‫ݐ‬ሻ to determine how many days it will take the company to break even on the purchase of
the truck.

Solution:
a. ‫ܥ‬ሺ‫ݐ‬ሻ ൌ ͳǡ͹ͲͲǡͲͲͲ ൅ ͳʹͷͲ‫ݐ‬
b. Here ܴሺ‫ݐ‬ሻ ൌ ͷͷͲͲ‫ݐ‬
c. Since ܲሺ‫ݐ‬ሻ ൌ ܴሺ‫ݐ‬ሻ െ ‫ܥ‬ሺ‫ݐ‬ሻ we have
ܲሺ‫ݐ‬ሻ ൌ ͷͷͲͲ‫ ݐ‬െ ሺͳ͹ͲͲͲͲͲ ൅ ͳʹͷͲ‫ݐ‬ሻ ൌ ͶʹͷͲ‫ ݐ‬െ ͳ͹ͲͲͲͲͲ
41

BUSINESS MATHS & STATISTICS (TC3)



d. At break-even point ܲሺ‫ݐ‬ሻ ൌ Ͳ i.e 4ʹͷͲ‫ ݐ‬െ ͳ͹ͲͲͲͲͲ ൌ Ͳ. Solving for ‫ ݐ‬we obtain ‫ ݐ‬ൌ ͶͲͲ. 

4.5.4 QUADRATIC FUNCTIONS

a) Form of a quadratic function

Quadratic functions are sometimes known as second order functions. The main feature is that the
independent variable has power equal to 2.

Generally a quadratic function will take the form:

‫ ݕ‬ൌ ܽ‫ ݔ‬ଶ  ൅ ܾ‫ ݔ‬൅ ܿ where ܽ ് ͲǤ

Examples 28 The following are typical quadratic equations:

i. f(x) = x2 – 5x + 6
ii. y = x2 + 9x +8
iii. g(x)=2x2 – 11x + 22
iv. h(x)=3x2 + 7x + 4

b) Solving of a quadratic equation

A quadratic equation can be solved by


i. factorization
ii. formula
iii. graph

Example 29: Solve the following equation:


x2 – 5x + 6 = y
Using the three methods

Solution Note: solving a quadratic equation in general means finding the “roots” of the expression.
This means finding the value of x when y = 0.

Thus we find the value of y when x = 0


Or find the value of x in x2 – 5x + 6 = 0

(a) Solution by factorization


‫ ݔ‬ଶ Ȃ ͷ‫ ݔ‬൅ ͸ ൌ Ͳ
ሺ‫ ݔ‬െ ʹሻሺ‫ ݔ‬െ ͵ሻ  ൌ Ͳ
‫ݔ‬Ȃ ʹ ൌ Ͳ or ‫ݔ‬Ȃ ͵ ൌ Ͳ
‫ ݔ‬ൌ ʹ or ‫ ݔ‬ൌ ͵

(b) Use of the formula


ି௕േξ௕మ ିସ௔௖
Given ܽ‫ ݔ‬ଶ  ൅ ܾ‫ ݔ‬൅ ܿ ൌ Ͳ then ‫ ݔ‬ൌ .
ଶ௔
42

BUSINESS MATHS & STATISTICS (TC3)


So if ‫ ݔ‬ଶ Ȃ ͷ‫ ݔ‬൅ ͸ ൌ Ͳ, a = 1 b = -5 and c = 6

ିሺିହሻേඥሺିହሻమ ିସሺଵሻሺ଺ሻ ହേξଶହିଶସ


Therefore ‫ݔ‬ൌ ൌ ൌ ͵or ʹ.
ଶሺଵሻ ଶ

(c) Graph
To solve a quadratic equation using we need a table of values, plot the points and then read off values of
where the curve crosses the x-axis. See the graph below.

Figure 4.7 Graph


y of a quadratic equation
y 12 6 2 0 0 2 6 12
14

12

10

6 Solution
4

0
x
Ͳ2 Ͳ1 Ͳ2 0 1 2 3 4 5 6 7

Thegraphcutsthexaxiswherex=2and3

Thereforex=2or3

Example 30: A company invests in a particular project and it has estimated that after x months of
running, the cumulative profit in thousands of kwacha from the project is given by the expression
͵ͳǤͷ‫ ݔ‬െ ͵‫ ݔ‬ଶ െ ͸Ͳ where ‫ ݔ‬represents time in months, the project can run for a number of months.

(a) Draw a graph to present the profit function.


(b) Calculate the break even time point for the project
(c) What is the initial cost for the project?
(d) Use the graph to estimate the best time to end the project,

Solution a) The graph is shown overleaf.


b) For break even points Profit =0 i.e. ‫ ݕ‬ൌ ͵ͳǤͷ‫ ݔ‬െ ͵‫ ݔ‬ଶ െ ͸Ͳ ൌ Ͳ

ି௕േξ௕మ ିସ௔௖
Using the general form ‫ݔ‬ൌ
ଶ௔

ିଷଵǤହേඥሺଷଵǤହሻమ ିସሺିଷሻሺି଺଴ሻ
‫ݔ‬ൌ
ଶሺିଷሻ
43

BUSINESS MATHS & STATISTICS (TC3)


െ͵ͳǤͷ േ ξͻͻʹǤʹͷ െ ͹ʹͲ
‫ݔ‬ൌ
െ͸
െ͵ͳǤͷ േ ͳ͸Ǥͷ
‫ݔ‬ൌ
െ͸
ିସ଼ ିଵହ
‫ݔ‬ൌ or ‫ ݔ‬ൌ
ି଺ ି଺

‫ ݔ‬ൌ ͺ‘”ʹǤͷ

Thus the break even is arrived at in 2.5 months or 8months

Graphically:

Figure 4.8 Breakeven and quadratic equation

xistimeinMonths y=profitinK'm

x 0 1 2 3 4 5 6 7 8 9 10
y Ͳ60 Ͳ32 Ͳ9 7.5 18 22.5 21 13.5 0 Ͳ20 Ͳ45

Y 30
Breakeven
20

10

0
0 2 4 6 8 10 X
Ͳ10

Ͳ20

Ͳ30
Profit
Ͳ40 function

Ͳ50

Ͳ60

c) The initial cost is at 0 months. This is K60m (equivalent to profit of -K60m


d) The best time to end the project is month 8. Beyond this the profit is negative

44

BUSINESS MATHS & STATISTICS (TC3)


4.5.5Exponential Functions

a) Definition
An exponential function is a function of the form ݂ሺ‫ݔ‬ሻ ൌ ܽ ௫ where ܽ ൐ Ͳ, ܽ ് ͳ. Here the independent
variable occurs as a power of the base ܽ.

Example 31 The following are examples of exponential functions


a) ݂ሺ‫ݔ‬ሻ ൌ ʹ௫
ଵ ௫
b) ݂ሺ‫ݔ‬ሻ ൌ ቀ ቁ

c) ݂ሺ‫ݔ‬ሻ ൌ Ͷି௫

We restate the properties of exponents for convenience in terms of variable exponents. We assume that
ܽǡ ܾ ൐ Ͳ and that ‫ݔ‬and ‫ ݕ‬are real numbers. We have:

a) ܽ ௫ ܽ ௬ ൌ ܽ ௫ା௬
b) ሺܾܽሻ௫ ൌ ܽ ௫ ܾ ௫
௔ೣ
c) ൌ ܽ ௫ି௬
௔೤
௔ ௫ ௔ೣ
d) ቀ ቁ ൌ ೣ
௕ ௕
e) ሺܽ௠ ሻ௫ ൌ ܽ௠௫ 

ସషయ
Example 32 Use the laws of exponents to rewrite and simplify a) b) ͺయ
ଶషఴ
ସషయ
Solution: a) ൌ Ͷିଷ ൊ ʹି଼ ൌ ሺʹଶ ሻିଷ ൊ ʹି଼ ൌ ʹି଺ ൊ ʹି଼ ൌ ʹି଺ା଼ ൌ ʹଶ ൌ ͶǤ
ଶషఴ
ర భ ସ
b)ͺయ ൌ ቀͺయ ቁ ൌ ʹସ ൌ ͳ͸ .

b) The natural exponential function

In mathematics there is a special number which when used as a base for an exponentialfunction yields
several useful results. The number is ʹǤ͹ͳͺʹͺͳͺ (to 7 dp)and is usually represented by the letter ‘݁’.
Thus the natural exponential function is the function݂ሺ‫ݔ‬ሻ ൌ ݁ ௫ and has the same properties as an
ordinary exponential function.

4.5.6 Logarithmic Functions

a) Definition

A logarithmic function is a function of the form ݂ሺ‫ݔ‬ሻ ൌ ݈‫݃݋‬௔ ‫ ݔ‬where ܽ ൐ Ͳ and ܽ ് ͳ. Exponential
functions and logarithmic are closely related. Generally, if ‫ ݕ‬ൌ ݈‫݃݋‬௔ ‫ݔ‬, then ‫ ݔ‬ൌ ܽ ௬ . This tells us that the
logarithm of ‫ ݔ‬to the base ܽ is the power to which ܽ must be raised to obtain ‫ݔ‬. In the same way if ‫ ݔ‬ൌ
ܽ ௬ , then ‫ ݕ‬ൌ ݈‫݃݋‬௔ ‫ݔ‬. What is the logarithm of Ͷ to base ʹ?

Example 33Without using a calculator, give the exact value of each of the following logarithms.
a) ݈‫݃݋‬ଶ ͳ͸

45

BUSINESS MATHS & STATISTICS (TC3)


b) ݈‫݃݋‬భ ͵͸

ଶ଻
c) ݈‫݃݋‬య
మ ଼
Solution
a) ݈‫݃݋‬ଶ ͳ͸ ൌ ݈‫݃݋‬ଶ ʹସ ൌ Ͷ.
ଵ ௬
b) Let ݈‫݃݋‬భ ͵͸ ൌ ‫ ݕ‬then ͵͸ ൌ ቀ ቁ this can be written as ͸ଶ ൌ ሺ͸ିଵ ሻ௬ ൌ ͸ି௬ . Hence ‫ ݕ‬ൌ െʹ after
ల ଺
equating indices.
c) Left to the reader.

b) Bases of logarithms
Logarithms can be expressed in different bases but the most commonly used are base 10 and base ࢋ.A
logarithm with a base 10 is called a common logarithm and if the base is eit is called the natural
logarithm. These logarithms have their own abbreviations:

a) ݈‫݃݋‬ଵ଴ ‫ ݔ‬is written as Ž‘‰ ‫ݔ‬


b) ݈‫݃݋‬௘ ‫ ݔ‬is written as Ž ‫ ݔ‬read as ‘lin of x’.

Both forms of logarithms can be found on a calculator.

c) The Operations

Operations on logarithms follow rules similar to those of ordinary index numbers. The rules of
logarithms can be summarized into:
x Product rule
x Quotient rule
x Power rule
x Change of base rule

Product Rule
The logarithm of a product is the sum of the logarithms of the factors.

݈‫݃݋‬௔ ሺ‫ݕݔ‬ሻ ൌ  ݈‫݃݋‬௔ ‫ ݔ‬൅  ݈‫݃݋‬௔ ‫ݕ‬

Example 34 ݈‫݃݋‬ଵ଴ ሺʹͲ ൈ ͷሻ ൌ ݈‫݃݋‬ଵ଴ ʹͲ ൅ ݈‫݃݋‬ଵ଴ ͷ ൌ ͳǤ͵ͲͳͲ ൅ ͲǤ͸ͻͻͲ ൌ ʹ

Quotient Rule
The logarithm of a quotient is the logarithm of the numerator minus the logarithm of the denominator.

݈‫݃݋‬௔ ሺ‫ݔ‬Ȁ‫ݕ‬ሻ  ൌ  ݈‫݃݋‬௔ ‫ݔ‬Ȃ݈‫݃݋‬௔ ‫ݕ‬



ଵ଺
Example 35 ݈‫݃݋‬ଶ ቀ ቁ ൌ ݈‫݃݋‬ଶ ͳ͸ െ  ݈‫݃݋‬ଶ Ͷ ൌ Ͷ െ ʹ ൌ ʹ


Power Rule
݈‫݃݋‬௔ ‫ ݔ‬௡  ൌ ݈݊‫݃݋‬௔ ‫ݔ‬

46

BUSINESS MATHS & STATISTICS (TC3)


Example 36 ݈‫݃݋‬ଵ଴ ͳ͸ଶ ൌ ʹ݈‫݃݋‬ଵ଴ ͳ͸

Change of Base Rule

Sometimes, you may be required to convert between bases. Using some simple algebra, a formula can be
derived for changing bases:
Example Given a theoretical problem
ܽ௠  ൌ ܾ
Solve it for m (i.e. make m the subject).

Solution There are two ways to solve it

Method (a) Method (b)


Use the log base a: Take log base c:
݈‫݃݋‬௔ ሺܽ௠ ሻ ൌ ݈‫݃݋‬௔ ܾ ݈‫݃݋‬௖ ሺܽ௠ ሻ ൌ  ݈‫݃݋‬௖ ܾ
݈݉‫݃݋‬௔ ܽ ൌ  ݈‫݃݋‬௔ ܾbut ݈‫݃݋‬௔ ܽ ൌ ͳ ݈݉‫݃݋‬௖ ܽ ൌ  ݈‫݃݋‬௖ ܾ
So we have m = ݈‫݃݋‬௔ ܾ ݉ ൌ ݈‫݃݋‬௖ ܾȀ݈‫݃݋‬௖ ܽ

Example 37: Find an expression, in terms logarithms to base e, for݈‫݃݋‬ହ ͳͲ and give an approximate value
for the quantity.
௟௡ଵ଴
Solution: From the change-of-base formula, ݈‫݃݋‬ହ ͳͲ ൌ ൎ ͳǤͶ͵
௟௡ହ

d) Application of Exponential and Logarithmic Functions

It is worth noting that exponential and logarithmic functions are inverse operations of one another so a
problem involving exponentials may require the use of logarithms. It is for this reason that we treat the
applications of exponential and logarithmic functions in one section.

We now consider some of the applications in which exponential and logarithmic functions are used.

e) Compound Interest:

Ifa principal ofܲkwacha is invested at an annual rate of interest ‫ݎ‬, and the interest is compounded ݊
times per year, then the amount of money ‫ܣ‬ሺ‫ݐ‬ሻ generated at time ‫ݐ‬is given by the formula:
‫ ݎ‬௡௧
‫ܣ‬ሺ‫ݐ‬ሻ ൌ ܲ ቀͳ ൅ ቁ
݊

Example 38: If K600,000 is invested for 3 years at 8% interest compounded annually at the end of each
year,what will the final value of the investment be?

Solution: Here P=K͸ͲͲǡͲͲͲ, ‫ ݎ‬ൌ ͲǤͲͺ, ‫ ݐ‬ൌ ͵ and ݊ ൌ ͳ. We now plug these values into the formula
above.
଴Ǥ଴଼ ଵሺଷሻ
‫ܣ‬ሺ͵ሻ ൌ ͸ͲͲͲͲͲ ቀͳ ൅ ቁ ൌ ͹ͷͷǡͺʹ͹ǤʹͲ.

47

BUSINESS MATHS & STATISTICS (TC3)


Example 39: An account earning 10% compound interest has K450,000 as the amount. It is known that
the holder invested K180,000 at the beginning, he never added any other deposits and he did not
withdraw any money from the account. How long has the account been running to accumulate the stated
amount?
Solution ‫ ܣ‬ൌ ʹͷ͹ͷͷͲ ܲ ൌ ͳͺͲǡͲͲͲǡ ݊ ൌ ͳ and ‫ ݎ‬ൌ ͲǤͳ
The question is about finding‫ݐ‬. Let us plug the values into the compound interest formula.
ͶͷͲǡͲͲͲ ൌ ͳͺͲͲͲͲሺͳ ൅ ͲǤͳሻ௧ ൌ ͳͺͲͲͲͲሺͳǤͳሻ௧

It is easy to see that the value we are looking for is an exponent. To find ‫ ݐ‬therefore we need to use
logarithms.
ସହ଴ǡ଴଴଴ ହ
We have ͶͷͲǡͲͲͲ ൌ ͳͺͲͲͲͲሺͳǤͳሻ௧ from which we obtain  ൌ ൌ ሺͳǤͳሻ௧ .
ଵ଼଴଴଴଴ ଶ

Now taking natural logs of both sides we get: ŽሺͳǤͳሻ௧  ൌ ŽሺʹǤͷሻ.
୪୬ሺଶǤହሻ
Therefore, ‫ ݐ‬ൌ ൌ ͻǤ͸ͳyrs.
୪୬ሺଵǤଵሻ

CHAPTER SUMMARY

In this chapter we have looked at the following


x Functions in general (including the definition of a function, domain range , evaluating and
combining functions)
x Types of functions
x linear functions (general form of linear function, formulating equations from some scenario, the
graph of a linear function, using a table of values: the slope of a straight line graph, finding the
equation of a line, linear simultaneous equation, solving simultaneous equations system of
equations with 3 unknowns and 3 equations and business applications
x Quadratic functions (including form of a quadratic function solving of a quadratic equation
x Exponential functions
x Logarithmic functions (definition, product rule, quotient rule, power rule, change of base rule and
application of exponential and logarithmic functions

STUDENT EXERCISES

1. Given ݂ሺ‫ݔ‬ሻ  ൌ ͹‫ ݔ‬ଶ Ȃ ͵‫ ݔ‬and ݃ሺ‫ݔ‬ሻ  ൌ  െͷ‫ ݔ‬ଶ  െ ʹ‫ ݔ‬൅ ͸ find

i) ݂ሺ‫ݔ‬ሻ  ൅ ݃ሺ‫ݔ‬ሻ ii) ݂ሺ‫ݔ‬ሻ  െ ݃ሺ‫ݔ‬ሻ iii) ݂ሺ‫ݔ‬ሻሺ݃‫ݔ‬ሻ

2. Solve the equation: Ͷ‫ ݔ‬ଶ  ൅ ͵‫ ݔ‬െ ʹǤͷ ൌ ͲǤ

3. True or False? If R(x) = 124x, and C(x) = 78-5x + 5005 then the profit function is P(x)=45.5x +
5005.

4. The length of a rectangle is 1cm more than twice its width. Represent the width of the rectangle
by ‫ ݓ‬and write a function to express the perimeter of the rectangle in terms of ‫ݓ‬.

48

BUSINESS MATHS & STATISTICS (TC3)


5. The functions ܴሺ‫ݔ‬ሻ  ൌ ͳͲͲ‫ ݔ‬and ‫ܥ‬ሺ‫ݔ‬ሻ  ൌ ʹͲͲͲ‫ ݔ‬൅ ͷͲ‫ ݔ‬represent revenue and costs of an
organisation. Write out the profit function.

6. Given that the price of an item is K3.50 when 250 items are demanded, but when only 50 are
demanded the price rises to K5.50 per item, identify the linear demand function and calculate the
price per item at a demand level of 115.

7. Assume that the supply function is represented by P=0.05q +10,and the demand function is
represented by P=17-0.02q, where Pis the price in K’000 and q is the quantity.

Required:
If equilibrium price is defined as a situation where demand equals supply, find the equilibrium
price and quantity.

8. The supply of a commodity is related to the price by the relation ൌ ͷඥʹ‫ ݌‬െ ͳͲ. Show that the
supply curve is a parabola. Find its vertex and the price below which supply is 0?

9. An air freight company has determined that its cost of delivering x parcels per flight is
‫ܥ‬ሺ‫ݔ‬ሻ  ൌ ʹͲʹͷ ൅ ͹‫ݔ‬.
The price it charges to send x parcels is
‫݌‬ሺ‫ݔ‬ሻ ൌ ʹʹ െ ͲǤͲͳ‫ݔ‬Ǥ
Determine
(a) The revenue function.
(b) The profit function.
(c) The company's maximum profit.
(d) The price per parcel that yields the maximum profit.
(e) The minimum number of parcels the air freight company must ship to break even.

49

BUSINESS MATHS & STATISTICS (TC3)


CHAPTER 5 SEQUENCES AND SERIES

OBJECTIVES

By the end of this chapter students should be able to:

i. Define a sequence and series


ii. Give examples of a sequence
iii. Give examples of a series
iv. Define an Arithmetic Progression
v. Identify an Arithmetic Progression
vi. Identify a geometric progression
vii. Determine the ݊௧௛ ‫ ݏ݉ݎ݁ݐ‬of arithmetic progressions and geometric progressions
viii. Find the sum of a given AP
ix. Find the sum of a given GP

5.0 INTRODUCTION
In general mathematics deals with patterns, whether they are visual patterns or numerical patterns. For
example, exponential growth is a growth pattern that is shared by populations, bank accounts etc.
Sequences and Series deal with numerical patterns. In this chapter we will look at two types of
sequences- arithmetic and geometric sequences, but first we start with a general overview of sequences
and series.

5.1 SEQUENCES AND SERIES OVERVIEW

5.1.1 Sequences
A sequence is a list of numbers, called terms arranged in a definite order.

Example 1

i. 3, 7, 11, 15, . . .
In this sequence each term is obtained by adding 4 to the previous term. So the nextterm would be
19.

ii. 4, 9, 16, 25, . . .


This sequence can be rewritten as ʹଶ ǡ ͵ଶǡ Ͷଶ ǡ ͷଶ ǡ ǥ . The next term would be ͸ଶ or 36.

The dots (…) indicate that the sequence continues indefinitely – an infinite sequence.

A sequence such as 3, 6, 9, 12 (stopping after a finite number of terms) is a finite sequence.

Suppose we write ܽଵ for the first term of a sequence, ܽଶ for the second and so on. There maybe a
formula for the݊௧௛ ‫ܽ݉ݎ݁ݐ‬௡ :

50

BUSINESS MATHS & STATISTICS (TC3)


Example 2
The ݊௧௛ term of a sequence
(i) 4, 9, 16, 25, . . . The formula for the ݊௧௛ term is ܽ௡ ൌ ሺ݊ ൅ ͳሻଶ .

(ii) The sequence : 5, 7, 9, 11, . . ., would be given by the formula: ܽ௡ ൌ ʹ݊ ൅ ͵

(iii) ܽ௡ ൌ ʹ݊ ൅ ͳ. The sequence is: 3, 5, 7, 9, . . .

5.1.2 Series
A series is formed when the terms of a sequence are added together. The Greek letter σ
(pronounced “sigma”) is used to denote “the sum of”:

෍ ܽ௜ ‡ƒ•ܽଵ ൅ ܽଶ ൅ ‫ ڮ‬൅ ܽ௡
௜ୀଵ
Examples 3
(i) In the sequence 3, 6, 9, 12, . . . , the sum of the first five terms is the series is

͵ ൅ ͸ ൅ ͻ ൅ ͳʹ ൅ ͳͷ

(ii) σ଺௜ୀଵሺʹ݅ ൅ ͵ሻ ൌ ͷ ൅ ͹ ൅ ͻ ൅ ͳͳ ൅ ͳ͵ ൅ ͳͷ

5.2 ARITHMETIC PROGRESSION (AP)


An arithmetic progression is a sequence in which each term, after the first, can be obtained by adding a
fixed number, called the common difference, to the previous term.

Example 4

The two sequences below are both arithmetic progressions

i. ͳǡ Ͷǡ ͹ǡ ͳͲǡ ǥ Common difference is 3.

ii. ͳ͵ǡ ͹ǡ ͳǡ ǥ Common difference is െ͸

5.2.1 ࢔࢚ࢎ ࢚ࢋ࢘࢓ of an arithmetic progression

Suppose an AP has the first term ܽ and a common difference ݀ then the AP will take the form

ܽǡ ܽ ൅ ݀ǡ ܽ ൅ ʹ݀ǡ ܽ ൅ ͵݀ǡ ǥ

ൌ ܽǡ ܽ ൅ ሺʹ െ ͳሻ݀ǡ ܽ ൅  ሺ͵ െ ͳሻǡ ܽ ൅ ሺͶ െ ͳሻ݀ǡǤǤǤ

From above we can therefore deduce that given the first term ܽ and common difference ݀ of an AP, the
term on any position ݊ of the progression would be given by

ܽ௡ ൌ ܽ ൅ ሺ݊ െ ͳሻ݀

51

BUSINESS MATHS & STATISTICS (TC3)


Where ܽ௡ is the ݊௧௛ term of the AP.

Example 5 Find the ͸௧௛ term of the AP: ͳǡ Ͷǡ ͹ǡ ͳͲǡ ǥ

Solution

݊௧௛ ‫ ݉ݎ݁ݐ‬ൌ ܽ ൅ ሺ݊ െ ͳሻ݀


ܽ ൌ ͳǢ ݀ ൌ ͵
Hence
͸௧௛ ‫ ݉ݎ݁ݐ‬ൌ ͳ ൅ ሺ͸ െ ͳሻ͵ ൌ ͳ͸

Example 6Find the ͳͶ௧௛ term of the AP ʹͲǡ ͳ͹ǡ ͳͶǡ ͳͳǡ ͺǡ ǥ

Solution

ܽ ൌ ʹͲǢ ݀ ൌ െ͵
௧௛
ͳͶ ‫ ݉ݎ݁ݐ‬ൌ ʹͲ ൅ ሺͳͶ െ ͳሻ ൈ ሺെ͵ሻ
ൌ െͳͻ

Example 7The Salary of a teacher grows by K3,000 each year. If the starting salary in his grade is
K18,000, find the teacher’s salary in the fifth year.

Solution

The teacher’s salary at the start of the start of each year would be

‫ͳܭ‬ͺǡͲͲͲǡ ‫ͳܭ‬ͺǡͲͲͲ ൅ ‫͵ܭ‬ǡͲͲͲǡ ‫ͳܭ‬ͺǡͲͲͲ ൅ ‫ܭ‬͸ǡͲͲͲǡ ǥ,

which is an AP with a common difference ‫͵ܭ‬ǡͲͲͲ

Hence the salary at the start of the fifth year would be:

‫ͳܭ‬ͺǡͲͲͲ ൅ ሺͷ െ ͳሻ ൈ ‫͵ܭ‬ǡͲͲͲ ൌ ‫Ͳ͵ܭ‬ǡͲͲͲ

5.2.2 Sum of an Arithmetic Progression

For an AP with a first term ܽ and common difference ݀ the sum of the first ݊ terms of the progression is
given by
݊
ܵ௡ ൌ ሾʹܽ ൅ ሺ݊ െ ͳሻ݀ሿ
ʹ

Where ܵ௡ is the sum of the first ݊ terms of the progression.

Example 8 Find the sum of the following sequence.

2, 5, 8, 11, 14, 17.

52

BUSINESS MATHS & STATISTICS (TC3)


Solution

Adding each of the terms we get the sum


2 + 5 + 8 + 11 + 14 + 17
= 57

However, we note that the above sequence is an AP of 6 terms with the first term ʹ and common
difference ͵

Using the formula

݊
ܵ௡ ൌ ሾʹܽ ൅ ሺ݊ െ ͳሻ݀ሿ
ʹ

To find the sum of those six terms of the AP we have

͸
ܵ଺  ൌ ሾʹ ൈ ʹ ൅ ሺ͸ െ ͳሻ ൈ ͵ሿ
ʹ
ൌ ͵ ൈ ሺͶ ൅ ͷ ൈ ͵ሻ
ൌ ͷ͹

Example 9 A firm rents out its premises and the rental agreement provides for a regular annual
increase at K10,000. If the rent in the first year is K85,000.00

a) What is the rent in the 10th year?


b) How much will the firm have paid cumulatively in the 10 years.

Solution

a) 10th year rent is the 10th term of the sequence of annual payments which form an AP with first
term ‫ܭ‬ͺͷǡͲͲͲ and common difference ‫Ͳͳܭ‬ǡͲͲͲ

ͳͲ௧௛ ‫ ݉ݎ݁ݐ‬ൌ ‫ܭ‬ͺͷǡͲͲͲ ൅ ሺͳͲ െ ͳሻ ൈ ‫Ͳͳܭ‬ǡͲͲͲ


ൌ ‫ͳܭ‬͹ͷǡͲͲͲ

b) Cumulative rental is the sum of the first 10 rentals.



ܵ௡ ൌ ሾʹܽ ൅ ሺ݊ െ ͳሻ݀ሿ

ଵ଴
ܵଵ଴ ൌ ሾ‫ܭ‬ͺͷǡͲͲͲ ൈ ʹ ൅ ሺͳͲ െ ͳሻ ൈ ͳͲǡͲͲͲሿ

ൌ ͷሺ‫ͳܭ‬͹ͲǡͲͲͲ ൅ ‫Ͳͻܭ‬ǡͲͲͲሻ
ൌ ‫ͳܭ‬ǡ͵ͲͲǡͲͲͲ

53

BUSINESS MATHS & STATISTICS (TC3)


5.3 GEOMETRIC PROGRESSION (GP)

A geometric Progression is sequence in which each term can be obtained by multiplying the previous
term by a fixed number, called the common ratio.

Examples of Geometric Progression

i. ͳǡ ʹǡ Ͷǡ ͺǡ ǥ
Each term is double the previous one. The common ratio is 2.

ii. ͺͳǡ ʹ͹ǡ ͻǡ ͵ǡ ͳǡ ǥ The common ratio is

5.3.1 The ࢔࢚ࢎ term of a geometric progression


Suppose a geometric progression has a first term ܽ and a common ratio ‫ݎ‬ǡ then the progression will take
the form

ܽǡ ܽ‫ݎ‬ǡ ܽ‫ ݎ‬ଶ ǡ ܽ‫ ݎ‬ଷ ǡ ǥ

ൌ ܽǡ ܽ‫ ݎ‬ଶିଵ ǡ ܽ‫ ݎ‬ଷିଵ ǡ ܽ‫ ݎ‬ସିଵ ǡ ǥ

It can therefore be deduced from above that the ݊௧௛ term of the progression at any position ݊ ൐ ͳ will be
given by

ܽ௡ ൌ ܽ‫ ݎ‬௡ିଵ

Example 10

a) Find the sixth term of the sequence 1, 2, 4, 8, . . .

b) The first term in a GP is 1000 and its common ratio is 0.8. Find the 5th term.

Solution

We note that each term is twice its predecessor hence the sequence is a geometric progression with the
first term 1 and common ratio 2.

a) ݊௧௛ ‫݉ݎ݁ݐ‬ǣ ܽ௡ ൌ ܽ‫ ݎ‬௡ିଵ

Hence the ͸௧௛ term of the progression is

ܽ଺ ൌ ͳ ൈ ʹ଺ିଵ
ൌ ͳ ൈ ʹହ
ൌ ͵ʹ

54

BUSINESS MATHS & STATISTICS (TC3)


Solution
݊௧௛ ‫݉ݎ݁ݐ‬ǣܽ௡ ൌ ܽ‫ ݎ‬௡ିଵ

ͷ௧௛ ‫ ݉ݎ݁ݐ‬ൌ  ͳͲͲ ൈ ͲǤͺହିଵ

ൌ ͶͲǤͻ͸

5.3.2 Sum of a Geometric Progression


For a geometric progression with a first term ܽ and a common ratio ‫ ݎ‬the sum of the first ݊ terms of the
progression is given by

ܽሺ‫ ݎ‬௡ െ ͳሻ
ܵ௡ ൌ
‫ݎ‬െͳ

ܽሺͳ െ ‫ ݎ‬௡ ሻ

ͳെ‫ݎ‬

If the common ratio ‫ ݎ‬has a numerical value lying between െͳ and ͳ that is ȁ‫ݎ‬ȁ ൏ ͳ, then the sum of an
infinite number of terms of the series denoted ܵஶ or ܵ௡՜ஶ is given by
ܽ
ܵ௡՜ஶ ൌ
ͳെ‫ݎ‬

Example 11
Find the sum of the first 5 terms in the GP 2, 4, 8,...

Solution Sum of a geometric progression:

ܽሺ‫ ݎ‬௡ െ ͳሻ
ܵ௡ ൌ
‫ݎ‬െͳ

ܽ ൌ ʹǢ ‫ ݎ‬ൌ ʹǢ ݊ ൌ ͷ

ʹሺʹହ െ ͳሻ
ܵ௡ ൌ
ʹെͳ

ʹሺ͵ʹ െ ͳሻ

ͳ

ൌ ͸ʹ

Example 12

Find the sum of the infinite sequence 1000, 500, 250, 125, ...

Solution

Note that each term in the sequence after the first can be obtained by multiplying its predecessor by

55

BUSINESS MATHS & STATISTICS (TC3)


Hence ܽ ൌ ͳͲͲͲǢ ‫ ݎ‬ൌ ͲǤͷǡ ݊ ՜ λ
And
ܽ
ܵ௡՜ஶ  ൌ 
ͳെ‫ݎ‬
ͳͲͲͲ

ͳ െ ͲǤͷ

ൌ ʹͲͲͲ

CHAPTER SUMMARY

In this chapter, we have looked at sequences and series with a special focus on

x Arithmetic Progression
ƒ Definition of an Arithmetic Progression
ƒ Finding the ݊௧௛ ‫ ݉ݎ݁ݐ‬of an Arithmetic Progression
ƒ Sum of an Arithmetic Progression
x Geometric Progression
ƒ Definition of an Geometric Progression
ƒ Finding the ݊௧௛ ‫ ݉ݎ݁ݐ‬of an Geometric Progression
ƒ Sum of an Geometric Progression

END OF CHAPTER EXERCISES

1. Insert 5 numbers between 8 and 26 such that the resulting series is an AP.

2. The fifth term of an arithmetic progression is 26 and twelfth is 75. Find the eighth term.

3. Find the first term of an arithmetic progression given that the ʹͲ௧௛ ‫ ݉ݎ݁ݐ‬ൌ ͳͲͲ and the
ʹʹ௡ௗ ‫ ݉ݎ݁ݐ‬ൌ ͳͲͺ.

4. An employee, who received fixed annual increments had a final salary of K90,000 after 10 years.
If her total salary was K650,000 over the 10 yearswhat was her initial salary?

5. James has just been offered a new job. His initial salary is K1, 500,000 per annum, with an
annual increment of K50,000. If this does not change how much will he earn in total over a period
of 12 years?

6. Which term in the GP 2, 4, 8, 16 is 1024

7. Find the sixth term of the geometric progression

a. 128, 96, 72, ...

56

BUSINESS MATHS & STATISTICS (TC3)


b. 3, -6, 12, . . .

8. The third term of a geometric progression is 6, and the eighth term is 192. Find the first term of
the progression and the common ratio,

9. A business man saves from his profits as follows: K3,000 in the first week, K6,000 in the second
week , K12,000 in the third week, K24,000 in the fourth week, and so on. If he keeps on with this
pattern of savings,

a. How much would he save in the tenth week?

b. How much would be his total savings in the first ten weeks?

57

BUSINESS MATHS & STATISTICS (TC3)


CHAPTER 6 INEQUALITIES

LEARNING OBJECTIVES

By the end of this chapter the student should be able to:

i. Define an inequality
ii. Formulate simple linear inequalities
iii. Solve linear inequalities
iv. Sketch a feasible region given linear constraints
v. Apply inequalities to simple commercial situations

6.0 INTRODUCTION

The concept of inequalities is similar to that of equations. The difference is that in an equation one side is
identical in value to the other. In inequalities one side is not necessarily identical to the other. It may be
‘greater than’, ‘greater than or equal to’, ‘less than’, or ‘less than or equal to’.

Just like with equations, the solutionto an inequalityis a value that makes the inequality true. You can
solve inequalities in the same way you solve equations, by following the rules.

Inequalities may be linear or non-linear expressions.

6.1 SYMBOLS USED

< means “less than”


” means “less than or equal to”
> means “greater than”
• means “greater than or equal to”

Example 1 The following expressions are all inequalities

a. ͵‫ ݔ‬൅ Ͷ ൐ ʹ


b. ‫ ݔ‬െ ͷ ൒ Ͳ
c. ʹ െ ͵‫ ݔ‬൏ ͺ
d. ‫ ݔ‬൅ ͵ ൑ ͷ
e. ‫ ݔ‬ଶ  ൅ ‫ ݔ‬െ ͸ ൐ Ͳ

Inequalities a) to d) are all linear while e) is non-linear

6.2 SOLVING LINEAR INEQUALITIES.

58

BUSINESS MATHS & STATISTICS (TC3)


6.2.1 Addition and subtraction of a number

Any positive or negative number may be added to both sides of an inequality.

Example 2 Solve a. ‫ ݔ‬െ ͷ ൒ Ͳ b. ‫ ݔ‬൅ ͵ ൑ ͷ

Solution: a. Add 5 to both sides to find ‫ݔ‬:


‫ ݔ‬െ ͷ ൅ ͷ ൒ Ͳ ൅ ͷ
‫ ݔ‬൒ ͷ

b. Subtract 3 from both sides to find x:


‫ ݔ‬൅ ͵ െ ͵ ൑ ͷ െ ͵
‫ ݔ‬൑ ʹ

6.2.2 Multiplication or division of an inequality by a number


An inequality can be multiplied or divided by a positive number on both sides in the process of
simplifying the expression without affecting the direction of the inequality symbol. However, if an
inequality is multiplied or divided by a negative number, the direction of the inequality sign is reversed.

Example 3

Solve

a. ൏ ʹ

b. ͸‫ ݔ‬൐ ͳͺ
c. െ‫ݔ‬Ȁ͸ ൐ Ͷ
d. െͶ‫ ݔ‬൐ ʹͶ

Solution

௫ ௫
a. ൏ ʹ, ‡ multiply both sides by 3 to get ͵ ൈ ଷ ൏ ʹ ൈ ͵ ֜ ‫ ݔ‬൏ ͸Ǥ

଺௫ ଵ଼
b. ͸‫ ݔ‬൐ ͳͺ. Here we divide both sides by 6, that is, ൐ ֜ ‫ ݔ‬൐ ͵.
଺ ଺


c. െ ൐ ͶǤ To make ‫ ݔ‬the subject we have to multiply both sides by െ͸, doing that gives െ͸ ൈ


െ ൏ Ͷ ൈ െ͸ ֜ ‫ ݔ‬൏ െʹͶ.

ିସ௫ ଶସ
d. െͶ‫ ݔ‬൐ ʹͶ. Here we will divide both sides by -4Ǥ ‡–—•†‘–Šƒ–, ൏ ֜ ‫ ݔ‬൏ െ͸.
ିସ ିସ

6.2.3 Taking the reciprocal of an inequality


Given an inequality, the direction of the sign changes if both sides are inverted.

59

BUSINESS MATHS & STATISTICS (TC3)


Example 4 Take the reciprocal of the following inequality:

‫ ݔ‬൐ ͷ

Solution ͳȀ‫ ݔ‬൏ ͳȀͷ

Key points

i) Adding or subtracting any number does not change the direction of the sign.
ii) Multiplying or dividing by a positive sign does not change the direction of the sign.
iii) Multiplying or dividing by a negative number reverses the inequality symbol.
iv) The reciprocal of the inequality reverses the sign.

6.2.4. Solving General Inequalities


As pointed out earlier, inequalities are solved the same way we solve equations but we have to remember
that when we multiply or divide an inequality by a negative number or take the reciprocal of an
inequality, we have to change the direction of the inequality sign.

Example 5 Solve the following inequalities

ƒሻ Ͷ‫ ݔ‬െ ͵ ൐ ͳͲ


„ሻ ͷ െ ͵Φ‫ ݕ‬൏ ʹͳ

Solution a) Ͷ‫ ݔ‬െ ͵ ൐ ͳͲ Ͷ‫ ݔ‬൐ ͳ͵


‫ ݔ‬൐ ͵Ǥʹͷ

b) ͷ െ ͵Φ‫ ݕ‬൏ ʹͳ െ͵Φ‫ ݕ‬൏ ʹͳ


͹
െ ‫ ݕ‬൏ ʹͳ
ʹ
െ͹‫ ݕ‬൏ Ͷʹ
‫ ݕ‬൐  െ͸ (Note the change in the sign)

6.2.5 GRAPHS OF INEQUALITIES


While the graph of ‫ ݕ‬ൌ ݉‫ ݔ‬൅ ܿ is a line passingthrough all points which satisfy‫ ݕ‬ൌ ݉‫ ݔ‬൅ ܿ, the
graph of an inequality, for example ‫ ݕ‬൏ ݉‫ ݔ‬൅ ܿdefines a region containing solutions to the inequality.

To draw the graph of a linear inequality;

Step 1: Proceed the same way we graph a linearfunction.


Step 2: Draw a dotted line if the inequality is strict(< or >). Else draw a solid line.
Step 3: Now choose a test point.
Step 4: Plug the test point into the given inequality to test if it satisfies the given inequality.
Step 5: If it does, shade the opposite region ELSEshade the region containing the test point.

Example8 Draw the graph of x > 2

Solution First draw the graph of x = 2. This is a vertical line through x = 2


60

BUSINESS MATHS & STATISTICS (TC3)


Figure 6.1 Graph of x < 2

The un-shaded area to the right of the graph contains solutions to x>2.

You will have noticed in the graph of the previous inequality that we shaded the unwanted region but
other authors prefer to shade the wanted region. As such do not be surprised if you come across a book in
which the region containing solutions to the inequality is shaded. In an exam, where all working is done
by hand, we advise that you shade the unwanted side as doing that leaves the region containing solutions
to the inequality clear and not messy.

Let us look at an example where the wanted side is shaded.

Example9 Draw the graph of x>y.

Solution:

Step 1: Draw the graph of ‫ ݔ‬ൌ ‫ݕ‬. This will be a dotted line through the origin.
Step 3: Now choose (2,0) as a test point.
Step 4: (2,0) means ‫ ݔ‬ൌ ʹ and ‫ ݕ‬ൌ Ͳ so the inequality becomes ʹ ൐ Ͳ which is true. So the side
containing ሺʹǡͲሻ is the wanted side.
Step 5: Since in this case we are shading the wanted side, we will shade the region containing the point
ሺʹǡͲሻ as shown below.

61

BUSINESS MATHS & STATISTICS (TC3)


Figure 6.2 Graph of x > y

Example. 10 Draw the graph ofy< 2x + 2

Solution:
Step 1: Draw the graph of ‫ ݕ‬ൌ ʹ‫ ݔ‬൅ ʹ. This will be a solid line through ሺͲǡʹሻ and ሺെͳǡͲሻ.
Here we are just plotting the intercepts.
Step 3: Now choose ሺͳǡͲሻ as a test point.
Step 4: (ͳǡͲሻ means ‫ ݔ‬ൌ ͳ and ‫ ݕ‬ൌ Ͳ so the inequality becomes Ͳ ൏ ͵ which is true. So
the side containing ሺͳǡͲሻ is the wanted side.
Step 5: Since in this case we are shading the unwanted side, we will shade the region not
containing the point ሺͳǡͲሻ as shown below.
The following is the graph:

62

BUSINESS MATHS & STATISTICS (TC3)


Figure 6.3 Graph of y < 2x + 2

CHAPTER SUMMARY

In this chapter we have looked at the following:

x Symbols used when describing an inequality.


x Solving inequalities.
x Graphing linear inequalities

END OF CHAPTER EXERCISES

1 Solve the following inequalities for x

a) x + 5 > 3 b) 5 - 3x ” 14

c) 9x + 9 • 2x - 19 d) 3(2x + 7) – 5 ” 4(x + 1) – x

e) 10x – 5 <45

2 Solve a) 3x + 2 - 1<x
6 3

b) 15/4 > 1/
BUSINESS MATHS & STATISTICS (TC3)
3 Zione the Sells The Daily times by direct delivery to homes in Area 47, Lilongwe. For each
Newspaper delivered she earns K5 and The Newspaper company gives her K5000 each regardless
of how manyNewspapers she is selling.IfZionewants to earn at least K40000 this week, what is
the minimum number of subscriptions she needs to sell?

4 Wezi a self boarding Accountancy student and studying in Blantyre has K30,000 in her account at
the start of the semester which has 21 weeks. She would like to save at K5000 for her transport
back home in Lilongwe. Write out an inequality to express how much Wezi should withdraw
from the account per week for her upkeep.

64

BUSINESS MATHS & STATISTICS (TC3)


CHAPTER 7 LINEAR PROGRAMMING

OBJECTIVES

By the end of this chapter the students should be able to:

i. Find the objective function for a given problem.


ii. Come up with constraints from a given word problem.
iii. Plot inequalities and determine a feasible region for a given word problem.
iv. Find the optimal value of a function for a given word problem.

7.0 INTRODUCTION
This chapter seeks to introduce you to linear programming which is concerned with the problem of
optimizing (maximizing or minimizing) some variable (profit,output,loss etc) subject to some constraints
(costs, availability etc). Linear programming therefore enables a manager to calculate the profit-
maximizing output mix of a multi-product firm subject to restrictionson input availability, or the input
mix that will minimize costs subject to minimum quality standardsbeing met. As such linear
programming is an extremely useful tool for managerial decision-making.

7.1 DEFINITION OF TERMS

In the solution of a linear programming problem one usually meets the terms defined below:
Objective Function:This is the function to bemaximised or minimised.
Constraints: Linear equations or inequalitieswhich restrict the values of the variables.
Feasible Region:A region in the x-y plane which satisfies all of the constraints under consideration.
Let us consider the case of amanager who wishes to decide on the product mixwhich will maximize
profits when his firm has limited amounts of the various inputs required forthe different products that it
makes. The firm’s objective is to maximize profit and so profit is the objective function. He will try to
optimize this function subject to theconstraint of limited input availability. The constraints (there will be
many) determinea feasible region when plotted.

7.2 LINEAR PROGRAMMING MODEL


A general linear programming model takes the following form:

To do: Optimise ܽ‫ ݔ‬൅ ܾ‫ݕ‬.


Subject to:
ܽଵ ‫ ݔ‬൅  ܾଵ ‫ ݕ‬൏ ܿଵ
ܽଶ ‫ ݔ‬൅  ܾଶ ‫ ݕ‬൏ ܿଶ
‫ڭ‬
ܽ௡ ‫ ݔ‬൅  ܾ௡ ‫ ݕ‬൏ ܿ௡ 
where ൏can also be ൑ǡ ൐ǡ ൒ or ൌ.
This tells us that the optimal value of the objective function ܲ ൌ ܽ‫ ݔ‬൅ ܾ‫ ݕ‬will be found subject to a given
number of constraints (݊ in this case).
There are various ways of solving a linear programming problem, the only one relevant to this course is
the graphical approach.

65

BUSINESS MATHS & STATISTICS (TC3)


7.3 GRAPHICAL APPROACH

We give the procedure for solving a linear programming problem.

Input: Objective functionܲ ൌ ܽ‫ ݔ‬൅ ܾ‫ ݕ‬and a set of constraints.


Output: Optimal value of the objective function.

Step 1:Plot the lines corresponding to each of the given constraints.


Step 2: Determine the feasible region. This will be the region satisfying all the given inequalities.
Step 3: The maximum or minimum value of the objective function occurs at avertex of the
feasible region. So to work out the optimum solution, substitute values of ‫ ݔ‬and ‫ ݕ‬from
each vertex of the feasible region into the given objective function. Now determine the
maximum value or minimum value by considering values of the objective function. The
largest value of the objective function corresponds to a vertex which maximizes the
objective and the smallest value of the objective function corresponds to a vertex which
minimizes the objective function.

Note: It is not always the case that the optimal solution lies, as some students would like to believe,
where the constraints they have drawnintersect. See problem 6 in the exercises.

Example 1:Find the maximum and minimum values ofʹ‫ ݔ‬െ ‫ ݕ‬subject to the constraints
ʹ‫ ݔ‬൅ ‫ ݕ‬൑ ͳͲ,‫ ݔ‬൅ ‫ ݕ‬൑ ͸ and ‫ ݔ‬൒ Ͳǡ ‫ ݕ‬൒ Ͳ.

Solution:

Here we would like to optimize ܲ ൌ ʹ‫ ݔ‬െ ‫ݕ‬

Subject to the constraints ʹ‫ ݔ‬൅ ‫ ݕ‬൑ ͳͲǡ ‫ ݔ‬൅ ‫ ݕ‬൑ ͸ and ‫ ݔ‬൒ Ͳǡ ‫ ݕ‬൒ Ͳ.

We will plot the inequalities to obtain the feasible and then plug vertices of the feasible region into ܲ ൌ
ʹ‫ ݔ‬െ ‫ݕ‬. The diagram below shows the feasible region.

66

BUSINESS MATHS & STATISTICS (TC3)


Figure 7.1 Graph of constraints and feasible region

The feasible region has four vertices ሺͲǡͲሻ,ሺͲǡ͸ሻ, ሺͷǡͲሻ and ሺͶǡʹሻ. Now to optimize the objective
functionܲ ൌ ʹ‫ ݔ‬െ ‫ ݕ‬we will plug each vertex intoܲ ൌ ʹ‫ ݔ‬െ ‫ݕ‬. Let us do that.

Table 7.1 Feasible region vertices

Vertex ܲ ൌ ʹ‫ ݔ‬െ ‫ݕ‬


ሺͲǡͲሻ‹Ǥ ‡‫ݔ‬ ൌ Ͳƒ†‫ ݕ‬ൌ Ͳ ܲ ൌ ʹሺͲሻ െ Ͳ ൌ Ͳ
ሺͲǡ͸ሻ‹Ǥ ‡‫ݔ‬ ൌ Ͳƒ†‫ ݕ‬ൌ ͸ ܲ ൌ ʹሺͲሻ െ ͸ ൌ െ͸
ሺͶǡʹሻ‹Ǥ ‡‫ݔ‬ ൌ Ͷƒ†‫ ݕ‬ൌ ʹ ܲ ൌ ʹሺͶሻ െ ʹ ൌ ͸
ሺͷǡͲሻ‹Ǥ ‡‫ݔ‬ ൌ ͷƒ†‫ ݕ‬ൌ Ͳ ܲ ൌ ʹሺͷሻ െ Ͳ ൌ ͳͲ

We can see that the maximum value is ͳͲ and minimum value െ͸.

Example 2: A company manufactures two types of boxes,corrugated and ordinary cartons. The
boxesundergo two major processes: cutting and pinning operations. The profits per unit are K 6and K 4
respectively. Each corrugated boxrequires 2 minutes for cutting and 2 minutes forpinning operation,
whereas each carton boxrequires 3 minutes for cutting and 1 minute forpinning. The available operating
time is 120minutes and 60 minutes for cutting and pinningmachines respectively. Determine the optimum
quantities ofthe two boxes to maximize the profits.

Solution:

Key Decision: To determine how many (numberof) corrugated and carton boxes are to bemanufactured.

67

BUSINESS MATHS & STATISTICS (TC3)


TABLE:We present the information given in a table. You will notice that if we use a table coming up
with the constraints is straight forward.

Table 7.2 Resource unit requirements and unit profits

Process Corrugated Ordinary Available


Cutting 2 3 120
Pinning 2 1 60
Profit K6 K4

Decision variables:Let x and y be the number of corrugated andcarton boxes to be manufactured


respectively.

Objective Function: The objective is to maximize the profits. Given profits on corrugated box andcarton
box are K 6 and K 4 respectively. Therefore the objective function is, ܲ ൌ ͸‫ ݔ‬൅ Ͷ‫ݕ‬Ǥ

Constraints: The available machine-hours foreach machine and the time consumed by eachproduct are
given. Therefore, the constraints are,
ʹ‫ ݔ‬൅ ͵‫ ݕ‬൑ ͳʹͲ
ʹ‫ ݔ‬൅ ‫ ݕ‬൑ ͸Ͳ
As the company cannot produce negative quantities of the two goods, we can also add the twonon-
negativity constraints on the solutions for the optimum values‫ ݔ‬൒ Ͳƒ†‫ ݕ‬൒ Ͳ.
We now plot the inequalities to determine the feasible region. See the graph below.

Figure 7.2 Inequalities and Feasible region

68

BUSINESS MATHS & STATISTICS (TC3)


Note that the coordinates for B can be read off from the graph or can be found by solving the equations
ʹ‫ ݔ‬൅ ͵‫ ݕ‬ൌ ͳʹͲ andʹ‫ ݔ‬൅ ‫ ݕ‬ൌ ͸Ͳ.

The feasible region is clear from the graph and contains the vertices ሺͲǡͲሻ, ሺͲǡͶͲሻ, ሺͳͷǡ͵Ͳሻ and ሺ͵ͲǡͲሻ.
The next thing is to plug these into the objective function. We will do this using a table.

Table 7.2 Feasible region vertices and profit levels

Vertex ܲ ൌ ͸‫ ݔ‬൅ Ͷ‫ݕ‬


ሺͲǡͲሻ‹Ǥ ‡‫ݔ‬ ൌ Ͳƒ†‫ ݕ‬ൌ Ͳ ܲ ൌ ͸ሺͲሻ ൅ ͶሺͲሻ ൌ Ͳ
ሺͲǡͶͲሻ‹Ǥ ‡‫ݔ‬ ൌ Ͳƒ†‫ ݕ‬ൌ ͶͲ ܲ ൌ ͸ሺͲሻ ൅ ͶሺͶͲሻ ൌ ͳ͸Ͳ
ሺͳͷǡ͵Ͳሻ‹Ǥ ‡‫ݔ‬ ൌ ͳͷƒ†‫ ݕ‬ൌ ͵Ͳ ܲ ൌ ͸ሺͳͷሻ ൅ Ͷሺ͵Ͳሻ ൌ ʹͳͲ
ሺ͵ͲǡͲሻ‹Ǥ ‡‫ݔ‬ ൌ ͵Ͳƒ†‫ ݕ‬ൌ Ͳ ܲ ൌ ͸ሺ͵Ͳሻ ൅ ͶሺͲሻ ൌ ͳͺͲ

It is not difficult to see that 15 corrugated boxes and 30 ordinary boxes will yield the maximum profit.

CHAPTER SUMMARY

In this chapter we have looked at the following:

x Definitions of Constraint, Objective function and Feasible Region.


x Graphing a system of linear inequalities to obtain a feasible region.
x Solving linear programming problems using the graphical method.

END OF CHAPTER EXERCISES

1. Find the values‫ ݔ‬and ‫ ݕ‬that maximize the function ܲ ൌ ʹ‫ ݔ‬൅ Ͷ‫ ݕ‬subject to the constraints:
‫ ݔ‬൑ ͶͲͲǡ ‫ ݕ‬൑ ͵ͲͲǡ ‫ ݔ‬൅ ‫ ݕ‬൑ ͸ͲͲ and ‫ ݔ‬൒ Ͳǡ ‫ ݕ‬൒ Ͳ.

2. Show that more than one solution exists if one tries to maximize the objectivefunction
P = 4x + 4y subject to the constraints:
ʹͲ‫ ݔ‬൅ ʹͲ‫ ݕ‬൑ ͸Ͳ
ʹͲ‫ ݔ‬൅ ͺͲ‫ ݕ‬൑ ͳʹͲ
‫ ݔ‬൒ Ͳ‫ ݕܤ‬൒ Ͳ

3. Minimize the objective function ‫ ܥ‬ൌ ͳʹ‫ ݔ‬൅ ͺ‫ ݕ‬subject to the constraints
ͳͲ‫ ݔ‬൅ ͶͲ‫ ݕ‬൒ ͶͲ
ͳʹ‫ ݔ‬൅ ͳ͸‫ ݕ‬൑ Ͷͺ
‫ ݔ‬ൌ ͳǤͷ

4. Find the minimum value of the function C = 40A+20B subject to the constraints
ͳͲ‫ ܣ‬൅ ͶͲ‫ ܤ‬൒ ͶͲ
͵Ͳ‫ ܣ‬൅ ʹͲ‫ ܤ‬൒ ͸Ͳ

69

BUSINESS MATHS & STATISTICS (TC3)


ͳͲ‫ ܣ‬൒ ͳͲ
‫ ܣ‬൒ Ͳǡ ‫ ܤ‬൒ Ͳ

5. Wezi a self boarding Accountancy student and studying in Blantyre has K30,000 in her account at
the start of the semester which has 21 weeks. She would like to save at K5000 for her transport
back home in Lilongwe. Write out an inequality to express how much Wezi should withdraw
from the account per week for her upkeep.

6. A company uses inputs K and L to manufacture goods A and B. It has available 200 unitsof K and
180 units of L and the input requirements are:
10 units of K plus 30 units of L for each unit of A
25 units of K plus 15 units of L for each unit of B
If the per-unit profit is K80 for A and K30 for B, what combination of A and B should it
produce to maximize profit and how much of K and L will be used in doing this?

7 Mr Chiwoza is a farmer in Chitipa, a wheat and paprika growing area. He has 10 acres to plant in
wheat and puprica. Mr. Chiwoza has to plant at least 7 acres. However, he has only K360,000 to
spend and each acre of wheat costs K60,000 to plant and each acre of paprika costs K30,000 to
plant. Moreover, the farmer has to get the planting done in 12 hours and it takes an hour to plant
an acre of wheat and 2 hours to plant an acre of paprika.

Required

a) Identify the constraints for Mr Chiwoza and write them as inequalities.


b) Graph the inequalities and identify the feasible region.

70

BUSINESS MATHS & STATISTICS (TC3)


CHAPTER 8 MATRICES
LEARNING OBJECTIVES

By the end of this chapter the student should be able to:

i. Define a matrix
ii. Distinguish types and orders of matrices
iii. Understand the equality of matrices
iv. Represent data with matrices
v. Add, subtract and multiply matrices
vi. Apply the concept of matrices in manipulating commercial data
vii. Find the determinant of matrices up to 3 by 3.
viii. Find the inverse of 2 u 2 and 3 by 3 matrices..
ix. Solve systems of linear equations (up to 3 variables) using the inverse method and Cramer’s rule.

8.0 INTRODUCTION

You might have seen that there are so many ways in which numerical information is given or stored.
One of such ways is by using tables. Below is a table showing student grades in Mathematics,
Communication, Accounts and Computing for 3 best students at Payelepayele Business School

Table 8.1: Semester I results

Name Mathematics Communication Accounts Computing


Chimwemwe 80 78 55 68
Atupele 82 70 50 65
Ayali 56 80 70 50

Note that in the above table, the grades have been arranged in 3 rows and four columns.

In the second semester, the three pupils obtained the following results:

Table 8.2: Semester II results

Name Mathematics Communication Accounts Computing


Chimwemwe 76 70 65 72
Atupele 70 80 60 55
Ayali 68 70 70 60

We will now look at ways in which we can analyse this information given in the tables.

Example 1
A class tutoris interested in finding the average grades of the students. Show the final grades of the three
students in a table.

71

BUSINESS MATHS & STATISTICS (TC3)


Solution

It is easy to see that to get a grade of a pupil in a particular subject, we will add the two grades of a pupil
corresponding to each subject and then divide by 2.

Table 8.3: Average grades

Name Mathematics Communication Accounts Computing


Chimwemwe 80  76 76  70 55  65 68  72
2 2 2 2
Atupele 82  70 70  80 50  60 65  55
2 2 2 2
Ayali 56  68 80  70 70  70 50  60
2 2 2 2

Simplifying, we get:

Table 8.4

Name Mathematics Communication Accounts Computing


Chimwemwe 78 73 60 70
Atupele 76 75 55 60
Ayali 62 75 70 55

Note again that by adding and dividing by 2 the corresponding entries of the two tables with 3 rows and 4
columns, we have ended up with a table having the same 3 rows and 4 columns.

The method of keeping information using tables can be simplified by simply presenting the numbers in
rows and columns rounded with brackets.

Thus the information, say from Table 8.4, could simply appear as shown below

§ 78 73 60 70 ·
¨ ¸
¨ 76 75 55 60 ¸
¨ 62 75 70 55 ¸
© ¹

Thus we have conveniently represented Table 8.4 as a rectangular array of numbers, a mathematical word
of describing this is MATRIX and to show that the matrix has 3 rows and 4 columns, we say that it is a 3
by 4 matrix. The following definitions summarise the ideas.

8.1 DEFINITION OF KEY TERMS

A matrix(plural matrices)is a rectangular array of numbers arranged by rows and columns. These
numbers are the elements of the matrix and usually a matrix is written by enclosing the elements in
72

BUSINESS MATHS & STATISTICS (TC3)


brackets. It is also usual to name a matrix using a letter. A matrix M with r rows and c columns is said to
be an (or a) r u c which is read as ‘r by c’.

The following are illustrations of matrices.

Example 2

We look at the following matrices

§1 3 4 5·
¨ ¸
a) A ¨ 4 2 5 3¸ , b) B  2 0 3 4
¨ 6 1 0 2¸
© ¹

§ 4 ·
¨ ¸
¨  2¸
¨ 1 ¸ §3 9·
c) C d) X ¨¨ ¸¸
¨ ¸ ©8 7 ¹
¨ 3 ¸
¨ 5 ¸
© ¹

Thus since matrix A has 3 rows and 4 columns, it is a 3 u 4 matrix. While B has one row and 4 columns
hence it is a 1u 4 matrix and such matrices are referred to as row matrices. C has 5 rows and 1 column
and so it is a 5 u 1 matrix. By virtue of having one column, it is referred to as a column matrix. Lastly, X
is a 2 u 2 matrix and because the number of rows is equal to the number of columns, such matrices are
referred to as square matrices.

8.2 IDENTIFICATION OF ELEMENTS

Recall that a matrix is a rectangular array of numbers arranged in rows and columns. This means each
element will be in a particular row and column. The rows and columns make a grid which can be used to
uniquely identify an element.

The practice is to start with row then column.

Example 3

§3 9·
In matrix X ¨¨ ¸¸ , 3 can be identified as the element in row 1 column 1 and 8 is in row 2 column 1
©8 7 ¹
and so on.

Using this way of identifying elements, a matrix can be written in general as follows

§ a11 a12 ·
Matrix X ¨¨ ¸¸ and, in general, a i j represents an element in the ith row and jth column.
© a 21 a 22 ¹
73

BUSINESS MATHS & STATISTICS (TC3)


8.3 ORDER OF MATRICES
The term “order” (or dimension) in the context of matrices refers to the size of the matrix and it is
measured in terms of the number of rows and columns.

In section 8.1above, A above is a 3 u 4 (3 by 4) matrix, B is a 1u 4 matrix, C is a 5 u 1 matrix while X is a


2 u 2 matrix.

Example 4

State the order of the following matrices and, where possible, what is the value of element a 2 3 in each of
the matrices?

§2 1 1 ·
§  2 0  1 2 1· ¨ ¸
(a) A ¨¨ ¸¸ b) B ¨3 2 0 ¸
© 3 5  2 3 0¹ ¨ 4 1  2¸
© ¹
§ 3·
¨ ¸
¨ 2¸
c) C ¨ 0¸ d) D 4 3 0  1
¨ ¸
¨  1¸
© ¹

Solution

a) A is a 2 u 5 matrix while a 2 3 = -2
b) B is a 3 u 3 matrix while a 2 3 = 0
c) C is a 4 u 1 matrix while a 2 3 does not exist since there is no 3rd column.
d) D is a 1u 4 matrix while a 2 3 does not exist since there is no 2nd row.

8.4 IMPORTANT TYPES OF MATRICES

8.4.1 The square matrix


A square matrix is a matrix which has its number of rows equal to the number of columns.

Example 5

The following are all square matrices

§2 1 1 ·
§ 2  1· ¨ ¸
a) M ¨¨ ¸¸ b) Q ¨3 2 0 ¸
©0 3 ¹ ¨ 4 1  2¸
© ¹

74

BUSINESS MATHS & STATISTICS (TC3)


§ 1 0 1 2 ·
¨ ¸
¨ 12 9 1  3¸
c) B ¨
3 7 2 1 ¸
¨ ¸
¨ 2 0 3 4 ¸¹
©

8.4.2 The identity matrix


This is a square matrix whose elements consist of ones in the leading diagonal and zeros for the other
elements. It is usually denoted by letter I. The identity matrix has the property

Example 6

The following are all identity matrices

§1 0 0 0·
§1 0 0· ¨ ¸
§1 0· ¨ ¸ ¨0 1 0 0¸
a) I ¨¨ ¸¸ b) I ¨0 1 0¸ c) I ¨0
© 0 1 ¹ ¨0 0 1¸ 0 1 0¸
© ¹ ¨ ¸
¨0 0 0 1 ¸¹
©

8.4.3 The Zero matrix


The ‘Zero’ matrix is a matrix where every element in it is a zero.

§0 0·
Example 7: ¨¨ ¸¸ is a zero matrix of order 2 u 2 .
©0 0¹

0 0 is also a zero matrix of order 1u 2


§ 0 0 0·
¨ ¸
¨ 0 0 0 ¸ is a zero matrix of order 3 u 3
¨ 0 0 0¸
© ¹

8.4.4 Vectors
A vector is either a row or column matrix. A row matrix is a matrix that is made up of only one row. Its
order is 1 u n where n is the number of columns of a particular matrix. On the other hand, a column
matrix is a matrix that is made up of only one column. It has order m u 1 where m is the number of rows
of a particular matrix.

Example 8

The following are vectors:

a) A 4 2 1 is a row vector with order 1u 3

75

BUSINESS MATHS & STATISTICS (TC3)


§8·
¨ ¸
¨ 2¸
b) ¨ 7 ¸ is an example of a column vector with order 5u 1 .
¨ ¸
¨9¸
¨5¸
© ¹

8.4.5 Scalar
In matrix notation, the term scalar refers to a constant (any real number)

Example 9

The following are normal matrices

§8·
¨ ¸
§ 2  1 0· ¨ 2¸
§ 4 3· ¨ ¸ ¨7¸
A ¨¨ ¸¸ , B ¨ 5 1 9 ¸ and C
© 5 1¹ ¨ ¸
¨ 3 12 7 ¸ ¨9¸
© ¹
¨ 5¸
© ¹
The following are scalars

2, 13, 6.5, -5 0.37

Note: Scalars are constants.

8.5 USING MATRICES TO REPRESENT REAL LIFE DATA


Matrices can be used to represent real life data when data is capable of being viewed in two dimensions

Example 10

A company has two branches one located in Blantyre and the other in Mzuzu.There are 10 male and 6
female employees in Blantyre. There are 4 male and 2 female employees in Mzuzu. Represent the data in
matrix form.

Male Female
Solution: Blantyre §10 6 ·
¨ ¸
Mzuzu ¨© 4 2 ¸¹

This is 2 u 2 matrix.
Example 11

The table below shows the average number of passengers on three AXE Bus Service between two towns,
Blantyre and Lilongwe. Also shown is the price of a ticket per person for each service:

76

BUSINESS MATHS & STATISTICS (TC3)


Depature Service
Time Comfort Excursion Economy
6:30 15 35 60
11:30 12 40 70
16:00 8 50 34
Price (K) 4,000.00 2,500.00 2,000.00
Required:
a) Find the total number of passengers departing at 11:30 am
b) Construct two Matrices to represent the information from the table.

Solution

a) Passengers departing at 11:30: 12 + 40 + 70 = 122

b) Let the matrices be named N and P where N represents number of passengers and P represents
prices.

§15 35 60 · § 4000 ·
¨ ¸ ¨ ¸
Then N ¨12 40 70 ¸ and P ¨ 2500 ¸
¨ 8 50 34 ¸ ¨ 2000 ¸
© ¹ © ¹

Note matrix P is a column matrix. It could have as well been written as a row matrix depending
on the desired multiplication result.

8.6 EQUALITY OF MATRICES


Matrices A and B are said to be equal if they are of the same order and corresponding elements are
equal.

This is denoted A B .

§ a11 a12 · §b b ·
Thus if A ¨¨ ¸¸ and B ¨¨ 11 12 ¸¸ , then A B if a11 b11 , a12 b12 , a 21 b21 and a 22 b22
© a 21 a 22 ¹ © b21 b22 ¹

Example 12
§2 4 · §2 4 ·
The following matrices are equal: A ¨¨ ¸¸ and B ¨¨ ¸¸
© 1  3¹ © 1  3¹
i.e. A B .

77

BUSINESS MATHS & STATISTICS (TC3)


Example 13

Given matrix M and N as outlined below:

§x 3 · § 2 3·
¨ ¸ ¨ ¸
M ¨ 1  4 ¸ and N ¨ 1 y¸
¨ 3z 7 ¸ ¨12 7 ¸
© ¹ © ¹

Find the values of x , y and z for which M N.

Solution: Using equality of matrices, corresponding elements must be equal.

i.e. x 2 , y  4 and 3 z 12 i.e. z 4

8.7 ARITHMETIC OPERATIONS ON MATRICES

8.7.1 Addition and subtraction of matrices


When two matrices are being added or subtracted, it is the corresponding elements that get added or
subtracted to form a new matrix. This means matrix addition or subtraction can only be defined if the
matrices being added or subtracted are of the same order.

§ a11 a12 · §b b ·
Thus if A ¨¨ ¸¸ and B ¨¨ 11 12 ¸¸
© a 21 a 22 ¹ © b21 b22 ¹

§ a11 a12 · § b11 b12 · § a11  b11 a12  b12 ·


Then A  B = ¨¨ ¸¨ ¸=¨ ¸
© a 21 a 22 ¸¹ ¨© b21 b22 ¸¹ ¨© a 21  b21 a 22  b22 ¸¹

Note that it is the corresponding elements that add

Example 14
§1 2· §5 6 ·
§ 1 3· § 2 1· ¨ ¸ ¨ ¸
Given that A ¨¨ ¸¸, B ¨¨ ¸¸, C ¨ 6 0 ¸, D ¨2 1 ¸
© 2 4¹ © 3 2¹ ¨1 4¸ ¨ 3  1¸
© ¹ © ¹
Find where possible

(a) A B
(b) CD
(c) A D
(d) BA
(e) B A
(f) DB
(g) A B
78

BUSINESS MATHS & STATISTICS (TC3)


Solution
§ 1 3· §  2 1· §1  (2) 3  1 · § 1 4·
(a) A B ¨¨ ¸¸  ¨¨ ¸¸ ¨¨ ¸¸ ¨¨ ¸¸
© 2 4¹ © 3 2¹ © 2  3 4  2¹ © 5 6¹

§1 2· § 5 6 · §1 5 26 · § 6 8·
¨ ¸ ¨ ¸ ¨ ¸ ¨ ¸
(b) CD ¨6 0¸  ¨2 1 ¸ ¨6  2 0 1 ¸ ¨ 8 1¸
¨ 1 4 ¸ ¨ 3  1¸ ¨ 1  3 4  (1) ¸ ¨ 4 3¸
© ¹ © ¹ © ¹ © ¹

(c) A  B is not possible as A & D have different orders ( A has order 2 u 2 while D has 3u 2 )

§  2 1· §1 3· §  2 1 1 3 · §  3  2·
(d) B A ¨¨ ¸¸  ¨¨ ¸¸ ¨¨ ¸¸ ¨¨ ¸¸
© 3 2¹ © 2 4¹ © 3  2 2  4¹ © 1  2¹

§  2 1· § 1 3· § 2 1 1 3 · § 1 4·
(e) B A ¨¨ ¸¸  ¨¨ ¸¸ ¨¨ ¸¸ ¨¨ ¸¸
© 3 2¹ © 2 4¹ © 3  2 2  4¹ © 5 6¹

(f) D  B is not possible. The two have different orders.

§ 1 3· §  2 1· §1  (2) 3  1 · § 3 2·
(g) A B ¨¨ ¸¸  ¨¨ ¸¸ ¨¨ ¸¸ ¨¨ ¸¸
© 2 4¹ © 3 2¹ © 2  3 4  2¹ © 1 2¹

Notice that A  B B  A and that A  B z B  A .

We,say that Matrix Addition is Commutative i.e. If A and B are matrices of the same order,
A  B B  A . Notice also that Matrix subtraction is NOT commutative.

Example 15

Given matrix H, G and Q as outlined below

§x 3 · §3 3· §y 6 ·
¨ ¸ ¨ ¸ ¨ ¸
M ¨ 1  2¸ , N ¨ 1 y ¸ and Q ¨ 2 2 x ¸
¨2 7 ¸ ¨6 2¸ ¨8 9 ¸
© ¹ © ¹ © ¹

Find the values of x and y if Q M N.

Solution

Now Q M  N implies
79

BUSINESS MATHS & STATISTICS (TC3)


§ y 6 · § x 3 · §3 3·
¨ ¸ ¨ ¸ ¨ ¸
¨ 2 2x ¸ ¨ 1  2¸  ¨ 1 y ¸
¨ 8 9 ¸ ¨2 7 ¸ ¨6 2¸
© ¹ © ¹ © ¹

Using equality of matrices, then

3 x y
y2 2x

By substitution, 3  x  2 2 x
Therefore 3  x  2 2 x
x 1
So y 3  1 4

8.7.2 Multiplication of matrices


Given two matrices A and B. The product C = AB is defined only when the number of columns in A (the
multiplicand) and the number of rows in B the multiplier are the same:

Illustration

§ b11 b12 ·
§ a11 a12 a13 · ¨ ¸
If A ¨¨ ¸¸ and B ¨ b21 b22 ¸
© a 21 a 22 a 23 ¹ ¨b ¸
© 31 b32 ¹

Then the product matrix C = AB is defined because in A, there are 3 columns and in B there are 3 rows.
The product is found as follows:

§ b11 b12 ·
§ a11 a12 a13 · ¨ ¸
C = AB = ¨¨ ¸ ¨ b21 b22 ¸
© a 21 a 22 a 23 ¸¹ ¨b b ¸
© 31 32 ¹
§ a11b11  a12 b21  a13 b31 a11b12  a12 b22  a13 b32 ·
= ¨¨ ¸
© a 21b11  a 22 b21  a13 b31 a 21b12  a 22 b22  a 23 b32 ¸¹

Note
Row 1 elements in A multiply column 1 elements in B. These are added to form element in row one
column 1 in C

The same row 1 element then multiply corresponding elements in column 2 of B. These are added to
form the element in row 1 and column 2 of C

80

BUSINESS MATHS & STATISTICS (TC3)


Row 2 elements in A multiply column 1 elements in B. These are added to form element in row 2 column
1 element in C

The same row 2 elements then multiply corresponding elements in column 2 of B. These are added to
form the element in row 2 and column 2 of C.

Example 16

§4 9 · § 3  2·
Given that A ¨¨ ¸¸ , B ¨¨ ¸¸ .
© 3  6 ¹ © 7 4 ¹

a) Find matrix C such that C AB .


b) Is AB BA ?

Solution

§ 4 9 · § 3  2· § 4 u 3  9 u 7 4 u (2)  9 u 4 ·
a) C AB ¨¨ ¸¸ ¨¨ ¸¸ = ¨¨ ¸¸
© 3  6 ¹© 7 4 ¹ © 3 u 3  ( 6) u 7 3 u ( 2)  ( 6) u 4 ¹
§ 75 28 ·
= ¨¨ ¸¸
©  33  30 ¹

§ 3  2 ·§ 4 9 · § 3 u 4  (2) u 3 3 u 9  (2) u (6) ·


b) BA ¨¨ ¸¸¨¨ ¸¸ = ¨¨ ¸
© 7 4 ¹© 3  6 ¹ © 7 u 4  4 u 3 7 u 9  4 u (6) ¸¹

§ 6 39 ·
= ¨¨ ¸¸
© 40 39 ¹

which is different from AB as calculated above.

NOTE:In normal algebra xy yx but in matrix algebra AB z BA

Example 17

Given the two matrices M and N below. Which of the products, MN or NM, is defined?

§ 3  2·
§4 9 · ¨ ¸
M ¨¨ ¸¸ , N ¨7 4 ¸
© 3  6¹ ¨5 1 ¸
© ¹
Carry out the defined multiplication.

81

BUSINESS MATHS & STATISTICS (TC3)


Solution

Following the rules of matrix multiplication, NM is defined because number of columns in N = number
of rows in M

§ 3  2· § 6 39 ·
¨ ¸§4 9 · ¨ ¸
Then NM ¨ 7 4 ¸ ¨¨ ¸¸ = ¨ 40 39 ¸ .
¨ 5 1 ¸ © 3  6 ¹ ¨ 23 39 ¸
© ¹ © ¹

On the other hand, MN is not possible since the number of columns in M (2) is different from the
number of rows (3) in N.

Example 18

A business man purchases 20 tables, 80 chairs, and 2 water dispensers for his conference room. The
respective unit prices are K35,000, K18,000 and K65,000.

a) Express the quantities and prices in terms of matrices Q and P in such a way that they can be
multiplied.
b) Multiply the two matrices so that the interim result gives the expenditure in each item and the
final result provides the total expenditure.

Solution

From the data the two matrices will be vectors.

a) For multiplication to be possible, one must be a column matrix and the other a row matrix:
§ 35000 ·
¨ ¸
Then Q 20 80 2 , P ¨ 18000 ¸ .
¨ 56000 ¸
© ¹

b) Total expenditure:

§ 35000 ·
¨ ¸
Now 20 80 2 ¨ 18000 ¸ 20 u 35000  80 u 18000  2 u 56000
¨ 56000 ¸
© ¹
= 2,252,000
i.e. total expenditure is K2,252,00

Example 19

Product X has fixed costs of K12,000 and variable costs of K1,200 per product, product Y has fixed cost
of K7,000 and variable cost of K1,600 per product.

82

BUSINESS MATHS & STATISTICS (TC3)


Required:

§12000 1200 · §1·


a) If C ¨¨ ¸¸ and Q ¨¨ ¸¸ , where C is the cost coefficient matrix, evaluate the matrix
© 7000 1600 ¹ ©q¹
product CQ and interpret.

b) Given that product X and Y sell at K1,800 and K2,000 respectively:

i. Write down the revenue matrix R


ii. Evaluate the matrix RQ – CQ

c) Putting P = RQ – CQ and A = (1 0), solve the matrix equation AP = BP and interpret the value
of q obtained.

Solution

a) Note that this is matrix multiplication: C has 2 columns and Q has 2 rows. So multiplication is
well defined.

§12000 1200 · § 1 · §12000  1200q ·


CQ ¨¨ ¸¸ ¨¨ ¸¸ ¨¨ ¸¸
© 7000 1600 ¹ © q ¹ © 7000  1600q ¹

Results:

The first element is a cost function of product X and the second element is a cost function of product Y.

§ 0 1800 ·
b) Revenue coefficient matrix R ¨¨ ¸¸
© 0 2000¹

This is prompted by the fact that when multiplied by Q there will be no notion of “fixed revenues”.

§ 0 1800 ·§ 1 · § 0  2000q · § 2000q ·


RQ ¨¨ ¸¸¨¨ ¸¸ = ¨¨ ¸¸ = ¨¨ ¸¸
© 0 2000 ¹© q ¹ © 0  1800q ¹ © 1800q ¹

§ 2000q · §12000  1200q · § 600q  12000·


P RQ  CQ = ¨¨ ¸¸ - ¨¨ ¸¸ = ¨¨ ¸¸
© 1800q ¹ © 7000  1600q ¹ ©1200q  7000¹
The element in the matrix P are profit functions for product X and Y respectively.

Given that A 1 0 and B 0 1 .

83

BUSINESS MATHS & STATISTICS (TC3)


§ 600q  12000 ·
Then AP 1 0 ¨¨ ¸¸ = 600q  12000
© 200q  7000 ¹

§ 600q  12000·
and BP 0 1 ¨¨ ¸¸ = 200q  7000
© 200q  7000 ¹

Now AP = BP Ÿ 600 q  12000 = 200 q  7000


Ÿ 600 q  200 q 12000  7000
i.e. 400 q 5000
q 12.5
q 12.5 is the point at which profits on x and profits on y are the same.

8.7.3 Scalar multiplication


Scalar Multiplication is the multiplication of a matrix by a scalar (a constant). This is done by
multiplying each element of the matrix by a scalar.

Example 20

§4 9 · § 3  2· §  1 10 ·
Let A ¨¨ ¸¸ , B ¨¨ ¸¸ and C ¨¨ ¸¸
© 3  6¹ ©7 4 ¹ © 0 3¹

Evaluate the following:

a) 3A
b) 2B  4C
c) 2C  A  3B

Solution

§4 9 · § 3u 4 3 u 9 · §12 36 ·
a) 3 A = 3 ¨¨ ¸¸ ¨¨ ¸¸ = ¨¨ ¸¸ .
© 3  6¹ © 3 u 3 3 u (6) ¹ © 9  18 ¹

§ 3  2· §  1 10 ·
b) 2B  4C = 2 ¨¨ ¸¸  4 ¨¨ ¸¸
©7 4 ¹ © 0 3¹

§ 2 u 3 2 u (2) · § 4 u (1) 4 u 10 ·
= ¨¨ ¸ ¨ ¸
©2u 7 2 u 4 ¸¹ ¨© 4 u 0 4 u 3 ¸¹

§ 6  4 · §  4 40 ·
= ¨¨ ¸¸  ¨¨ ¸¸
©14 8 ¹ © 0 12 ¹

84

BUSINESS MATHS & STATISTICS (TC3)


§ 6  (4)  4  40) ·
= ¨¨ ¸¸
© 14  0 8  12 ¹

§ 6  4  4  40 ·
= ¨¨ ¸¸
© 14  0 8  12 ¹

§10  44 ·
= ¨¨ ¸¸
©14  4 ¹

§  1 10 · § 4 9 · § 3  2·
c) 2C  A  3B = 2 ¨¨ ¸¸  ¨¨ ¸¸  3 ¨¨ ¸¸
© 0 3 ¹ © 3  6 ¹ © 7 4 ¹

§ 2 u (1) 2 u10 · § 4 9 · § 3 u 3 3 u (2) ·


= ¨¨ ¸¸  ¨¨ ¸¸  ¨¨ ¸¸
© 2 u 0 2 u 3 ¹ © 3  6 ¹ © 3 u 7 3 u 4 ¹

§  2 20 · § 4 9 · § 9  6 · §  2  4  9 20  9  6 ·
= ¨¨ ¸¸  ¨¨ ¸¸  ¨¨ ¸¸ = ¨¨ ¸¸
© 0 6 ¹ © 3  6 ¹ © 21 12 ¹ © 0  3  21 6  6  12 ¹

§3 5·
= ¨¨ ¸¸
©18 24 ¹

8.7.4 Determinant of a matrix


a) Determinant of a 2 x 2 matrix

Determinant of matrix, denoted by two vertical lines binding the matrix

§a b ·
Given a matrix A ¨¨ ¸¸
© c d ¹
a b
The determinant of A, denoted by A or det(A) is = ad  bc .
c d

BUSINESS MATHS & STATISTICS (TC3)


Example 22

§3 2 ·
Given that D = ¨¨ ¸¸ , find D .
©1  2¹

Solution

3 2
Determinant of D, D = = 3 u (  2 )  2 u 1  6  2 8 .
1 2
b) Determinant of a 3 x 3 matrix or higher

The easiest way of finding the determinant of a 3 x 3 matrix (or higher order) is to use sarrus’ rule:

Step 1 Given a matrix, create an augmented matrix by writing the same matrix side by side.
Step 2 Multiply the elements of each lead diagonal (without repetition). There will be a product
for each set of elements in the diagonal. Add these products
Step 3 Repeat step 2 for the non-lead diagonals.
Step 4 Subtract the sum in step 3 from the sum in step 2, this is the determinant.

Example 23
§ 3 2  1·
¨ ¸
Find the determinant of K ¨1 1 1 ¸
¨3 1 2 ¸
© ¹
Solution

Step 1: Create an augmented matrix by writing the same matrix side by side.

3 2 1 3 2 1

1 1 Ͳ1 1 1 Ͳ1

3 1 Ͳ2 3 1 Ͳ2

Step 2: Multiply the elements of each lead diagonal


3 2 1 3 2 1

1 1 Ͳ1 1 1 Ͳ1

3 1 Ͳ2 3 1 Ͳ2

The lines show elements in three lead diagonals.

The products are: 3x1x(-2) = -6, 2x(-1)x3 = -6 and 1x1x1 = 1


86

BUSINESS MATHS & STATISTICS (TC3)


The sum is (-6) + (-6) + 1 = -11

Step 3: Repeat step 2 for the non-lead diagonals

3 2 1 3 2 1

1 1 Ͳ1 1 1 Ͳ1

3 1 Ͳ2 3 1 Ͳ2

The lines are on the non-leading diagonals.


The products are 3x(-1)x1 = -3; 2x1x(-2) = -4; 1x1x3 = 3
The sum is -3 + (-4) + 3 = -4

Step 4: Find the determinant of K This = -11 – (-4) = -7

Note: This process can be used for a 4 x 4 matrix or higher.

8.7.5 The inverse of a matrix


A. Definition
The inverse of a square matrix A denoted by A-1 is a matrix defined as
A-1 = AA-1 = A-1A = I; I being an identity matrix.

B. Finding the inverse of a 2 x 2 matrix

§a b ·
Given a matrix A = ¨¨ ¸¸
©c d ¹
1 § d  b·
The inverse of A = A-1 = ¨¨ ¸¸
A © c a ¹

1 § d  b·
= ¨¨ ¸¸
ad  bc © c a ¹

Note: As we cannot divide by 0, we insist that A ad  bc z 0 . It turns out that A 0 , we shall


conclude that the inverse does not exist.

Example 24

§ 7  3·
Let A ¨¨ ¸¸ , find the inverse of A.
© 9  3 ¹

87

BUSINESS MATHS & STATISTICS (TC3)


Solution

1 §  3  ( 3) · 1 §  3 3·
A 1 ¨¨ ¸¸ = ¨ ¸
A © 9 7 ¹ 7 u (3)  (3) u 9 ¨©  9 7 ¸¹

1 §  3 3·
= ¨ ¸
6 ¨©  9 7 ¸¹

We can check if this is indeed the inverse matrix by finding the product A A 1

§ 7  3· 1 §  3 3 · 1 § 6 0 · § 1 0 ·
A A 1 ¨¨ ¸¸ ¨¨ ¸¸ = ¨¨ ¸¸ ¨¨ ¸¸ = I
© 9  3¹ 6 ©  9 7 ¹ 6 © 0 6 ¹ © 0 1 ¹

Example 25

§ 3  2·
Let D ¨¨ ¸¸ . Find D 1 .
©  6  4¹

Solution

Now we need D 3 u ( 4)  ( 2) u ( 6) = -12 + 12 = 0. Since D 0 , we conclude that D 1 does not
exist i.e. matrix D does not have an inverse.

8.8 SOLVING SYSTEMS OF SIMULTANEOUS LINEAR EQUATIONS USING MATRICES

8.8.1 Overview.
Matrices are quite handy tools for solving systems of linear equations. Under matrix algebra it is possible
to solve a system of linear equations. This can be done by:

a) The inverse method.


b) Using the Cramer’s rule

The latter is shorter especially for 3 x 3 matrices than those of higher order.

8.8.2 Solution by inverse method


Consider the system of linear equations
a11 x  a12 y r
a 21 x  a 22 y s .

This system of linear equations can be expressed in matrix form as follows :


AX B,

88

BUSINESS MATHS & STATISTICS (TC3)


§ a11 a12 · § x· §r·
where A ¨¨ ¸, X ¨¨ ¸¸ and B ¨¨ ¸¸ .
© a 21 a 22 ¸¹ © y¹ ©s¹

§a a12 · § x· §r·
Thus A X B Ÿ ¨¨ 11 ¸ ¨¨ ¸¸ = ¨¨ ¸¸
© a 21 a 22 ¸¹ © y¹ ©s¹

Note that if matrix multiplication is carried out and equality of matrices used, the left side and the right
yield the system of linear equations:

a11 x  a12 y r
a 21 x  a 22 y s .

If the inverse of A is defined, the system of the equations can be solved as follows:

Given that A X B and that the inverse, A-1 , exists.

Then A 1 A X A 1 B
I X A 1 B
or X A 1 B

§ x·
Recall that X ¨¨ ¸¸ i.e. x and y . The solution therefore is actually found by multiplying the inverse of
© y¹
A and matrix B.

Example 26

Solve the simultaneous equations y = 3x + 5; and 2y + 3x = 28 by

a) Ordinary algebraic method


b) By matrix method (using the inverse)

Solution

a) By ordinary algebra

Substitute y into equation 2:


ʹሺ͵‫ ݔ‬൅ ͷሻ  ൅ ͵‫ ݔ‬ൌ ʹͺ
͸‫ ݔ‬൅ ͳͲ ൅ ͵‫ ݔ‬ൌ ʹͺ
ͻ‫ ݔ‬ൌ ͳͺ
‫ ݔ‬ൌ ʹ

Substitute x into equation 1: ‫ ݕ‬ൌ ͵ሺʹሻ ൅ ͷ ൌ ͳͳ


89

BUSINESS MATHS & STATISTICS (TC3)


b) Using matrix algebra (the inverse):

The equations: ‫ ݕ‬ൌ ͵‫ ݔ‬൅ ͷ; and ʹ‫ ݕ‬൅ ͵‫ ݔ‬ൌ ʹͺ can be rearranged as:
െ͵‫ ݔ‬൅ ‫ ݕ‬ൌ ͷ
͵‫ ݔ‬൅ ʹ‫ ݕ‬ൌ ʹͺ

and be written as
§ 3 1· § x · §5·
¨¨ ¸¸ ¨¨ ¸¸ ¨¨ ¸¸ i.e. A X B
© 3 2¹ © y ¹ © 28 ¹

If A 1 exists, then X A 1 B .

1 § 2  1· 1 § 2  1· 1 § 2  1·
Now A 1 ¨¨ ¸¸ ¨ ¸= ¨ ¸
A ©  3  3¹  6  3 ¨©  3  3 ¸¹  9 ¨©  3  3 ¸¹

1 § 2  1· § 5 · 1 §  18 · § ( 19 ) u (18) ·
Then X A 1 B ¨ ¸¨ ¸ ¨ ¸=¨ ¸
 9 ¨©  3  3 ¸¹ ¨© 28 ¸¹  9 ¨©  99 ¸¹ ¨© ( 19 ) u (99) ¸¹

§2·
= ¨¨ ¸¸
©11¹
§ x· §2·
So ¨¨ ¸¸ ¨¨ ¸¸
© y¹ ©11¹

Using equality of matrices, x = 2, y = 11

8.8.3 Solution using Cramer’s Rule

Given a system of linear equations, Cramer's Rule is a handy way to solve for just one of the variables
without having to solve the whole system of equations.

Cramer’s Rule uses determinants as illustrated below.

Consider the system of linear equations:

a11 x  a12 y r
a 21 x  a 22 y s .
r a12 a11 r
s a 22 a12 s
Provided A z 0 , then x , y
A A

90

BUSINESS MATHS & STATISTICS (TC3)


Example 27

Use the cramer’s rule to solve the system of equations

y = 3x + 5; and 2y + 3x = 28 by

Solution

The equations: y = 3x + 5; and 2y + 3x = 28 can be rearranged as:

-3x + y = 5
3x + 2y = 28

and be written as

§ 3 1· § x · §5·
¨¨ ¸¸ ¨¨ ¸¸ ¨¨ ¸¸ i.e. A X B
© 3 2¹ © y ¹ © 28 ¹

3 1
Now A = ( 3) u 2  1 u 3 = - 9
3 2

5 1 3 5
28 2 10  28 3 28  84  15  99
Then x 2 and y 11 .
9 9 9 9 9

In summary, the solutions (variable by variable) are found as follows:

Step 1: Find the determinant of the coefficient matrix A

Determinant of Matrix A as calculated above is -9

Step 2: Create a matrix Ax by replacing the x column in matrix A by the solution vector (i.e. elements of
the right hand side). Find the determinant of this new matrix.

Step 3 Create another matrix Ay by replacing the y column in A with the solution vector

Using Cramer’s Rule, solve the following system of linear equations:

2x + y + z = 3
x–y–z=0
x + 2y + z = 0

91

BUSINESS MATHS & STATISTICS (TC3)


Solution

§2 1 1· 2 1 1 2 1
¨ ¸
Now A ¨ 1  1  1¸ , Need A 1  1  1 1 1
¨1 2 1 ¸ 1 2 1 1 2
© ¹

Then

A 2 u ( 1) u 1  1 u ( 1) u 1  1 u 1 u 2  1 u ( 1) u 1  2 u ( 1) u 2  1 u 1 u 1

= -2 – 1 + 2 +1 + 4 -1 = 3.

§ 3·
¨ ¸
The solution vector is B ¨0¸ .
¨0¸
© ¹

3 1 1
0 1 1
Ax 0 2 1
Then x 1
A A

2 3 1
1 0 1
Ay 1 0 1
y 6
A A

2 1 3
1 1 0
Az 1 2 0
z 3
A A

So x 1 , y 6 , z 3.

BUSINESS MATHS & STATISTICS (TC3)


CHAPTER SUMMARY

A matrix is a table of values conveying numerical information. Having defined what matrices are, the
chapter looked at different types of matrices and presented algebraic operations associated with matrices.
Under types of matrices, examples presented included the zero matrix, identity matrix, square matrices,
row matrices and column matrices. On the other hand, algebraic operations included addition,
subtraction, multiplication, determinants and inverses (in matrix algebra, there is no division!).

In matrix addition or subtraction, matrices have to be of identical size and involves adding or subtracting
corresponding elements. Under multiplication, the chapter presented scalar multiplication, which
involved multiplying every element of the matrix by a scalar, and matrix multiplication which involved
two matrices in which the number of columns of one matrix is equal to the number of rows of the other,
otherwise it will not be possible to perform matrix multiplication. In addition to this, the chapter also
presented the concept of determinant which is cardinal when finding inverses of matrices and solving
systems of linear equations.

Matrices are incredibly useful algebraic structures that have numerous applications in mathematics and
other sciences. Someof them merely take advantage of the compact representation of a set of numbers in
a matrix. In this chapter, matrices were used in solving systems of linear equations. The methods for
solving systems of linear equations were the matrix inversion method and Cramer’s Rule. The method of
matrix inversion involves multiplying the right hand side by the inverse of the coefficient matrix while
Cramer’s Rule used determinants.

END OF CHAPTER EXERCISES

§12  4 · § 5  2·
¨ ¸ ¨ ¸ § 2 0 3·
1. Consider the following matrices: A ¨ 8 3 ¸, B ¨ 3 3 ¸ and C ¨¨ ¸¸
¨1 6 ¸¹ ¨1 7 ¸ © 1  3 1¹
© © ¹

Find the following, where possible:

a) A–B
b) A+B
c) 3A
d) 3A-2B
e) AC
f) CA
g) BC

2 Given two matrices

§ 2 5·
¨ ¸ § 3  2·
A ¨ 3 2 ¸ and B ¨¨ ¸¸
¨1 0¸ ©4 1 ¹
© ¹
93

BUSINESS MATHS & STATISTICS (TC3)


a) Which of these, AB or BA is defined?
b) Carry out the defined multiplication.

§ 2 3· § 5 6· § 3  1·
3. Let A ¨¨ ¸¸, B ¨¨ ¸¸, C ¨¨ ¸¸
© 1 4¹ ©7 8¹ © 2 4 ¹

Find the matrices (i) A B


(ii) BC
(iii) A  B  C
(iv) A  B  C

What conclusion can you draw from (iii) and (iv)?

4. (a) Evaluate AB, given that matrices A and B are defined as :

§ 2  4·
§ 2  1 3· ¨ ¸
  A ¨¨ ¸¸ and B ¨ 1 4 ¸ 
© 5 7 2¹ ¨5  1¸
© ¹

(b) The table below shows the number of passengers on three AXE Bus Services between
two towns Blantyre and Lilongwe and the price of a ticket per person for each service:

Departure Class Class Class


time Comfort Excursion Economy
6.30 15 35 60
11.30 12 40 70
16.00 8 50 34
Price (K) 4,000 2,500 2,000

Required:

(i) Find the total number of passengers departing at 11:30 a. m.


(ii) Construct two matrices to represent the information from the table.
(iii) Use the matrices from part (ii) to obtain the total revenue from each Departure time.

5. Find x and y in each of the following

(a) 2 x 3y 8 6

§ x  1 4· § 2 4·
(b) ¨¨ ¸¸ ¨¨ ¸¸
© y  3 5¹ ©1 5¹

94

BUSINESS MATHS & STATISTICS (TC3)


A housewife makes the following purchases during one week

Monday 2 packets of milk and 1 loaf of bread; Tuesday 3 litres of milk; Wednesday 1 litre of milk
and 2 loaves of bread; Thursday 2 loaves of bread; Friday 3 litres of milk; Saturday 2 litres mil
and 2 loaves of bread; Sunday 3 loaves of bread. Arrange her purchases in a matrix with two
rows and seven columns.

Solve each of the following equations for A, where A is a 2 u 2 matrix.

§ 4 10 ·
(a) 2A ¨¨ ¸¸
© 6 14 ¹

§  2 6· § 19 18 ·
(b) ¨¨ ¸¸  3 A ¨¨ ¸¸
© 5 2¹ © 20 11¹

Solve the following systems of equations using the inverse method

y = 2x + 1
y = -2x + 9

3x  4 y 5
2x  y 0

3x  4 y 2
5x  3 y 4

Solve the following using the Cramer’s rule

a) 3x  4 y 5
2x  y 0

b) 3x - y + z = 5
2x + 2y + 3z = 4
x + 3y – z = 11
c) 2x  5 y  z 8
3x  y  2 z 7
2 x  10 y  2 z 16

d) 3x  3 y  4 z 1
5 x  9 y  17 z 4
3x  5 y  9 z 2

95

BUSINESS MATHS & STATISTICS (TC3)


10. Product x has fixed costs of K12,000 and variable costs of K1,200 per product; product y has
fixed costs of K7,000 and variable costs of K1,600 per product.

§12000 1200 · §1·


a) If C ¨¨ ¸¸ and Q ¨¨ ¸¸ , where C is the cost coefficient matrix, evaluate the
© 7000 1000 ¹ ©0¹
matrix product CQ, and interpret.

b) Given that products x and y sell at K1,800 and K2,000 respectively:


(i) Write down the revenue coefficient matrix, R and
(ii) Evaluate the matrix RQ – CQ and explain its significance.

b) Putting P=RQ-CQ, A= (1 0) and B = (0 1), solve the matrix equation AP=BP


and interpret the value of q obtained.

96

BUSINESS MATHS & STATISTICS (TC3)


CHAPTER 9 DATA AND DATA COLLECTION
LEARNING OBJECTIVES

At the end of this chapter the student should be able to:

i.Distinguish between data and information


ii.Classify data
iii.Distinguish between data collection methods and select a suitable method
iv.Distinguish between different types of data
v.State stages in statistical investigation

9.0 INTRODUCTION

This chapter is concerned with the various methods employed in choosing the subjects for an
investigation and the different ways that exist for collecting data. The advantages and disadvantages of
the data collection methods have been outlined.

9.1 DATA AND INFORMATION

9.1.1 Definitions

Data is a term used to mean raw facts which one can use to produce information which can address a
question or explain a situation.

Information is processed data that is meaningful to the user.

Processing of data can be any of the following.

a. Classification
b. Computation
c. Sorting
d. Categorisation

9.1.2 Classification of data

Overview

Data can be classified into many groups. The bases include nature of measurement and source.

If nature of measurement is considered, data can be classified as discrete, continuous, qualitative or


quantitative, while if the source of data is taken into account, terms like primary or secondary data can be
used.

97

BUSINESS MATHS & STATISTICS (TC3)


It should be noted that some of these classifications are not mutually exclusive, they do overlap. For
example, quantitative data can be secondary or primary.

Quantitative and Qualitative

Quantitative data are numerical measurements expressed in terms of numbers and the numbers stand for
specific values. They are not mere labels.

Example 1

x Balance at a bank K2344.2


x Number of students in a class 35
x Height of a person 1.8m

Qualitative data comprises labels, opinions or expressions of conclusions. As opposed to quantitative data
which is in numeric form, qualitative data is expressed by means of a natural language descriptions.
Sometimes qualitative data is referred to as "categorical" data.

Example 2

x Today’s weather: “bad”


x Level of attendance: “high”
x Comment on Project: “successful”
x Grade in Maths at MSCE: “9”
x Expressions of size: “small, medium and large”
x Sociologist conclusion: “poverty fuels spread of HIV”
x Storm category: “5”

Note that use of numbers in data does not necessarily make that data quantitative.

Example 3: The number at the back of a football player: 11

Scores representing opinions: 1 (poor)


5 (Excellent)

Storm category 5 (bad)

The numbers above are only labels (much as some can be described as some form of ranking). These
numbers can be changed without changing the meaning or what it represents. For instance, a player
wearing jersey 9 can put on jersey 12. He remains the same person. Excellent” can be a 4 and other time
a 5 can be used. A 9 is a label for “fail” on the Malawi School Certificate level. In other exam situations a
“0” means fail. Contrast this with a bank balance of MK17.00. If the entry is MK0.00, the meaning is
very different.

98

BUSINESS MATHS & STATISTICS (TC3)


Discrete and continuous

Discrete data is data relating to “countable variables.” The data is mainly in the form of integers.

Examples: 2 cars, 5 people, 272 tablets of soap

Continuous data is data which is part of a continuum. The user simply decides on a limit.

Items whose data is normally continuous include

Age, Time, Length, Height, Weight

Continuous data is characterized by the presence of fractions in its measurement, although some will be
expressed as whole numbers.

Examples
i. The age of a child: 5.7 yrs
ii. The distance between towns: 23.8 km
iii. A bag of maize 50 kg

Primary and Secondary Data

Primary data means original data that have been collected specially for the purpose in mind. Primary data
can also be defined as data collected from an original source.

It means when an authorized organization, an investigator or an enumerator collects the data for a
specific purpose and from an original source himself or with the help of an institution or an expert then
the data thus collected are called primary data.

Usually primary data is collected through surveys, observations or participation. Various tools for data
collection (questionnaires, schedules or even check lists) exist.

Research where one gathers this kind of data is referred to as field research.

Secondary data are data that have been collected for another purpose. Examples of Secondary data
include

Data from magazine, newspapers, books, or any publication.

Research where one gathers this kind of data is referred to as desk research.

9.1.3 Other important terms

Parameter

A parameter is a constant or variable term in a function that determines the specific form of the function.

99

BUSINESS MATHS & STATISTICS (TC3)


Statistic
A statistic can be any specific value of interest that can be collected in relation to data of interest. Thus a
value of age can be a statistic and so is the population of a town. Alternatively a statistic is a value
obtained from a variable entering into a mathematical form of any distribution such that the possible
values of the variable corresponding to different distributions can be determined. Thus an average
calculated using some function or formula is a statistic.

Stages in Statistical investigation

A statistical investigation involves a number of stages:

i. Definition of the problem or issue


ii. Collection of relevant data
iii. Classification and analysis of the collected data
iv. Presentation of the results

Before the collection of data starts, there are important points to consider when planning a statistical
investigation.

Preliminary Considerations
It is extremely important to consider the following points before you start collecting data

x Aim

This is necessary in order to ensure that nothing important is omitted from the enquiry and the effort is
not wasted by collecting irrelevant data. Many people fail to think clearly about the problem being
investigated. You must write down the objective of the survey, stating exactly what information you want
to get out of the data that you are planning to collect, then you will collect only relevant data

x Units
It is essential that on what units to use before data collection starts. The results must appear in
comparable units for any analysis is to be valid. The choice of units should be influenced by the possible
need to compare sets of data collected from different sources. Frequently data will be collected by a
number of people all using different units. Some conversion factors would therefore be needed.

x Scope of the enquiry


No investigation should be undertaken without defining the field to be covered. Should the study cover
all the departments of the organisation or only some. Should it cover more than one organisation.

x Accuracy of the data


If the level of accuracy is not defined beforehand then it is difficult to know the amount of detail to be
collected. To what degree of accuracy is data to be recorded?. For example are ages of individuals to be
given to the nearest year or month or as the number of completed years?. If some of the data is to come
from measurement, then the accuracy of the measuring instrument will affect the accuracy of the results.
The degree of precision required in an estimate might affect the amount of data that we need to collect. In

100

BUSINESS MATHS & STATISTICS (TC3)


general, the more precisely we wish to estimate a value, the more readings we need to take. If the level of
accuracy is stated beforehand it is easier to estimate the cost of the data collection

9.1.4 Requirement of Statistical data

Attributes of good data

For data to be classified as of high quality, it must have the following attributes:

a) Accuracy

Data should be sufficiently accurate for the intended use. For example day to day operations planning
require that data should be detailed to the last unit of measurement (e.g. a car costing K7,850,330.26).
Whereas data for long term planning requires rounded figures (e.g. the same car can be quoted as costing
8 million kwacha). Data should be captured only once, although it may have multiple uses. Data should
be captured at the point of activity.

b) Validity

Validity refers to correctness and reasonableness of data. Correctness looks at connection to the subject
matter data is to relate to and reasonableness includes issues like age falling within expected ranges,
account or phone numbers having certain digits and within specific ranges, and names spelt correctly.
Data should also be correct and be recorded in compliance with relevant requirements, including the
correct rules or definitions.

c) Reliability

Data should reflect stability and consistency in collection processes (methods), across collection points
and over time.

d) Timeliness

Data should be captured as quickly as possible after the event or activity and must be available for the
intended use within a reasonable time period. Data must be available quickly and frequently enough to
support information needs and to influence service or management decisions.

e) Relevance

Data captured should be related to the purposes for which it is collected. This will require a periodic
review of requirements to reflect changing needs.

f) Completeness

Data should be able to address all needs for which it is collected. This means data requirements should be
clearly specified based on the information needs of the organisation and data collection processes
matched to these requirements.

101

BUSINESS MATHS & STATISTICS (TC3)


9.2 DATA COLLECTION METHODS

9.2.0 Overview

The following are the most common ways of collecting primary data.

Data can be collected in a number of ways. The following are most common methods:

i.Personal interviews
ii.Observations
iii.questionnaires.
iv.Use of published statistics

The choice of a data collection method will be influence by a number of factors including cost, speed,
volume of data, type of data, expertise, etc.

9.2.1 Personal Interviews

Definition

These are face to face interviews where the interview collects data by asking questions directly to the
respondent.

Personal interviews can be Structured, Unstructured and Semi-structured

Structured

These are interviews guided by a structured questionnaire. One must have clear objectives of an enquiry
and develop a questionnaire with question which are logically arranged. The interviewer uses these
question and the flow to collect data.

The aim of structured interview is to ensure that each interview employs exactly the same questions and
in the same order. This ensures that answers can be reliably aggregated and that comparisons can be
made with confidence between sample subgroups or between different survey periods.

Unstructured

Unstructured Interviews are a method of interviews where the interview has a topic to explore and a set
of key questions (not necessarily in an order. These questions can be changed or adapted to meet the
respondent's intelligence, understanding or belief. Unlike a structured interview they do not offer a
limited, pre-set range of answers for a respondent to choose, but instead they encourage listening to how
each individual person responds to the question.

The interview mostly proceeds in a “friendly manner”. However it may lack the reliability and precision
of the structured interview.

102

BUSINESS MATHS & STATISTICS (TC3)


Semi structured – it is a bit of structured and unstructured.

Semi -structured interviews are in-depth interviews. They take on features of both structured and semi
structured interviews in that the interviewer uses predetermined questions and follows a pattern while
being flexible enough to probe deeper depending on responses and at the same time allowing the
interviewee the options to take different paths and explore different thoughts, feelings. The interview has
however got to control the interview and should bring the interview back to the subject matter.

Advantages and Disadvantages of Personal Interviews

Personal interviews have the following advantages:

a. The interviewer can achieve a high response rate.


b. The method enables flexibility in terms of interpreting questions to the respondent.
c. There is usually better depth of data.

The disadvantages are that:

a. Responses may be distorted by wrong emphasis on the part of the interviewer.


b. The interviewer may lose control
c. Personal interviews may be expensive because travel may be involved.

9.2.2 Observations

Definition

In this method data is collected through participation and taking measurement then documenting the
results. Sometimes observations can be carried out through experiments.

Advantages and Disadvantages of Observations

The method has the following advantages.

x A high degree of depth of the data can be achieved


x Data is usually very accurate because the results are a first hand account of a subject matter.

The disadvantages are:


x Cost of observations can be high
x Observations require a lot of time.
x Some situations may not render themselves easy to experimentation.

9.2.3 Questionnaires

Definition
This is where one designs a questionnaire and either hands it in person or mails to the respondent. The
respondent completes the questionnaire and sends it back to the one collecting data.

103

BUSINESS MATHS & STATISTICS (TC3)


Principles of good questionnaire design

i. Keep the questionnaire as short as possible.


ii. Keep the individual questions as simple as possible.
iii. Questions should not be capable of different interpretations.
iv. Avoid questions that require the respondent to make calculations.
v. Arrange questions in a logical manner.
vi. Avoid leading questions.
vii. If possible use closed questions.
viii. Explain the purpose of the investigation so as to encourage people to respond

Advantages and disadvantages of self completion questionnaires

Advantages include:

i.Low cost. If mailed the major costs include stamps and paper
ii.Convenience (the respondent can fill the question at any time)
iii.Depth of data because of the time allowed, detailed questions can be asked.

Major disadvantages of self completion Questionnaire are:

i.High non response rate. The very fact of trying to answer the questionnaires
ii.Distortion of data due to varying interpretation.
iii.Difficulty to come up with a good questionnaire.

9.2.4 Published Statistics


Sometimes we may be attempting to solve a problem that does not require us to collect new data but to
reassemble and reanalyse data which has already been collected by someone else for some other purpose.
We can make good use of the amount of statistical data published by government, nationalised industries,
chambers of commerce and so on. When using this method it is particularly important to be clear on the
definition of the terms and units used and the accuracy of the data. The information must be reliable and
up-to-date.
The information you require may not be found in one source but parts may appear in several different
sources

Advantages of published statistics


i. The cost is low in acquiring the data since the data is already available.
ii. The information can easily be accessed.
iii. It can be used to clarify the research questions.
iv. It can be used to facilitate in aligning the focus of primary research at large scale.

Disadvantages
i. It is time consuming in searching for appropriate data
ii. The definitions used for variables and units may not be the same as you wish to use.
iii. The data may be not be presented in the form the researcher needs
iv. The data may not give full version of the data as required by the researcher

104

BUSINESS MATHS & STATISTICS (TC3)


v. There might be a problem of time, as data collected five years ago may not match with data
collected now.
vi. The data is largely self-governed therefore the data must be scrutinised closely

CHAPTER SUMMARY

In this chapter you have leaned:

x Definition of data and information.


x Classification of data into quantitative and qualitative, for quantitative discrete and continuous.
x The difference between primary and secondary data.
x Attributes of good information.

x Data collection methods including their advantages and disadvantages.

END OF CHAPTER EXERCISES

1. Explain the meaning of the following in the context of statistics and in each case give an example:

a) Quantitative variable.
b) Qualitative variable.
c) Statistic.
d) Parameter

2.Classify the following as qualitative or quantitative data:

a) Employees salaries on the payroll


b) Employees, employment numbers on the same payroll
c) Electricity units purchased for a prepaid meter
d) Competition winners telephone numbers copied from a news paper.
e) Number of jobs handled by a garage
f) The number at the back of a marathon runner
g) Receipt numbers written out by an auditor from a sales file.

1. Classify the data in question 1 as primary or secondary.

2. An officer in the Ministry of Agriculture is asked to evaluate the benefits of the fertiliser subsidy
programme to the ordinary farmer. What data Collection method will he/she use and why.

3. Using principles of a good questionnaire, design a questionnaire to be used by a human resources


manager to collect data on employees responsibilities at work in the light of ones qualification
and experience.

105

BUSINESS MATHS & STATISTICS (TC3)


CHAPTER 10 SAMPLING

LEARNING OBJECTIVES
At the end of this chapter, the student should be able to

i. Distinguish between sampling and taking a census


ii. Outline the major sampling techniques
iii. Be able to suggest a suitable sampling technique for a situation and take a sample using any of the
major techniques.

10.0 INTRODUCTION

In data collection, an ideal situation is to canvas or collect data from all elements of a population. This is
taking a census (or complete “count”) of a population. However this is not always practical for many
reasons which include.

x Cost: censuses are vast undertakings requiring large amounts of equipment, materials,
and human resource)
x Time: Censuses require time in planning, data Collection and analysis.
x Depth of data: Because of the size of the undertaking, it is not possible to ask in depth questions

Instead of a census, a sampling is used. A sample is a group of selected elements of the population.
Sampling refers to the process of selecting the elements from the population.

For many enquiries, sampling is preferred because of costs are lower compared to censuses, it takes
shorter time and in-depth data can be collected. However sampling has the problem of representation, and
accuracy. These can be mitigated by using an appropriate sampling technique (see below) and increasing
the sample size (the number of items selected into a sample.)

10.1 IMPORTANT TERMS

10.1.1 Population
A population is the collection of all people or items with the characteristics that one wishes to understand
in an enquiry.

Example

For a survey collecting data on family size in a district. The populations are all the families in the district.

10.1.2 Population size


The numbers elements in the population. This number is often denoted by N.

10.1.3 Sample
A sample is a collection of items selected from the population. A sample is therefore a part of the
population.
106

BUSINESS MATHS & STATISTICS (TC3)


10.1.4 Sampling frame
This is an itemized list of the items from which a sample is to be drawn. The term sampling frame is
different from population because it refers to a situation where the elements of a population can be and
have been individually identified for purposes of sampling. With a sampling frame it is possible to state
exactly the proportion of the population/frame which has been sampled. Not all populations render
themselves to the construction of a sampling frame.

10.1.5 Sample size


This is the number of items in a sample. Sample size is often denoted by n. For example, given a
population has 250 element and 25 items are selected into a sample, then the sample size n is 25.

10.2 SAMPLING TECHNIQUES

10.2.1 Major groupings of sampling techniques.


a) Random or scientific techniques:

These are techniques where sample elements have a defined chance of being selected into the
sample.

b) Non Random or non scientific:

Sample items do not have a defined probability of being selected.

10.2.2 Sampling techniques.


A. Overview

The most common sampling designs are simple random sampling, stratified random sampling, and
multistage random sampling.

B. The simple random sampling (SRS)

Simple random sampling is a random (scientific) sampling technique where each individual is chosen
entirely by chance and each member of the population has an equal chance of being included in the
sample.

A simple illustration of simple random sampling is what happens with raffle ticket draws. Several tickets
are crumpled thrown into a drum and reshuffled. A person is blind folded and asked to pick a ticket or
tickets. The tickets so picked are picked purely at random

In the case of sample surveys random numbers are used to achieve this randomness. Random numbers
are number often generated by computer and they are such that one cannot predict any particular number
from the set. They do not have a pattern nor is it possible to fit a formula to them.

Example 1:

Given a list of people in the following table clearly identified and numbered from 01 to 20, select a
simple random sample.
107

BUSINESS MATHS & STATISTICS (TC3)


Table 10.1 Sampling frame
1 D Magwera 11 C Ndlovu
2 L Mphande 12 M Mitepa
3 E Muyandira 13 I Jemu
4 W Mhango 14 M Chapola
5 P Chisambo 15 E Frackson
6 T Tilenga 16 M Mlotho
7 H Ali 17 J Nyirenda
8 A Lwanda 18 P Tambala
9 J Sikwese 19 E Kankodo
10 L Ndawambe 20 T Majamanda

Solution

Since the people in the table are clearly numbered, the list is the sampling frame. The desired sample size
is 5, therefore the first 5, two-digit random numbers are selected from a random number table as shown.

Table 10.2 Selected random numbers

60 21 6 70
14 22 38 55
34 22 6 30
21 92 19 43
33 59 56 25
5 96 17 30
54 8 83 75
59 50 89 20
19 69 52 63
67 77 32 81

From the random number table, going vertically, the following are the first 5 random numbers between
01 and 20: 17, 01, 13, 07, and 10

Thus the elements of the sampling frame selected into the sample are:

01 D Magwera*
07 H Ali*
10 L Ndawambe*
13 I Jemu*
17 J Nyirenda*

Simple random sampling has the following advantages and disadvantages:

108

BUSINESS MATHS & STATISTICS (TC3)


Advantages
i. It minimizes bias
ii. Because of low bias the sample is usually representative.
iii. The method is generally accepted by layman as fair
Disadvantages
i. The sample is venerable to sampling error
ii. The elements selected may not reflect the population.
iii. SRS may be Cumbersome and tedious.
iv. SRS will always require a sampling frame.
v. There is chance that certain significant attributes of the population are over or under represented

C. Systematic sampling

Systematic sampling is a technique where sample items are selected at a regular interval from a random
start. The interval at which the population elements are picked is called a sampling interval. That is
sampling is carried out by picking say every 5th, 8th, etc item depending on the sampling interval
determined.

The sampling interval is calculated as follows

Let k = the sampling interval


N = the population size
n = the sample size
Then k = N/n

Example 2:

Select a sample of 5 people from the list shown in 3.2.1 above using systematic sampling

Solution

N = 20, n=5 the sampling interval k = 20/5 = 4

In such a case, sampling proceeds by taking every 4th item fro a random start in the first 4 elements of the
sampling frame (the in the first interval).

Using the same random number table the first number between 01 and 04 is 01.
This means starting with item 01 selecting every 4th element from the population, we have:

01, 05, 09, 13, and 17

The names are:

109

BUSINESS MATHS & STATISTICS (TC3)


1 D Magwera
5 P Chisambo
9 J Sikwese
13 I Jemu
17 J Nyirenda

Advantages

i. The method is practical and easy to carry out on the ground.


ii. It does not require a strict sampling frame. It only requires the arrangement of the elements in the
population.

Disadvantage

i. Systematic sampling can produce biased results if the population has periodicity in the elements. That is
the arrangement forms a pattern and the sampling interval resonates with that pattern then only elements
of a similar type will be selected.

ii. The method is not truly random since ( once a random starting point has been selected ) all subjects are
pre-determined.

D. Stratified sampling

This is a sampling technique where the population is first divided into categories according to pre
existing characteristics in the population. The categories are called “strata” (Single stratum).

A sample is then drawn from each stratum and results combined later. For example, a survey on
industrial turn over industries can be stratified as follows:

Stratum 1 Large industries


Stratum 2 medium
Stratum 3 small

Stratified sampling techniques are generally used when the population is heterogeneous, or dissimilar,
and certain homogeneous, or similar, sub-populations can be isolated (strata). Simple random sampling is
most appropriate when the entire population from which the sample is taken is homogeneous. Any
appropriate sampling technique can then be used in the individual stratum:

Thegeneral procedure for taking a stratified sample is:


i. Stratify the population, defining a number of separate partitions.
ii. Calculate the proportion of the population lying on each partition.
iii. Split the total sample size up into the above proportions.
iv. Take a separate sample (normally simple random) from ech partition, using the sample sizes as defined in
(iii).
v. Combine the results to obtain the required stratified sample.

110

BUSINESS MATHS & STATISTICS (TC3)


Advantages

i. It enables researchers to draw inferences about specific sub groups. That may be lost in a more general
random sample.
ii. It leads to better statistical estimates.
iii. Stratified sampling makes data more readily available for pre existing subgroup within a population.
iv. If strata are treated independently different sampling technique can be used depending on the data
required and a situation on the ground.

Disadvantages

i. The technique requires selection of a relavant clarification criteria.


ii. It can be expensive due to extra time and manpower required for the organisation and implementation of
the sample.
iii. It requires an extensive sampling frame.
iv. Strata levels of importance can only be selected subjectively.
v. It is not useful when there are no homogeneous subjects.

E. Multistage sampling

A multistage sample is constructed by taking a series of samples in stages starting with wider area or
definition of units to narrower and more specific units. The method is used where a population is spread
over a relatively wide geographical area and SRS would require travelling to all parts of the area.

The method is as follows:

i. Split the area up into a number of regions


ii. Randomly select a small number of regions
iii. Confine sub-samples to these regions alone with size of each sub-sumple proportional to the size of the
area.
iv. The above procedure can be repeated for sub-regions within regions.

Once the final regions have been selected the final sampling technique could be random or systematic
depending on the existence or otherwise of a sampling frame.

Example 3:

A researcher wants to estimate the proportion of household above a poverty line in terms of income in
rural Malawi. Help the researcher design a sample.

Solution:

Taking the country as a whole the sample design would as follows:

Stage 1 Pick districts using SRS


Stage 2 From each district select a number of Traditional Areas
Stage 3 From each T/A select a number of Village headmen
111

BUSINESS MATHS & STATISTICS (TC3)


Stage 4 From each Village headman select the required number of households.

The number of units (districts, traditional authorities, village headmen and households) being sampled at
each stage depends on the accuracy required. Larger samples tend to give better accuracy.

Advantages

i. Cost of canvassing is low


ii. Data can be collected with relative speed due to ease of organisation the technique can give.
iii. It has convenience of finding the survey sample
iv. Normally more accurate than cluster sampling for the same size sample

Disadvantages

i. Is not as accurate as SRS if the sample is the same size


ii. More testing is difficult to do
iii. Possible bias if a very small number of regions is selected.
iv. No member of the population in any other region can be selected.

F. Cluster sampling
Cluster sampling is used where it is difficult or impossible to isolate the target responding units (elements
of the population from which data is collected). In such cases it is more cost-effective to select
respondents in groups ('clusters'). Grouping or clustering is often based on geography, or by time periods.
Data is then collected from all respondents in a selected cluster.

Example 4

An NGO has read in the papers that very few people use mosquito nets in Ndirande-Zambia slam
township. To confirm this, the NGO would like to collect first hand data necessary for intervention.
However the Director is at pains as to what sampling method can be used because the dwelling units are
not numbered and they are not arranged in proper rows. He has turned to you for help in the matter.
Suggest a sampling method.

Solution

The proper sampling technique is cluster sampling. Much as the dwelling units are not numbered or
arranged in rows, there are small access roads which form a network. The roads form boundaries of
groups or clusters of houses. These clusters can be identified and an SRS can be picked and data
collected from every household in a selected cluster

Example 5:
A manager in Peoples Trading Centre would like to find out from shoppers whether or not they like the
new arrangement of goods. Help him draw a sample of the shoppers.

112

BUSINESS MATHS & STATISTICS (TC3)


Solution
It is difficult to identify all hoppers individually before hand and therefore a sampling frame cannot be
constructed. The best way is to cluster shoppers on time. Possible clusters are:

Treating shoppers that enter the shop starting at 08:00 can be grouped in periods of 15 minutes up to say
close of the shop at 20:00. A systematic sample of these groupings can be made. One possible such
sample is to pick the groupings every two hours as follows

08:00 – 08:15

10:00 – 10:15

12:00 – 12:15

14:00 – 14:15

16:00 – 16:15

18:00 – 18:15

Note that a SRS sample can also be drawn since the time clusters can be uniquely identified.

Advantages

i. Clustering can reduce travel and administrative costs. An interviewer can make a single trip to
visit several responding units in one block, rather than having to drive to a different block for
each unit.
ii. It also means that one does not need a sampling frame listing all elements in the target population.
Instead, clusters can be chosen from a cluster-level frame, with an element-level frame created
only for the selected clusters.

Disadvantages

i. Cluster sampling requires a larger sample than SRS to achieve the same level of accuracy because
of the variability among clusters.
ii. The fact that the method is not random sampling then the selection bias could be significant.

G. Quota sampling
Quota samplingis one of the non-scientific techniques of sampling. The researcher decides on the number
of sampling units (i.e sample size) according to the required coverage (purely from judgement). The data
collector then collects data from any responding units in the population up to the number arrived at
earlier.

Advantages

i. The technique is simple and convenient


ii. There is no non-response
113

BUSINESS MATHS & STATISTICS (TC3)


iii. The cost of data collection is very low,
iv. Quota sampling does not require a sampling frame.

Disadvantage

i. The technique produces very biased results because of the human interference in the sample
selection.
ii. Severe interviewer bias can be introduced into the survey by inexperienced or untrained
interviewers, since all the data collection and recording rests with them

CHAPTER SUMMARY

In this chapter you have learned that:

x Census is a survey which examines every member of the population


x A sample is a relatively small subset of the population with advantage over census that cost, time
and other resources are much lower
x A sampling frame is a structure which lists or identifies the members of the a population.
x Simple random sampling is a technique which ensures that each and every of the population has
an equal chance of being chosen for the sample.
x Stratified random sampling ensures that every significant group in the population is represented
in proportion in the sample.
x Multi-stage (quasi-random) sampling is normally used in homogeneous population spread over a
wide area.
x Stratified (quasi-random) sampling involves selecting a random starting point and then sampling
every nth member of the population.
x Cluster (quasi-random) sampling exhaustive sampling from a few well chosen areas
x Quota (quasi-random) sampling normally involves teams in of interviewers who obtain
information from a set quota of people.

END OF CHAPTER EXERCISES

1 Define and give examples of the following terms

a) Population
b) Sampling frame
c) Sampling unit
d) Random numbers
e) Responding unit

2 Compare and contrast probability and non-probability sampling techniques.

3 Systematic sampling may produce a biased sample. Discuss

114

BUSINESS MATHS & STATISTICS (TC3)


4 How can you collect a sample using the following sampling techniques

Simple random sampling


Cluster sampling
Multistage sampling
Stratified sampling

5 Suggest a suitable sampling technique in each of the following situations Describe the technique
and state reason for its suitability.

(i) In an audit you wish to form an opinion from a number of invoices received throughout
the year.

(ii) You are trying to estimate maize field over a 5 hectare field.

(iii) You want to gather opinions from spectators on the strength of two soccer teams just
before a match.

115

BUSINESS MATHS & STATISTICS (TC3)


CHAPTER 11 DATA PRESENTATION

Learning Objectives:

By the end of this chapter, you should be able to:


i. explain the need to represent statistical information
ii. understand the various techniques for presenting data
iii. select appropriate data presentation techniques for specific types of data
iv. construct diagrammatic displays of data
v. interpret graphical displays of data

11.0 INTRODUCTION
Before looking at all different techniques for presenting data, it is necessary to consider the purpose for
which the data was collected. For instant, the data you collected might have been wanted for your
company’s annual report. A straightforward list of all data values could be presented but, particularly if
there were a lot of items, this would not be very helpful and even very boring.

Data presentation therefore simplifies large amounts of data, shows key facts and patterns, and display
data in an interesting and easily understandable way. This generally involves sorting and grouping,
illustration and summary statistics. In this chapter, we will focus on sorting, grouping and illustrating
data using tables, charts and graphs.

Common data presentation techniques include:


x Frequency table/distribution
x Pictogram
x Bar charts: Simple, multiple (compound) and component (stacked) bar charts
x Pie chart
x Histogram
x Frequency polygon
x Cumulative frequency graph (ogive)
x Lorenz curve
x Z-chart

11.2 FREQUENCY DISTRIBUTION

Definition
A frequency is the number of observations or items or data values that belong to each category or class
of data. For qualitative or discrete quantitative data, a frequency is simply a record of how many of each

116

BUSINESS MATHS & STATISTICS (TC3)


type were present. Consequently, a Frequency Distribution can be said to be a grouping of data into
mutually exclusive classes showing the number of observations or items in each class.

The main aim of a frequency distribution is to summarise data in a logical manner that enables an overall
perspective of data to be obtained quickly. A frequency distribution can be represented in form of a table,
a graph or a formula. In this module, however, we shall only look at frequency distributions that take the
forms of a table and/or graph.

Frequency distributions can be classified into simple and grouped distributions. While simple
distributions involve qualitative and discrete quantitative data, grouped distributions involve continuous
quantitative data.

Table 11.1 is a frequency distribution showing the frequency with which some ports of exit were used by
departing visitors in 2009.

Table 11.1: Number of visitors departing Malawi by port of exit, 2009

Table 11.1 Visitors by port of exit - 2009


Port of exit Number of visitors
Chileka 76,308
Chiponde 52,836
Chitipa/Chisenga 6,816
Dedza 10,916
Kaporo/Songwe 90,350
Lilongwe (KIA) 174,652
Mchinji/Chimaliro 82,304
Muloza 41,203
Mwanza 141,956
Nayuchi 14,759
Nsanje/Marka 17,322
Other 45,609
Total 755,031
Source: 2009 Tourism Report - NSO
From the table above, it is easy to see that visitors departing Malawi frequently used Kamuzu
International Airport (KIA) as their port of exit in 2009. A total of 174,652 visitors exited Malawi
through Lilongwe (KIA). The simple frequency distribution in Table 10.1 can be transformed into a
complex distribution by among other things including the reasons for visiting and the gender of the
visitor as shown in Table 11.2.

117

BUSINESS MATHS & STATISTICS (TC3)


Table 11.2: Number of visitors departing Malawi by port of exit, gender and reason for visit

Reason for visit


Port of exit Holiday/vacation Work/business Visit friends/relatives Conference
Female Male Female Male Female Male Female Male
Chileka 7226 8353 9378 42638 3587 5022 52 52
Chiponde 10250 28391 2511 8968 1537 1179 0 0
Chitipa/Chisenga 512 769 256 4254 51 974 0 0
Dedza 4049 5688 154 307 462 256 0 0
Kaporo/Songwe 7533 17065 3075 35772 8712 18142 0 51
Lilongwe (KIA) 17117 28084 17527 89837 6560 15221 102 204
Mchinji/Chimaliro 5842 11018 5074 41716 7790 10864 0 0
Muloza 8251 17527 1845 9327 1537 2716 0 0
Mwanza 16758 23523 8866 62010 12351 18448 0 0
Nayuchi 2716 6047 1230 4561 0 205 0 0
Nsanje/Marka 564 2716 1947 9532 1281 1230 0 52
Other 2716 12813 3382 16963 3588 5943 204 0
Total 83534 161994 55245 325884 47456 80200 358 359
Source: 2009 Tourism Report – NSO

Let us now construct our own frequency distribution.

Example 1
A BMS class at an Accountancy College has 42 students. Of these, 19 have a Certificate in Financial
Accounting (CIFA) background. There are 4 male students with CIFA background while 15 female
students have the CIFA background. In total there are 19 female and 23 male students. Present the data
on a table.

Solution:
Number of students in a BMS class
CIFA background Other Total
Male 4 15 19
Female 15 8 23
Total 19 23 42

Constructing a Frequency Distribution for Quantitative Data

We can construct a frequency distribution from an array or table of numbers (data vales) using the
following steps as a guide:

118

BUSINESS MATHS & STATISTICS (TC3)


Step 1:Decide on the number of classes, k
We are supposed to use just enough classes or groups of data values so that we reveal the shape of the
distribution. The number of classes depends on the number of observations (n) in the data set.
Traditionally, the number of classes must vary from 5 to 20 i.e. 5 d k d 20
A quick hint to decide on the number of classes is the “2 raised to the k rule”. The rule states that we
must selected the smallest k number of classes such that 2 k ! n

Step 2: Determine the class width or interval


The class interval or width must, in general, be the same for all the classes. The class width is expressed
as:
x  xmin
i t max
k
where i is the class interval/width, x max is the largest or highest data value, x max is the smallest or lowest
data value in the raw data and k is the number of classes.

Step 3: Determine the individual class boundaries


Set the class boundaries such that each data value can be put into only one category. This means we must
avoid overlapping class boundaries. For example, the classes 10 – 20 and 15 – 25 should not be used in
the same distribution because they are overlapping.

In order to get the classes right, you need to set up the correct lower boundary of the first or lowest class.
The lower boundary for the first class can be the lowest data value ( xmin ) in the data set or any number
slightly lower than the lowest value. For instance, if the lowest data value is 21 then the lower boundary
of the first class can be 21 or any number slightly lower that 21 e.g. 20. The rest of the class boundaries
are then determined based on this lower boundary.

Note that as you decide on the classes they will be adjustments.Ensure that figures used for class
boundaries are numbers which are easy to work with e.g.numbers divisible by 2, 5 or 10.

Step 4: Tally the data values


Use tallies to allocate counts of data values for each class.
Step 5: Count the number of items
Count the tally marks in each class to obtain the class frequencies. The frequency distribution/table can
then be rewritten so that it is presented without the tally marks.

Example 2
A fisherman using a line and rod recorded his catch per day for 50 days and his records are as following.

119

BUSINESS MATHS & STATISTICS (TC3)


Table 11.3 Fish (catch) per day
23 27 24 7 28 15 25 29 15 46
5 9 12 10 17 22 23 17 16 32
11 12 18 20 38 13 27 38 18 22
22 20 13 14 26 14 19 19 40 31
33 17 34 21 23 26 18 14 21 27

Construct a frequency distribution for these data.

Solution
Step 1: Number of classes
Since there are n 50 data values, we need to have k classes such that 2 k t 50 . For k 5,
5
2 32 which is less than 50. Consequently, 5 classes are not enough for this data set. If we set
k 6 , then 2 64 , which is greater than 50. So we
6
recommend k 6 classes for the data set.

Step 2: Class width/interval


Since the lowest catch/day is 5 and the highest catch/day is 46, and that we need 6 classes,
46  5
the class width should be at least i 6.8 . Usually, such a class width would be rounded up to
6
some convenient number such as the next integer, a multiple of 5, 10 or 100. In our case, we round 6.8
to 7.

Step 3: Individual class boundaries


Since the lowest catch/day is 5, the lowest boundary for the first class can be 5 or any number
slightly smaller than 5 such as 4 and 3. We are of the view that ‘5’ is much easier to work with, so
we set the lower boundary of the first class at 5. Consequently, here are the 6 classes for this data set:
5 – 12, 12 – 19, 19 – 26, 26 – 33, 33 – 40, 40 – 47

Step 4: Tallying Table 11.4 Fish catch tallies


Daily catch Tallies
5 – 12 ////
12 – 19 //////////// /
19 – 26 //////// ////
26 – 33 //// ////
33 – 40 ////
40 – 47 //
Note: Each class includes the lower boundary but not the upper boundary. For example, the data value 26
is in the class 26 – 33 and not in the class 19 – 26.

Step 5: Counting the number of items or tallies to obtain the class frequencies

120

BUSINESS MATHS & STATISTICS (TC3)


Table 11.5 Fish catch frequency table
Daily catch Frequency
5 – 12 5
12 – 19 16
19 – 26 14
26 – 33 9
33 – 40 4
40 – 47 2

11.2.1 General Forms of Frequency Distributions


The following examples show typical forms of frequency distributions. Each example tries to depict a
special feature that one may find in a frequency distribution.

i) A simple frequency distribution shows singles values and their frequencies (counts), and it is
useful when summarizing simple discrete data over a limited range.

Table 11.6 Simple frequency distribution


x f
10 2
14 7
17 8
21 6
25 3
30 5

ii) The following frequency distribution shows the number of orders that a company received per
week over a 40-weeks period. Notice that the classes are continuous and of equal width

Table 11.7 Orders received over 40 weeks


Number of order Number of weeks (f)
5 but less than 10 1
10 but less than 15 5
15 but less than 20 7
20 but less than 25 10
25 but less than 30 7
30 but less than 35 4
35 but less than 40 3

121

BUSINESS MATHS & STATISTICS (TC3)


iii) Here the classes are not continuous. There are gaps for example between 14 and 15, between 19
and 20, etc. These are known as class limits and not boundaries. This is typical for a frequency
distribution where data is discrete.

Table 11.8 Students Age distribution


Age (years) Number of students
10 – 14 50
15 – 19 80
20 – 24 45
25 – 29 12
30 – 34 10

We can transform the class limits to boundaries by simply averaging the adjacent limits. The
class boundary between the classes 10 – 14 and 15 – 19 would be the average of 14 and 15,
14  15
i.e. 14.5 , and the boundary between the classes 15 – 19 and 20 – 24 would be the
2
19  20
average of 19 and 20 i.e. 20 .5 . We can therefore represent the distribution above
2
using class boundaries as shown below:

Table 11.9 Students age distribution


Age (years) Number of students
9.5 – 14.5 50
14.5 – 19.5 80
19.5 – 24.5 45
24.5 – 29.5 12
29.5 – 34.5 10

Note: The classes are now continuous

iv) The classes in the following distribution do not have equal width. In this case, the width will
depend on the requirements of the individual presenting the data.

Table 11.10 Frequency distribution: unequal class width


Days F
0 – 10 8
10 – 15 12
15 – 20 10
20 – 22 16
22 – 25 11

122

BUSINESS MATHS & STATISTICS (TC3)


v) This distribution has two open ended classes: under 100 and 250 and over. Open ended classes are
used when data at the ends of the distribution are too widely and sparsely spread.

Table 11.11 Frequency distribution with open ended classes.


Income (in K’000) Number of employees
Under 100 116
100 and under 120 89
120 and under 150 184
150 and under 170 142
170 and under 200 159
200 and under 250 124
250 and over 124

11.3 CLASS INTERVALS/WIDTH AND MIDPOINTS


In this module, the terms ‘class interval/width’ and ‘class midpoints’ are used frequently. You need,
therefore, to have a clear understand of the two. The class midpoint or class mark is halfway the
between the lower and upper boundaries (or limits) of the same class. It is computed by adding the lower
and upper boundaries and dividing the results by 2. Refer to the fisherman’s example, the midpoint for
first class (5 – 12) with 5 as the lower boundary and 12 as its upper boundary is 8.5 found by (5+12) y 2.
The rest of the midpoints are:

Table 11.12 Frequency distribution: class mid-point calculated


Daily catch f Midpoint, x Midpoint found by
5 – 12 5 8.5 (5+12) y 2
12 – 19 16 15.5 (12+19) y 2
19 – 26 14 22.5 (19+26) y 2
26 – 33 9 29.5 (26+33) y 2
33 – 40 4 36.5 (33+40) y 2
40 – 47 2 43.5 (40+47) y 2

To determine the class interval/width, subtract the lower class boundary from the upper class boundary.
We can also determine the class interval/width by finding the difference between consecutive midpoints.
The class interval for the fisherman’s example was determined as 7, which we find by subtracting the
lower boundary of the first class, 5, from the upper boundary of the same class, 12 i.e. 12 – 5 = 5.
Alternatively, since the midpoint of the first class is 8.5 and the midpoint of the second class is 15.5, then
the class width is 15.5 – 8.5 = 7.

123

BUSINESS MATHS & STATISTICS (TC3)


11.4 RELATIVE AND CUMULATIVE FREQUENCIES
It may be desirable to convert frequencies to relative and/or cumulative frequencies. A relative
frequency is the ratio of the class frequency to the total number of observations. It shows the fraction (or
percentage) of the total number of data values in each class.

Mathematically, a relative frequency of each class/data value is defined as:


Frequency of a class or data value f
Sum of the frequencies ¦f
In our fisherman example, we may want to show the proportion of days on which specific intervals of
fish catch were recorded as shown below:

Table 11.13 Relative frequency distribution


Daily catch Frequency Relative frequency Found by
5 – 12 5 0.10 5 y 50
12 – 19 16 0.32 16 y 50
19 – 26 14 0.28 14 y 50
26 – 33 9 0.18 9 y 50
33 – 40 4 0.08 4 y 50
40 – 47 2 0.04 2 y 50
Total 50 1

Note: We can convert the relative frequencies to percentages simply by multiplying them by 100 (%).

Definition: Less-than and more-than cumulative frequencies


A less-than cumulative frequency is the number of data values/observations that are less than a specific
data value fall or below the upper boundary of a specific class in case of grouped frequency distribution.
The less-than cumulative frequency is the default cumulative frequency. However, a more-than
cumulative frequency is the number of data values/observations that are equal to or greater than a
specific data value or the lower boundary of specific class in case of grouped frequency distribution.
Referring to our fisherman example, we may want to show the number of days on which he recorded less
than 19 fishes. In this case we would simply add 5 and 16 to give us 21 days. And for less than 26 fishes,
we add 5, 16 and 14 to get 35 days. Furthermore, if we wanted the number of days on which he recorded
either equal to or more that 26 fishes, we would simply add 9, 4 and 2 to get 15 days. The cumulative
frequencies for the remaining classes are shown in the table below:

124

BUSINESS MATHS & STATISTICS (TC3)


Table 11.14 “Less-than” cumulative frequency distribution.
Daily catch Frequency “Less-than” Found by
f cumulative, F
5 – 12 5 5 5
12 – 19 16 21 5+16
19 – 26 14 35 5+16+14
26 – 33 9 44 5+16+14+9
33 – 40 4 48 5+16+14+9+4
40 – 47 2 50 5+16+14+9+4+2

Table 11.15 “More-than” cumulative frequency distribution


Daily catch Frequency Less-than Found by
f cumulative, F
5 – 12 5 50 2+4+9+14+16+5
12 – 19 16 45 2+4+9+14+16
19 – 26 14 29 2+4+9+14
26 – 33 9 15 2+4+9
33 – 40 4 6 2+4
40 – 47 2 2 2

11.5 ADVANTAGES AND DISADVANTAGES OF TABULAR PRESENTATION


Advantages of Tables
x Enhanced level of accuracy and precision
x Data can be presented in more than one dimension (more than one variable). For example in
Table 10.2, we have the following variables presented: Port of exit (name), reason for visit and
gender of the visitors.

Disadvantage of a Table
x Tables lack visual impression of the distribution
x The figures may be cumbersome (especially in large tables)

11.6 GRAPHICAL AND DIAGRAMMATIC PRESENTATION OF FREQUENCY


DISTRIBUTION
Managers and other busy individuals often need a quick picture of the trend in variables of interest such
as sales, expenditure, cost, revenue, and profit. These trends are usually easy to depict using charts and
graphs.

125

BUSINESS MATHS & STATISTICS (TC3)


11.6.1 Pictogram
A pictogram is a technique where pictures or symbols are used to represent data. For example, if 500
houses were built in Ndirande and 250 houses in Zolozolo, then using a scale of 1 picture of a house to
50 houses, we can represent the information as follows:

Figure 11.1 Pictogram

Ndirande:
Zolozolo:

Key: = 50 houses

Example 3
A survey was carried out to find the number of school buses operating in the major towns in Malawi. The
results are presented in the distribution below. Present the distribution using a pictogram.

Table 11.16 School buses


Town Number of school buses
Blantyre 200
Lilongwe 150
Zomba 50
Mzuzu 20

Solution: Figure 11.2 Pictogram

Blantyre:

Lilongwe:

Zomba:

Mzuzu:

Key: = 50 school buses


Advantage of a Pictogram
x A pictogram provides a quick visual impression of the data (magnitude)

126

BUSINESS MATHS & STATISTICS (TC3)


Disadvantages of a Pictogram
x It lacks precision specially interpreting fractions of the pictures/symbols used. For example, the
immediate problem that comes out is how to express the 20 buses in Mzuzu in the above problem.
x The technique has limited dimensions in which data can be presented.

11.6.2 Bar Chart


A bar chart or bar graph is a chart with non-joining rectangular bars. Data is represented by the bars and
the lengths or heights are proportional to the values or frequencies that they represent. The bars can be
plotted vertically or horizontally.

There are three common forms of bar charts namely:


a) The simple bar chart
b) The multiple or compound bar chart
c) The component or stacked bar chart

How to Construct a Bar Chart


1) Place the data categories on the x-axis (no scaling is required)
2) Place the frequencies/counts/percentages on y-axis and must be scaled accordingly. The height or
length of each bar will show the frequency/percentage/count for each category.
3) The bars must be equal equally spaced and of equal width.

The Simple Bar Chart


A simple bar chart presents data only in two dimensions.

We typify the construction of a simple bar chart using the information in the table below on the levels of
imports into Malawi in 2010.

Table 11.2: Malawi’s imports by Broad Economic Category (BEC) in 2010

Table 11.17 Malawi Imports - 2010


BEC description CIF Value (MK’Billion)
Food and beverages 26.0
Consumable goods 38.2
Fuel and lubricants 30.0
Capital goods 42.9
Passenger motor cars 7.1
Parts and accessories 8.2
Total 152.4

Source: Annual Statistics Trade Report 2010

127

BUSINESS MATHS & STATISTICS (TC3)


To construct the bar chart, we shall place BEC item descriptions on the x-axis and the CIF values in the
y-axis. The bar graph is presented belo.

Figure 11.3 Malawi Imports bar chart

2010 Imports by Broad Economic Category for


Malawi
Import(CIF)value(MK'Billion)

Broad Economic Category (BEC)

With this chart it is easy to see which BEC registered the highest CIF value in 2010. We can also see that
CIF value for consumable goods was more than four times the CIF value for passenger cars in 2010.

Multiple or Compound Bar Chart

A multiple bar chart is similar to a simple bar chart only that it does show more than one aspect of the
data/variable. In a multiple bar chart, each bar represents a specific of the major category of a variable.

Example 4
The following data shows the annual external trade values for Malawi from 2005 to 2011.

Table 11.18: Annual external trade values for Malawi

Trade values (MK’Billion)


Year Exports Imports
2005 59.6 138.8
2006 93.8 174.6
2007 122.2 194.6
2008 123.6 285.8
2009 168.0 298.2
2010 159.4 325.6
2011 223.3 381.3

128

BUSINESS MATHS & STATISTICS (TC3)


Source: NSO-July/December 2012 Trade Brief
Present the trade values on a multiple or compound bar chart.

Solution
We proceed by constructing the bar chart clustered by year (i.e. years appear along the x-axis) and the
charts is presented below.

Figure 11.4 Annual external trade values

Annual external trade values for Malawi


Trade value (MK'Billion)

Exports
Imports

Year
One can tell from the chart that Malawi had been importing more than it had been export over the period
2005-2011.

It is also possible to construct a multiple bar chart clustered by trade type. In this case we shall put the
trade in the x-axis. The bar chart would look like the one below:

Table 11.5 External trade values

Annual external trade values for Malawi

2005
Trade value (K'Billion)

2006
2007
2008
2009
2010
2011

External Trade

129

BUSINESS MATHS & STATISTICS (TC3)


We can tell from the chart that while exports had generally increased from 2005 to 2011, imports into
Malawi had drastically increased over the same period.

Component Bar Chart


A component or stacked bar chart is more like the simple bar chart except that each bar, while
representing the total (frequency/count/value), it is split up into the specific components of the variable
category which are stacked on top of one another in the same order. For instance, if the bars are
representing the number of students in a class, the bars would be split up show the number of male and
female students in the class.

The following table shows holiday locations booked through a travel agent in Malawi.

Table 11.19 Number of bookings for holiday locations


Annual bookings
Holiday location 2001 2002 2003 2004 2005
Mangochi 385 350 326 341 286
Liwonde 186 178 224 212 195
Kasungu 140 156 187 188 184
Other 112 65 156 143 112
Total 823 749 893 884 777

We shall now proceed to construct a component bar chart for the data clustered by year and leave for
your practice to construct a component bar chart clustered by holiday location.

The first step in constructing component bar chart is to find the totals. Luckily, the totals for each year
(since we are clustering by year) are already provided in the table above. We place the years in the x-axis
and number of bookings in the y-axis, scaled according. The component bar chart is presented below.

Figure 11.6 Annual booking for holiday locations

130

BUSINESS MATHS & STATISTICS (TC3)


Annual bookings for holiday locations
Number of bookings

Other
Kasungu
Liwonde
Mangochi

Year

Note that for each year the four segments of the bar are stacked together to give the total number of
bookings for each particular year. For 2001, the bars for Other (112), Kasungu (140) and Liwonde (186)
are stacked onto the Mangochi bar (385) resulting into a combined bar for 2001 whose height/length is
823.

Quite often component bar charts are presented in terms of percentages. Unlike the absolute component
bar chart we just constructed which requires the use of raw data frequencies, to construct a percentage bar
chart we need to express the composition for each year as percentage of the total for that year. We
explain the process of constructing a percentage component bar chart using the information in Table 11.5,
which shows number of holiday bookings from 2001 to 2005.

The first step in constructing a percentage component chart is to express the components as a percentage
of the total (for each year since we are clustering by year). For example, the Mangochi bookings are
385 y 823 u 100 46.7 | 47% of 2001 total. In 2005, Kasungu bookings are
184 y 777 u 100 23.7 | 24% of the total. The rest of the percentages are given in the following table.

Table 11.20 Holiday bookings for locations (%)


2001 2002 2003 2004 2005
Location Raw % Raw % Raw % Raw % Raw %
Mangochi 385 47 350 47 326 37 341 39 286 37
Liwonde 186 23 178 24 224 25 212 24 195 25
Kasungu 140 17 156 21 187 21 188 21 184 24
Other 112 14 65 9 156 17 143 16 112 14
Total 823 100 749 100 893 100 884 100 777 100

131

BUSINESS MATHS & STATISTICS (TC3)


We then construct a component bar chart just as before but this time around we will have percentages in
the y-axis instead of raw number of bookings.
Table 11.7 Annual holiday bookings,

The chart shows that where as Mangochi had above 40% in 2001 and 2002 of the total number of
bookings, its number of bookings dropped in percentage terms while that of Kasungu and Liwonde
gained prominence in the subsequent years.

11.6.3 Pie Chart


A pie chart is normally useful for illustrating nominal level variables or data. In a pie chart data is
presented on sectors of a circle. The size of a particular sector is proportional to the magnitude of a data
item or the frequency of a data class. The proportionality is determined by assigning an appropriate angle
at the centre of the circle.

How to construct a Pie Chart


Step 1: Find angles
Convert the data frequencies/values to angular measurements. This is done by dividing each
frequency/value by their sum and then multiplying by 360 q

Step 2: Draw a circle (pie)


Draw a circle of reasonable radius and divide it into segments/sectors proportional to the frequencies (or
counts) as measured by the angles. It is expected that the sum of the sectors or angular measurements will
be 360 q (or 100%)

Step 3: Shade the sectors or segments different and these must be explained in a “legend” or “key”

We reproduce the 2010 Malawi imports data from Table 11.4. Our intention is to construct a pie chart for
the data.

132

BUSINESS MATHS & STATISTICS (TC3)


Table 11.21 Malawi Exports
BEC description CIF Value (MK’Billion)
Food and beverages 26.0
Consumable goods 38.2
Fuel and lubricants 30.0
Capital goods 42.9
Passenger motor cars 7.1
Parts and accessories 8.2
Total 152.4

The first step is to convert the CIF values to angles.

Table 11.22 Import values and angles of sectors of a pie chart


BEC description Value Angle at the centre
Food and beverages 26.0 26
u 360 q | 61.4 q
152 .4
Consumable goods 38.2 38 .2
u 360 q | 90 .2
152 .4
Fuel and lubricants 30.0 30
u 360 q | 70.9 q
152 .4
Capital goods 42.9 42.9
u 360 q | 101 .3 q
152 .4
Passenger motor cars 7.1 7 .1
u 360 q | 16 .8 q
152 .4
Parts and accessories 8.2 8 .2
u 360 q | 19.4 q
152 .4
Total 152.4 360 q

The second step involves using a compass to draw a circle and a protractor to measure and assign angles
at the centre of the circle. And finally the in third step you shade/colour the pie chart sectors. The final
pie chart is as follows:

133

BUSINESS MATHS & STATISTICS (TC3)


Figure 11.8 Value of imports – Pie chart
CIFValue
(MK’Billion), CIFValue(MK’Billion)
Passengermotor
cars,7.1,5%
CIFValue
(MK’Billion),Food
andbeverages,26,
18%
Foodandbeverages
CIFValue
(MK’Billion),Capital Consumablegoods
goods,42.9,30%
Fuelandlubricants
CIFValue
(MK’Billion), Capitalgoods
Consumablegoods,
Passengermotorcars
38.2,26%
CIFValue
(MK’Billion),Fueland
lubricants,30,21%

Because the area of the pie represents the relative share of each import item, we can easily compare them.
From the pie chart, it is clear that capital goods accounted for the largest proportion of imports for
Malawi in 2010. ‘Passenger car’ category was the least at 5% of the five broad economic categories in
terms of their CIF values.

Advantages:
i. A pie chart can give a visual impression of the comparative sizes of the components
ii. A pie chart is relatively easy to understand
iii. Much as it is meant to provide a comparative picture of the components of the data, a pie chart
can have actual data and/or percentages embedded in the diagram to show magnitudes

Disadvantages:
i. Lack of accuracy in general if figures are not embedded
ii. Limited dimensions in which data can be presented since data can only be presented in one
dimension
iii. Need to calculate the angles, drawn the circle and draw sectors

11.6.4 Histogram
A histogram is one of the most common ways of presenting grouped frequency distribution, pictorially. A
histogram is very similar to a bar chart. The difference is that while the bar chart has spaces between
bars, the bars in a histogram are drawn adjacent to each other.
134

BUSINESS MATHS & STATISTICS (TC3)


Example 5
We illustrate the construction of a histogram by recalling the frequency distribution in our fisherman
example. Below is the frequency distribution.

Table 11.23 Daily fish catch


Daily catch Number of days
5 – 12 5
12 – 19 16
19 – 26 14
26 – 33 9
33 – 40 4
40 – 47 2
Total 50

Construct a histogram for the distribution

Solution
Since all the class intervals are then same, therefore the frequencies (number of days) will be represented
by the heights of the bars. So we proceed by placing the daily catch values along the x-axis scaled to
ensure that it fits the values from 5 to 47 and the frequencies are scaled along the y-axis.

Figure 11.9 Daily catch for a fisherman

Daily catch for a fisherman


Number of days

Daily Catch
Points to note on the drawing of a histogram
x Each bar represents just one class, the bar width corresponds to the class width i.e. each bar
extends from the lower boundary to the upper boundary of the class.
x The bars are joined together (i.e. the values on the x-axis should be continuous)

135

BUSINESS MATHS & STATISTICS (TC3)


x If class width vary, then the areas of the bars (width xheight) corresponds to the class frequencies.
frequency f
In this case the height of bar, referred to as the frequency density, is given by .
Width i
Examples 6
An accountant of a progressive tea estate has gathered data on wages paid to tea pickers in the month and
has presented the data on a frequency distribution as follows

Table 11.24
Wage (MK’000) Number of pickers
10 – 15 20
15 – 18 12
18 – 25 35
25 – 30 27
30 – 40 8
40 and over 5

Construct a histogram for the distribution.

Solution
Note that the last class is open-ended and that the class widths are not equal. The open ended class is
dealt with by assigning it the most common width or the width of the preceding class. In our case we
assign it the class width of 5 and the class becomes 40 45.

The presence of unequal class width requires that we use frequency densities for the heights of the bars.
The frequency densities are calculated in the table below.

Table 11.25 Wage frequency density


f
Frequency density,
Wage (MK’000) f i
10 – 15 20 20 y 5 4
15 – 18 12 12 y 3 4
18 – 25 35 35 y 7 5
25 – 30 27 27 y 5 5.4
30 – 40 8 8 y 10 0.8
40 – 45 5 5y5 1
Here is the histogram for the distribution

Example 7
Below is age frequency distribution for students from Tayamba Pvt. Secondary School. Represent the
distribution on a histogram.

136

BUSINESS MATHS & STATISTICS (TC3)


Table 11.26 Students age students
Age (years) Number of students
10 – 14 55
15 – 19 82
20 – 24 45
25 – 29 28
30 – 34 8
Total 218

Solution
This is case data classes are not continuous. We need therefore to close the gaps between the age groups
by converting the class limits into boundaries (see section ____ ) as shown below

Table 11.27 Students age distribution


Age (years) Boundaries Number of students
10 – 14 9.5 – 14.5 55
15 – 19 14.5 – 19.5 82
20 – 24 19.5 – 24.5 45
25 – 29 24.5 – 29.5 28
30 – 34 29.5 – 34.5 8
Total 218

137

BUSINESS MATHS & STATISTICS (TC3)


The histogram is presented below.
Figure 11.10 Students age distribution
F
80

70

60

50

40

30

20

10

1 2 3 4 5 6 7 8 9 10 11 12 13 14 15 16 17 18 19 20 21 22 23 24 25 26 27 28 29 30 31 32 33 34 35 36
AGE

Although a histogram provides a strong visual appeal, one cannot read the exact data values since data
values are grouped.

11.6.5 Frequency Polygon


A frequency polygon is similar to a histogram. It consists of line segments connecting the points formed
by the intersections of the class marks (midpoints) and the class frequencies i.e. connecting the top
centres of the bars in a histogram.

To construct a frequency polygon, we scale the class midpoints along the X-axis and the frequencies
along the Y-axis. We illustrate the construction of the frequency polygon using the data from the
previous example. The data is reproduced below.

138

BUSINESS MATHS & STATISTICS (TC3)


Table 11.28 Students age distribution
Age (years) Number of students
10 – 14 55
15 – 19 82
20 – 24 45
25 – 29 28
30 – 34 8
Total 218

To construct the frequency polygon we first need to find the class midpoint. The midpoints are given in
the following table.

Table 11.29 Students age distribution


Age (years) Midpoints Number of students
10 – 14 12 55
15 – 19 17 82
20 – 24 22 45
25 – 29 27 28
30 – 34 32 8
Total 218

Next we plot the midpoints and their corresponding frequencies on the Cartesian (X-Y) plane. The points
to be plotted have the following coordinates (12,55), (17,82), (22,45), (27,28) and (32,8). The points are
then connected in order as shown below

Figure 11.11 Students age distribution-frequency distribution


90
80
70
Numberofstudents

60
50
40
30
20
10
0
0 5 10 15 20 25 30 35 40
AgeGroup(MidͲpoints)

139

BUSINESS MATHS & STATISTICS (TC3)


This does look like a polygon. In order to complete the frequency polygon, the ends must be connected to
the x-axis at zero frequencies. To do this, add two imaginary classes, one at either end, and connected the
ends of the line graph to the X-axis at their midpoints. In our case the classes are 5 – 9 (midpoint is 7)
and 35 – 39 (midpoint is 37). The ends of the line graph will then be connect to the points (7,0) and
(37,0) to complete our frequency polygon as shown below.

Figure 11.12
90
80 Number
Age Midpoin
70
of
(years) ts
Numberofstudents

students
60
5–9 7 0
50
10 – 14 12 55
40
15 – 19 17 82
30
20 – 24 22 45
20 25 – 29 27 28
10
1,722,000.00 30 – 34 32 8
0 35 – 39 37 0
0 5 10 15 20 25 30 35 40
AgeGroup(MidͲpoints)

Both the histogram and frequency polygon allow us to get quick picture of the main characteristics of the
data i.e. highs, lows, points of concentration etc. However, the frequency polygon has an advantage over
the histogram in the sense that it allows for direct comparison of two or more distributions.

Figure 11.13 Age distribution frequency polygons


a)

Ages of students from Tayamba Pvt


School
Number of students

Age(Year)
Tayamba alone
b)

140

BUSINESS MATHS & STATISTICS (TC3)


Ages of students from Takwera
Number of students Community

Age (Year)
Takwera alone
c)
Ages of students
Number of students

TayambaPvt

Takwera
Community

Age (Year)
Tayamba and Takwera

11.6.6 Ogive
An ogive is a cumulative frequency graph. There are two forms of an ogive and these are: The “less than” o
give and the “more than” ogive. We illustrate the steps followed in constructing an ogive using this
example.

Example 8
We shall once again use the data on the daily catches of fish. The distribution is below:

Table 11.30 Daily catch for a fisherman


Daily catch Number of days
5 – 12 5
12 – 19 16
19 – 26 14
26 – 33 9
33 – 40 4
40 – 47 2
Total 50

141

BUSINESS MATHS & STATISTICS (TC3)


Construct “less-than” and more-than ogives.

Solution:
As the description of an ogive suggests, an ogive requires cumulative frequencies. We present the
cumulative frequencies in the table below.

Table 11.31 Daily catch for a fisherman


Daily catch f “Less-than” cum “More-than” cum
frequency F frequency, F
5 – 12 5 5 50
12 – 19 16 21 45
19 – 26 14 35 29
26 – 33 9 44 15
33 – 40 4 48 6
40 – 47 2 50 2
Total 50

The next step is to plot the points. For a less-than ogive, we plot the class upper boundaries and the
corresponding cumulative frequencies. In our case the first plot is at (12,5) and the next is (19,21). The rest
of the points follow in this order (26,35), (33,44), (40,48) and (47,50). Once plotted the points are connected
to produce a less-than ogive. To complete the less-than ogive, the lower suspended end is connected to the
X-axis at the lower boundary of the first class. A completed less-than ogive is given below.

Figure 11.14 Daily catch “less than” ogive


60

50

40
Numberofdays

30

20

10

0
0 5 10 15 20 25 30 35 40 45 50
Daily catch

For a more-than ogive, we plot the class lower boundaries and the corresponding more-than cumulative
frequencies. In our case the first plot would be at (5, 50) and the next is (12,45). The rest of the points
142

BUSINESS MATHS & STATISTICS (TC3)


follow in this order (19,29), (33,6), and (40,2). Once plotted the points are connected to produce a more-
than ogive. To complete the more-than ogive, the lower suspended end is connected to the X-axis at the
upper boundary of the last class. A completed more-than ogive is given below.

Figure 11.15 Daily catch “more than” ogive


60

50

40
Numberofdays

30

20

10

0
0 5 10 15 20 25 30 35 40 45 50
Daily catch

11.7 LORENZ CURVE

The Lorenz curve is a curve that shows how even or uneven the distribution of some variable such as
income and wealth is. It is therefore primarily used to give a visual display of the degree of inequality of any
given data.

In an ideal situation and given a number of employees at an organisation, we would expect that 20% of the
employees would command 20% of the wage bill, 50% would command 50% of the wage bill and so on.
The graph of such an ideal situation would look as follows
Figure 11.16 Lorenz curve (diagonal)

143

BUSINESS MATHS & STATISTICS (TC3)


Cumulative number of employees
(%)

Cumulative wage bill (%)

However, in reality, incomes and other issues are rarely evenly distributed and the graph would not be a
straight line. The graph above represents a line of even distribution or line of equality/uniformity. A Lorenz
curve therefore shows how degree of deviation from the line of absolute equality.

Construct a Lorenz Curve

We can easily construct a Lorenz curve using the following steps

Step 1: Calculate cumulative percentage frequency for each class

Step 2: Calculate cumulative percentage class totals for each class

Step 3: Plot the cumulative percentage frequency (Y-axis) against cumulative percentage class totals (X-
axis) scaled accordingly.

Step 4: Connect the points with smooth curve to obtain a Lorenz curve

Step 5: Fit in a line equality by joining the origin (0,0) to the point (100,100). This helps to show the
degree of inequality

Example 8
Laponda Ltd has the following employee and wage distribution. The CEO has been claiming that his
organisation does not have huge imbalances in employees pay. Draw a Lorenz curve of the distribution to
prove or disprove the CEO’s claim.

144

BUSINESS MATHS & STATISTICS (TC3)


Table 11.32 Laponda Wage distribution

Salary (K’000) Number of Total wage bill


employees (MK’000)
0 – 50 26 1300
50 – 100 65 9750
100 – 150 50 12500
150 – 200 40 14000
200 – 250 30 13500
250 – 300 25 13750
300 – 350 15 9750
350 – 400 10 7500
400 – 450 8 6800
450 – 500 5 4750
500 – 550 3 3150
550 – 600 2 2300
600 – 650 1 1250

Solution
Since the wage bill totals for each class have been given, we do not need to be estimated estimate them. So
we proceed to calculate the cumulative percentages.

Table 11.33 Laponda wage and employee distribution


Number of Total wage Cum F% Cum total Cum total
employees bill employees, wage bill wage bill %
(MK’000) F
26 1300 26 9.3 1300 1.3
65 9750 91 32.5 11050 11.0
50 12500 141 50.4 23500 23.4
40 14000 181 64.6 37550 37.4
30 13500 211 75.4 51050 50.9
25 13750 236 84.3 64800 64.6
15 9750 251 89.6 74550 74.3
10 7500 261 93.2 82050 81.8
8 6800 269 96.1 88850 88.6
5 4750 274 97.9 93600 93.3
3 3150 277 98.9 96750 96.5
2 2300 279 99.6 99050 98.8
1 1250 280 100.0 100300 100.0

145

BUSINESS MATHS & STATISTICS (TC3)


To construct a Lorenz curve, we plot the cumulative frequencies (F%) against the cumulative total wage bill
(%) on a Cartesian plane and the join the points. The points to be plotted are; (1.3, 9.3), (11,32.5),
(23.4,50.4). . .(98.8,99.6) and (100,100). We must remember to include the origin, (0,0) as we plot the
points. We will complete the Lorenz curve by fitting in a line of equality or even distribution. The Lorenze
curve is given below.

Figure 11.17 pLorenz


y curve ( ) g g
s, F
100 %
0 0
26 1300 26 9.3 1300 1.3
65 80
9750 91 32.5 11050 11
CumNo.ofemployees(%)

50 12500 141 50.4 23500 23.4


40 14000 181 64.6 37550 37.4
30 13500 211 75.4 51050 50.9
60
25 13750 236 84.3 64800 64.6
15 9750 251 89.6 74550 74.3
10 7500 261 93.2 82050 81.8
40
8 6800 269 96.1 88850 88.6
5 4750 274 97.9 93600 93.3
3 3150 277 98.9 96750 96.5
20
2 2300 279 99.6 99050 98.8
1 1250 280 100 100300 100

0
0 20 40 60 80 100
Cumwagebill(%)

Comments:

Generally, the closer the Lorenze curve is to the line of even distribution or line of equality, the more even
the distribution is. For Laponda Ltd, the Lorenz curve bulges significantly from the line of equality. It is
clear that the wage distribution for Laponda is uneven. In fact, we can see from the curve that 30% of the
lowly paid employees share just about 10% of the total wage bill.

11.8 Z CHART
The Z chart displays time series data showing 3 major aspects. The three aspects when plotted roughly form
the letter Z. These aspects are:
i) Actual figures or data values for a chosen period
ii) Cumulative figures ordate values for the chosen period
iii) Moving totals to date for the same period

We illustrate the construction of a Z chart using the following example.

Example 9
The production manager of a local firm has recorded the following monthly production figures in order to
assess the production levels over time.

146

BUSINESS MATHS & STATISTICS (TC3)


Table 11.33 Local firm’s monthly production
Jan Feb Mar Apr May Jun Jul Aug Sep Oct Nov Dec
2011 150 154 183 162 181 149 130 152 180 199 193 186
2012 162 163 171 158 175 145 121 138 172 175 163 152

Construct a Z chart for 2012

Solution

The actual production figures for 2012 are shown in Column 2 below. We cumulate the figures for 2012 and
show the cumulative figures in Column 3 below. To achieve this, we start with 162 for January, for
February we add 163 to 162 and obtain 325, For March 171 to 325 to obtain 496, and so on. . .

Since there are 12 time points in each period (i.e. year), the moving totals will be calculated using period,
P 12 .

The moving total for January 2012 is the total from February 2011 to January 2012, i.e. 154 + 183 + 162 +
... +193 + 186 + 162 = 2031.

The moving total for February 2012 is the total from March 2011 to February 2012, i.e. 183 + 162 + 181+
...+ 186 + 162 + 163 = 2040.

An easier way to find the moving total for February 2012 is 2031 – 154 + 163 = 2040. If we proceed in this
way, we should obtain 1895 for December 2012. This figure must be the same as the cumulative total for
December 2012. The moving totals are given in Column 4 below.

Table 11.34 Monthly production, cumulative and moving totals


Month Actual figures Cumulative Moving
2012 figures, 2012 totals 2012
Jan 162 162 2031
Feb 163 325 2040
Mar 171 496 2028
Apr 158 654 2024
May 175 829 2018
Jun 145 974 2014
Jul 121 1095 2005
Aug 138 1233 1991
Sep 172 1405 1983
Oct 175 1580 1959
Nov 163 1743 1929
Dec 152 1895 1895

147

BUSINESS MATHS & STATISTICS (TC3)


To construct a Z chart, we shall plot the figures from the above table against time points i.e. months. Each
month shall have two data points plotted against it. The Z chart is shown below.

Figure 11.18 Z-chart

Production figures for a local firm Monthlyfigures

Cumulative
figures
Movingannual
totals
Production

Year: 2012

Comments on the Z chart:


Actual figures show some fluctuations in the level of production from month to month. The cumulative total
shows a steady rise in production. The year to-date (moving totals) figures are indicating that production is
declining over time.

CHAPTER SUMMARY

In this chapter we have discussed


x A variety of methods for presenting data pictorially. These include frequency distributions, charts for
presenting qualitative data such as the pie chart and bar chart (simple, component, and multiple bar
charts), and the pictogram.
x We also discussed graphs that are well suited for quantitative data namely the histogram, the
frequency polygon and ogives which are basically cumulative frequency polygons.
x Other curves including the Lorenz curves and Z charts which are primarily used to assess levels of
inequality in a distribution and display time series data respectively.

148

BUSINESS MATHS & STATISTICS (TC3)


ND OF CHAPTER EXERCISES

The following are sales for XYZ Ltd for the years 2010 to2012. Present them on a bar chart:

Year
Item 2010 2011 2012
Food Item 500 800 1000
Drinks 600 400 200
Clothing 400 400 800
TOTAL 1500 1600 2000

Students at a certain school were surveyed to find out the mode of transport they used when going to
school. The results were:

Walking: 9, Bicycle: 10, Car: 6 and Bus: 15.


a) Construct
i. a pie chart of radius 4cm to present this information
ii. a simple bar chart of the same data
b) Comment on both the charts

A company decided to research the price of laptops on the market. An analysis of advertisements in
the press and specialised IT bulletins produced the following information:

Price (MK’000) Number of laptops


200 – 250 5
250 – 300 12
300 – 400 15
400 – 420 9
420 – 450 6
450 – 680 3
raw a histogram of the prices.

The following set of data represents the age distribution of a company’s workforce of 170
employees.
Age (Year) Frequency
18 but under 30 56
30 but under 40 44
40 but under 50 35
50 but under 60 27
60 but under 70 7
70 and over 1

149

BUSINESS MATHS & STATISTICS (TC3)


a) Draw a histogram of the data
b) Draw an ogive
c) How are the two charts (the histogram and ogive) different

5. About 60% of small and medium sized businesses are family owned. An international survey asked
chief executive officers (CEOs) of family owned businesses how they became CEO. Responses
were that the CEO inherited the business; the CEO built the business, or the CEO was hired by the
family owned firm. A sample of 26 CEOs of family owned business provided the following data on
how each one of them became CEO.
Built Built Built Built Built Built
Inherited Built Inherited Built Inherited Inherited
Inherited Built Built Hired Built Built
Built Hired Hired Built
Inherited Inherited Inherited Hired

a) Construct a frequency distribution table of the above data


b) Draw a bar chart for the data
c) What could be the main reason a person becomes CEO of a family owned business?

6. A food processor makes chambo fillet from chambo supplied from the lake. The bigger the mass the
better (larger and juicer) the fillet and therefore the higher the price. It is known that fish of weight
greater than 1 Kg make good fillets which fetch better prices. In order to forecast his revenue he
decides to weigh the fish coming in. He asks his buyer to take a sample of 100 fish from different
consignments in accordance to a predetermined scientific technique. The results are as follows:

Weights of 100 fish in grams:


393 1327 776 1383 1179 913 1958 651 1307 1909
426 592 1691 412 1233 1490 1010 1481 1400 1280
722 1612 628 1483 718 1313 642 1478 630 1454
818 881 1227 929 1341 865 982 1488 983 833
1145 1444 634 1010 1727 1076 1137 1439 1149 1074
1296 1321 1690 1364 1916 1206 1309 1756 1536 1366
1439 810 1478 1532 784 1182 1573 429 1519 1303
1639 1689 1083 1670 1617 1610 1144 1370 1762 1748
1840 3890 806 1733 1917 1435 1845 1380 1631 214
2027 1212 1332 1482 1442 1553 1416 1170 1763 1129

a) Suggest suitable classes of a frequency distribution for the weights of fish


b) Construct the frequency distribution with one open ended class and state a reason for your
choice
c) Construct a less than ogive of the data
d) Construct a less than cumulative diagram of the weights

150

BUSINESS MATHS & STATISTICS (TC3)


7. The following are sales figures of a small scale grocery shop.

Actual 2008 Actual 2009


January 150 162
February 154 163
March 183 171
April 162 158
May 181 175
June 149 145
July 130 121
August 152 138
September 186 172
October 199 175
November 193 163
December 168 152

a) Present the figures on a Z chart


b) Comment on what the chart shows

8. The following is a distribution of monthly incomes among farmers in Maganga Village.


Incomes (MK’000) Number of farmers
Women Men
Less than 30 2 0
30 up to 40 3 1
40 up to 50 17 4
50 up to 60 17 24
60 up to 70 8 21
70 up to 80 3 7
80 or more 0 3
Construct separate Lorenz curves for women and men on the same Cartesian plane. Comment on the
distribution of income among women and men in Maganga.

9. The following data shows the distribution of wealth in Chindozwa.

Wealth owned by: Percentage of total wealth


most wealth 1% 18
most wealth 5% 37
most wealth 10% 51
most wealth 25% 72
most wealth 50% 93
all wealth owners 100
Represent the information on a Lorenz curve. Briefly comment on your diagram.

151

BUSINESS MATHS & STATISTICS (TC3)


CHAPTER 12 STATISTICAL MEASURES

LEARNING OBJECTIVES

By the end of this chapter students should be able to:


i. Calculate measures of central tendency from simple data
ii. Calculate measures of central tendency for frequency distributions
iii. Interpret measures of central tendency
iv. Calculate measures of dispersion
v. Interpret measures of dispersions
vi. Compare distributions using summary measures
vii. Determine the skewness of a distribution
viii. Interpret the skewness of a distribution

12.0 INTRODUCTION
Data summarization is a process of combining scores into a single number, called a statistic. Statistics serve
two functions: they estimate parameters in population models and they describe the data. This chapter looks
at measures that typify the data, called the measures of central tendency or averages, and measures of
dispersion which show how compact or spread the data values are.

12.1 MEASURES OF CENTRAL TENDENCY


The commonly used measures of central tendency include

x Arithmetic mean
x The mode
x The median
x Geometric Mean

12.1.1 The Arithmetic Mean


The Arithmetic mean, commonly called mean, is the sum of all scores in a data set (distribution) divided by
the total number of such scores.

i.e if x1 , x 2, ..., x n is a data set of n scores then:

x1  x 2  ......  x n
Mean
n

Notation:
The arithmetic mean is usually denoted by ‫ ݔ‬and the sum

x1  x 2  ...  x n ¦x
Hence mean:
σ௫
‫ݔ‬ҧ ൌ

152

BUSINESS MATHS & STATISTICS (TC3)


Example 1
Given data values (x) to be 4, 2, -3, 7, 5 find the mean.

σ௫
Solution Mean: ‫ݔ‬ҧ ൌ

Ͷ ൅ ʹ ൅ ሺെ͵ሻ ൅ ͹ ൅ ͷ

ͷ
ͳͷ
ൌ
ͷ

ൌ͵
Mean for frequency distributions
When data is in form of a frequency distribution, the mean is calculated using the following formula.

σ ݂‫ݔ‬
‫ݔ‬ҧ ൌ
σ݂

Where ‫ ݔ‬are the data values and ݂ their corresponding frequencies.


Note that for grouped frequencies distributions the data values ሺ‫ ) ݔ‬are the class midpoints

Example 2

The values 0, 1, 2, 3, 4, and 5 have been presented in a simple frequency distribution as follows:

Table 12.1
Data values(x): 0 1 2 3 4 5
Frequency: 2 3 7 8 6 4

Calculate the arithmetic mean


Solution
Table 12.2 Table of calculations

x f fx
0 2 0
1 3 3
2 7 14
3 8 24
4 6 24
5 4 20
30 85

σ ݂‫ݔ‬
‫ݔ‬ҧ ൌ
σ݂

153

BUSINESS MATHS & STATISTICS (TC3)


= 85/30
= 2.833

Note that the above is a simple frequency distribution where values of x can easily be identified.

In the following example, values of x are not identifiable. The calculation of mean in the following will
need the estimation of x because x is lost in grouping the data.

Example 3
Calculate the arithmetic mean of the following

Table 12:3 Distribution of order sizes


Order frequency
5-10 1
10 – 15 5
15 – 20 7
20 – 25 10
25 – 30 7
30 – 35 4
35 – 40 3
40 – 45 2
45 – 50 1
40

Solution

Table 12.4 Table of calculations


Order Frequency x fx
5 - 10 1 7.5 7.5
10 - 15 5 12.5 62.5
15 - 20 7 17.5 122.5
20 - 25 10 22.5 225
25 - 30 7 27.5 192.5
30 - 35 4 32.5 130
35 - 40 3 37.5 112.5
40 - 45 2 42.5 85
45 - 50 1 47.5 47.5
40 985

x values are estimated by the midpoints of each class of data

σ ௙௫ ଽ଼ହ
Mean = ‫ݔ‬ҧ ൌ ൌ =24.625
σ௙ ସ଴

154

BUSINESS MATHS & STATISTICS (TC3)


Example 4:

Mr. Selemani has 60 of salesmen in his business empire. He is interested in finding out the average sales
they make and to do so he has tallied the number of sales each one makes and has grouped them as follows:

Table 12.5 Sales


Number of sales Number of
salesmen
0-4 1
5-9 10
10-14 18
15-19 16
20-24S 11
25 and above 4
60

Find the arithmetic mean of the sales for the salesmen as a suitable average.

Solution
Note that the frequency distribution has an open ended class (25 and above). This is closed by assigning it
the most common class interval. In this case 4 and it becomes 25 to 29.

Table 12.6 Sales (table of calculations


Number of sales x f fx

0 Ͳ 4 2 1 2
5 Ͳ 9 7 10 70
10 Ͳ 14 12 18 216
15 Ͳ 19 17 16 272
20 Ͳ 24 22 11 242
25 Ͳ 29 27 4 108
60 910

σ ݂‫ݔ‬
‫ݔ‬ҧ ൌ
σ݂

ͻͳͲ

͸Ͳ

=15.16667

The average number of sales per salesman is 15

Advantages and disadvantages of the Arithmetic Mean

The arithmetic mean has the following advantages and disadvantages.

155

BUSINESS MATHS & STATISTICS (TC3)


Advantages

(i) The mean is easy to understand


(ii) It is a true representative of the data (it uses all data values)
(iii) The mean can be used in further statistical analysis.

Disadvantages

(i) The mean is influenced by the extreme values


(ii) It can result in unrealistic figures

Note: These advantages and disadvantages help the user to decide whether or not the mean is the
appropriate average.

12.1.2 The Mode


Definition

Mode of a distribution of data is the data item that has the highest frequency of occurrence

A. Mode for ungrouped data or a simple frequency distribution.


If data is not grouped, the mode can be found by inspection or simple count of the individual values. The
same applies when data is in form of a simple frequency distribution.

Example 5 Given x values as: 4, 0, 2, 5, 4, 8, 3. Find the mode

Solution: By inspection mode = 4

Example 6 Given x =4, 0, 2, 5, 4, 2, 8, 3

Solution: Mode = 4 and 2

Note that it is possible for a distribution to have more than one mode as in the second example. A
distribution with one mode is said to be unimodal and bimodal when it has two modes.

Example 7 Consider the simple frequency distribution given below and find the mode.

Table 12.7 table of x values


x f
0 2
1 3
2 7
3 8
4 6
5 4

Solution The mode is 3 (item with the highest frequency)

156

BUSINESS MATHS & STATISTICS (TC3)


B. Mode for grouped data(Using a formula)

When data is grouped, the mode can be estimated by using a formula or histogram.

The starting point is to identify the modal class. Given that individual data items of a frequency distribution
cannot be identified, the mode is assumed to be contained in the class with the highest frequency. Having
identified the modal class, the mode can then be estimated using the formula:

݂ଵ െ ݂଴
‫ ݁݀݋ܯ‬ൌ ‫ ܮ‬൅ ൬ ൰‫ܥ‬
ʹ݂ଵ െ ݂ଶ െ ݂଴
Where f1 = frequency of modal class
f0 = the frequency immediately before the model class
f2 = frequency immediately after the modal class
C = modal class width
L = lower boundary of modal class

Example 8

Kamkaka Farms supplies milk directly to shops, hotels and lodges in the Capital City. The MD is interested
in the most likely number of orders that the farm receives in order to prepare for the supplies realistically.

The following are the figures tabulated by the sales office. Calculate the mode as his guide.

Table 12.8 Weekly orders


Orders in a week Number of weeks
10-15 6
15-20 7
20-25 8
25-30 10
30-35 5
35-40 3

Solution First identify the modal class


Highest frequency = 10.
Modal = 25 - 30.

L (Lower limit of modal class) = 25


f1 ( the frequency of the modal class) = 10
f0 (The frequency of class immediately before modal class) = 8
f2 (Frequency of the class immediately after the modal class) = 5
C = modal class width =30 – 25= 5
݂ଵ െ ݂଴
‫ ݁݀݋ܯ‬ൌ ‫ ܮ‬൅ ൬ ൰‫ܥ‬
ʹ݂ଵ െ ݂ଶ െ ݂଴

ଵ଴ି଼
ൌ ʹͷ ൅ ቀ ቁൈͷ
ሺଶൈଵ଴ሻିହି଼

= 26.43

157

BUSINESS MATHS & STATISTICS (TC3)


C. Mode for grouped data (Using histogram)

Example 9

Consider Kamkaka’s figures above. Draw the histogram to represent the number of orders and use it to
estimate the mode.

Solution:

Figure 12.1 Mode using histogram


Weeks Kamkakaorders

10
9
8
7
6
5
4
3
2
1
0
10 15 20 25 30 35 40 Orders

Modeapprox26.5

Note the two lines drawn in the modal class bar. The mode is estimated by dropping a vertical line from the
intersection point of the two lines to the x-axis

Advantages and Disadvantages of the Mode

Advantages of the mode


i) It is easy to understand
ii) The mode is not easily influenced by extreme values
iii) The mode is realistic (it gives realistic figures) and it will usually be one of the data values except
for the ones estimated through formula or histogram.

Disadvantages

i) The mode cannot be used in further statistical analysis it does not use all the data values.
ii) The mode may not be unique.

158

BUSINESS MATHS & STATISTICS (TC3)


12.1.3 Median
Definition

If data is arranged in some ascending or descending order the median is the data item at the middle of the
distribution. Alternatively the median can be described as the numerical value separating the higher half of a
data in a distribution from the lower half.

Example 10

Given the x-values as: 3, 10, 4, 5, 8, 15, 3. Find the median

Solution:
The data rearranged: 3 3, 4, 5, 8, 10, 15

The value in the middle is 5. This is the median

Methods of finding the median

In example 10 the median was determined by a simple count because the number of data items n is small. If,
however, n is large the following techniques can be used to determine the median
௡ାଵ
a. If n is odd the median is the item on position when the values data values are arranged

in order.

Example 11

Use the technique outlined above to determine the median of example 1.

Solution: ݊ൌ͹
௡ାଵ
The median is therefore the item on position ൌͶ

Rearranged data: 3, 3, 4, 5, 8, 10, 15


The 4th item is 5
Hence median = 5
௡ ௡
b. If ݊ is even the median is the arithmetic mean of the items on positions and ൅ ͳ
ଶ ଶ

Example 12

Given the x-values as: 22, 10, 4, 5, 8, 15, 3, 17. Find the median

Solution
݊ ൌ ͺ, which is even
଼ ଼
Therefore we find items on positions ൌ Ͷand ൅ ͳ
ଶ ଶ

Rearranged data: 3, 4, 5, 8, 10, 15, 17, 22

159

BUSINESS MATHS & STATISTICS (TC3)


The 4th and 5th items are 8 and 10
଼ାଵ଴
Median = ൌͻ

Example 13

Consider the following simple frequency distribution

Table 12.9 x values

x f
0 2
1 4
2 6
3 10
4 7
5 4
6 1 Find the median.

Solution

The distribution shows that the x values are already arranged in an ascending order. A cumulative frequency
column can be used to count the values and determine the median.

Table 12.10 Table of calculations


x f Cumf n is 34 which is even. The median is
0 2 2 therefore the mean of items 17 and 18.
1 4 6
2 6 12 From the cumulative frequency
3 10 22
column:
17th item = 10
4 7 29
18th item = 10
5 4 33
6 1 34 ͳͲ ൅ ͳͲ
Median for grouped data ൌ ͳͲ
ʹ
Since in a grouped distributions individual values are lost, themedian
median can
= only
10 be estimated and they are
two ways this which this can be done

(a) Using formula

Given a frequency distribution the median is estimated by the interpolation formula:

σ ݂ൗ
‫ ݊ܽ݅݀݁ܯ‬ൌ ‫ ܮ‬൅ ቌ ʹ െ σ ݂ቍ ‫ܥ‬
݂

Having already identified a median class,


L= Lower boundary of the median class
160

BUSINESS MATHS & STATISTICS (TC3)


f = Frequency of the median class
C=median class width
f0= Cumulative frequency up to the class immediately before the median class

Example 14 Consider the following table regarding Kamkaka’s orders

Table 12.10 Weekly orders


Orders in a week Number of weeks
10-15 6
15-20 7
20-25 8
25-30 10
30-35 5
35-40 3

Estimate the median number of orders

Solution:

Table 12.11 Weekly orders cumulative frequencies

Orders in a week Number of Cum f


weeks
10 15 6 6
15 20 7 13
20 25 8 21
25 30 10 31
30 35 5 36
35 40 3 39

Cumulative frequency 19.5 falls in class 20 to 25. This is the median class

L=20 f =8 C=5
f0= 13

= 20+ (6.5/8) x 5 = 20 + 4.06


= 24.06

161

BUSINESS MATHS & STATISTICS (TC3)


(b) Using a cumulative frequency curve (Ogive).

The median can also be estimated graphically by using the ogive.

Example 15

Consider the orders received by Kamkaka farms in example 4. Draw an ogive for the distribution of the
orders and use it to estimate the median.

Solution

Table 12.12 Weekly order cumulative frequency


Orders f Cumf
"Lesthan"
10 0 0
10 15 6 6
15 20 7 13
20 25 8 21
25 30 10 31
30 35 5 36
35 40 3 39

Figure 12.2 The ogive and median

Advantages and disadvantages of the median

Advantages
162

BUSINESS MATHS & STATISTICS (TC3)


i. It is easy to understand
ii. Its not influenced by extreme values
iii. It will often assume a value equal to the original data
Disadvantages

i. It cannot be used in advanced statistical analysis


ii. It does not use all the data values and therefore it’s not truly representative of the distribution.
iii. For ungrouped data, the median requires that data to be rearranged.

12.1.4 Geometric Mean


Unlike the arithmetic mean which uses sums of data values, the geometric mean uses products of the data
values. For a data set with nvalues, the geometric mean is the ݊௧௛ root of the product of the data values.

i.e. If ‫ݔ‬ଵǡ ‫ݔ‬ଶǡ ‫ݔ‬ଷǡ ǥ ‫ݔ‬௡ǡ is a set of n data values, then their geometric mean is given by

‫ ܯܩ‬ൌ ೙ඥ‫ݔ‬ଵ ൈ ‫ݔ‬ଶ ൈ ‫ݔ‬ଷ ൈ ǥ ൈ ‫ݔ‬௡

Notation

If ‫ݔ‬ଵǡ ‫ݔ‬ଶǡ ‫ݔ‬ଷǡ ǥ ‫ݔ‬௡ǡ is a general set of n values, then the product ‫ݔ‬ଵ ൈ ‫ݔ‬ଶ ൈ ‫ݔ‬ଷ ൈ ǥ ൈ ‫ݔ‬௡ is denoted ς௡௜ୀଵ ‫ݔ‬௜
hence the geometric mean

‫ ܯܩ‬ൌ ೙ඥ‫ݔ‬ଵ ൈ ‫ݔ‬ଶ ൈ ‫ݔ‬ଷ ൈ ǥ ൈ ‫ݔ‬௡



ൌ ሺς௡௜ୀଵ ‫ݔ‬௜ ሻ೙

Example 16

Find the geometric mean of 4, 5, 12 8, and 2

Solution
‫ ܯܩ‬ൌ ೙ඥ‫ݔ‬ଵ ൈ ‫ݔ‬ଶ ൈ ‫ݔ‬ଷ ൈ ǥ ൈ ‫ݔ‬௡

Since there are 5 values, the geometric mean is the ͷ௧௛ root of their product.

‫ ܯܩ‬ൌ ξͶ ൈ ͷ ൈ ͳʹ ൈ ͺ ൈ ʹ

ൌ ξ͵ͺͶͲ
ൌ ͷǤʹͳͲ͵Ͷ

12.2 MEASURES OF DISPERSION


The measures of central tendency we have looked at in the previous chapter are used to represent data or
compare them, but on their own they do not give sufficient information about distributions. Here we will
consider the following measures of dispersion.

i. Range
ii. Mean deviation
iii. Variance and standard deviation
163

BUSINESS MATHS & STATISTICS (TC3)


v. Coefficient of skewness
12.2.1 Range
Definition
The range of a distribution of data is defined as the difference between the highest value and lowest value in
a data set

i.e. Range =The highest value – the lowest value

Example 17 Find range, given the following values of x

a) 5, 4, 15, 7, 8, 2, 1

b) 5, 4, -50, 7, 8, 2, 1

Solution a) Highest value = 15; Lowest value =1


Range = 15 – 1=14

b) Range = 8 - (-50) = 58

12.2.2 Mean Deviation


Definition
The mean deviation of a data items is defined as the arithmetic mean of the absolute differences between the
arithmetic mean and each of the data value.

Thus, if ‫ݔ‬ҧ is the mean of some ‫ ݔ‬values then


σȁ‫ ݔ‬െ ‫ݔ‬ҧ ȁ
‫ ݊݋݅ݐܽ݅ݒ݁ܦ݊ܽ݁ܯ‬ൌ
݊

Where ݊ is the number of data values.

Example 18

The following are ages (in years) of pupils in standard three:11, 7, 6, 8, 10, 7, 9, 8, 6, and 8. Calculate
the mean deviation of the ages

Solution
ͳͳ ൅ ͹ ൅ ͸ ൅ ͺ ൅ ͳͲ ൅ ͹ ൅ ͻ ൅ ͺ ൅ ͸ ൅ ͺ
‫ݔ‬ҧ ൌ
ͳͲ
଼଴
ൌ = 8
ଵ଴
‫݊݋݅ݐܽ݅ݒ݁ܦ݊ܽ݁ܯ‬
ȁͳͳ െ ͺȁ ൅ ȁ͹ െ ͺȁ ൅ ȁ͸ െ ͺȁ ൅ ȁͺ െ ͺȁ ൅ ȁͳͲ െ ͺȁ ൅ ȁ͹ െ ͺȁ ൅ ȁͻ െ ͺȁ ൅ ȁͺ െ ͺȁ ൅ ȁ͸ െ ͺȁ ൅ ȁͺ െ ͺȁ

ͳͲ
͵൅ͳ൅ʹ൅Ͳ൅ʹ൅ͳ൅ͳ൅Ͳ൅ʹ൅Ͳ

ͳͲ

164

BUSINESS MATHS & STATISTICS (TC3)


ͳʹ
ൌ ൌ ͳǤʹ
ͳͲ

The mean deviation is simple to understand but the problem is that one has to remember to use only the
absolute values (ignoring negative signs).

12.2.3 Variance and Standard Deviation


Definition
Standard deviation is a statistic that measures the spread of data around the arithmetic mean. The variance is
the square of the standard deviation. It is normal as it will be seen below to calculate the variance first then
the standard deviation. The variance is generally taken to be an interim measure/step in the calculation of
standard deviation.
The standard deviation which is often denoted by Sis calculated as:

σሺ௫ି௫ҧ ሻమ
ܵ ൌ ට for simple data.

For data in a frequency distribution



σ ݂‫ ݔ‬ଶ σ ݂‫ݔ‬
ܵൌඨ െቆ ቇ
σ݂ σ݂

Example 19

Find the standard deviation of 4, 3, 5, 7, 6

Solution
σሺ‫ ݔ‬െ ‫ݔ‬ҧ ሻଶ
ܵ ൌ ඨ
݊

Ͷ൅͵൅ͷ൅͹൅͸
‫ݔ‬ҧ ൌ
ͷ

=5

ሺͶȂ ͷሻଶ  ൅ ሺ͵Ȃ ͷሻଶ  ൅  ሺͷȂ ͷሻଶ  ൅  ሺ͹Ȃ ͷሻଶ ൅  ሺ͸Ȃ ͷሻଶ
ܵൌ
ͷ

= 2

Note: For easy computation Standard deviation for simple data is also calculated using the formula:


σ ‫ݔ‬ଶ σ‫ݔ‬
ඨ െቆ ቇ
݊ ݊

BUSINESS MATHS & STATISTICS (TC3)


Example 20
A day care facility in a small town takes care of 30 under-five children of working class parents. The ages of
the children are distributed as follows
Table 12.13 Children’s Age distribution
Age of children (x) Number of children
1 5
2 8
3 10
4 5
5 2
Total 30

Find the standard deviation of the children’s ages.

Solution:

Table 12.14 Table of calculations

Age of Number of fx 2
fx
children (x) children (f)

1 5 5 5
2 8 16 32
3 10 30 90
4 5 20 80
5 2 10 50
Total 30 81 257


σ ݂‫ ݔ‬ଶ σ ݂‫ݔ‬
ܵൌඨ െቆ ቇ
σ݂ σ݂


ʹͷ͹ ͺͳ ଶ
ൌඨ െ൬ ൰
͵Ͳ ͵Ͳ

= 1.13 years

Example 21
The following set of data represents a frequency distribution of accidents recorded on 50 road stretches
selected throughout the country in the month of December.

Table 12:15 Accidents recorded

No. of accident recorded Frequency


0 but under 2 5
2 but under 4 10
4 but under 6 15
166

BUSINESS MATHS & STATISTICS (TC3)


6 but under 8 10
8 but under 10 6
10 but under 12 4

(a) Calculate the mean number of accidents

(b) Calculate the standard deviation and interpret it.

Solution:

Table 12.16 Table of calculations (accidents)


2
Recordedaccidents x f ™fx ™fx
2 Ͳ 4 3 5 15 45
4 Ͳ 6 5 10 50 250
6 Ͳ 8 7 10 70 490
8 Ͳ 10 9 6 54 486
10 Ͳ 12 11 4 44 484
35 233 1755

b) Mean:
σ ݂‫ݔ‬
‫ݔ‬ҧ ൌ
σ݂
ଶଷଷ
ൌ = 6.66
ଷହ

σ ௙௫ మ σ ௙௫ ଶ
b) ܵൌට σ௙
െቀσ ቁ

ͳ͹ͷͷ ʹ͵͵ ଶ
ൌඨ െ൬ ൰
͵ͷ ͵ͷ
= 2.406

The mean number of accidents is 6.66. However the number of accidents spread about the mean by 2.41
accidents.

12.2.4 Quartile Deviation


Definition
The quartile deviation (sometimes called the semi inter-quartile range) is a value that measures the spread of
data around the median. To calculate the quartile deviation one must first of all estimate the quartiles.

Quartiles are data values that divide a distribution into sections of data of 25% (quarter) of the data each.

167

BUSINESS MATHS & STATISTICS (TC3)


Illustration

Figure 12.3 Quartiles


25% 25% 25% 25%ofdata

Q1 Q2 Q3 Note that Q2 is the median.

Example 22
Identify the three quartile points in the following set of data values

X: 2, -4 10, 20, 12, 8, 5, 25, 17, 11, 3, 19, 26

Solution
Rearranged: -4, 2, 5,10, 11, 12, 17, 19, 20, 25, 26

Q1 = 5, Q2 = 12, Q3 = 20
Calculation of the quartile deviation

The quartile deviation of data is calculated as; half the deference between the 3rd and 1st quartiles.

If we can denote the quartile deviation with QD

Then
ܳଷ െ ܳଵ
ܳ‫ ܦ‬ൌ
ʹ

where Q1 = 1st quartile


Q2 = 3rd quartile

Example 23
Consider the data in example 6 above and calculate the quartile deviation

Solution: 
ܳଷ െ ܳଵ
ܳ‫ ܦ‬ൌ
ʹ

ʹͲ െ ͷ

ʹ

= 7.5

Quartile deviation for grouped data

Example 24

Consider the following delivery times (days) of orders.

168

BUSINESS MATHS & STATISTICS (TC3)


Table 12.17 Deliver time distribution

Delivery time 0 1 2 3 4 5 6 7 8 9 10 11

No of orders 4 8 11 12 21 15 10 4 2 2 1 1
Find the (a) the median and
(b) the quartile deviation for the delivery times

Solution

Table 12.18 Delivery times-Cumulative frequency distribution


X f Cum. f
0 4 4
1 4 12
2 11 23
3 12 35
4 21 56
5 15 71
6 10 81
7 4 85
8 2 87
9 2 89
10 1 90
11 1 91

b. Median:

n is odd and therefore median is item on position


σ݂ ൅ ͳ
ʹ

ଽଵାଵ
ൌ = 46th item.

From cumulative frequencies, 46th item is in the group that takes the frequencies from a cumulative number
of 35 to 56.

This data item is 4.


Median = 4

c. The quartile deviation:

There is need to find the ܳଵ and ܳଷ items


௡ାଵ ଽଵାଵ
ܳଵ the item on position ൌ
ସ ସ
= 23rd item

169

BUSINESS MATHS & STATISTICS (TC3)


ܳଵ ൌ ʹ (using cum frequencies as a guide)

ଷሺଽଵାଵሻ
ܳଷ is item on position = 69thitem

ܳଷ ൌ ͷ
ହିଶ
Therefore ܳ‫ ܦ‬ൌ

= 1.5
Example 25
Consider the orders received by Kamkaka farms in example 4 of section 5 above.
a) Calculate the quartile deviation using a formula.
b) Estimate the quartile deviation graphically

Solution:

Table 12.19 Orders less than cumulative frequencies


Orders f Cumf
"Lesthan"
10 0 0
10 15 6 6
Q1class 15 20 7 13
20 25 8 21
Q3class 25 30 10 31
30 35 5 36
35 40 3 39


σ ݂ൗ
ܳଵ ൌ ‫ ܮ‬൅ ቌ Ͷ െ σ ݂଴ ቍ ‫ܥ‬
݂

Where L = lower limit of Q1 class f = frequency of Q1 class


σ௙ ଷଽ
™f0 = Cum f up to class before Q1 class. ܳଵ class is on position ൌ
ସ ସ
= 9.75

ଽǤ଻ହି଺
 ܳଵ ൌ ͳͷ ൅ ቀ ቁͷ

ൌͳ͹Ǥ͸ͺ

͵ σ ݂ൗ
ܳଷ ൌ ‫ ܮ‬൅ ቌ Ͷ െ σ ݂଴ ቍ ‫ܥ‬
݂

Where L = lower limit of class ܳଷ


f = frequency of ܳଷ class
™f0 = Cum f up to ܳଶ class before class.
BUSINESS MATHS & STATISTICS (TC3)
ଷσ௙ ଷൈଷଽ
ܳଷ class is on position ൌ = 29.25
ସ ସ

ଷൈଽǤ଻ହିଶଵ
ܳଷ ൌ ʹͷ ൅ ቀ ቁ ͷൌʹͻǤͳ͵
ଵ଴

ொయ ିொభ
ܳ஽ ൌ

ʹͻǤͳ͵ െ ͳ͹Ǥ͸ͺ

ʹ

= 5.72

ƒ Using the graph requires first drawing the Ogive and locating the ܳଵ and ܳଶ values

Figure 12.4 The ogive and quartiles

c) From the graph, median = 23.5

d) For the quartile deviation the process is the same as for locating the median. The difference is that
for the quartile deviation indicators are ¼ of the frequencies (to find ܳଵ and ¾ of the cumulative
frequencies to findܳଷ .

σ௙ ଷଽ
ܳଵ is positioned on = = 9.75
ସ ସ
Draw a horizontal line from 9.75 to the graph. Then drop a perpendicular line from where line above meets
the graph.

ܳଵ is where the perpendicular line meets the x axis.


171

BUSINESS MATHS & STATISTICS (TC3)


And this point is 16.5

ଷൈσ ௙ ଷൈଷଽ
ܳଷ is positioned on = .=29.25
ସ ସ
The process is the same as for ܳଵ
Therefore ܳଷ = 26.5

ʹ͸Ǥͷ െ ͳ͸Ǥͷ
ܳ‫ ܦ‬ൌ
ʹ
= 5

12.2.5 Coefficient
Definition
A Coefficient of variation (CV) of a distribution is a measure of the relative spread of data in distributions.
It basically expresses the standard deviation as a proportion of the mean. That way it can be used as tool for
comparing the spread of two or more distributions.

For a data set with mean ‫ݔ‬ҧ and standard deviation ܵ the Coefficient of Variation is given by :

‫ ܸܥ‬ൌ
௫ҧ

Example 26

An auditor examines two batches of invoices. To get a feel of the values he tabulates the values and
calculates the mean and standard deviations as follows:
Distribution1 Distribution 2
MK’000 MK’000
Mean 1260 25.2
Std Deviation 52 15.2

Calculate the coefficients of variation and interpret.

Solution

ͷʹ
‫ܸܥ‬ଵ ൌ
ͳʹ͸Ͳ

=0.041

ଶହǤଶ
‫ܸܥ‬ଶ ൌ
ଵହǤଶ

=0.603

Distribution 2 has a higher CV, therefore it is more variable.

172

BUSINESS MATHS & STATISTICS (TC3)


Example 27

A production manager orders an item to be used in production from two suppliers, as a way of ensuring
consistent supply. In order to judge the reliability in terms of delivery times, he has compiled the time in
days it took to receive 10 orders in the past two weeks from each supplier. The figures are as follows:

Table 12.20 Supplier delivery times

Order SupplierA SupplierB


No. deliverytime deliverytime
1 13 4
2 16 10
3 25 3
4 9 7
5 15 15
6 19 2
7 7 18
8 18 5
9 19 2
10 10 9

Advise the manager on the reliability of the suppliers.

Solution:

The reliability here can be judged from mean, standard deviation and the coefficient of variation.

Table 12.21 Table of calculations (delivery times)

2 2
X X X X
13 169 4 16
16 256 10 100
25 625 3 9
9 81 7 49
15 225 15 225
19 361 2 4
7 49 18 324
18 324 5 25
19 361 2 4
10 100 9 81
Totals 151 2551 75 837

Supplier A Mean:
ଵହଵ
‫ݔ‬ҧ ൌ ൌ ͳͷǤͳ
ଵ଴

173

BUSINESS MATHS & STATISTICS (TC3)


Standard deviation:
ଶହହଵ
ܵൌට െ ͳͷǤͳଶ  ൌ  ξʹ͹ǤͲͻ
ଵ଴
ൌ ͷǤʹͲ

Coefficient of variation:

ହǤଶ଴
‫ ܸܥ‬ൌ  ൌ ͲǤ͵Ͷ
ଵହǤଵ

Supplier B Mean:
଻ହ
‫ݔ‬ҧ ൌ ൌ ͹Ǥͷ
ଵ଴

Standard deviation:
଼ଷ଻
ܵൌට െ ͹Ǥͷଶ
ଵ଴

ൌ ξʹ͹ǤͶͷ
ൌ ͷǤʹͶ

ହǤଶସ
Coefficient of variation: ‫ ܸܥ‬ൌ ൌ ͲǤ͹Ͳ
଻Ǥହ

Supplier B has a lower average meaning the average delivery time is lower, in other words he can deliver
quicker on average.

The standard deviations are more or less the same (5.2), However the CV for B is almost two times that of
A. This means the delivery times for B are relatively more variable. This makes them less predictable
therefore, not very reliable.

12.2.6 Skewness
A distribution in which the values of mean, median and mode coincide (i.e. mean = median = mode) is
known as a symmetrical distribution. Conversely, when values of mean, median and mode are not equal the
distribution is known as asymmetrical or skewed distribution. In moderately skewed or asymmetrical
distribution a very important relationship exists among these three measures of central tendency. In such
distributions the distance between the mean and median is about one-third of the distance between the mean
and mode

Mode = mean - 3 [mean - median]

‫ ݁݀݋ܯ‬ൌ ͵‫ ݊ܽ݅݀݁ܯ‬െ ʹ‫݊ܽ݁ܯ‬

Median = mode + (2/3)(mean-mode)

Example28

Given median = 20.6, mode = 26. Find mean.

Solution: Mode = 3 Median – 2 Mean


174

BUSINESS MATHS & STATISTICS (TC3)


ͳ
‫ ݊ܽ݁ܯ‬ൌ ሾ͵‫ ݊ܽ݅݀݁ܯ‬െ ‫݁݀݋ܯ‬ሿ
ʹ

ൌ ሾ͵ ൈ ʹͲǤ͸ െ ʹ͸ሿ = 17.9

Pearson’s measure of skewness


While the degree of skewness could be measured by the difference between the mean and mode, for most
practical purposes it is required that a measure of skewness be unit free. The pearson’s measure of skewness
is used in this regard.

Pearson’s measure of skewness:


‫ ݊ܽ݁ܯ‬െ ‫݁݀݋ܯ‬
ܲ௦௞ ൌ
ܵ‫݊݋݅ݐܽ݅ݒ݁݀݀ݎܽ݀݊ܽݐ‬
ଷሺெ௘௔௡ି௠௢ௗ௘ሻ

ௌ௧௔௡ௗ௔௥ௗௗ௘௩௜௔௧௜௢௡

If ܲ௦௞ ൐ Ͳ the distribution is said to be positively skewed

ܲ௦௞ ൏ Ͳ the distribution is said to be negatively skewed

ܲ௦௞ ൌ Ͳ there is no skewness (the distribution is symmetric)

Example 29

Given that the distribution in the above example ( with median = 20.6, mode = 26) has a standard deviation
of 6.1 calculate the Pearson,s Measure of skewness

Solution.

‫ ݊ܽ݁ܯ‬െ ‫݁݀݋ܯ‬
ܲ௦௞ ൌ
‫݊݋݅ݐܽ݅ݒ݁݀݀ݎܽ݀݊ܽݐݏ‬
ͳ͹Ǥͻ െ ʹ͸Ǥ

͸Ǥͳ
ൌ െͳǤ͵ʹͺ
The negative Pearson’s measureof skewness shows that the distribution is negatively skewed.

CHAPTER SUMMARY

In this chapter we have looked at various ways of summarising, describing and comparing distributions of
data using the following:

x Measures of Central Tendency


Arithmetic mean, median, mode and geometric mean: definitions, calculations and interpretation

x Measures of Dispersion:
Range, mean deviation, standard deviation, variance and inter quartile range: definitions,
calculations and interpretations

x Coefficient of Variation: Definition, calculation and interpretation


175

BUSINESS MATHS & STATISTICS (TC3)


x Measures of Skewness
Definition of skewness
Empirical relationship between mean, median and mode
Pearson’s measure of skewness: Calculation and interpretation

END OF CHAPTER EXERCISES

1 Distinguish between the arithmetic mean, median and the mode of a set of data.

2 An enquiry on 20 families shows the following number of children in the families

Number Number
of Children of families
1 2
2 4
3 8
4 4
5 2

Calculate the mean, mode and median number of children in the families

3. What is the best “average” if any, to use in each of the following situations? Justify each of your
answers.
a. If a shop sold television sets and wants to find the price of average television set.

b. To establish a typical wage to be used by an employee in wage negotiations for a small


company of 300 employees, a few of whom are very highly paid specialists.

c. To state the amount to be paid to each employee when the company introducing a profit
sharing scheme requires that each employee receives the same amount.

d. To determine the height of a bridge to be constructed (not a drawbridge) where the


distribution of the heights of all ships which would pass under it is known.

e. To ascertain the average annual income of all workers when it is known that the mean annual
income of skilled workers is K900,000 while the mean annual income of unskilled workers is
K700,000.00.
4. Consider the following set of measurements which represent the unemployment rates (%) of 12
countries:

7, 9, 11, 5, 3, 5, 10, 12, 5, 6, 7, 4

Find the arithmetic mean, median, mode and geometric mean of these 12 unemployment rates.

5. The following set of data represents a frequency distribution of the unemployment rates (%) of 50
countries:

Unemployment rate(%) Frequency


0 but under 2 5

BUSINESS MATHS & STATISTICS (TC3)


4 but under 6 15
6 but under 8 10
8 but under 10 6
10 but under 12 4

Calculate the mean mode and median of the distribution of the unemployment rate above.

6. Given the following simple data set: 9, 8, 3, 4, 7, 5, 3, find the range, mean deviation, and standard
deviation

7. The following set of data represents the annual acquisition expenses in K million (Km) incurred by
100 insurance companies in 2009:

Acquisition Expenses (Km) Frequency

Less than 20 10
20 but less than 40 35
40 but less than 60 40
60 but less than 80 10
80 but less than 100 3
100 but less than 120 2

(a) Find
i. the arithmetic mean
ii. median
iii. mode for this distribution
iv. the Pearson’s coefficient of skewness and say whether the distribution is positively
or negatively skewed.

(b) Calculate the standard deviation of the acquisition expenses

(c ) Calculate the coefficient of variation for the acquisition expenses

8. Incomes of females in 2006 by highest educational qualification were as follows

Range of Weekly income Degree O-level


(K’000) % %
40 and under 60 5 15
60 and under 80 7 30
80 and under 100 7 28
100 and under 120 18 14
120 and under 140 23 7
140 and under 160 14 3
160 and under 180 10 2
180 and under 220 16 1

Given that there were 100 women with degrees and 500 women with O-level
177

BUSINESS MATHS & STATISTICS (TC3)


(i) Calculate the mean and standard deviation for weekly income for those with a degree
qualification.
(ii) Calculate the mean and standard deviation for weekly income for those with a O-level
qualification
(iii) Using coefficients of variation compare the variability in the distributions.

9. A food processor makes chambo fillet from chambo supplied from the Lake. It is known that fish of
weight greater than 1 Kg make good fillets which fetch better prices. Th e processor further knows
that to make a profit, fish weighing at least 1Kg must make up 59% of a consignment. In order to
forecast his revenue and therefore profit, he decides to take a sample of 100 fish from different
consignments in accordance to a predetermined scientific technique. The following is the frequency
distribution of the fish:

Weights of 100 fish in grams:

Weght of fish Number of fish


• 200 but < 400 2
• 400 but < 600 4
• 600 but < 800 9
• 800 but < 1000 10
• 1000 but < 1200 13
• 1200 but < 1400 19
• 1400 but < 1600 20
• 1600 but < 1800 15
• 1800 but < 2000 6
• 2000 2

Required

a) Draw a histogram of the above distribution


b) Estimate the mean, mode and median fish weight.
c) Which one is the suitable average for the food processor and why?
d) Calculate the standard deviation, and the coefficient of skewness
e) Explain what the statistics calculated in (b) and (c) tell the food processor.
f) Draw an Ogive of the data.
g) Identify the weight range in which the middle 50% of the fish lie.
h) What is the proportion of fish that have weight of 1Kg and above? Advise the food
processor on what your calculation shows regarding making a profit.

10. The monthly salaries earned by 30 randomly-selected primary school teachers in a city (to the
nearest K’000) are as follows:

20 25 26 30 32 40
22 25 26 30 32 48
22 25 26 30 36 52
23 26 28 30 36 60
25 26 28 32 38 61

178

BUSINESS MATHS & STATISTICS (TC3)


(c) Form a simple frequency distribution to represent the distribution of salaries

(d) Find the arithmetic mean, median and mode for the salaries

(e) Calculate the standard deviation.

(f) Calculate the coefficient of skewness and comment on the result.

(g) The 30 teachers have expressed a concern that gaps between their salaries are too big
compared to teachers in an adjacent town which has a mean of K20,000 but a lower standard
deviation of 8300. Use an appropriate measure to compare the salaries of the two groups of
teachers and comment on the concern.



179

BUSINESS MATHS & STATISTICS (TC3)


CHAPTER 13 COMBINATORIAL ARITHMETIC

LEARNING OBJECTIVES

By the end of this chapter the student should be able to:

i. Differentiate between permutations and combinations.


ii. Carry out arithmetic computations involving permutations and combinations
iii. Understand the concepts well enough for use in sample space under probabilities

13.0 INTRODUCTION

Permutations and Combinations are techniques of quantifying arrangement or selections of items from a
group.

13.1 FACTORIALS

13.1.1 Definition
For each positive integer n, the quantity n factorial denoted n!,is defined to be theproduct of all the integers
from 1 to n:Ǩ  ൌ ͳ ൈ ʹ ൈ ͵ ൈ Ͷ ൈ ͷ ൈ ǥ ൈ ሺ െ ͳሻ  ൈ .By definition,ͲǨ  ൌ ͳǤ

Example 1
1 factorial or 1! = 1
2 factorial or 2! = ͳ ൈ ʹ ൌ ʹ
3 factorial or 3! = ͳ ൈ ʹ ൈ ͵ ൌ6
7 factorial or 7! = ͳ ൈ ʹ ൈ ͵ ൈ Ͷ ൈ ͷ ൈ ͸ ൈ ͹ ൌ ͷͲͶͲ

13.1.2 Arithmetic involving factorials

A. Multiplication and division


Factorials can be multiplied and divided in the normal way numbers are multiplied.

Example 2
1) Evaluate a) 4! x 3! b) 1! x 0! x 3! x 2!

2) Find the values of a) 4! ÷ 3! b) 26! ÷ 24!


n! .
c) Where n= 12 r = 2
(n-r)!*r!

3) Simplify (n+2)!
( n-1)!

180

BUSINESS MATHS & STATISTICS (TC3)


Solution
1 a) 4! x 3! = (1 x 2 x 3 x 4) x (1 x 2 x 3) = 24 x 6 = 144
b) 1! x 0! x 3! x 2! = 1 x 1x 6 x 2 = 12

2 a) 4! x 3! =
1 x=2 x43 x 4
b) 26! ÷ 24! = 25 x 126x 2 =x650
3
c) 12! .= 12! .
= 11 x 6
(12-2)!*2! (10)!*2!
= 66

3 (n+2)! = 1×2×3×4×...×(n – 1)×(n)×(n + 1)×(n + 2)

(n-1)! = 1×2×3×4×...×(n – 1)

ሺ݊ ൅ ʹሻǨ ͳ ൈ ʹ ൈ ͵ ൈ Ͷ ൈ ǥ ൈ ሺ݊Ȃ ͳሻ ൈ ሺ݊ሻ ൈ ሺ݊ ൅ ͳሻ ൈ ሺ݊ ൅ ʹሻ



ሺ݊ െ ͳሻǨ ͳ ൈ ʹ ൈ ͵ ൈ Ͷ ൈ ǥ ൈ ሺ݊Ȃ ͳሻ

ൌ ݊ሺ݊ ൅ ͳሻሺ݊ ൅ ʹሻ

ൌ ݊ଷ ൅ ͵݊ଶ ൅ ʹ݊

B. Addition and subtraction

Addition and subtraction involve the end result or product of the figures being multiply.

Example3 1 Simplify ͷǨ  ൅ ͵Ǩ

2 Subtract 3! From 4!

Solution 1 5! + 3! = 120 + 6 = 126

2 4! - 3! = 24 - 6 = 18

Factorials are often used in permutations and combinations

13.2 PERMUTATIONS

13.2.1 General concept

The concept of permutations relates to arranging objects or items in an ordered manner.

Thus a permutation of r objects at a time taken from n is the number of ways r objects can be arranged out
of n where the order is important

Example 4 Given 3 books which on a shelf, find the number of ways in which the three books can be
arranged

Solution Let the books be those of Algebra, Biology and Chichewa (A, B, C)

181

BUSINESS MATHS & STATISTICS (TC3)


If arranged systematically the arrangements are:

ABC, ACB

BAC, BCA

CAB CBC

There are 6 arrangements (permutations)

Example 5 Given the three books in a box, find the number of ways the books taking two at a time can
be arranged on the shelf

Solution If the books are ABC the following are the arrangements

AB BA
AC CA
AD DA
BC CB
BD DB
CD CD

There 12 arrangements (permutations)

13.2.2 Formula
A permutation of say r objects at a time from say n can be calculated using a formula.

A permutations of r objects out of n is denoted by nPr which is also written as Pn,r


n
Pr = n!/(n-r)!

Example 6 Find the number of permutations in

a) Example 1 and
b) Example 2 above

Solution

a) 3
P3 = 3!/(3-3)! = 3!/0! = 6/1 = 6

b) n
Pr = 3
P2 =
3!/(3-2)! = 6

Example 7 Find the permutations of 2 letters out of 4

Solution 4!/(4-2)! = 4!/2! = 4 x 3 = 12

182

BUSINESS MATHS & STATISTICS (TC3)


13.3 COMBINATIONS

13.3.1 Definition
A combination is a selection of r objects out of n where the order is not important.
Example 8 Find the number of way two items can be selected from 4
Solution Let the item be A, B, C, and D
The arranging these systematically in twos the combinations are:
AB AC, AD
BC, BD
CD
6 ways

13.3.2 Formula
A combination of say r objects at a time out of n is denoted by
n
n
Cr ; Cn,r or r
n
Cr= n .
(n-r)! r!

Example 9 Researchers send teams of two data collectors to collect data from a particular town. If they
have a total of 6 data collectors, determine the possible number of possible teams than can be
selected.

Solution This is a combination of two collectors at a time out of 6.


n
Cr ; = nCr ; = 6!/((6-2)!x2!)
= 6!/(4! x 2!)
= 6 x 5 /2 = 15 possible teams

CHAPTER SUMMARY

This chapter covered

x Permutation (the concept and formula and application)


x Combinations (the concept, formula and application)

STUDENT EXERCISES
1 Calculate a) 4! b) 0! c) 7!
d) 12! e) 1! f) 21!

2 Simplify the following factorial problems:


a) 3! + 2!
b) 3! - 2!
c) 3! × 2!

183

BUSINESS MATHS & STATISTICS (TC3)


d) 10!/3!
e) 12!/(12-4)
f) 59!/58!

3 Simplify nPn.

4 Express 10P5 in terms of factorials and evaluate.

5 Six different books are on a shelf. In how many different ways could you arrange them?

6 How many permutations are there of the letters wxyz?If four are taken at a time

7 Evaluate 8C6.
ళ஼ ൈ ల௉
ర య
8 Evaluate భబ ஼

9 A committee of three is supposed to be selected from a group of 5 people (two ladies and three
gentlemen. How many different committees are possible?

184
BUSINESS MATHS & STATISTICS (TC3)
CHAPTER 14 PROBABILITY THEORY
LEARNING OBJECTIVES
By the end of the chapter, the student should be able to:
i. Define probability
ii. Describe the role of probability in decision making
iii. Describe the classical, empirical, and subjective approaches to probability
iv. Distinguishexperiment, event and outcome
v. Calculate marginal and conditional probabilities
vi. Apply the rules of probability including addition and multiplication rules
vii. Apply a tree diagram to organize and compute probabilities

14.0 INTRODUCTION
The emphasis in the first chapters is on descriptive statistics. Descriptive statistics is concerned with
summarizing data collected from past events. Now we turn to a different facet of statistics. This facet allows
us to compute the chance that something will occur in future. Seldom does a decision maker have complete
information from which to make a decision. In situations like these, one takes a change or some risks to
make a decision. Because there is uncertainty in decision making, it is important that all the known risks be
statistically evaluated. Helpful in this evaluation and quantification of uncertainty is probability theory,
which is broadly referred to as the science of uncertainty. Probability allows a decision maker to analyze the
risks and minimize the gamble inherent. Furthermore, probability allows for the determination of reliability
and validity of generalisation of sample results to a population.

This chapter introduces the basic probability language, including terminology such as experiment, event,
and basic concepts of probability. We also discuss the main rules and principles of probability.

14.1 DEFINITIONS OF KEY TERMS


Probability is the numerical measure of the likelihood that an event in the future will happen. It can only
assume a value between 0 and 1 inclusive (i.e. 0 d P ( A) d 1 , where P(A) is the probability of the event A).
A value near zero means the event is not likely to happen and a value near one means it is more likely.

Key Terms
a) Experiment:
This term means an undertaking or a process that leads to the occurrence of one and only one of several
possible observations. This definition is more general than our understanding of an experiment from
physical science. In probability, an experiment has two or more possible results, called outcomes, and it is
uncertain which will occur. Examples include tossing a coin, drawing a card, making orders of supplies,
hiring out specific vehicles, and exploration/drilling to find oil.

b) Outcome:
An outcome is a particular result of an experiment. For example, if we asked 500 form four students
whether they would pursue a career in accounting. We would be uncertain as how many would say ‘yes’.
One possible outcome is therefore that ‘321 students indicate that they would pursue a career in accounting’.

185

BUSINESS MATHS & STATISTICS (TC3)


Another outcome is that ‘210 students would pursue a career in accounting’. Still another possibility is that
“all of them would pursue a career in accounting’.

c) Event:
An event is a collection of one or more outcomes of an experiment.

Experiment Roll a die Count the number of


companies in liquidation
Outcomes Observe a 1 None in liquidation
Observe a 2 One in liquidation
Observe a 3 Two in liquidation
Observe a 4 …
Observe a 5 67 in liquidation
Observe a 6 …
Some possible Observe an even number More than 21 in liquidation
events Observe a prime number 9 or fewer in liquidation
Observe an odd number
Observe a number less than 4

14.2 TYPES OF EVENTS


Considering the definition of an event above it is possible to distinguish two types of event in the context of
probability. These are mutually exclusive event and collectively exhaustive.

Mutually exclusive events are events which do not occur together. The occurrence of one precludes the
other. For instance, when a coin is tossed you either get head and tails. These are therefore mutually
exclusive. Another example is gender of an individual. One is either male or female.

Figure 14.1 Mutually exclusive events

Event A Event B

Mutually exclusive events: No common outcomes

Events that share some common outcomes or can occur simultaneously are said to be no-mutually exclusive
or overlapping events.

Figure 14.2 Overlapping events

Event A
Event B

Common outcomes
186

BUSINESS MATHS & STATISTICS (TC3)


Collectively exhaustive events are events from the same sample space whose probabilities add up to 1. If an
experiment is conducted, then at least one of these events must occur.

Sample space
A sample space is the collection of allpossible outcomes of an experiment. When a coin is tossed, the
sample space comprises a head and a tail (H, T). Sometimes the sample space is easy to determine, while at
other times it is complex. When elements of the sample space are counted, the result is the number of total
possibilities.

14.3 APPROACHES TO PROBABILITIES


Probabilities are generally of two types based on the approach followed to assigning probabilities. These
types are:
x Subjective
x Objective

14.3.1 Subjective Probabilities


This is the likelihood (probability) of an event occurring that is assigned based on whatever information is
available. The source of information can be expert opinion, intuition, or a pure guess. If there is little or no
information on which to base a probability, it may be arrived at subjectively. To a larger extent, subjective
probability describes a situation where one is simply expressing his or her strength of belief in the way
events will turn out.

Some examples of subjective probability


x An ardent supporter of a football team putting the probability of winning the next game high even
when the team has lost more games than it has won in the past.
x A candidate estimating the probability that she complete PAEC Technician programme within 18
months.
x A ‘new’ political party estimating the probability of winning the next presidential election.

14.3.2 Objective Probabilities


If a probability can be established scientifically, it is referred to as objective probability. Objective
probability is subdivided into classical and empirical probability.

Classical Probability
This is based on the assumption that the outcomes of an experiment are equally likely. Using the classical
approach, the probability of an event is computed by dividing the number of favourable outcomes by the
number of possible outcomes i.e.
Number of favourable outcomes
Probability of an event
Total number of possible outcomes
In classical probability approach we do not need to carry out an experiment rather we only need to know the
nature of the experiment. This is called a priori condition of the experiment. For instance the probability that
of obtaining head when a fair coin is tossed is 12 . We do not need to actually toss a coin to find this. All we
need is to know that a coin has two faces, head and tail, and are equally likely.

187

BUSINESS MATHS & STATISTICS (TC3)


Example 1
Consider an experiment of selecting an accountant from finance department to attend training in a new
accounting package. If there are 4 female and 10 male accountants, what is the probability that a female
accountant attends the training?

Solution:
We notice that any of the accountants has an equal chance of being selected for the training. Hence the
sample space has 14 individual or possibilities

Since we are interested in female accountant, there are 4 favourable outcomes in the collection of 14 equally
likely accountants. Therefore:
4 m Number of favourable outcomes
Probability of female accountant
14 m Total number of possible outcomes
= 0.26 (to 2 dec. Places)
Empirical Probability
Another way to determine probability is based on relative frequencies. In this approach, the probability of an
event occurring is determined by observing what fraction of the time similar events occur in the past i.e.
Number of times event occured in the past f
Probability of an event
Total number of observatio ns ¦f
The empirical approach to probability is based on what is called the law of large numbers which states over
large numbers of observations or experiments the empirical probability of an event will approach its true
probability.

Example 2
Business person notes that out of 10 orders he made to a supplier only 2 were delivered late. Find the
probability that an order he is making now will be late.

Solution:
The probability that the current order will be late is
Number of times event occured in the past o 2 1
Total number of observatio ns o 10 5

Notethat classical approach has an element of frequency being used. The difference is that under a classical
approach the denominator is fixed by the nature of the experiment, while under empirical approach the
denominator is not fixed but depends on empirical data from trials.

14.4 MATHEMATICAL DEFINITION OF PROBABILITY


Let A be an event, then the probability of the event A occurring, denoted P(A), is defined mathematically as:
n( A)
P( A)
n( S )
where n(A) is the number of outcomes in even A, and n(S) is the number of total possible outcomes.
188

BUSINESS MATHS & STATISTICS (TC3)


Example 3
Ziko Ltd is running a promotion involving customers. There are 13 tickets selected from the North, 15 from
the Center, 22 from the South and 15 from the Eastern region. These are placed in one drum for a final draw
and the draw will be a random process.
a) If there is one grand price, a car, what is the probability the car goes to the Center?
b) Suppose there are two grand prices, where the East and Southern regions compete as an entity while
the North and Center compete as another entity, find the probability that:
i) In East and south entity the car goes to the South and
ii) In the North and Central entity the car goes to the North

Solution
a) Let C be the event that the car is won by a customer from the Centre
n(C ) 15 15
? P(C ) 0.23
n( S ) 13  15  22  15 65
b)
i) Let T be the event that the car is won by a customer from the South in the East-South entity
Number of customrs from South
P (T )
Number of customers from East and South

22 22
? P (T ) 0.59
15  22 37

ii) Let N be the event that the car is won by a customer from the North in the North-Centre
entity
Number of customrs from North
P( N )
Number of customers from North and Centre

13 13
? P( N ) 0.46
13  15 28

14.5 BASIC RULES OF PROBABILITY


There are three basic rules probability
i) Probability limits:
The rule states that the probability of any event A lies between 0 and 1inclusive
0 d P ( A) d 1
where P ( A) 0 means event A cannot occur (i.e. an impossible event) and P( A) 1 means event A is
certain to occur.
ii) Total probability rule:
The rule states that the sum of the probabilities of all events/outcomes in a sample simple is 1
n

¦ P( A )
i 1
i 1

189

BUSINESS MATHS & STATISTICS (TC3)


iii) Complement rule of probability:
The rule states that if the probability of an event A occurring is P(A), the probability of the event not
occurring is:
P( A ) 1  P( A)
Note: P( A ) { P( Ac) { P( A c )

For example, if it is known that 43% of accountants in Malawi are female. We can therefore write;
P ( Female ) 0.43 , and P ( Male ) 1  P ( Female )  1  0.43 0.57

Example 4
The following table shows the number of visitors that exited Malawi in 2009.

Table 12.1 Visitors to Malawi - 2009


Port of exit Number of visitors
Chileka 76,308
Chiponde 52,836
Chitipa/Chisenga 6,816
Dedza 10,916
Kaporo/Songwe 90,350
Lilongwe (KIA) 174,652
Mchinji/Chimaliro 82,304
Muloza 41,203
Mwanza 141,956
Nayuchi 14,759
Nsanje/Marka 17,322
Other 45,609
Total 755,031
Source: 2009 Tourism Report - NSO

If we randomly select a visitor, what is the probability that the visitor


a) exited through Lilongwe (KIA)
b) did not exit through Chileka
c) exited either through Kaporo, Dedza or Mwanza?

Solution:
Let L = visitor exited through Lilongwe, C = visitor exited through Chileka, K = visitor exited through
Kaporo, D = visitor exited through Dedza, and M = visitor exited through Mwanza.
a) P (L) :
n( L ) 174652
P ( L) 0.23
n( S ) 755031
b) P (C ) :

190

BUSINESS MATHS & STATISTICS (TC3)


P (C ) 1  P (C )
n(C ) 76308
P(C ) 0.1
n( S ) 755031
? P(C ) 1  0.1 0.9
c) P ( K or D or M )
n( K )  n( D )  n( M ) 90350  10916  141956
P( K or D or M )
n( S ) 755031
243222
0.32
755031

Intersection of Events
An intersection of events A and B is the collection of all outcomes that belong to both events A and B. The
intersection of events A and b is denoted ( A ˆ B ) i.e. (A and B). We illustrate the intersection of event A
and B below.

Figure 14.3 Intersection of events

A
B

( A ˆ B)
Union of Events
A union of events A and B is the collection of all outcomes that belong to either event A alone or B alone or
to both A and B. The union is denoted ( A ‰ B ) i.e. (A or B). We illustrate the union of events A and B
below:
Figure 14.4 Union of events
A
B

( A ‰ B)

Independent Events
Events are independent if the occurrence of one event does not affect the occurrence of another. They may
or may not occur together. For example, person tosses a coin two times. The result on the first toss and of
the second are independent, and the height of a person and position held at work are also independent.

191
BUSINESS MATHS & STATISTICS (TC3)
Conditional events
In this case, one event is influenced, to a certain extent by a preceding event. For example, rain and clouds
in the sky are conditional. Furthermore, profit of a company will be affected or influenced by the level of
sales.

Classify the following events as mutually exclusive, independent or conditional,


a) The price of cooking oil rising in shops and floods in Karonga
b) Mr Phiri catching the Axa Coach from Lilongwe to Blantyre and Flying Air Malawi.
c) Being the chief accountant and the height of the person.
d) A consignment arriving late and the same consignment arriving on time from RSA.
e) Age of person and being chief executive.

Solution
a) Independent
b) Mutually exclusive
c) Independent
d) Mutually exclusive

Some Rule for Computing Probabilities


Now that we have defined probability and looked at most types of events, we turn our attention to
calculating the probabilities of two or more events by applying the rules of addition and multiplication.

14.5.1 Addition Rule of Probability (The OR rule)


The rule states that, given events A and B which are mutually exclusive, the probability of their union
( A ‰ B ) written as P ( A ‰ B ) is given by
P ( A ‰ B ) P ( A)  P ( B ) or P ( A or B ) P ( A)  P ( B )
This can be referred to as the special rule of addition.

If the events are not mutually exclusive, then the probability of their union (i.e. A or B) is given by:
P ( A or B ) P ( A)  P ( B )  P ( A ˆ B )

where P ( A ˆ B ) is the probability of the overlap/intersection or the chance that the two occur together.

Note that the addition rule has here been termed the “OR” rule because it is applied where the probability of
one event “OR” the other is being sought. The word ‘OR’ suggests that A may occur or B may occur. This
also includes the possibility that A and B may occur. The idea of “OR” can be explicit (stated) in the
question or it can be implied or deduced from the context.

Example 6
A card is drawn from a pack of 52 playing cards. What is the probability that the card is a 10 or a picture
card (Q, J, and K)

Solution
Let A = drawing a 10 and there are 4 10s in the pack and B = drawing a picture card and there are 12 picture
cards

192

BUSINESS MATHS & STATISTICS (TC3)


P(A) = 4/52 and P(B) = 12/52, A and B are mutually exclusive (we don’t have a picture card that is also a
10)
4 12 4
? P ( A ‰ B ) P ( A)  P ( B ) 
52 52 13

Example 7
A card is drawn from a pack of playing cards. Find the probability that it is a ten or a red card.

Solution
Let A = drawing a 10, and R = Red card. Note that there are two 10s which are red cards (diamond and red
heart). This is the overlap or events happening together.

? P ( A ‰ R ) P ( A)  P ( R )  P ( A ˆ R )
4 26 2 7
 
52 52 52 13

14.5.2 Multiplication Rule of Probability, The “AND” rule


This rule is used when a probability of two or more events occurring together is required. The condition is
that the events must be independent.

The rule states that:Given two events A and B which are independent, the probability of their intersection,
P ( A ˆ B ) is given by:
P ( A ˆ B ) P ( A) u P ( B )

Example 8
A die is rolled two times. Find the probability of obtaining two 6s.

Solution
Let A1 and A2 be the events of obtaining a 6 on the 1st and 2nd rolling respectively.
1 1 1
? P ( A1 ˆ A2 ) P ( A1 ) u P ( A2 ) u
6 6 36
Example 9
The Government has advised for consultancy work in the fields of finance and at the same time a new
company is also requesting for bids for a financial consultant. Mr. J.J. Kayange (FCCA) weighs his chances
of winning the government consultancy to be 0.25 and the new company one to be 0.3. Since the two
requests for bids are not related he decides to bid for both.What is the probability that he wins both?
Solution:
Let G = wining the government consultancy and C = winning the one from the private company
P ( A ˆ B ) P (G ) u P (C ) 0.25 u 0.3 0.075

14.6 CONDITIONAL PROBABILITY


A conditional probability is the probability of a particular event occurring, given that another event has
occurred.The probability of the event A given that the event B has occurred is written P ( A B ) and is given
by:

n( A ˆ B ) P( A ˆ B)
P( A | B)
n( B ) P( B)

193

BUSINESS MATHS & STATISTICS (TC3)


From this definition we can state the general rule of multiplication. Given two event, A and B, the joint
probability that both will happen is
P( A ˆ B) P( A | B) u P( B)
or
P ( A ˆ B ) P ( B | A) u P ( A)
Example 10
A card is drawn from a deck of 52 cards. Find the probability that the card is an Ace given it is a red heart.

Solution
Let A = Ace, and R = Red card.
The probability of an Ace given a red heart is given by dividing the number of Aces of red heart by the total
number of red hearts i.e.
n( A ˆ R ) 1
P( A | R)
n( R ) 13

Conditional probability can also be calculated from a contingency table. A contingency table is a two-way
table used to classify observations according to two or more identifiable characteristics.

Example 11
A sample of 1000 persons screened for a certain disease is distributed according to height and disease status
resulting from a clinical examination as follows:

Table14:3 Disease status


Disease Status
None Mild Moderate Severe Total
Tall 122 78 139 61 400
Height Medium 74 51 90 35 250
Short 104 71 121 54 350
Total 300 200 350 150 1000

A person is chosen at random, find the probability that the person is


a) short
b) in the severe status of the disease
c) short and of severe disease status
d) short given that he/she is of severe disease status

Solution
Let H be the event of being short and E that of severe disease status

n( H ) 350
a) P( H ) 0.35
n( S ) 1000
n( E ) 150
b) P( E ) 0.15
n( S ) 1000
n( H ˆ E ) 54
c) P( H ˆ E ) 0.054
n( S ) 1000

194

BUSINESS MATHS & STATISTICS (TC3)


n( H ˆ E ) 54
d) P( H | E ) 0.36
n( E ) 150
or
P( H ˆ E ) 0.054
P( H | E ) 0.36
P( E ) 0.15

14.7PROBABILITY TREES
A probability tree is a diagram used to show outcomes of an undertaking and their probabilities. The
diagram is in form of a tree and the braches represent the outcomes and attached probabilities

Example 12
A coin is tossed two times. Express the outcomes and their probabilities on a tree diagram.

Solution:
The tree diagram is shown below:

Figure 12.5 Probability tree


1st throw 2nd throw Events Join probabilities

HH 0.5x0.5=0.25
H
0.5
T
H 0.5 0.5x0.5=0.25
0.5 HT

TH 0.5x0.5=0.25
T H
0.5 0.5
T
0.5 HH 0.5x0.5=0.25

Note the first throw has two possible outcomes: H or T. Each one has its probability indicated.Given a head
on the first throw, there are two further outcomes possible (H or T). Because of the independence of results
after tossing a coin, the probabilities are still 0.5 on either of the outcomes. The same is true when the
outcome is T on the first toss.

The tree diagram also shows the overall set of possibilities of the undertaking. These are

a) Head on the first toss and Head on the 2nd : HH, P(HH) = 0.5 x 0.5 = 025

b) Head on first toss and Tail on second toss: HT, P(HT)=0.25

c) Tail on the first toss and Head on the second: TH, P(TH) =0.25

d) Tail on the first toss and Tail on the second: TT, P(TT)= 0.25

Note that the joint probabilities add up to 1

195

BUSINESS MATHS & STATISTICS (TC3)


Example 13

Maruta travels to South Africa frequently to buy a special chemical used in higher grade chicken feed.
Maruta knows that the price of the chemical at his sources can be high if demand is high and low if the
demand is equally low. From his past experience he is certain that there is a 0.6 chance that the demand for
the chemical is high in RSA. When Maruta sells the chemical at home, he can either make a profit or loss
depending on the combination of price he pays in RSA and the demand for chickens. Again using his
experience he has calculated that there is a 0.3 probability of making a profit if the cost price in RSA was
high and can make a profit with a probability of 0.8 give he bought the chemical at a low price.

Calculate the probability of making a loss

Solution:

We outline the outcomes and their probabilities on a tree diagram.

Let H = high price and L = low price in RSA. Let Pr = the event of making a profit at home, and Lo =
making a loss.

P(H) = 0.6, Then P(L) = 0.4 (complementary rule)

The conditional probability P(Pr|H) = 0.3 and P(Lo/H) = 0.7

The conditional Probability P(Pr|L) = 0.8 and P(Lo/L) = 0.2

Figure 12.6 Probability tree

RSA: Cost Home: Profit Events Join probabilities

H and Pr 0.6x0.3=0.18
Pr
0.3
Lo
H 0.7 H and Lo 0.6x0.7=0.4
0.6
L and Pr 0.4x0.8=0.32
L Pr
0.4 0.8
Lo
0.2 L and Lo 0.4x0.2=0.0

The probability of making a loss is given by the joint events (H and Lo)or (L and Lo)

P((H and Lo) or (L and Lo)) = P(H and Lo) + P(L and Lo) Mutually exclusive
= 0.42 + 0.08 = 0.5

196

BUSINESS MATHS & STATISTICS (TC3)


14.8EXPECTED VALUE
The expected value, sometimes called the mathematical expectation, of a set of values which have
associated probabilities of occurrence is calculated as:
E ( X ) ¦ xP ( x )
where P (x ) is the probability of a value x. Expected value is therefore the sum of the products of data
values and their respective probabilities.

Example 14

During holidays a mathematics student has been fishing in a river close to his village and putting his
mathematics to practice by weighing his catch of fish and calculating some probabilities of specific
quantities (in weight) he can catch. After a month of in fishing he reckons that his catch ranges from 2 Kg to
8 Kg with certain pattern of probabilities which he tabulates as follows:

Table 12.4 Fish weights


Weight of fish (kg) Probability
2 0.075
3 0.15
4 0.2
5 0.25
6 0.175
7 0.125
8 0.025
Total 1

Find the expected weight of the catch.

Solution
Let the weight of the catch be represented by X then,
E ( X ) ¦ xP ( x )
=(2 x 0.075)+(3 x 0.15)+(4 x 0.2)+(5 x 0.25)+(6 x 0.175)+(7 x 0.127)+(8 x 0.025) = 4.775 kg

Note that an expected value is actually a weighted average

Example 15
Business man calculates that it is possible to make profits of K2,000,000 if the market conditions are
perfect, K800,000 if the market conditions are moderate and a loss of K500,000 if the conditions are poor.
The probabilities of perfect, moderate and poor conditions are 0.3, 0.5 and 0.2 respectively. Calculate his
expected profit.

Solution
Let X = Profit
? E( X ) ¦ xP( x)
0.3 u 2000000  0.5 u 800000  0.2 u ( 500000 ) 900000

His expected profit is K900,000.00


197

BUSINESS MATHS & STATISTICS (TC3)


CHAPTER SUMMARY

In this chapter we introduced probability asa measure of chance and discussed a number of issues.
Specifically we wish to highlight the following:
x The probability values lie between zero and one, i.e. 0 d P ( A) d 1 where A is an event.
x When they are several possible outcomes of an event and it is impossible for more than one outcome
to occur at any one time, then the outcomes are said to be mutually exclusive.
x When two or more events take place, these events are said to be independent when the outcome of
one event does not affect the outcome of the other event/s.
x The multiplication rule of probability is applied when all events under consideration must occur. The
addition rule of probability is applied when we have an either/or situation.
x A Venn diagram can be used to organize and find probabilities for none-sequential events when the
categories or situations overlap. Venn diagrams are usually helpful when number elements are
provided rather than probabilities.
x A tree diagram can be used to organise and calculate probabilities when there are sequential events
and alternative, non-overlapping situations, each with a different outcome. The situations and
outcomes must all have probabilities.

END OF CHAPTER EXERSCISES

1. John goes to College each morning by bus and he catches it at 07:10 each morning. Much as the bus
sticks to time but of late John has been left stranded at the bus stop on three days in the past 12 days
because the bus had been coming full. Tomorrow morning he will attempt to catch the same bus.
Calculate John’s probability of catching the bus.

2. A committee of three is supposed to be selected from a group of 5 people (two ladies and three
gentlemen.
a) Find the probability that the committee will comprise ladies only

b) What is the probability that the committee will be made up of at least 1 lady.
3. Two dice are thrown. What is the probability that
a) the sum of the scores is greater than 10,
b) the sum of the scores is greater than 10 given that the first die is 6

4. Over the last few years, an internet company has recorded the number of ‘hits’ to its website at
600,000 and the number of customers who made follow-up enquiries at 40,000, a quarter of whom
actually made a purchase. Calculate the probability of a ‘hit’ leading to:

a) An enquiry
b) A purchase

5. Students at a certain school were surveyed to find out the mode of transport they used when going o
school. The results were:

198

BUSINESS MATHS & STATISTICS (TC3)


Modeoftransport Numberofstudents
Walking 9
Bicycle 10
Car 6
Bus 15
40

A student is picked at random,


a) What is the probability that the student comes to school by bus?

b) What is the probability that the student comes to school by car or bus?

6. It is estimated that the probability that an energy saver bulb will last more than 2 is 0.8. If ESCOM
installs 500,000 bulbs in one of the towns, estimate the expected number of bulbs that will last more
than 2 years.

7. 100 students sat for a particular examination of which 60 were boys. The number of students who
passed this examination was 40, of whom 20 were girls. Find the probability of:
a) A student passing the examination
b) A girl passing the examination
c) A selected student who is a boy, failing the examination

8. Goliati Ltd must decide which of two alternative strategies to adopt. The company has asked you to
predict the probabilities of different profit and loss levels resulting from the two strategies. The
results are shown below:
Probability
Profit/Loss Strategy A Strategy B
£1.25 million profit 0.5 0.7
£250,000 profit 0.4 0
£250,000 loss 0.1 0.3

a) Calculate the expected value of profit for each strategy.


b) Advise the company on the better strategy to follow.

9. Georgina goes to the library. The probability that she checks out (a) a work of fiction is 0.40, (b) a
work of non-fiction is 0.30, and (c) both fiction and non-fiction is 0.20. What is the probability that
the student checks out a work of fiction, non-fiction, or both?
10. In a library box, there are 8 Accounting, 8 Mathematics, and 8 Communication books. If Chisomo
selects two books at random, what is the probability of selecting two different kinds of books in a
row?
11. What is the probability of drawing an ace from a standard deck of cards, given that the card is a
diamond

12. A survey of Chimasula Primary School senior (Standard 6, 7 and 8) students asked the
question: What is your favorite sport? The results are summarized below:

199

BUSINESS MATHS & STATISTICS (TC3)


Class Favorite Sport
Football Netball Volley ball TOTAL
6 68 41 46 155
7 84 56 70 210
8 59 74 47 180
TOTAL 211 171 163 545

Using these 545 students as the sample space, a student from this study is randomly selected.

a) What is the probability of selecting a student whose favorite sport is volley ball?
b) What is the probability of selecting a Standard 6 student?
c) What is the probability of selecting a student who likes netball and is in standard 8?
d) What is the probability of selecting a student who likes volley ball or a standard 7 student?
e) If the student selected is in 7, what is the probability that the student prefers Netball?
f) If the student selected prefers Football, what is the probability that the student is in standard
6?
g) If the student selected is in standard 8, what is the probability that the student prefers
Football?

200

BUSINESS MATHS & STATISTICS (TC3)


CHAPTER 15 REGRESSION AND CORRELATION ANALYSIS

LEARNING OBJECTIVES
At the end of this chapter, students should be able to:

i. explain the meaning of regression analysis


ii. identify practical examples where regression analysis can be used
iii. plot scatter diagrams
iv. construct a simple linear regression model
v. prepare estimates of the unknown variable using the regression model
vi. compute and interpret the correlation coefficient
vii. compute and interpret the coefficient of determination
viii. create an understanding of linear regression

15.0 INTRODUCTION
In practice many entities which are measured appear to be related to or they influence each other. Easy
examples of such situations are given below:

x Advertising expenditure is assumed to have an influence on sales volumes.


x Share price is influenced mainly by a company’s return on investment.
x Hours of operator training is likely to impact positively on productivity.
x Operating speed of a bottling machine affects the reject rate of under-filled bottles.

Because the entities change in terms of quantum, they are referred to as variables. Relationships between
variables need to be investigated in terms of existence, nature and strength of the relationship (association).
Asituation broadly known as correlation. One reason for such analyses is that in many business decisions, it
is necessary to predict the unknown values of a numeric variable using other numeric variables for which
values are known.

Techniques for establishing and measuring the strength of the relationship include use of a scatter diagram
and calculation of what is termed a correlation coefficient. Prediction is carried out by using among other
techniques, regression analysis. The validity and dependability of predictions are dependent on the strength
of correlation.

Regression and correlation analysis require that the data type for all variables e.g. marketing, economic,
financial, production, human resources, etc.) must be numeric.

This chapter looks and regression analysis first then considers correlation analysis. Specifically it looks at
the following topics:
x Simple linear regression analysis, which examines the relationship between two numeric variables
only;
x Correlation analysis, which computes the strength of a relationship.

Note: It does not consider, multiple linear regression analysis, where numerous numeric measures are used
to influence the outcome of a single numeric measure.
201

BUSINESS MATHS & STATISTICS (TC3)


15.1 REGRESSION ANALYSIS

A scatter plot of pairs of data between two numeric random variables, x and y , visually displays the
relationship between the two variables, as illustrated graphically in Figure 9.1

A scatter plot between pairs of x and y data values

Figure 15.1 Advertising expenditure v Sales volume

SalesVol AdsExp Thescatterdiagram


x'000) yK'000)
2 10 30
3 14
25
5 14
ExponAds(K'000)

7 16 20
10 23
12 21 15
13 25
10

0
0 2 4 6 8 10 12 14
SalesVolume('000)

Correlationanalysis measures the strength of this identified association between the variables.

If a structural (mathematical) relationship exists between two numeric random variables – and can be
measured and quantified - then knowing the values of one of the variables, x , can be used to predict (or
estimate) the outcome of the other variable, y , for which values are generally unknown. This is the primary
purpose of regression and correlation analysis. These techniques can provide managers with a powerful tool
for prediction purposes.
Regression analysisis a statistical process for estimating the mathematical relationships among numeric
random variables as a mathematical equation (usually a straight-line equation). The relat

In Figure 15.1, the straight-line equation would be fitted using regression analysis. The degree of closeness
of the plots to the straight line is measured by correlation analysis. The straight equation can be used to
estimate the y  values based on known x  values. Correlation analysis provided a measure of the
confidence a manager can have in the estimated y  values.

15.2 SIMPLE LINEAR REGRESSION ANALYSIS

Simple linear regression analysis finds a straight-line equation between the values of two numeric random
variables only. The one variable is called the independent or predictor variable, x , and the other is termed
the dependent or response variable, y .

202

BUSINESS MATHS & STATISTICS (TC3)


15.2.1 Independent variableሺ࢞ሻ
The independent variable is represented by the symbol x . It is the variable influencing the outcome of the
other variable. For this reason it is also called the predictor variable. Its values are usually known or easily
determined. In certain instances, the independent variable’s values can be controlled or manipulated. In the
examples in section 9.1 above, the independent variables would be: advertising expenditure; company return
on investment, hours of operator training; and bottling machine speed.

15.2.2 Dependent variable ሺ࢟ሻ


The other numeric random variable is called the dependent variable and is represented by the symbol y .
The dependent variable is influenced by (or responds to) the independent variable. Hence it is also called the
response variable. Values for the dependent variable are not readily known and need to be estimated from
values of the independent variable x . In the examples in section 9.1 above, the dependent variable are:
sales volumes; share price; productivity; and reject rate.

In simple linear regression, only one independent variable, x , is used to estimate or predict values of the
dependent variable, y unlike in multiple regression where two or more independent variables are used to
estimate the value of the dependent variable.

To build a simple linear regression mode, a number of steps are followed, as illustrated below.

Identify the Dependent and Independent Variables


An essential prerequisite is to correctly identify the independent and dependent variables. This is necessary
to ensure that a valid relationship is established. A useful rule of thumb is to ask the following question:
“Which variable is to be estimated?”

The answer to this question will identify the dependent variable, y . Thus the logic of relationship must be
checked before proceeding with regression analysis.

Use a Scatter Plot to Graphically Examine the Relationship Between the Dependent and Independent
Variables
Consider the following data on volume of sales and advertising expenditure.

The first step towards identifying a possible relationship between two numeric random variables is to
prepare a visual plot of their data values. This is done through a scatter plot or scatter diagram.

For the data on advertising expenditure and sales volume the scatter plot is shown along side the data in
figure 15.1. .

A scatter plot (or scatter graph) graphically displays all pairs of data values of the independent and
dependent variables on an x  y axis. The x values are recorded along the horizontal axis and the y values
along the vertical axis, as was shown in Figure 15.1.

A visual inspection of the scatter plot will show whether there is a relationship between the two variables, x
and y , and how strong it is likely to be. The scatter in Figure 15.1 show an upward pattern. As x is rising so
is y. There is likely to be a strong relationship between the two variables. These initial insights are likely to
be reflected in the regression and correlation analysis findings.

BUSINESS MATHS & STATISTICS (TC3)


If for example, the data points are widely scattered, then a linear regression equation will be of little value in
estimating the y -variable. The correlation measure will also show almost no association.

Figures 15.2 and 15.3 show various possible patterns of relationships between a dependent numeric
variable, y , and an independent numeric random variable, x .

Figure 15.2 Figure 15.3


35 20

Y
Y

x y xy x2 y2 18
30 x y xy x2 y2
1 1 1 16 1 18 18
25 2 24 14 2 19 19
3 7 12 3 15 15
20
4 26 10 4 16 16
15 5 1 8 5 10 10
6 12 6 6 10 10
10
7 30 7 7 7
4
5 8 19 8 3 3
2
9 2 9 3 3
0 0
0 2 4 6 8 10 0 2 4 6 8 10
X X

No linear relationship exists if values of x and y are randomly scattered (i.e. for any given x value, y can
have any value over a wide range). This is the case in Figure 15.2. On the other hand, we observe Inverse
linear relationship with small dispersion (i.e. for any given x value, the range in y values is small) in Figure
15.3.

From a manager’s perspective, the pattern shown in Figures 15.1 and 15.3 are the most desirable as they
show strong linear relationships between x and y . Estimate of y based on these relationships will be highly
reliable. However, the pattern shown in Figure 15.2 is evidence of no statistical relationship between the
two numeric measures. In such cases, there is no value in using regression analysis to estimate y based on
x values. The estimates will be unreliable.

15.2.3 Calculating the Linear Regression Equation

Regression analysis finds the equation of the best-fitting straight line to represent the actual data points.

Formula
A straight line graph is defined as follows


ab x
Where x = values of the independent variable
ŷ = estimated values of the dependent variable
a = the y intercept coefficient (where the regression line cuts the y axis)
b = the slope (gradient) coefficient of the regression line
(i.e. for every one unit change in x, y will change by b )

204

BUSINESS MATHS & STATISTICS (TC3)


15.2.4 Techniques for estimating the regression equation

We will consider three techniques for estimating the regression equation. These are:

a) Scatter graph and line of best fit


b) The high low method
c) The least squares method

Example 1

XYZ ltd manufactures rubber shoe soles often bought by retailers and who then resale them to shoe
repairers in town. XYZ can make any order size in thousands depending on the customers’ needs.

The managing director wants a simple model he can use to predict the cost of any order made by a customer
before that order is worked on. For this he has extracted records on the last 10 orders made and delivered.
The data is as follows:

Table 15.1 Order sizes and cost


Order number 1 2 3 4 5 6 7 8 9 10
Order size 5 10 50 15 2 30 60 30 3 50
'000 (x)
Cost '000 (y) 190 240 350 250 300 310 395 335 300 300

Estimate the regression line using

a) Scatter graph and line of best fit


b) The high low method
c) The least squares method

Solution

a) Using the scatter diagram.

The scatter diagram appears as follows:

205

BUSINESS MATHS & STATISTICS (TC3)


Figure 15.4

The regression equation is

y = a + bx (in general)

Where a = y intercept i.e. value of y when x = 0


= 245 from the line of best fit

b = slope and can be determined by picking any two points on the graph

Let the points be (x1, y1) and (x2, y2)

y 2  y1 change in y
b i.e.
x 2  x1 change in x

Using the graph let (x1, y1) = (0, 245) and (x2, y2) = (50, 345)
345  245
b 2
50  0

The regression equation is y = 245 + 2x

b) Using the high low method:

h hi h dl d fi d h i d d i bl
BUSINESS MATHS & STATISTICS (TC3)
x y
High 60 395
Low 5 190
Change 55 205

b=Slope=changeiny
changeinx

Thereforeb=205/5= 3.73

The constant “a” can be determined by substituting values of one of the coordinates and b into the general
linear equation and solving for it.

Generally y = a + bx

Using the “low” coordinate (5, 190) we have

190 = a + 3.73 x 5

A = 190 - 18.65
= 171.35

The regression equation is y = 171.35 + 3.73x

c) The method of least squares (MLS)

Regression analysis uses the method of least squares to find the best-fitting straight-line equation to the
plotted data points. The method of least squares is a mathematical technique which finds values for the
coefficients, a and b , such that:

“the sum of the squared deviations of the data points from the fitted line is minimized.”

A brief explanation of the rationale is that it considers the vertical deviations between the actual values y i
and the estimated values ŷ i .
i. A deviation (error) (written as e i ), which is a measure of the vertical distance from an actual y -
value to the fitted line, is first computed for each yi -value.

ei y i  yˆ i
ii. Each deviation is now squared to avoid positive and negative deviations canceling each other out
when summed.

e 2 i y i  yˆ i
2

iii. A measure of total squared deviations is then found by summing the individual squared deviations.
¦ e 2 i ¦ yi  yˆ i
2

207

BUSINESS MATHS & STATISTICS (TC3)


iv. Values for a and b are now found, which will minimize the sum of these squared deviations in (iii).
The mathematical calculation that will minimize the sum of these squared deviations is called the
method of least squares.

Without showing the mathematical calculations, the coefficients a and b that result from the method of
least squares are given as follows.

n¦ xy  ¦ x¦ y
b
n¦ x 2  ¦ x
2

a
¦y b ¦x or a y bx
n

The values of a and b that are found from the above formulae define the best-fitting linear regression line.
This means that no other straight-line equation can be found that will give a better fit (i.e. a smaller sum of
squared deviations) than the regression line.

In this example,

Table 15.2 Calculation table (order sizes and cost)


Order Order size '000 Cost MK'000 xy x
2

number (x) (y)

1 5 190 950 25
2 10 240 2400 100
3 50 350 17500 2500
4 15 250 3750 225
5 2 300 600 4
6 30 310 9300 900
7 60 395 23700 3600
8 30 335 10050 900
9 3 300 900 9
10 50 300 15000 2500
255 2970 84150 10763

n = 10 (pairs of observations)

n¦ xy  ¦ x¦ y
Now b
n¦ x 2  ¦ x
2

10 u 84150  255 u 2970


= 1.98
10 u 10763  255 2

208

BUSINESS MATHS & STATISTICS (TC3)


a y bx

2970 255
 1.98 u = 246.63
10 10

The regression equation is y = 246.63 + 1.98x

15.2.5 Estimating y  values using the Regression Equation


The regression equation can now be used to estimate values of y from (known) x -values. Estimates of y
are found by substituting a given x  value into the regression equation. The values of x that can be
substituted in the regression equation should lie only within the domain of the x variable. The domain of the
x variable is defined as the range of x  values from the data set that were used to calculate the regression
line.

15.2.6 The dangers of extrapolation

Extrapolation occurs when y  values are estimated using x  values that lie outside the domain of the x 
values. Valid estimates of y are produced only from x  values that lie within its domain. If the values of
y are estimated for x  values outside the limits of the domain (i.e. extrapolation has taken place), the
estimates may be invalid, as the relationship between x and y beyond these limits is unknown (or has not
been defined). The relationship may in fact be quite different from that which is defined between x and y
within the x -domain. Extrapolation can sometimes lead to absurd and meaningless estimates of y . The
following example illustrates the above process of building a simple linear regression model.

Example 2

The least squares regression equation of cost against order size in the example above is

y = 246.63 + 1.98x where x = order size, y = Cost

Estimate the costs of order sizes of

i) 60,000
i) 55,000

Solution

Note that the question is about predicting the cost of the stated order sizes. The answer is found substituting
the value of x the independent value (order size) into the regression equation.

i) x = 60, y = 246.63 + 1.98 x 60


= 364.43 or K364,430

A predicted value will usually be different from an actual observation because the former is a result
of the formula.

ii x = 55 y = 246.63 + 1.98 x 55

209

BUSINESS MATHS & STATISTICS (TC3)


= 355.53 or K355,530

15.2.7 Validity and reliability of a prediction


a. Validity

The equation and therefore any predicted variables are valid only within the range of the original
observations

The reason is that a regression equation is calculated using a set of observations which have a range. The
mathematical relationship may not hold true outside the range of the data used. Another way of looking at
the same thing is a linear regression is described as such because the relationship between the variables is
linear. This is true for the data at hand. The relationship may not be linear outside the range of the given
data.

Reliability and accuracy of a prediction.

Dependability and accuracy of a prediction is defined by the extent of correlation between the two variables.
The higher the correlation the more reliable and therefore accurate the predicted value is.

Thus a higher correlation coefficient (say r = 0.9) is indicative of good relationship and therefore the fact
that a prediction is on a sound basis.

Reliability and accuracy are also strengthened by the coefficient of determination. The coefficient of
determination indicates the percentage of the dependent variable which is explained by the other (the
independent variable). In other words, it shows the degree to which the variables vary together.

The higher this proportion the better the accuracy in prediction.

15.3 CORRELATION ANALYSIS

The reliability of the estimate of y is determined by the strength of the relationship between the x and the
y variables. A strong relationship will result in a more accurate and reliable estimate of y .
Correlation analysis measures the strength of the linear association between dependent and independent x
and y .

Examples

a) Revenue and profit levels are related. The higher the revenues, the higher the profits will be.
b) height and weight of people are related; taller people tend to be heavier than shorter people.
c) The time it takes to cover a distance by car and the speed of the car.

15.3.1 Techniques for determining correlation


There are several techniques and measures to determine correlation. The most common ones include

a) Inspection of a scatter diagram


b) The Pearson’s product moment correlation coefficient
c) The spearman’s rank correlation coefficient

We will look at each technique on the following sections.


210

BUSINESS MATHS & STATISTICS (TC3)


A. Scatter diagram technique

This involves plotting a scatter graph. The pattern in the points of the scatter graph would reveal a pattern in
the data that may suggest the degree of the relationship between the variables.

Example 3

The manager of Tikwere Ltd has been wondering whether or not there is a relationship between turnover
and profit before tax. To confirm this he has gone to past records and extracted turnover and profit figures as
follows but he does not know how to use them to show the correlation:

Table 15.3 Turn over and profit before tax


Year 2005 2006 2007 2008 2009 2010

Turn over (millions) 106 125 147 167 187 220

Profit before tax( millions) 10 12 16 17 18 22

Draw a scatter diagram and advise the manager on what it reveals

Solution

a) Figure 15.5 The scatter diagram (turnover and profit before tax)

25
Year 1977 1978 1979 1980 1981 1982
20
Turn over 106 125 147 167 187 220
(millions)
Profitaftertax

Profit 15 10 12 16 17 18 22
before
tax(
10
millionS)

0
0 50 100 150 200 250

Turnover
The scatter of point shows a pattern (upwards). This indicates that there is correlation between the
two variables

B. The Pearson’s Product Moment Correlation Coefficient (r)

The Pearson’s Product moment Correlation coefficient indicates whether or not there is correlation but it
also gives the strength of that relationship.
211

BUSINESS MATHS & STATISTICS (TC3)


If the correlation coefficient is constructed using the full population data of x and y , it is represented by
the symbol U (rho). If only sample data was used to compute the correlation coefficient, then the sample
correlation coefficient is represented by the symbol r . In practice, only a sample correlation coefficient is
usually computed, hence the term r is commonly used.

Interpretation of correlation coefficient


A correlation coefficient is a proportion that takes on values between  1 and  1 only.

 1 d r d 1

a) A value of r = 1 means perfect correlation


b) 0 < r < 1 means positive partial (as opposed to perfect) correlation
c) r = 0 stands for no correlation
d) -1 < r < 0 means negative partial correlation
e) When r = -1, its perfect but negative correlation

Any interpretation should take the following two points into account:

i. A low correlation does not necessarily imply that the variables are unrelated, but simply that the
relationship is poorly described by a straight line. A non-linear relationship may well exist. The
correlation coefficients that we consider here do not measure non-linear relationships.
ii. A correlation does not imply a cause and effect relationship. It is merely an observed statistical
association.

C. Computation of Pearson’s Correlation Coefficient

Pearson’s coefficient represents the correlation between two numerical random variables only and is
computed as follows:

Formula

n¦ xy  ¦ x ¦ y
r
>n¦ x  ¦ x @u >n¦ y  ¦ y @
2 2 2 2

Where r = the sample Pearson’s correlation coefficient


x = the values of the independent variable
y = the values of the dependent variable
n = the number of paired data points in the sample

Pearson’s correlation coefficient formula is derived from the least squares regression approach, hence its
formula has similar terms to the regression coefficients.

Example 4

Consider the turnover and profit figures for Tikwere Ltd. Calculate the product moment correlation
Coefficient and interpret the result.

212

BUSINESS MATHS & STATISTICS (TC3)


Solution

Table 15.4 Turn over and proft (calculation table)


Year T/O Profit
2 2
x y xy x y
2005 106 10 1 060 11 236 100
2006 125 12 1 500 15 625 144
2007 147 16 2 352 21 609 256
2008 167 17 2 839 27 889 289
2009 187 18 3 366 34 969 324
2010 220 22 4 840 48 400 484
Totals 952 95 15 957 159 728 1 597

n¦ xy  ¦ x ¦ y
r
>n¦ x  ¦ x @u >n¦ y  ¦ y @
2 2 2 2

Now n 10 , ¦x 952 , ¦ xy 15957 , ¦x 2


159728 , ¦y 2
1597 , ¦y 95

10 u 15957  952 u 95
So r 0.98
>10 u 159728  952 @u >10 u 1597  95 @
2 2

A correlation coefficient of r 0.98 indicates a very strong correlation between the variables.

Note: Due to the relationship among scatter plots, linear regression and correlation, it is normal to consider
them together as the following example illustrates.

Example 5

In the following set of data, y represents the number of annual claims for flood damage received by an
insurance company (in thousands) and x represents the annual rainfall (in centimeters) over a period of 10
years.

Table 15.5 Flood damage claims and rainfall


y (000s) x (cm)
2.0 210
0.5 150
0.2 120
4.0 450
4.0 400
2.5 200
2.0 210
4.0 430
0.1 190
4.0 400

(a) Plot the data on a scatter diagram and comment on the likely relationship between x and y.

213

BUSINESS MATHS & STATISTICS (TC3)


(b) Find the equation of the least-squares regression line, assuming that insurance claims for flood
damage depend on the amount of rainfall.
(c) Calculate the correlation coefficient and comment on the result.
(d) Use your results to predict the number of flood damage claims in years with 50cm of rainfall and in
years with 250cm of rainfall. Comment on the likely validity and accuracy of your predictions.

Solution

a) Figure 15.6 Scatter diagram

Much as points appear in clusters on the scatter diagram, there is a general upward trend which
shows correlation. There is a pattern which indicates correlation.

Table 15.6 Workings:


2 2
x(cm) y(K'm) xy x y
210 2 420 44100 4
150 0.5 75 22500 0.25
120 0.2 24 14400 0.04
450 4 1800 202500 16
400 4 1600 160000 16
200 2.5 500 40000 6.25
210 2 420 44100 4
430 4 1720 184900 16
190 0.1 19 36100 0.01
400 4 1600 160000 16
Totals 2760 23.3 8178 908600 78.55

n = 10

214

BUSINESS MATHS & STATISTICS (TC3)


n¦ xy  ¦ x¦ y
Now b
n¦ x 2  ¦ x
2

10 u 8178  2760 u 23.3


0.119
10 u 908600  2760 2

and a
¦ y b¦ y = 23 .3
 0.119 u
2760
= - 0.95
n n 10 10

The regression equation: y 0.95  0.119 x

c) The correlation coefficient

n¦ xy  ¦ x ¦ y
r
>n¦ x  ¦ x @u >n¦ y  ¦ y @
2 2 2 2

10 u 8178  2760 u 23.3


r = 0.926
>10 u 908600  2760 @u >10 u 78.5  23.3 @
2 2

The coefficient of determination r2 = (-0.926)2 = 0.85

Both the coefficients are high. There is strong correlation between rainfall and claims while 85% of
claims are explained by the rainfall.

d)
When x = 50 When x = 250
y = -0.95 + 0.119 x 50 y = -0.95 + 0.119 x 250
y = 5 y = 28.8
This prediction may be accurate because of The predition is accurate and valid.
high r but not valid because 50 is outside There is high coefficient of determination
the range of observed data and 250 is in the range of the
observations.

215

BUSINESS MATHS & STATISTICS (TC3)


A graphical presentation of the interpretation of values of the Correlation Coefficient

Figure 15.7 Scatter diagram and associated values of correlation coefficient


 25 X Y 25 X Y
Y

Y
20 1 8 1 8
20
2 11 2 15
15 15
3 14 3 9
10 4 17 10 4 17
5 5 20 5 5 20
0 6 23 6 18
0
0 2 4 6 8 0 2 4 6 8
X X
r = + 1 perfect positive correlation 1• r • 0Partialpositivecorrelation
Scatter of points fall on a straight Corelation is partial since points do not
line rising together. fall on a straight line. Positive because
as one vaiable rises other rises as well
Y

X Y X Y

Y
30 30
25 1 8 25 1 26
20
2 24 20 2 20
3 12 15
3 20
15
4 5 4 21
10 10
5 27 5 8
5 5
6 17 6 11
0 0
7 2 7 6
0 2 4 6 8 10 0 2 4 6 8 10
8 23 X 8 7 X
r=0 Scatter of points does not show any 0шrшͲ1 Negative, partial correlation
pattern downward sloping
X Y
Y

30
25 1 26
20 2 23
15
3 20
10
4 17
5
5 14
0
6 11
0 7 2 84 6 8 10
X
8 5
R=Ͳ1 Negative, perfect correlation
Points fall on a staight line but variables
vary in oposite directions.

Measure of the strength of correlation

Perfect correlation (indicated by values of r = 1 or -1) is the strongest level a relationship


(correlation) between variables can be.

The other extreme is where there is no correlation indicated by r = 0

Ranges of r between 0 and 1 indicate positive correlation while ranges of r between -1 and 0 indicate
negative correlation. The closer r is to 1 or -1, the stronger the measure of correlation.

216

BUSINESS MATHS & STATISTICS (TC3)


Generally r = 0.5, for positive correlation and -0.5 for negative correlation are taken to be cut off
points.

The following diagram illustrates the measures:

Figure 15.8 Strengths of correlation

Nocorr.
Strongcorr. Weakcorr. Weakcorr. StrongCorr.

1 0.5 0 0.5 Ͳ1

Example 6

Describe the strengths of the following coefficients of correlation calculated on various sets of data.

a) r = - 0.98
b) r = 0.8
c) r = 0.4
d) r = - 0.6
e) r = - 0.2

Solution

a) r = - 0.98 correlation is very strong


b) r = 0.8 correlation is strong
c) r = 0.4 this is weak correlation
d) r = - 0.6 fairly strong correlation
e) r = - 0.2 weak correlation

15.3.2 Coefficient of Determination


When the sample correlation coefficient, r , is squared r 2 , the resultant statistical measure is called the
coefficient of determination, denoted R 2 .

The coefficient of determination, r 2 , defined as the proportion (or percentage) of variation in the
dependent variable, y , that is explained by the independent variable, x .

The coefficient of determination ranges between 0 and 1 (or 0% and 100% )

i.e. 0 d r2 d1

To interpret the coefficient of determination

The proportion (or percentage) of variation y that x can explain is a measure of how strongly x and y are
associated. If x can explain a high proportion (or percentage) of the variation in y , then x and y are
strongly associated and vice versa.

217

BUSINESS MATHS & STATISTICS (TC3)


When r 2 0 no variation in y can be explained by the x variable.
This corresponds to the scatter plot in which r = 0, where x is of no value in estimating y . There is no
association between x and y .

When r 2 1 the values of y are completely explained by the x  values. There is perfect association
between x and y . This is where where x  values exactly estimate the y  values.

When 0  r  1

x Values of r 2 that lie closer to zero (or 0% ) indicate a low percentage of variation in y explained by
the x variable. This represents a weak association between x and y .
x Alternatively, values of r 2 that lie closer to 1 (or 100% ) show that the x variable is of real value in
estimating the actual values of the y variable. This represents a strong association between x and y
.
Example 7

Consider the following situations

Kgs of sugar Cost


purchased (MK)

Table 15.7 a) Sugar purchases b) Time and fish catch


Situation A: Situation B:
Sugar Purchased Cost Time fishing Catch
Kgs MK (Hrs) Kg
1 200 0.5 3
2 400 1 7
3 600 2 8
4 800 3 5
5 1000 4 15
6 1200 5 14

For each situation:


a) plot the scatter diagrams
b) calculate the coefficients of determination using the correlation coefficients shown and interpret the
coefficients of determination.

218

BUSINESS MATHS & STATISTICS (TC3)


Solution

a) Figure 15.9 Scatter diagrams:

SituationA: SituationB:

1400 SugarPurchased Cost Timefishing


16 Catch
Kgs MK (Hrs) Kg
1200 1 200 14 0.5 3
2 400 12
1 7
1000 3 600 2 8
4 800 10 3 5
CostofSugar

CatchinKgs
800
5 1000 4 15
8
6 1200 5 14
600
6
Correlationcoefficient=
400 1 Correlationcoefficient= 0.83
4
200 2

0 0
0 2 4 6 8 0 2 4 6
Kgsofsugar Hrs

b) Coefficients of determination:

Situation A:

r = 1:
It is perfect correlation. The scatter-graph shows perfect goodness of fit in that all point lie on the
straight line

The coefficient of determination r2 = 12 = 1


All increases in the cost are accounted for or explained by increases in the quantity of sugar.

Situation B
r = 0.83:
Correlation is partial and the scatter diagram shows an imperfect fit

r2 = 0.63 meaning only 63 % of variations in fish caught are explained by variation in time spent
fishing.

The scatter diagram also shows that there are other factors explaining the catch in addition to time. Fo
example 3 hours resulted in 8 kgs while 4 hours had a catch of 5 kgs.

Example 8

An economist has put it that the rate of unemployment is related to its Gross Domestic Product (GDP). In a
bid to prove this an economist collects data on GDP and associated rates of unemployment for the past 10
years where the rate of inflation has been relatively stable. The figures are as follows:
219

BUSINESS MATHS & STATISTICS (TC3)


Table 15.8 GDP and rate of unemployment

GDP Rate of Unemployment


15 8
14 9
14 10
12 11
11 12
12 8
15 7
19 5
22 3
22 3

Required.

a) Taking X to represent GDP, and Y to represent unemployment plot a scatter diagram and comment
on the likely relationship between GDP and unemployment.

b) Calculate the Pearson correlation coefficient of the data and comment on the result.

c) Calculated the coefficient of determination and comment on its likely use.

Solution

a) Figure 15.5 The scatter diagram

x y
14
Y

15 8
1412 9
14 10
1210 11
11 12
8
12 8
15 6 7
19 5
22 4 3
22 3
2

0
X
0 5 10 15 20 25

The scatter of points produce a pattern which is slopping downwards to the right signifying that
unemployment is negatively related to GDP.

Further the points are not on a straight line meaning the correlation is not perfect but partial.
However since the points a close to forming a straight line, the correlation is strong.
220

BUSINESS MATHS & STATISTICS (TC3)


b) The correlation coefficient

n¦ xy  ¦ x ¦ y
r
>n¦ x  ¦ x @u >n¦ y  ¦ y @
2 2 2 2

Table 15.9 Table of calculations (GDP) and unemployment


2 2
x y XY X Y
15 8 120 225 64
14 9 126 196 81
14 10 140 196 100
12 11 132 144 121
11 12 132 121 144
12 8 96 144 64
15 7 105 225 49
19 5 95 361 25
22 3 66 484 9
22 3 66 484 9

Totals 156 76 1078 2580 666

n= 10

10 u 1078  156 u 76
So r = - 0.95 (2dp)
>10 u 2580  156 @u >10 u 666  76 @
2 2

c) Coefficient of determination R 2 is r2 = (-0.95)2 = 0.90

90% of the variations in unemployment are explained by the variations in GDP. This will form a
sound basis for predicting one variable (e.g. unemployment) from the other.

15.3.3 Rank Correlation Coefficient

DEFINITION

Rank correlation measures the correlation between variables which have been expressed in form of ranks

221

BUSINESS MATHS & STATISTICS (TC3)


There are several ways of calculating the rank correlation coefficient but the most common one is the
Spearman’ Rank Correlation Coefficient.

Often denoted by R the Spearman’s rank correlation coefficient is calculated as:

6¦ d 2
R 1 , where d = difference between corresponding ranks and

n n2 1
n = number of pairs of ranks
Example 9

The mid semester examination results in Mathematics and Costing of a sample of 6 students were as follow:

Table 15.10 Examination marks


Students Mathematics Costing
John 98 77
Annie 72 84
Peter 52 50
Chikondi 65 64
Mary 45 49
George 50 20
Required:

Use the Spearman’s rank correlation coefficient to investigate whether or not there is a relationship between
ability in Mathematics and Costing.

Solution

Since the data given is not in form of ranks, there is need to create ranks and then the rank correlation
coefficient formula can be used.

Table 15.11 Workings on ranks


Students Rank in Rank in d 2
d
Mathematics Costing
John 1 2 Ͳ1 1
Annie 2 1 1 1
Peter 4 4 0 0
Chikondi 3 3 0 0
Mary 6 5 1 1
George 5 6 Ͳ1 1
Total 4

6¦ d 2
Now R 1 

n n2 1
6u 4
= 1 = 0.886
6 6 2  1
222

BUSINESS MATHS & STATISTICS (TC3)


This is very high correlation which means ability in costing and mathematics are strongly related.

Tied ranks

It is possible to have a situation where ranks are tied. For instance there could be 2 number threes of three
number fours and so on. Where there are tied ranks, the ties are replaced by the mean of the ranks which
would have been there had the ties not occurred.

Example 10

A safari operator offers 8 products of adventure to his customers. In order to put more attention to te most
popular be has asked two top guides rank the products in terms of excitement generated in customers. The
results are as follows:

Table 15.12 Product ranks


Product Guide one Ranks Guide two Ranks

A 1 2
B 2 1
C 2 3
D 4 6
E 5 3
F 6 3
G 7 8
H 8 7

Do the rankings of the two guides corroborate?

Solution
Tied ranks for Guide One are: 2 and 2
If no ties these would have been: 2, and 3
23
Mean (to replace the ties) 2.5
2

Tied ranks for Guide Two: 3, 3 and 3


These took the place of 3, 4 and 5
3 45
Mean to replace the ties 4
3

223

BUSINESS MATHS & STATISTICS (TC3)


Table 15.13 Product ranks workings
Guide one Guide two d d
2

Ranks Ranks
1 2 Ͳ1 1
2.5 1 1.5 2.25
2.5 4 Ͳ1.5 2.25
4 6 Ͳ2 4
5 4 1 1
6 4 2 4
7 8 Ͳ1 1
8 7 1 1
16.5
Shadedranksarereplacements

6¦ d 2 6 u 16.5
Now R 1  = 1 0.804
2
n n 1 8 8 2  1

The rankings by the two Guides corroborate.

CHAPTER SUMMARY

This chapter established building blocks for investigating relationships between basically two variables,
called independent and dependent variables respectively. It studies two related concepts: regression and
correlation. Regression analysis searches for a relationship between a variable of interest (dependant
variable) and other variables (independent variables). The main goal of such relationship building is to
forecast the dependant variable in the future based on past values of the dependent and independent
variables. For example, we might want to predict the level of sales in the future for a company by studying
the relationship between the sales (dependant variable) and the level of marketing (independent variable).

Correlation, on the other hand, measures the degree or strength of the relationship between the variables.
This measure of the strength of the relationship is referred to as the correlation coefficient. Two correlation
coefficients were presented: Pearson’s product moment correlation coefficient and Spearman’s rank
correlation coefficient. These are calculated using the formula:

n¦ xy  ¦ x ¦ y
Pearson’s product moment correlation coefficient: r .
>n¦ x  ¦ x @u >n¦ y  ¦ y @
2 2 2 2

6¦ d 2
Spearman’s rank correlation coefficient: R 1 

n n2 1
The chapter also looked at the coefficient of determination, r 2 , which is defined as the proportion (or
percentage) of variation in the dependent variable, y , that is explained by the independent variable, x .
224

BUSINESS MATHS & STATISTICS (TC3)


Once the correlation coefficient, r, has been determined, the coefficient of determination is easily found by
squaring r, i.e. r 2 .

END OF CHAPTER EXERCISES

1. What is the difference between regression and correlation?

2. Describe any two ways of investigating relationships between variables.

3. In the following set of data, y represents ten finance companies’ total operating costs (in millions of
K) for a particular year and x represents the companies’ assets (in millions of K) for the same year.
y x
5 310
3 250
2 100
5 450
2 150
3 200
4 320
3 230
2 140
6 400
Required:

a. Draw the scatter diagram on graph paper and comment on the relationship between x and y.
b. Find the equation of the least-squares regression line, assuming that operating costs depend
on assets.

i. Use your results in (ii) above to predict the operating costs for a firm with assets of
K500 million.

ii. If the coefficient of correlation r is 0.93 comment on the likely accuracy of your
prediction.

4. A cost accountant has derived the following data on the running costs and distance travelling by
twenty of a company’s fleet of new cars used by its computer salesmen last year. Ten of the cars
are type F and ten are type L.

225

BUSINESS MATHS & STATISTICS (TC3)


Car F Car L
Distance Running Distance Running
Travelled Costs (K Travelled Costs (K
(Thousand million) (Thousand million)
km) km)
X y x y
4.0 5.3 3.5 6.9
4.6 6.7 4.6 7.6
5.9 7.5 5.3 7.9
6.7 8.8 6.0 8.3
8.0 8.0 7.2 8.8
8.9 9.1 8.4 9.2
8.9 10.5 10.1 9.6
10.1 10.0 11.1 10.3
10.8 11.7 11.5 10.1
12.1 12.4 12.3 11.3

Mean 8.0 9.0 8,0 9.0

Required:

(a) The least square regression lines were calculated using a standard computer package as
follows:

Car F : y = 2.650 + 0.79x


Car L : y = 5.585 + 0.427x

(i) Plot the two scatter diagrams and regression lines on the same graph, distinguishing
clearly between the two sets of points.

(ii) Explain the meaning of the two regression coefficients for each set of these data.

(b) For the car F data the following statistics were calculated:

™x2 = 704.34, ™xy = 771.07


™y2 = 854.38

Calculate the correlation coefficient and interpret its meaning.

(c) Predict the running costs for the two different types of cars if the average distance travelled is
12,000km.

5. Kay’s Confectionaries makes small cakes for functions and an analysis units made in the past and
associated costs are as follows

226

BUSINESS MATHS & STATISTICS (TC3)


Units Made Total cost (K'000
100 58
250 145
90 62
200 144
150 85
50 44
65 48
75 50

Required:

a) Calculate the Pearson’s product moment correlation coefficient and the coefficient of
determination.

b) Interpret your results.

6. (a) Define ‘correlation’.

(b) Calculate Spearman‘s rank correlation coefficient for the following data and comment on the
result:

X:1 2 3 4 5 6 7 8 9 10
Y: 2 4 5 1 3 7 9 6 10 8

7. A Stores supervisor and a Purchase manager were asked to rank the main suppliers (ABCDEFG and
H) in order of value to the company. The two managers ranked the eight suppliers and the following
are the results.

Stores supervisor: E C G H B D A F
Purchase manager: E G B D C A H F

a) Use the Spearman’n rank correlation coefficient to determine the amount of agreement
between the two managers.
b) Can any conclusion be drawn about the suppliers

8. Calculate Spearman’s rank correlation coefficient for the following ordinal data and comment on the
result:

x 1 2 3 4 5 6 7 8 9 10
y 10 8 5 6 9 7 2 3 1 4

9. In the following set of data, y represents ten finance companies’ total operating costs (in millions of
K) for a particular year and x represents the companies’ assets (in millions of K) for the same year.

227

BUSINESS MATHS & STATISTICS (TC3)


y x
5 310
3 250
2 100
5 450
2 150
3 200
4 320
3 230
2 140
6 400
35 2550

Required:

a) Draw the scatter diagram on graph paper and comment on the relationship between x and y.

b) If ™xy = 10300, ™x2 = 768500, ™y2 = 141

Find the equation of the least-squares regression line, assuming that operating costs
depend on assets.

c) Use your results in (ii) above to predict the operating costs for a firm with assets of £550
million. If r = 0.93 comment on the likely accuracy of your prediction.

10 Music Technologies, an electronics retail company in Durban, has kept records of the number of
Ipods sold within a week of placing advertisements in the Mercury. The following table shows the
number of ipods sold and the corresponding number of advertisements placed in the Mercury for 12
randomly selected weeks over the past year.

Ads 4 4 3 2 5 2 4 3 5 5 3 4
Sales 26 28 24 18 35 24 36 25 31 37 30 32

Required
a) Construct a scatter plot for the data.
b) Find the straight-line regression equation to estimate the number of ipods that Music Centre
can expect to sell within a week, based on the number of advertisement placed.
c) Estimate the likely mean sales of ipods when three advertisements are placed.
d) Calculate the product moment correlation coefficient and comment on the result obtained.
e) Calculate the coefficient of determination and interpret the result.

228

BUSINESS MATHS & STATISTICS (TC3)


CHAPTER 16 TIME SERIES
LEARNING OBJECTIVES:

By the end of this chapter the student should be able to:


i. Define a ‘time series’
ii. Plot time series data
iii. Described times series models
iv. Distinguish between components of a time series
v. Decompose a time series into its components
vi. Forecast time series values

16.0 INTRODUCTION
A time series is an ordered sequence of values of a variable at equally spaced time intervals. Simply put
time series is a set of data values that are recorded at successive and regular intervals over a period of time
e.g. daily, weekly, monthly, quarterly and annually. We must therefore caution you not to think of time
series as quarterly data only.

An analysis of history, a time series, can be used by management or business individuals to make current
and future plans based on long term time series data, and assuming that patterns would continue into the
future. Both short-term and long-term predictions are essential for a business entity to execute possible
development plans in terms of financing, procurement, manufacturing, sales, profits, revenue, new products,
new plants/machinery, market demand, and recruitment among others.

For forecasts based on time series to be meaningful, the rule of thumb is that the data must recorded over a
relatively longer period of time i.e. at least 10 observations. In this chapter we deal with the use of time
series data to forecast future events or activities. We first discuss the components of a time series. Then, we
explore basic techniques for analysing time series data in a process called decomposition. Finally, we
forecast future activities.

16.1 APPLICATIONS OF TIME SERIES

The analysis of time series has two major objectives namely to


a) Obtain an understanding of the underlying forces and structure that produced the observed data
b) Forecast using fitted time series model. The models can also be used to monitor a set of time series
components.

Time Series analysis many areas of application including economic forecasting, sales forecasting, budgetary
analysis, stock market analysis, yield projections, process and quality control, inventory studies , workload
projections, utility studies, and census analysis.

16.2 PLOTTING A TIME SERIES


A time series is plotted on graph called a histogram. The procedure for plotting time series is explained by
means of a worked example.

229

BUSINESS MATHS & STATISTICS (TC3)


Example 1
The following table shows water consumption (in litres) for a typical household in Kawale.

Table 16.1 Water consumption


Year Consumption
Jan – Mar Apr – Jun Jul – Sept Oct - Dec
2009 560 660 740 1200
2010 600 710 820 1340
2011 610 750 880 1400
2012 700 780

Construct a time series plot for the data

Solution:
To plot time series data, each data value is plotted against the corresponding time point and the points are
then joined by straight line segments as shown below.

Figure 16.1 Water consumption

Notice the up and down swings (peaks and troughs) which are typical of time series data. There is marked
seasonality in water consumption with the highest water consumption being in the last quarter (Oct-Dec) of
each year. Over the years, there is a slight increase in water consumption.

16.3 COMPONENTS OF A TIME SERIES


There are four main components to a time series: the secular trend or simply trend, the cyclical variation, the
seasonal variation, and the random variation.

16.3.1 Secular Trend (T):


This is defined as the long-term direction of a time series. The trend can be upward (increasing), downward
(decreasing) or constant (show no change) over time. In the previous example, we plotted a time series on a
230

BUSINESS MATHS & STATISTICS (TC3)


historigram and noted that there was a slight increase in water consumption over the four-year period. We
can confirm this observation by fitting in a trend line in the time series plot as shown below:

Figure 16.2 Trend

Trend

16.3.2 Cyclical Variation (C):


The wave-like movement of a time series caused by booms and slumps of an economy over long periods of
time. The periods usually correspond to business cycles in the economy.

16.3.3 Seasonal Variation (S):


These are patterns of change in a time series which occur over short repetitive periods. These represent
predictable deviation from the trend. For example, an ice creamer seller would expect a substantial increase
in sales in summer. Given a particular time point at which data is observed, a seasonal variation is therefore
the difference between an actual data value observed and the trend figure.

When we think of season in time series analysis, we must not confine ourselves to annual geographical
seasons of say spring, summer, autumn, and winter or rainy and dry seasons. We must note that in business
seasons can be monthly (month-end, mid-month), weekly (mid-week, weekend) or even daily (morning,
noon, afternoon, evening, night). All these seasons will have varying effects on business operations.

Graphically, we can show the seasonal variation as below:

231

BUSINESS MATHS & STATISTICS (TC3)


Figure 16.3 Seasonal variations

16.3.4 Random or Irregular Variations (R or I):


These are variation which occur over short intervals and are unpredictable. Factors that can cause these
variations include unexpected changes in weather, strikes, breakdowns, theft, war/political unrest, death, and
sickness.

16.4 TIME SERIES MODELS


Once the components of a time series are identified, we need to understand how they are combined to
provide an observed data value. There are two common models for combining time series components and
these are: The additive model and the multiplicative model.

The additive model assumes that each time series data value is a sum (algebraically) of the components:
Y T C S  R

Where Y = Actual observations or observed data value


T = Trend
C = Cyclical variation
S = Seasonal variation
R = Random/Irregular/Residual variation

At level of this text we shall ignore Cyclical variations. Random variations will be estimated as a residue in
the calculation of average seasonal variations.

Consequently, any time series value shall be assumed to be made up of the remaining two components
resulting into:

232

BUSINESS MATHS & STATISTICS (TC3)


Y T S
The multiplicative model also referred to as the classical time series model assumes that each time series
data value is as a result of multiplying (a product of) the components:
Y T C S R
If we ignore the C and R components, the model becomes
Y T S

Note: At Technician Level candidates are normally told which model to use. However, in the event that the
model is not specified, full marks are awarded if the time series is analysed correctly by using either model.
Mostly, candidates are expected to use the additive model. At Foundation Level, however, candidates are
expected to be able to decide from a plot of the data or from other methods which model to apply, and then
use the correct model.

16.5 TIME SERIES DECOMPOSITION


Decomposition is the process of breaking a time series into its components. As pointed out earlier, our focus
in this manual is on the trend and seasonal components.

16.5.1 The Trend


The objectives behind the study of the trend are:
To explain the general underlying tendency in the movement of data over time. This is usually
assumed to be in form of a linear pattern with a positive (increasing) or negative (decreasing)
gradients.
To remove the trend from the data in order to expose the movements in the other components.

The trend can be estimated using any of the following methods:


a) Semi-average method
b) Least squares method
c) Moving average method

A. Trend by Semi-Average Method


To find the trend using the semi-average method we follow the following basic steps:
Step 1:
Split the data into 2 equal parts (halves). If there is an odd number of observations/values, simply leave out
the median values
Step 2:
Calculate the arithmetic mean for each group to obtain x1 and x 2
Step 3:
Plot these two points against the median position of each group. Join the two points with a straight line. This
is the trend line.

The trend for each time point is then read from the linear plot. Alternatively, find the average
increase/decrease per year by dividing the difference between the two mean points by the number of time
points between them. We illustrate this by an example.

233

BUSINESS MATHS & STATISTICS (TC3)


Example 2
The following data shows the quarterly domestic exports (to the nearest billion of kwacha) for Malawi for
the years 2010 to 2012.

Table 16.2 Domestic Exports


Domestic exports 2010 – 2012
Year Jan-Mar Apr-Jun Jul-Sep Oct-Dec
2010 32 27 52 49
2011 48 46 53 76
2012 50 65 100 -
Source: 2011 & 2012 Malawi Trade Briefs - NSO

Find the trend by means of semi-average method. And hence assign trend values for all quarters.

Solution

Step 1: Splitting the data


The data set has 11 observations. Therefore it can be split into halves of 5 observations each. The first half
covers the period Jan-Mar 2010 to Jan-Mar 2011 with the following export values: 32, 27, 52, 49 and 48.
The second half covers the period Jul-Sep 2011 to Jul-Sep 2012 with the following export values: 53, 76,
50, 65 and 100.

Note: Since have an odd number of values, we have left out the median data observation of 46 for Apr-Jun
2011.

Step 2: Calculating the arithmetic means


32  27  52  49  48 53  76  50  65  100
x1 41 .6 , x 2 68 .8
5 5
Step 3: Plotting the mean points
We will plot the time series data just as before and then fit in a trend line by joining the two mean points. x1
will be plotted against Jul-Sep 2010 and x 2 against Jan-Mar 2012. These are the median positions for each
group.

234

BUSINESS MATHS & STATISTICS (TC3)


Figure 16.4 Domestic Exports – Trend by semi averages
120

100
Trend

SemiͲaverages
80
Exports

60

40

20

Q1 Q2 Q3 Q4 Q1 Q2 Q3 Q4 Q1 Q2 Q3
2010 2011 2012
YearsandQrts

The trend values for each time point can be read from the graph or calculated as below:
Difference between the mean po int s
Average change in trend
Number time po int s between them
x2  x1 68.6  41.6
4.53
6 6
We will proceed to find the trend values by adding or subtracting 4.53 to or from either of the mean points.
For example the trend for Apr-Jun 2010 is 41.6 – 4.53 = 37.1, for Oct-Dec 2010 is 41.6 + 4.53 = 43.1,
and for Apr-June 2012 is 68.8 + 4.53 = 73.3. The rest of the trend values are presented below.

Table 16.3 Trend


Year Month Trend
2010 Jan-Mar 32.5
Apr-Jun 37.1
Jul-Sep 41.6
Oct-Dec 46.1
2011 Jan-Mar 50.7
Apr-Jun 55.2
Jul-Sep 59.7
Oct-Dec 64.3
2012 Jan-Mar 68.8
Apr-Jun 73.3
Jul-Sep 77.9
Oct-Dec -

235

BUSINESS MATHS & STATISTICS (TC3)


B. Trend by Least Squares Method
To find the trend using the method of least squares we follow the following steps:

Step 1:
Treat time points as the independent X-variable. The time points are transformed into values through a
process of coding as follows: The first time point is coded 1, the second time point is coded 2, and so on
until the last time point is coded.

Step 2:
Obtain a least squares regression line between the x-codes and the time series data values (Y-values). The
trend line through the data is given by:
y a  bx
Where:
y trend value for a given period
n¦ xy  ¦ x¦ y
b
n¦ x 2  ¦ x
2

a
¦ y b¦x
n n
To calculate the trend value, substitute the appropriate x-code into the least squares equation and compute
the value of y.
Example 3
Find the trend by means of least squares method for the domestic export data from the previous example.

Solution
To find the least squares trend, first code the time point as follows: Jan-Mar 2010 – 1, Apr-Jun 2010 – 2,
Jul-Sep 2010 – 3, ... Jul-Sep 2012 – 11.

Table 16.4 Trend calculation (least squares)


Year Month X-codes Data value Trend
Y y 24.64  4.95 x
2010 Jan-Mar 1 32 29.6
Apr-Jun 2 27 34.5
Jul-Sep 3 52 39.5
Oct-Dec 4 49 44.4
2011 Jan-Mar 5 48 49.4
Apr-Jun 6 46 54.3
Jul-Sep 7 53 59.3
Oct-Dec 8 76 64.2
2012 Jan-Mar 9 50 69.2
Apr-Jun 10 65 74.1
Jul-Sep 11 100 79.1

236

BUSINESS MATHS & STATISTICS (TC3)


To find the least squares line, y a  bx , we need the following:
n 11, ¦x 66, ¦x 2
506, ¦y 598, ¦ xy 4133

11 u 4133  66 u 598 598 66


?b 4.9545 and a  4.9545 24 .6366
11 u 506  66 2 11 11

The least square trend is therefore given by y 24.64  4.95 x

We calculate the trend values by substituting the x-codes into the least squares equation y 24.64  4.95 x .
For example, for Jan-Mar 2011 (coded 5) the trend is y 24.64  4.95 u 5 | 49.4 . The rest of the trend
values are provided in the last column of the table above.
 Trend by Moving Averages Method
C. Trend by Moving Averages Method
This is an alternative and possibly standard method for finding the trend and requires no specific
mathematical formula. The method is non-linear (though fairly linear), in the sense that it does not result in
a straight line, but it does smooth out peaks and valleys (ups and downs) in a set of observations by
“removing” seasonal and random variations from to reveal the trend.

The moving average trend is accomplished by “moving” the arithmetic mean values through the time series
i.e. averaging sets of overlapping data observations through the time series data. In working out the moving
average trend, the number of observations to include or average depends on the periodicity of the time
series. With quarterly data, the period is 4; for data recorded daily 5 days a week, the period would be 5. If
the period is an even number (e,g. quarterly data or data recorded daily for 6 days/week) then a centred
moving average is required. However, if the period is an odd number (e.g. data recorded daily 5 or 7
days/week), then a simple moving average is appropriate.

Calculation of a moving average


As indicated earlier, a moving average is calculated by averaging a set of “consecutive” and overlapping
values at a time. We illustrate the procedure in the following example.

Example 4
The following are production figures over 9 days. Use them to find 2-point and 3-point moving averages.

Table 16.5 Production figures


Day Day 1 Day 2 Day 3 Day 4 Day 5 Day 6 Day 7 Day 8 Day 9
Production 6 4 10 8 5 12 9 7 15

Solution
For 2-point moving averages:

237

BUSINESS MATHS & STATISTICS (TC3)


We first find the moving totals by adding 2 adjacent figures at a time and allow the pairs to overlap through
the last figure. The first moving total is 6  4 10 , Next we drop the first figure 6, and then add 3rd figure
10: 4  10 14 , and so on. These moving totals must be placed at the median position as shown in the 3rd
column of the following table. To obtain the 2-point moving averages, we divide the moving totals by 2.
The moving averages are placed in column 4.

Table 16.6 Production figures – moving averages


Day Data value, y Moving totals 2-point moving average
Day 1 6
10 5
Day 2 4
14 7
Day 3 10
18 9
Day 4 8
13 6.5
Day 5 5
17 8.5
Day 6 12
21 10.5
Day 7 9
16 8
Day 8 7
22 11
Day 9 15

For 3-point moving averages:

We first find the moving totals by adding 3 adjacent figures at a time and allow the sets to overlap through
the last figure. The first moving total is 6  4  10 20 , Next we drop the first figure 6, and then add 4th
figure 8: 4  10  8 22 , and so on. These moving totals must be placed at the median position as shown in
the 3rd column of the following table. To obtain the 3-point moving averages, we divide the moving totals
by 3. The moving averages are placed in column 4.

Table 16.7 Production figures (moving averages)

Day Data value, y Moving totals 2-point moving average


Day 1 6
Day 2 4 20 6.67
Day 3 10 22 7.33
Day 4 8 23 7.67
Day 5 5 25 8.33
Day 6 12 26 8.67
Day 7 9 28 9.33
Day 8 7 31 10.33
Day 9 15
238

BUSINESS MATHS & STATISTICS (TC3)


Note: In a 2-point moving average the moving totals and averages fall between two time points (days). For
example the first is between the Day 1 and Day 2 being the median position of the first pair of figures. The
3-point moving totals and averages fall against/adjacent times points. For example the first is aligned against
Day 2 being the median position for the first three figures. Notice that the resulting moving averages are less
variable than the original data. The production figures have been ‘smoothed’.

The most common moving averages calculated in time series data for commercial purposes are 4, 5, 6 or 7
point ones because most data relates to 4 quarter/yearly cycles, and 5, 6 or 7 day/weekly cycles.

Example 5 (Data with an even period)


The following data shows the sales revenue (in MK’ million) of a local company from 2010 to 2013.

Table 16.8 Sales revenue


Sales revenue 2010-2013
Year Qtr 1 Qtr 2 Qtr 3 Qtr 4
2010 79 48 68 107
2011 97 66 85 134
2012 113 91 100 148
2013 136 105 125 174

Find the trend using the moving average method and plot it together with the time series data.

Solution
We proceed to arrange the data in a table that will help facilitate the calculation of trend values. Place the
data values in one column (in our case column 2). Then add in fours since the period is 4 (for quarterly
data): The first moving total is 79+48+68+107=302, and for the second we drop 79 and then add the fifth
value 97: 48+68+107+97=320 and so on. The rest of the moving totals are place in column 2 in the table.
The moving totals are then divided by 4 (the period) to obtain the moving averages. These are placed in
column 4. Notice that both the moving totals and averages are not aligned with time points (Qtrs) because
the period is even. Hence the need to find centred moving averages for the ‘trend’. To find the centred
moving averages we add the moving averages in pairs and divide by 2. The first centred moving average is
(75.5  80) y 2 77.75 | 77.8 , the second is (80  84.5) y 2 82.25 | 82.3 , and so on. Each centred
moving averages is placed between the two moving averages being averaged. This then aligns the centred
moving averages (Trend) with the time points (Qtrs) as shown in column 5 in the following table.

239

BUSINESS MATHS & STATISTICS (TC3)


Table 16.9 Sales revenue Trend by moving averages
Year/Qtr Data value Moving totals Moving average ‘Trend’
y (add in fours) (Total divide by 4) Centred average
(add in pairs and
divided by 2)
2010 Qtr 1 79

Qtr 2 48
302 75.50
Qtr 3 68  Average = 77.8
320 80.00
Qtr 4 107  82.3
338 84.50
2011 Qtr 1 97  86.6
355 88.75
Qtr 2 66  92.1
382 95.50
Qtr 3 85  97.5
398 99.50
Qtr 4 134  102.6
423 105.75
2012 Qtr 1 113  107.6
438 109.50
Qtr 2 91  111.3
452 113.00
Qtr 3 100  115.9
475 118.75
Qtr 4 148  120.5
489 122.25
2013 Qtr 1 136  125.4
514 128.50
Qtr 2 105  131.8
540 135.00
Qtr 3 125

Qtr 4 174

Both the time series data values and the trend values are plotted on the following graph.

BUSINESS MATHS & STATISTICS (TC3)


Figure 16.5 Sales revenue
160

140

120 SmoothenedSvs

100
REVENUE

80

60

40

20

0
Q1 Q2 Q3 Q4 Q1 Q2 Q3 Q4 Q1 Q2 Q3 Q4
2011 2012 2013
YearsandQrts

Notice that the moving average trend has smoothed out the time series and is fairly linear.

Example 7 (Data with an odd period)


The following data shows the output of a factory located in Zolozolo over a 3-week period.

Table 16.10 Zolozolo factory output


Production output
Week Monday Tuesday Wednesday Thursday Friday
Week 1 86 92 104 98 78
Week 2 90 99 110 102 80
Week 3 93 101 116 110 83

By means of moving average method, find the trend.

Solution:
We proceed as in the previous example. Note that the period is now 5 (an odd number); hence we do not
need to centre the moving averages. For instance the first moving total of 485 (i.e. 86+92+104+98+78) is
placed at the median position of the set which is alongside 104 i.e. Wednesday of Week 1. The second
moving total 462 (i.e. 92+104+98+78+90) is placed alongside 98 i.e. Thursday of Week 2, and so on. The
moving totals and simple moving averages are provided in the table that follows. The simple moving
averages are our trend values.
241

BUSINESS MATHS & STATISTICS (TC3)


Table 16.11 Zolozolo factory output trend
Week/Day Data value Moving totals ‘Trend’
y (add in fives) Moving average
(Total divide by 5)
Week 1 Mon 86
Tue 92
Wed 104 458 91.6
Thu 98 462 92.4
Fri 78 469 93.8
Week 2 Mon 90 475 95.0
Tue 99 479 95.8
Wed 110 481 96.2
Thu 102 484 96.8
Fri 80 486 97.2
Week 3 Mon 93 492 98.4
Tue 101 500 100.0
Wed 116 503 100.6
Thu 110
Fri 83

16.5.2 Seasonal Variation


As discussed earlier, many time series are affected by seasonal factors. It is necessary to find the value of a
seasonal component in order to obtain seasonally adjusted data and for forecasting purposes. The procedure
for finding the seasonal variations/factors depends on the model used.

Using the additive model:


x Obtain the seasonal variation for each time point by subtracting the ‘trend’ from the observed data value i.e.
S Y T
x Find the average seasonal variation for time point. Check that the sum of the average seasonal variations is
equal to zero (0).
x If the sum of the averages is not equal to zero, then the averages must be adjusted accordingly so that they
Sum of the averages
sum to zero. The adjustment factor is . If the sum is less than zero, we add the
Period
modulus of the adjustment factor to each average. However, if the sum is greater than zero, we subtract the
adjustment factor from each average to obtain the adjusted seasonal variations/factors.

Using the multiplicative model:


x Obtain the seasonal variation for each time point by dividing the data value by the ‘trend’ i.e. S Y y T
x Find the average seasonal variation for each time point. Check that the sum of the average seasonal
variations is equal to period. For quarterly data, they must sum to 4, for a 5 day-week, they must sum to 5. . .

242

BUSINESS MATHS & STATISTICS (TC3)


x If the sum of the averages does not equal the period, then the averages must be adjusted accordingly so that
Period
their sum equals the period. The adjustment factor is . The adjusted seasonal
Sum of the averages
variations/factors are then obtained by multiplying each average by the adjustment factor.

Determining which model to use


The additive model is used when the seasonal variation is approximately the same, irrespective of the trend
values. Consider the graph of sales revenue for a local company which we presented before and is
reproduced here.

Figure 16.6 Sales revenue of a local company 2011-2013


160

140

MagnitudeofSvinqrt4not
120 verydifferent

100
REVENUE

80

MagnitudeofSVinothercorresponding
60 quarters(egQ1,Q2,orQ3)arealso
similar

40

20

0
Q1 Q2 Q3 Q4 Q1 Q2 Q3 Q4 Q1 Q2 Q3 Q4
2011 2012 2013
YearsandQrts

We can see that the seasonal variations (differences between observed and trend values) for similar quarters
are almost the same. In this case the additive model is appropriate.

In the multiplicative model the season variation is approximately proportional to the trend. If the trend is
upward, then the variation increases. If the trend is downward, then the seasonal variation decreases.
Consider the following graph:

243

BUSINESS MATHS & STATISTICS (TC3)


ure 16.7 Number of Visitors

e can see that the trend is upward and the seasonal variations (differences between observed and trend
ues) for similar quarters are increasing (in absolute terms) as we move from 2010 to 2012. In this case
multiplicative model is used.

art from the graphical method, the other and quicker method for decide the right model to use is take the
ference between the smallest and largest value for each year (or week). If the differences are roughly the
me or do not follow a particular pattern, then additive model is appropriate. However, if the differences
increasing for an upward trend or decreasing for a downward trend, then the multiplicative model is
propriate. Consider the data sets whose graphs we just compared.

ble 16.12 Sale revenue figures


Sales revenue 2011-2013
Year Qtr 1 Qtr 2 Qtr 3 Qtr 4
2010 79 48 68 107
2011 97 66 85 134
2012 113 91 100 148
2013 136 105 125 174

e differences are: 2010: 107 – 48 = 59


2011: 134 – 66 = 68
2012: 148 – 91 = 57
2013: 174 – 105 = 69
hile the trend is upward, the differences show no obvious pattern. Therefore additive model is appropriate.

244

BUSINESS MATHS & STATISTICS (TC3)


Table 16.13 Number of visitors 2010 - 2012
Number of visitors 2010-2012
Year Qtr 1 Qtr 2 Qtr 3 Qtr 4
2010 24 28 35 48
2011 32 38 43 66
2012 36 50 54 96

The differences are: 2010: 48 – 24 = 24


2011: 66 – 32 = 34
2012: 96 – 36 = 58
As the trend is upward and the differences are increasing, the multiplicative model should be appropriate.

Note: In an examination the applicable model will be specified for TC3 paper.

Example 8 (Continuation of Example 5)


Using the sales revenue data and the trend calculated in the previous example find the quarterly seasonal
variation using the additive model.

Solution:
To find seasonal variations we need both the observations and trend values from the previous example. We
reproduce the table from the previous example here:

Table 16.14 Calculation table - Sales revenue seasonal variations


Year/Qtr Data value Moving Moving ‘Trend’ Seasonal variation
y totals average Centred average S Y T
2010 Qtr 1 79 -
Qtr 2 48 -
Qtr 3 68  77.8 68 – 77.8 = -9.8
Qtr 4 107  82.3 107 – 82.3 = 24.7
2011 Qtr 1 97  86.6 10.4
Qtr 2 66  92.1 -26.1
Qtr 3 85  97.5 -12.5
Qtr 4 134  102.6 31.4
2012 Qtr 1 113  107.6 5.4
Qtr 2 91  111.3 -20.3
Qtr 3 100  115.9 -15.9
Qtr 4 148  120.5 27.5
2013 Qtr 1 136  125.4 10.6
Qtr 2 105  131.8 -26.8
Qtr 3 125 -
Qtr 4 174 -

Notice that we have left out the moving totals and moving averages. This is so because we do not need them
at this stage.

245

BUSINESS MATHS & STATISTICS (TC3)


Having subtracted the trend values (T) from the time series values (Y), we must arrange the differences (i.e.
seasonal variations) such that all seasonal variations for similar quarters are in the same column.

Table 16.15 calculation of average seasonal variations


Seasonal variation (SV)
Year Qtr 1 Qtr 2 Qtr 3 Qtr 4
2010 - - -9.8 24.7
2011 10.4 -26.1 -12.5 31.4
2012 5.4 -20.3 -15.9 27.5
2013 10.6 -26.8
Total 26.4 -73.2 -38.2 83.6
Average SV 8.8 -24.4 -12.7 27.9 Sum of averages is -0.4
Adjustment factor +0.1 +0.1 +0.1 +0.1 The adjustment factor
Sum  0 .4 is then added to each
 0 .1
Period 4 SV average.
Adjusted SV 8.9 -24.3 -12.6 28.0 Sum is now 0
Seasonal variation are in MK’million

The final seasonal variations are:

Table 16.16 Adjusted average seasonal variation


Quarter1 Quarter2 Quarter3 Quarter4
MK9m MK24m MK13 MK28m

Note:
x To obtain the average, divide the totals by the number of seasonal variations in the respective column. In the
case above, the totals have been divided by 3 since each column has three seasonal variations. In some cases
the numbers will differ from column to column.
x The adjusted seasonal variations must be given be given to the same accuracy as the original data.

Seasonally Adjusted or Deseasonalized data


Most economic and business time series are published seasonally adjusted i.e. with seasonal variations
removed.

246

BUSINESS MATHS & STATISTICS (TC3)


The additive model

For additive model, seasonally adjusted values are obtained by subtracting the adjusted seasonal variations
from respective time series values (Actual)

: YSeasonally adjusted Y  S Adjusted

. For example, for 1st Quarter 2010, the seasonally adjusted value is 79 – 9 = 70 i.e. K70 million, and

for 2nd Quarter, 2010 is 48 – (-24) = 72 i.e. K72 million.

The Multiplicative model

In the case of the multiplicative model, seasonally adjusted values are obtained by dividing the time series
values by the respective adjusted seasonal variations:

Y
YSeasonally adjusted .
S Adjusted

Example 9 (Continuation of Example 5)


Using the sales revenue data, the trend and seasonal variations calculated in the previous examples,
seasonally adjust the sales revenue for 2013 (using the additive model).

Solution:
We need to subtract the average seasonal variations from the 2013 sales revenues

Table 16.17 Seasonally adjusted data


2013 Data, Y Seasonal Seasonally adjusted sales
variation, S Y–S
Qtr 1 136 9 136 – 9 = 127
Qtr 2 105 -24 105 – (-24) = 129
Qtr 3 125 -13 125 – (-13) = 138
Qtr 4 174 28 174 – 28 = 146

The seasonally adjusted sales for 2013 are: 1st Quarter – K127 million, 2nd Quarter – K129 million, 3rd
Quarter – K138 million and 4th Quarter – K146 million

Interpretation:

These are the ‘sales that the company would have made in each quarter of 2013 if it were not for the effects
of seasons.

247

BUSINESS MATHS & STATISTICS (TC3)


seasons tend to bring the sales down.

16.6 FORECASTING

To obtain forecasts, it is necessary to project/extrapolate the trend. Projected/extrapolated trend values are
obtained based on the method employed to find the trend.

Using least squares method:


x Extend the x-coding to the period whose forecast is sought.
x Substitute the appropriate x-code in the least squares equation to obtain the trend estimate.

Using moving average method:


x Find the average trend increase (or decrease) i.e.
Last trend  first trend values Tn  T1
Number of trends  1 n 1
x Add the average trend increase to the last trend values until the required trend estimates are obtained.

To forecast future values, either add the projected trends to the respective adjusted seasonal variations (for
additive model) or multiply the projected trends with the respective seasonal variations (for multiplicative
model).

Example 10 (Continuation of Example 9)


Using the sales revenue data, the trend and seasonal variations calculated in the previous examples, forecast
the sales revenue for each quarter of 2014 (using the additive model).

Solution:
We first find the average quarterly increase in trend. The average trend increase is (Last trend, 131.8 – first
trend, 77.8) divided by number of increments (12 – 1) where 12 is the number of trend.
131 .8  77 .8 54
Average increase in trend 4 .9
12  1 11
Since the last trend, 131.8, corresponds to the Qtr 2, 2013, to find the trend estimate for the first quarter, Qtr
1, of 2014, we need to add 4.9 three times to the 131.8. The projected trend value is given by
131.8  3 u 4.9 146.5 . For the remaining quarters of 2014, the projected trend values are:
Qtr 2 trend: 131.8  4 u 4.9 151.4
Qtr 3 trend: 131.8  5 u 4.9 156.3
Qtr 4 trend: 131.8  6 u 4.9 161.2

We proceed to obtain the forecast for each quarter by adding the adjusted seasonal variations as discussed
earlier.

248

BUSINESS MATHS & STATISTICS (TC3)


Table 16.18 Forecasting
Trend, Seasonal Forecast = T+S
2014 TEstimated variation,
S
Qtr 1 146.5 9 146.5 + 9 = 155.5
Qtr 2 151.4 -24 151.4 + (-24) = 127.4
Qtr 3 156.3 -13 156.3 + (-13) = 143.3
Qtr 4 161.2 28 161.2 + 28 = 189.2

Our forecasts for 2014 are: 1st Quarter – K156 million, 2nd Quarter – K127 million, 3rd Quarter – K143
million, and 4th Quarter – K189 million.

Note
The forecasts assume that there is a linear trend, and that the projection of a linear trend reflects the future.
The seasonal variations are also assumed to be stable. However, if these assumptions are not true, then our
forecasts are not correct.

This far we have successfully managed to look at problems involving quarterly data and the additive model.
We now shift out attention to time series data other quarterly data and the multiplicative model.

Example 11:
The owner of Zathu, a popular restaurant in Lilongwe wishes to study the patterns of customer numbers for
the days of the week in order to be able to forecast activities for the coming days. He then gathers data on
patronage over three weeks as follows.

Table 16.19 Customers to Zathu


Week Number of customers
Monday Tuesday Wednesday Thursday Friday
Week 1 113 72 65 95 145
Week 2 118 81 77 110 160
Week 3 129 100 95 120 166

a) Calculate the trend by using the method of moving averages.


b) Calculate the adjusted seasonal variations assuming the additive model.
c) Forecast the number customer for Monday and Tuesday of Week 4.
d) Calculate the adjusted seasonal variations assuming the multiplicative model.
e) Forecast the number of customers for Monday and Tuesday of Week 4.

249

BUSINESS MATHS & STATISTICS (TC3)


Solution:
a) Trend using moving average:

Since the time series is of period 5 and 5 is an odd number, simple 5-point moving averages are required for
the trend. We will add the data values in fives, place the total alongside the median position and then divide
each moving total by 5 as shown below

Table 16.20 Calculation table - Trend and SV


Week/Day Data value Moving totals ‘Trend’ SV
y (add in fives) Moving average
(Total divide by 5) S Y T
Week 1 Mon 113 - - -
Tue 72 - - -
Wed 65 490 98 -33
Thu 95 495 99 -4
Fri 145 504 100.8 44.2
Week 2 Mon 118 516 103.2 14.8
Tue 81 531 106.2 -25.2
Wed 77 546 109.2 -32.2
Thu 110 557 111.4 -1.4
Fri 160 576 115.2 44.8
Week 3 Mon 129 594 118.8 10.2
Tue 100 604 120.8 -20.8
Wed 95 610 122 -27
Thu 120 - - -
Fri 166 - - -

b) Seasonal variations, SV using additive model


Seasonal variation, S Y  T

Table 16.21 Seasonal variations using additive model


Seasonal variation (SV)
Week Mon Tue Wed Thu Fri
Week 1 - - -33 -4 44.2
Week 2 14.8 -25.2 -32.2 -1.4 44.8
Week 3 10.2 -20.8 -27 - -
Total 25 -46 -92.2 -5.4 89
Average SV 12.5 -23.0 -30.7 -2.7 44.5 Sum of averages is
0.6
Adjustment factor -0.12 -0.12 -0.12 -0.12 -0.12 Subtract adjustment
factor from each SV
Sum 0 .6 average.
0.12
Period 5
Adjusted SV 12.38 -23.12 -30.82 -2.82 44.38 Sum is now 0

250

BUSINESS MATHS & STATISTICS (TC3)


The seasonal variations are: Monday: 12 customers, Tuesday: -23 customers, Wednesday: -30 customers,
Thursday: -3 customers, and Friday: 44 customers.

c) Forecasting using additive model


122  98 24
Average increase in trend 2 .4
11  1 10

Week 4 Trend, T Seasonal variation, S Forecast, T + S


Monday 122  3 u 2.4 129.2 12 141 customers
Tuesday 122  4 u 2.4 131.6 -23 109 customers

d) Seasonal variation, SV, using the multiplicative model

Seasonal variations for multiplicative model are obtained by dividing the time series data values by the trend
values i.e. S Y y T

Table 16.22 calculation table, Trend and seasonal variation (multiplicative model)
Week/Day Data value Moving totals ‘Trend’ SV
y (add in fives) Moving average
(Total divide by 5) S Y yT
Week 1 Mon 113 - - -
Tue 72 - - -
Wed 65 490 98 0.663
Thu 95 495 99 0.960
Fri 145 504 100.8 1.438
Week 2 Mon 118 516 103.2 1.143
Tue 81 531 106.2 0.763
Wed 77 546 109.2 0.705
Thu 110 557 111.4 0.987
Fri 160 576 115.2 1.389
Week 3 Mon 129 594 118.8 1.086
Tue 100 604 120.8 0.828
Wed 95 610 122 0.779
Thu 120 - - -
Fri 166 - - -

Averaging the SVs: We arrange in a table as before and average them to sum to 5 (the period)

251

BUSINESS MATHS & STATISTICS (TC3)


Table 16.23 Average seasonal variaition – multiplicative model
Seasonal variation (SV)
Week Mon Tue Wed Thu Fri
Week 1 - - 0.663 0.960 1.438
Week 2 1.143 0.763 0.705 0.987 1.389
Week 3 1.086 0.828 0.779 - -
Total 2.229 1.591 2.147 1.947 2.827
Sum of averages
Average SV 1.115 0.796 0.716 0.974 1.414 is 5.015 not 5
Adjustment factor u 0.997 u 0.997 u 0.997 u 0.997 u 0.997 Multiply each SV
average by the
Period 5 adjustment factor.
0.997
Sum 5.015
Adjusted SV 1.112 0.793 0.714 0.971 1.410 Sum is now 5

The seasonal variations are:

Monday: 1.112 (or 111.2%),


Tuesday: 0.793 (or (79.3%),
Wednesday: 0.714 (or 71.4%),
Thursday: 0.971 (or 97.1%), and
Friday: 1.41 (or 141%)

e) Forecasting using the multiplicative model

We find the trend projections (i.e. extrapolate trend values) as before. However, the forecasts are obtained
by multiplying the trend estimates by the seasonal variations.

122  98 24
Average increase in trend 2 .4
11  1 10

Week 4 Trend, T Seasonal Forecast, T u S


variation, S
Monday 122  3 u 2.4 129.2 1.112 129.2 u 1.112 = 144 customers
Tuesday 122  4 u 2.4 131.6 0.793 131.6 u 0.793 = 104 customers

Note
Unlike the moving average method, if the least squares method is used all times point will have trend
values. The procedures for obtaining seasonal variations and forecast are the same as in moving average
method.

252

BUSINESS MATHS & STATISTICS (TC3)


CHAPTER SUMMARY

This chapter has covered the following concepts:

x A time series is a collection of data over a period of time has four components namely: trend, cyclic,
seasonal and random components.
x A trend is the long-term direction of the time series.
x The cyclic component is the fluctuation above and below the long-term trend line over a longer
period of time.
x The seasonal component is the pattern in the time series over shorter periods of time due to effects of
seasons. These patterns tend to repeat themselves from time to time.
x The random variations are unpredictable.
x The trend component of a times series can be obtained using either the semi-average, least squares or
moving average methods. The semi-average and least squares methods assume pure linearity of the
trend. The moving average method is used to smooth the trend in a time series
x To obtain the seasonal variations, we used two models. If the differences between the largest and
smallest values for each year (week) are roughly the same or do not follow a particular pattern, then
additive model is applicable. If the differences are increasing for an upward trend or decreasing for a
downward trend, then the multiplicative model is applicable.
x In the additive model the sum of the average seasonal variations must be zero. For the multiplicative
models the average seasonal variations add up to the period of the time series.
x The seasonal variations are used to seasonally adjust data observations, and to forecast future
activities, taking into account effects of season.

END OF CHAPTER EXERCISES

1. Briefly describe the components which make up a typical time series


2. The daily output of Chidziwitso Ltd over a four week period is shown in the table below:

Week Output
Monday Tuesday Wednesday Thursday Friday
Week 1 187 213 210 227 247
Week 2 207 218 215 234 256
Week 3 202 228 225 248 266
Week 4 208 255 245 257 278

a) Find the trend by five-point moving averages


b) Display on the same graph the actual data together with the trend figures

253

BUSINESS MATHS & STATISTICS (TC3)


c) Determine the daily deviations from the trend and use these to determine the average
adjusted daily variations
d) Forecast the daily output for the all days of week 5 to the nearest unit of production

3 a) Distinguish between the ‘additive model’ and the ‘multiplicative model’ in time-series
analysis.
b) The following set of data represents a mining company’s quarterly production levels (y), in
thousands of tonnes over 3 years:

Year Production
Jan- Apr- Jul- Oct-
Mar Jun Sep Dec
2011 20 50 40 100
2012 40 120 150 250
2013 100 220 280 450

i. Calculate a centered four-point moving average trend


ii. Using the multiplicative model and the trend estimated in (i), estimate the seasonal factors in
each quarter (to 3 decimal places)
iii. Use the trend estimated in (i) and the seasonal factors estimated in (ii) to forecast the
company’s production in all 4 quarters of 2014 (to the nearest whole numbers).
iv. Comment on the likely accuracy of your forecasts in (iii).

4 The consumer price index (CPI) for a village is shown quarterly over three years in following table.

Year Quarter CPI


2010 Jan -Mar 100
Apr-Jun 104
Jul-Sep 108
Oct-Dec 102
2011 Jan -Mar 104
Apr-Jun 110
Jul-Sep 112
Oct-Dec 106
2012 Jan -Mar 106
Apr-Jun 116
Jul-Sep 118
Oct-Dec 110

a) Find the least squares linear trend line. Use the equation to find the trend values for each
quarter for 2010 to 2012.
b) Using the additive model and the trend from part (a), estimate the average seasonal variations
in each quarter (to two decimal places)
254

BUSINESS MATHS & STATISTICS (TC3)


c) Deseasonalise the CPI values for 2011.
d) Use your results in (a) and (b) to forecast the CPI for the four quarters of 2013 (to the nearest
whole number). Suggest two reasons why your forecast may not be reliable.

5 The planning department of Bata Shoe has developed the following least squares trend equation for
sales, in thousands of pairs, based on five years quarterly data from 2009.
Y 3.30  1.75x
The following table gives the seasonal variations (SV) for each quarter.

Quarter
Qtr 1 Qtr 2 Qtr 3 Qtr 4
SV 1.10 1.20 0.80 0.90

Forecast the sales for each quarter of 2014.

6 A shop is open every day except Fridays and Sundays. The number of customers visiting the shop
each day for three weeks is show below

Week Day Customers


1 Monday 160
Tuesday 170
Wednesday 140
Thursday 180
Saturday 280
2 Monday 170
Tuesday 180
Wednesday 140
Thursday 190
Saturday 300
3 Monday 200
Tuesday 210
Wednesday 180
Thursday 230
Saturday 340

a) Find a five-day moving average trend


b) Using the multiplicative model, estimate the seasonal factors for each of the five days.
c) Seasonally adjust the number of customer for the days from wee 1 to week 3.
d) Forecast the number of customers for each day of the fourth week

255

BUSINESS MATHS & STATISTICS (TC3)


CHAPTER 17 INDEX NUMBERS
LEARNING OBJECTIVES

By the end of this chapter the student should be able to

i. Explain what an index number is


ii. Distinguish between base year and current year.
iii. Construct single item indices (price and quantity).
iv. Identify different types of index numbers.
v. Change the base of an index number
vi. Measure changes in economic data using indices
vii. Calculate the Laspeyres and Paasche Indices and explain the difference
viii. To adjust nominal money values into real terms (taking inflation into account)
ix. Explain how index numbers can be used in practice.

17.0 INTRODUCTION

In business, managers may be concerned with the way in which the values of most variables change over
time: prices paid for raw materials; numbers of employees and customers, annual income and profits, etc.
Index numbers are one way of describing such changes.

Index numbers were originally developed by economists for monitoring and comparing different groups of
goods. It is necessary in business to be able to understand and manipulate the different published index
series, and to construct your own index series.

17.1 DEFINITION

Index numbers are numbers which are used to measure changes in economic data over a period of time.

Index numbers measure the changing value of a variable over time in relation to its value at some fixed point
in time, the base period, when it is given the value of 100.
Such indexes are often used to show overall changes in a market, an industry or the economy. For example,
an accountant at a supermarket chain could construct an index of the chain's own sales and compare it to the
index of the volume of sales for the overall supermarket industry.

Examples of other economic data for which index numbers may be suitable include:
x Consumer prices
x Production levels
x Imports and exports
x Prices of shares

17.2 EXAMPLES OF INDEX NUMBERS (OR INDICES).

Several types of index numbers exist depending on the type of data whose changes are being
measured. Examples include:

x Consumer index numbers

256

BUSINESS MATHS & STATISTICS (TC3)


x Production index numbers
x Quantity index numbers
x Wholesale index numbers
x Share price index numbers

A typical consumer price index (CPI) would appear as follows

Table 17.1 Index numbers


Years Index
2000 100
2001 101
2002 109
2003 114
2004 121
2005 129

2000 = 100

Since the numbers are rising as it can be said that the data whose changes the index is measuring has an is
rising. The index (CPI) is about consumer prices, it can be said that consumer prices are rising.

Note: The notation 2000 = 100 means that the base year (the reference point) is 2000.

17.3 CONSTRUCTING AN INDEX


value in current period
Index for any time period n = u 100
value in base period
Thus an index number is basically a ratio of two values one for the current period and the other at some
reference point called base period (see below) and expresses as a percentage without showing the
percentage sign.

17.4 SINGLE ITEM INDICES

Single item index numbers are index numbers that measure changes in data relating to a single item,
The data could be price or quantity. A single item index is called a relative. A relative is actually the
basic index.

If the single item index is about price of the item, then it is called a price relative. It if relates to
quantities it is a quantity relative.

17.4.1 Price relatives


A price relative is the ratio of a current price to that of a base period.
If p1 denotes the current price and p 0 the base period price

p1
A price relative is calculated as: u 100
p0

257

BUSINESS MATHS & STATISTICS (TC3)


Example 1

The average price of bread in Malawi for the past three years has been as follows

Year 2009 2010 2011

Price (MK) 120 150 210

Calculate the price relatives for 2010 and 2011 with 2009 as base year (i.e. 2000 = 100)

Solution

Table 17.2 Calculation of price relatives


Year 2009 2010 2011

p0 p1 p1
Price (MK) 120 150 210

Price Relative P1 x 100 P1 x 100


P0 P0

150 x 100 210 x 100


120 120

100.0 125.0 175.0

The price relatives are: Year Relative


2009 100
2010 125.0
2011 175.0

17.4.2 Quantity Relatives


Like a price relative, a quantity relative is calculated as:
ொଵ
‫ͲͲͳݔ‬ Where Q1 = current quantity and
ொଶ
Q0 = base period quantity

Example 2

KK Confectionaries Ltd bought the following quantities of flour for her baking.

Month Qty of flour (Kgs)

July 11
A 8
BUSINESS MATHS & STATISTICS (TC3)
Calculate the quantity relatives with July = 100

Solution

Table 17.3 Calculation of quantity relative


Month July Aug Sept Oct

Q0 Q1 Q1 Q1
Quantity 11 8 12 16

Quantity Relative Q 1 x 100 Q 1 x 100 Q 1 x 100


Q0 Q0 Q0

150 x 100 210 x 100


120 120

100.0 72.7 109.1 145.5

Note that the drop in quantity of August is reflected in that the August relative is lower than that of July.

17.4.3 Finding values from Indexes

For index linked values (eg. salaries, prices of products), if we know all the index numbers and one of the
index linked values we can find all the other values by scaling accordingly.

Example 3
Table 17.4 Monthly price index for an item:

Month Jan Feb Mar Apr May June July Aug Sep Oct Nov Dec
Index 121 112 98 81 63 57 89 109 131 147 132 126

If the prices are index linked and if the price of a product in March is K240, what is its price in the other
months?

Price in March is K240


121
Ÿ Price in January is K 240 u K 296 .30
98

112
Ÿ Price in February is K 240 u K 274 .30
98

For the other months, the prices are calculated in a similar way and we summarise them, to the nearest
Kwacha, in the following table.

Table 17.5 Indices and estimated prices


259

BUSINESS MATHS & STATISTICS (TC3)


Month Jan Feb Mar Apr May June July Aug Sep Oct Nov Dec
Index 121 112 98 81 63 57 89 109 131 147 132 126
Price (K) 296 274 240 198 154 140 218 267 320 360 323 309

17.4.4 Changing the base period


The choice of base period can be any convenient time - it is periodically adjusted.

It is usual to update the base period when:


x Any significant change which makes comparison with earlier figures meaningless.
x The numbers are growing so large that the size of a points change is many times that of a percentage
change.

We need to know how to change a base period.


Published index numbers may include base changes: a series 470, 478, 485 continues 100, 103, 105, etc.
A common base year is needed before analysis as a continuous series can be undertaken.

In order to change from one index series to another we need values for both indexes in one period. The
ratio of these two values forms the basis of any conversion between them.

Formulae:
New Old
New Old u and Old New u
Old New

Example 4
The advertising expenditure by a supermarket is index linked and is described by the following indexes:

Table 17.6 Indices with different bases


Year 1 2 3 4 5 6 7 8

Index 1 100 138 162 196 220

Index 2 100 125 140 165

Adverts (K) 4860

Complete each index series, then, given the expenditure for year 3, calculate the expenditure for the other
years.

Solution
a) Base year for each index:
Index 1: Year 1 while Index 2: Year 5

260

BUSINESS MATHS & STATISTICS (TC3)


b) Missing values for Index 1 and Index 2: (Both values known for year 5)

Year Index11 220 2 3 increased


4 5 6 7 8
all values
Index 2 100
Index 1 100 138 162 196 220 275 308 363
Index 2 100
Index 2 45.5 62.7 all 73.6
values reduced
89.1 100 125 140 165
Index 1 220
Adverts (£) 3000 4140 4860 5880 6600 8250 9240 10890
Index 1 for Year 6
220 220
Index 1 = Index 2 u 125 u 275 .0
100 100

Index 2 for Year 1


100 100
Index 2 = Index 1 u 100 u 45 .45
220 220

a) Advertising expenditure is calculated for all years from the known year using the ratio of the relevant index
numbers from both series.

If advertising cost £4860 in year 3, that for:


100 45 .45
Year 1 = 4860 u 3000 or 4860 u 3000
162 73 .64
138 62 .73
Year 2 = 4860 u 4140 or 4860 u 4140
162 73 .64

Table 17.7 Linked indices

17.5 MULTI-ITEM INDICES


In the previous section, we dealt with single item indices (Relatives). In practice index numbers involve
many items. For example a consumer price index measures changes in prices of consumer goods. Consumer
goods may include various food stuffs, clothing, beverages or even transport services. The idea is that the
changes in the prices of the items must be measured by a single series of numbers which means these prices,
must be combined.

Techniques of combining the items include those where weights (see below) are used and those where
weights are not used.

17.5.1 Un-weighted Indices


An Un-weighted multi-item index can be calculated using any of the following methods.

i) The arithmetic mean of the price or quantity relatives


ii) Using the geometric mean of the relatives
iii) By calculation a simple aggregate index

We now look at each one of them using an example.


261

BUSINESS MATHS & STATISTICS (TC3)


i) Using the arithmetic mean of the relatives.

Example 5

KK confectionaries Ltd produces and sells four sizes of party cakes: the mini, standard, family and super.
The prices and quantities sold in the past 2 years are as follows:

Table 17.8 Price and quantities of selected commodities

Item 2009 2010


Price (K) Qty (’00) Price (K) Qty (’00)
Mini 10 20 12 15
Standard 20 40 25 25
Family 30 30 30 50
Super 50 10 55 10

Calculate a multi-item price index for 2010 taking 2009 as base year using

a) The arithmetic mean of the price relatives


b) The geometric mean of the price relatives.
c) The simple aggregate index

Solution

Let Q1 denote the current quantity and Q0 denote the base period quantity.
Let P1 denote the current price and P0 denote the base period price.

Table 17.9 Calculation table (Price relatives


Item 2009 2010 Price relatives

P0 Q0 P1 Q1
Mini 10 20 12 15 (12/10) x 100 = 120
Standard 20 40 25 25 (25/20) x 100 = 125
Family 30 30 30 50 (30/30) x 100 = 100
Super 50 10 55 10 (55/50) x 100 = 110

a) The arithmetic mean of the price relatives is

120  125  100  110


113 .75
4

b) The geometric mean of the price relatives is

4
120 u 125 u 100 u 110 113.34

c) Simple aggregate index is


262

BUSINESS MATHS & STATISTICS (TC3)


¦P n
u 100 =
12  25  30  55
u 100 = 110.91
¦P 0 10  20  30  50

17.5.2 Weighted indices


Concept of a weight

A weight is a figure that reflects the relative importance of a data value. Weights are important in that when
data items (prices or quantities) are being combined in a calculation, the weights reflect the importance of
that item. The larger the weight, the more the influence of that item will be in a sum.

Example 6

Jim and Zack are being assessed on the basis of there grades in Maths and Chichewa for a post of Accounts
Assistant. There grades are as follows.

Candidate Grade
Mathematics Chichewa
Jim 50% 95%

Zack 75% 55%

Work out a normal arithmetic mean.

a) Calculate a weighted average giving Mathematics a weight of 8 and Chichewa a weight of 2 (the
meaning here being) Mathematics is 4 times as important as Chichewa in the accounts work).
b) Which is the appropriate way of selecting the candidates?

Solution

a) Table 17.10 Calculation table - Arithmetic mean


Candidate Grade Arithmetic mean
Maths Chichewa
Jim 50% 95% (50 + 95)/2 = 73%

Zack 75% 55% (75 + 55)/2 = 65%


In this case Jim gets the job but indications are that he will struggle to perform as accounting
requires numeracy skills indicated by Mathematics

263

BUSINESS MATHS & STATISTICS (TC3)


b) Table 17.11 Calculation table Weighted mean
Grade Weighted mean
Maths Chichewa
Weights 8 2
Jim 50% 95% (50 x 8 + 95 x 2) 59%
8 + 2

Zack 75% 55% (50 x 8 + 95 x 2) 71%


8 + 2

c) The weight of 8 has amplified the grade in Mathematics. As long as the weighting is acceptable
this is the better way of selecting the candidates. Zack gets the Job.

Example 7

If the weights for KK confectionaries products are:

Mini 6
Standard 10
Family 8
Super 5

Calculate the 2010 index using:

a) a weighted mean of the price relatives


b) a weighted quantity index

Solution
Table 17.12 Calculation table- Mean of Price relatives
Item Price relatives Weight

Mini (12/10) x 100 = 120 6


Standard (25/20) x 100 = 125 10
Family (30/30) x 100 = 100 8
Super (55/50) x 100 = 110 5

Index = 120 x 6 + 125 x 10 + 100 x 8 + 110 x 5


6 + 10 + 8 + 5

= 114.48

a) A weighted index

A weighted price index is calculated as follows: I


¦ P W u 100 , where W
n
is a weight
¦PW 0

264

BUSINESS MATHS & STATISTICS (TC3)


A weighted quantity index is calculated as

¦ Q W u 100
n

¦Q W 0

Table 17.13 Weighted indices


Item 2009 2010 Weight
P0 Q0 Pn Qn

Mini 10 20 12 15 6
Standard 20 40 25 25 10
Family 30 30 30 50 8
Super 50 10 55 10 5

The weighted price index = (12 x 6 + 25 x 10 + 30 x 8 + 55 x 5) x 100


(10 x 6 + 20 x 10 + 30 x 8 + 50 x 5)

= 837 x 100
750

= 111.6

b) A weighted quantity index is calculated in a similar manner

Determination of weights

The weights cited in these examples may have resulted from an agreement or from an “educated guess”. In
reality when a price index is being calculated, the quantities become the weights and when a quantity index
is being calculated it is the prices that become the weights.

Prices and quantities occur both in the base year as well as current year. The question is which prices or
quantities are used as weights. Base or the current year quantities or prices may be used as weights.

When base year prices or quantities are used as weights the index
is called a LASPEYRES INDEX. While if current year weights are
used, the index is called the PAASCHE index.

17.5.3 The Laspeyres index


The Laspeyres Price index is calculated as follows:

¦P Q n 0
u 100 , where base year Quantities Q0 are the weights
¦P Q 0 0

265

BUSINESS MATHS & STATISTICS (TC3)


The Laspeyres Quantity is calculated as follows:

¦Q P n 0
u 100 , where now base year prices P0 are the weights.
¦Q P 0 0

Example 8

Use the KK Confectionaries data below to calculate the Laspeyres price and quantity index for 2010.

Table 17.14 Prices and quantities of confectionaries


Item 2009 2010
Price (K) Qty (’00) Price (K) Qty (’00)
Mini 10 20 12 15
Standard 20 40 25 25
Family 30 30 30 50
Super 50 10 55 10

Solution
Table 17.15 Laspeyres index
Po Qo Pn Qn PoQo PnQo QnPo

Mini 10 20 12 15 200 240 150


Standard 20 40 25 25 800 1000 500
Family 30 30 30 50 900 900 1500
Super 50 10 55 10 500 550 500
Totals 2400 2690 2650

a) The Laspeyres price index 2690 x 100


2400

= 112.08

b) The Laspeyres quantity index 2650 x 100


2400

= 110.42

17.5.4 The Paasche index

As stated in 17.5.3 a Paasche index is an index which uses current year weights. Like the case of Laspeyres
it is one can calculate a price or a quantity index.

The Paasche Price index =


¦P Qn n
u 100
¦P Q0 n

266

BUSINESS MATHS & STATISTICS (TC3)


The Paasche Quantity index =
¦P Qn n
u 100
¦P Qn 0

Example 9

Using the same figures relating to KK confectionaries, Calculate the Paasche price and quantity index for
2010

Solution

Table 17.16 Paasche Index


Item 2009 2010 Working
Po Qo Pn Qn PoQn PnQn QoPn

Mini 10 20 12 15 150 180 240


Standard 20 40 25 25 500 625 1000
Family 30 30 30 50 1500 1500 900
Super 50 10 55 10 500 550 550
Totals 2650 2855 2690

a) The Paasche price index = 2855 x 100


2650

= 107.74

b) The Paasche quantity index = 2855 x 100


2690

= 106.13

17.6 USES OF INDEX NUMBERS

Index numbers are very useful in measuring the relative changes in the value of money. They are very
helpful for the guidance and formulation of economic policies. Index numbers of imports, exports, wages,
and employment and population importance cannot be ignored.

The following are some of the main uses of index numbers:

a) Index numbers act as economic barometers

A barometer is an instrument that is used to measure atmospheric pressure. Index numbers are used to feel
the pressure of the economic and business behaviour, as well as to measure ups and downs in the general
economic condition of a country. For example, the composite index number of indexes of prices, industrial
output, foreign exchange reserves, and bank deposits, could act as an economic barometer.

267

BUSINESS MATHS & STATISTICS (TC3)


b) Importance for governments

In the industrialized world, an index number such an the Consumer Price Index (CPI) is the most widely
used measure of inflation and is sometimes viewed as an indicator of the effectiveness of government
economic policy. It provides information about price changes in a country’s economy to government,
business, labor, and private citizens and is used by them as a guide to making economic decisions.
Governments use trends in the CPI to aid in formulating fiscal and monetary policies.

c) Consumption standard

If we want to know the true consumption standard of a class in a locality we can compute the consumption
index number.

d) Fixation of wages

The money wages can be revised according the proportionate change in the cost of living. The cost of living
index number guides the government and the executives for the fixation and revision of wages. Employees
also use index numbers, such CPI, for wage negotiation.

f) Importance for the producer

Price index numbers may provide an indication to the producer whether to increase or reduce production or
If price level is rising it may suggest increasing profits.

g) Analysis of industry

If we want to judge the prospects of manufacturing concern the investment index number can be
constructed, to know the net yield of the industrial sector.

h) Comparison of developed and under developed Countries

International price index number can be used for comparing the general level of prices in the developed and
under developed countries.

i) Efficiency of labour
 In order to check the efficiency and per capita output of labour, index can be used to show this can be
shown by index number. Promotion and salary can be also considered keeping in view the index number.

j) Measures to remove inequality of income

Index numbers of wholesale prices also indicate about the regional disparity. So different measures can be
taken for the proper distribution of wealth and stabilizing of prices.

268

BUSINESS MATHS & STATISTICS (TC3)


CHAPTER SUMMARY

This chapter has covered the following under index numbers


x In general, index numbers provide a measure of change of a phenomenon from one time point,
referred to as base time, to another time point called current time.
x In particular, a quantity index shows how a quantity changes over a period of time while a price
index shows how the price of a product changes over a period of time.
x The chapter illustrated how an index is constructed and distinguished single item indices and multi-
item indices.
x In particular, the chapter introduced the idea of weighted indices. Two popular weighted indices
were explored: Laspeyres and Paasche index.
x When base year prices or quantities are used as weights the index is called a Laspeyres Index. While
if current year weights are used, the index is called the Paasche index.

END OF CHAPTER EXERCISES

1. a) What is an index number?


b) Give four examples of index numbers.

2. a) Explain why it may be necessary to change the base in index calculations.


b) Describe any three uses of index numbers.

3. A company buys four products with the following characteristics:


Number of units bought Price paid per unit (£)
Items Year 1 Year 2 Year 1 Year 2
A 20 24 10 11
B 55 51 23 25
C 63 84 17 17
D 28 34 19 20

a) Find the simple price indexes for the products for year 2 using year 1 as the base year.

b) Find the simple aggregate index for year 2 using year 1 as the base year.

c) Find the base-weighted aggregate index, the Laspeyres index, for year 2 using year 1 as the base
year.
d) Find the current period-weighted aggregate index, the Paasche index, for year 2 using year
1 as the base year.

269

BUSINESS MATHS & STATISTICS (TC3)


4. a) Change the base of the following index from year 2003 to year 2005:
Year Index
2003 100
2004 116
2005 120
2006 130
2007 145

(b) A factory produces 3 types of agricultural equipment: tractors, ploughs and balers. The following
table shows the prices of the 3 items and the quantities produced for the 2 years, 2008 and 2009:

Year 2008 2009


Items Price per Quantity Price per Quantity
Unit (Km) unit (Km)
Tractor 45 300 55 250
Plough 7 500 6 600
Baler 3 200 2 250

Required

(i) Calculate an unweighted aggregate price index for the factory’s agricultural equipment for
2009, using 2008 as the base year.
(ii) Calculate an unweighted geometric mean of price relatives for 2009, using 2008 as the base
year.
(iii) Construct a Laspeyres price index for 2009, using 2008 as the base year.
(iv) Construct a Paasche price index for 2009, using 2008 as the base year.

5. The Following information was recorded on the prices and consumption of tea coffee and
chocolate drinks

Year 0 Year 1 Year 2


Drink Price Qty Price Qty Price Qty

Tea 8 15 12 12 16 10
Coffee 15 3 17 3 18 4
Chocolate 22 1 23 3 24 5

Suppose the price is taken as the average amount paid in Kwacha for a cup and the quantity as the
average number of cups drunk per person per week.

Required

a) Using year 0 as the base year, determine the simple aggregate price index for
i) Year 1 and
ii) Year 2

b) using year 0 as base year, determine the average price relatives. for
i) Year 1 and
270

BUSINESS MATHS & STATISTICS (TC3)


ii) Year 2

Calculate the Laspeyres price index for year 1 only using year 0 as base year

(a) The following table shows an index of the annual number of advertisements placed by an
organization in the press and the index of the number of organizations products sold per
annum.

1988 1989 1990 1991 1992 1993 1994


NumberofAdvertplaced(1990=100) 90 96 100 115 128 140 160
Valueofsales(1980=100) 340 347 355 420 472 515 572
Required:

Rebase the index of volume of sales to 1990 and compare the two sets of relatives.

(b) The following table shows the quantities sold and total revenue on three key food items (P,
Q, and R) sold by a chain of supermarkets between 2007 and 2009.

ITEM 2007 2008 2009


Quantity Totalrevenue Quantity Totalrevenue Quantity Totalrevenue
(K'm) (K'm) (K'm)
P 2250kg 2.50 2500KG 2 3200kg 3.10
Q 12000litres 12.15 14500litres 15.00 15200litres 16.10
R 1250items 5.25 14500litres 5.30 1650items 5.55
Required:
(i) Construct a Laspeyre price index for the food items for 2008 and 2009 using 2007 as base
year.

(ii) Construct a Paasche quantity index for the food items for 2008 and 2009 using 2007 as base
year

Describe four uses of index numbers in practice.


.

BUSINESS MATHS & STATISTICS (TC3)


CHAPTER 18 FINANCIAL MATHEMATICS
LEARNING OBJECTIVES

By the end of this chapter, the student should be able to:

i. Distinguish between simple and compound interest


ii. Calculate interest, principal or period in given various combinations of parameters
iii. Should be able to compound at shorter periods and calculate the effective interest rate
iv. Describe the different techniques of depreciation.
v. Depreciate an asset using the various depreciation techniques
vi. Appraise an investment using pay back period
vii. Appraise an investment using Net Present Values
viii. Appraise an investment using Internal Rate of Return
ix. Compare the various techniques of investment appraisal
x. Calculate the maturity value of an annuity
xi. Calculate the fixed equal payment of annuity given the maturity value
xii. Define a sinking fund
xiii. Describe amortisation as a method of debt repayment

18.0 INTRODUCTION
Financial Mathematics is a collection ofmathematical techniques that find applications in Finance related
areas. This chapter looks at.

x Basic calculations on interests


x Investment decisions
x Calculation issues in asset management (e.g. of values of assets and depreciation)
x Loan issues

This text considers basic mathematical techniques useful in many computations, looks at the concepts of
interest,depreciation, capital appraisal and annuities.

18.1 INTEREST
Introduction
In the context of finance, interest is amount one earns when a sum of money is invested in a bank account or
otherwise. The term can also be used to mean what is paid over and above a sum of money one borrows.
The amount which is borrowed or invested is called the principal.

Either way there are two ways of dealing with (calculating) interest. These are:

a) Simple interest
b) Compound interest

18.1.1 Simple interest


Simple interest is interest calculated on a fixed principal amount.

For a principal P, invested at a rate of R% per annum, over T years, simple interest I is given by

272

BUSINESS MATHS & STATISTICS (TC3)


ܴܲܶ
‫ܫ‬ൌ
ͳͲͲ

Example 1

How much interest would K10,000 earn at 8% per annum simple interest over 15 years?

Solution: 
ܴܲܶ
‫ܫ‬ൌ
ͳͲͲ
௄ଵ଴ǡ଴଴଴ൈଵହൈ଼

ଵ଴଴

ൌ ͳʹǡͲͲͲ

Example 2

Jamadi wants to earn K500,000 in interest so that she can have enough money to buy a good used car. She
puts K800,000 into an account that earns 6.5% p.a. simple interest. How long will she need to leave her
money in the account to earn the K500,000 interest?

Solution

The question is about obtaining time (T) required for the principal to earn interest of K500,000.
௉்ோ
Making T subject of the formula ‫ ܫ‬ൌ , we obtain
ଵ଴଴

‫ ܫ‬ൈ ͳͲͲ
ܶൌ
ܲൈܴ
௄ହ଴଴ǡ଴଴଴ൈଵ଴଴

௄଼଴଴ǡ଴଴଴ൈ଺Ǥହ

ൌ ͻǤ͸‫ݏݎܽ݁ݕ‬

Example 3
A man invests 10,000 in a bank account for 3 years at 5% simple interest. After the 3 years he is enticed to
put all the money back into the account because the bank has offeres him 10% p.a. simple interest. If he
does not withdraw any money, calculate the total amount he will have in the account after a further 3 years.

Solution
This is a problem of calculating interest two times:
Accrued amount (A) after the first 3 years:
A= P+Interest
௉்ோ
ൌܲ൅
ଵ଴଴
‫Ͳͳܭ‬ǡͲͲͲ ൈ ͵ ൈ ͷ
ൌ ‫Ͳͳܭ‬ǡͲͲͲ ൅
ͳͲͲ
= K11,500

273

BUSINESS MATHS & STATISTICS (TC3)


Accrued amount the after a further 3 years:
௄ଵଵǡହ଴଴ൈଷൈଵ଴
‫ ܣ‬ൌ ‫ͳͳܭ‬ǡͷͲͲ ൅
ଵ଴଴
=K14,500

18.1.2 Compound interest

While simple interest calculates the interest on afixed principal period after period, compound interest arises
when interest earned in one period is added to the principal, so that the interest that has been added also
earns interest.

Example 4

Calculate total interest and the amount in the account earned when K1000 is invested at 10% compound
interest for 4 years.

Table 18.1 Calculation table – compound interest

Year Principal Rate Interst AmountinAccount


r P*i I+Pi

0 1000 0.1 100 1100

1 1100 0.1 110 1210

2 1210 0.1 121 1331

3 1331 0.1 133.1 1464.1

4 1464.1 0.1 146.41 1610.51

Compound interest formula and calculation

Assuming a sum P is invested in an account at a rate of r% for t years, the accrued (sometimes called
the terminal) amount A in the account after t years is given by:

‫ ܣ‬ൌ ܲሺͳ ൅ ሻ௧
ଵ଴଴

Example 5

K300,000 is invested for 4 years at 10% compound interest. Find the terminal value and interest gained after
the 4 years.

Solution

Terminal value: ‫ ܣ‬ൌ ܲሺͳ ൅ ሻ௧
ଵ଴଴

274

BUSINESS MATHS & STATISTICS (TC3)


ଵ଴
ൌ ‫ͲͲ͵ܭ‬ǡͲͲͲሺͳ ൅ ሻସ
ଵ଴଴
= K439,230

Compound Interest: ‫ ܫ‬ൌ‫ܣ‬െܲ


ൌ ‫ܭ‬Ͷ͵ͻǡʹ͵Ͳ െ ‫ͲͲ͵ܭ‬ǡͲͲͲ
=K139,230

Example 6

Kuganizira Ltd sells company vehicles to the officers using the vehicles when the car is 4 years old. The car
is sold to the employee at 10% the purchase cost. Charles is using one such vehicle which cost the company
Mk8,500,000 to buy and he is looking forward to buying it in three year from now. In order not to miss the
chance, he has been advised to set aside some money for the purpose. How much should he invest into an
account now to buy the vehicle if the fund he is to use pays 12% compound interest per annum.

Solution.

The problem is about finding the principal.


ଵ଴
ܲ‫ ݁ܿ݅ݎܲ݁ݏ݄ܽܿݎݑ‬ൌ ‫ܭ‬ͺǡͷͲͲǡͲͲͲ ൈ
ଵ଴଴
= K850,000

‫ ݎ‬௧
‫ ܣ‬ൌ ܲሺͳ ൅ ሻ
ͳͲͲ

Making P subject of the equation we obtain

‫ܣ‬
ܲ ൌ ௥
ሺͳ ൅ ሻ௧
ଵ଴଴

௄଼ହ଴ǡ଴଴଴
ൌ భమ య
ሺଵା ሻ
భబబ

=K605,013.21

Charles should invest K605,013.20 now.

Example 7

Ethel left K150,000 in her account at the bank, while going on study in the UK two years ago. She has come
back and has found a total amount of K174,960.00. What was the rate of interest assuming the rate never
changed in the period.

275

BUSINESS MATHS & STATISTICS (TC3)



Solution ‫ ܣ‬ൌ ܲሺͳ ൅ ሻ௧
ଵ଴଴

௥ ೟ ஺
ൌ ට െͳ
ଵ଴଴ ௉
ଵ଻ସǡଽ଺଴
ൌට െͳ
ଵହ଴ǡ଴଴଴

ൌ ͲǤͲͺ
Rate of interest= 8%

Example 8

Wonga has identified a fund which pays a good 14% compound interest. She has K450,000 which she can
invest. For how long should she invest the money to realise a total of K900,000 to buy a dream plot..

Solution:
The problem concerns working out the period t

௥ ௧
‫ ܣ‬ൌ ܲ ቀͳ ൅ ቁ
ଵ଴଴
‫ܣ‬ ‫ݎ‬
‫ ݃݋ܮ‬൬ ൰  ൌ ‫ ݃݋ܮݐ‬ቀͳ ൅ ቁ
ܲ ͳͲͲ
ͻͲͲǡͲͲͲ ͳͶ
‫ ݃݋ܮ‬൬ ൰  ൌ ‫ ݐ‬ൈ ‫ ݃݋ܮ‬൬ͳ ൅ ൰
ͶͷͲǡͲͲͲ ͳͲͲ

Ž‘‰ሺʹሻ
‫ ݐ‬ൌ
Ž‘‰ሺͳǡͳͶሻ

=5.3 years

Compounding at shorter periods

Apart from the annual basis, compounding can be at shorter periods like half-year, quarterly, monthly,
weekly, or even daily.

When compounding is done more frequently than once a year, it earns even higher interest amounts. As it
will be demonstrated in the example below, the shorter the compounding period in the year, the larger the
amount relatively.

Note:

In general if an amount P is invested under compound interest which is done n times in year at a rate of r%
per annum, over t years, then the accrued amount is given by

‫ݎ‬Τ ௡௧
݊
‫ ܣ‬ൌ ܲ ቆͳ ൅ ቇ
ͳͲͲ

BUSINESS MATHS & STATISTICS (TC3)


Example 9

300 000 is invested at 12% per annum. Find the amount at the end of 4 years if compounding is:

a) annually
b) ½ year
c) quarterly
d) monthly

Solution:
௥ ௧
a) ‫ ܣ‬ൌ ܲ ቀͳ ൅ ቁ
ଵ଴଴
ൌ ‫ͲͲ͵ܭ‬ǡ ͲͲͲሺͳǤͳʹሻସ = K472 055.81

ଵଶൗ ଶൈସ

b) ‫ ܣ‬ൌ ‫ͲͲ͵ܭ‬ǡͲͲͲ ൬ͳ ൅ ൰
ଵ଴଴
ൌ ‫ͲͲ͵ܭ‬ǡͲͲͲሺͳǤͲ͸ሻ଼ = K478,154.42

c) ‫ ܣ‬ൌ ‫ͲͲ͵ܭ‬ǡͲͲͲሺͳǤͲ͵ሻଵ଺ = K481,411.93

d) Aൌ ‫ͲͲ͵ܭ‬ǡͲͲͲሺͳǤͲͳሻସ଼ = KͶͺ͵ǡ͸͸͹Ǥͺʹ

The effective rate of interest(ERI)

Compound interest results in a higher terminal value because of the fact that interest earns further interest
when it is added back to the principal. The given annual rate is called the nominal rate and the rate final rate
resulting from compounding is called the effective rate of interest sometimes also called the Actual
percentage Rate (APR) .

Example 10

Given a nominal rate of r=12% per annum, find the effective interest rate ifcompounding isdone

a) Quarterly
b) Monthly for one year

Solution

a) ‫ ݁ݐܽݎ݈ܽݑ݊݊ܣ‬ൌ ͳʹΨ
ଵଶ
ܳ‫ ݁ݐܽݎݕ݈ݎ݁ݐݎܽݑ‬ൌ  Ψ ൌ ͵Ψ

‫ ݁ݐܽݎ݁ݒ݅ݐ݂݂ܿ݁ܧ‬ൌ  ሾሺͳǤͲ͵ሻସ െ ͳሿΨ =12.55%


ଵଶ
When compounding monthly ‫ ݁ݐܽݎݕ݈݄ݐ݊݋ܯ‬ൌ Ψ
ଵଶ
ൌ ͳΨ

‫݁ݐܽݎ݁ݒ݅ݐ݂݂ܿ݁ܧ‬ൌ ሾሺͳǤͲͳሻଵଶ െ ͳሿΨ


=12.68%

277

BUSINESS MATHS & STATISTICS (TC3)


18.2 DEPRECIATION
Any asset purchased and is being used, will get worn out or lose its value at a certain rate during its lifetime.
In business, depreciation is the term used to mean the part or proportion of the asset being consumed away
or the loss in value. For example a bus bought by Axa now at say K20,000,000 will be worth only
12,000,000 in 3 years time due to wear and tear. The 8,000,000 decline in value is the depreciation over the
3 years.

Note that when depreciation is allowed for (charged), the value of the asset or loan amount declines. The
resulting value is called The Net Book Value (NBV).

There are several techniques which can be used to calculate depreciation. These include

i. Straight line depreciation


ii. Sum of digits
iii. Reducing balance

18.2.1 Straight line depreciation


Straight line depreciation is when the value of an asset is reduced by fixed equal amounts over the life of the
asset.

The amount of depreciation charged each year under straight line depreciation,, also called the annual
depreciation value, is calculated as follows
ܱ‫ ݁ݑ݈ܸܽ݇݋݋ܤ݈ܽ݊݅݃݅ݎ‬െ ܵܿ‫݁ݑ݈ܸܽ݌ܽݎ‬
‫ ݁݃ݎ݄ܽܥ݊݋݅ݐܽ݅ܿ݁ݎ݌݁ܦ‬ൌ
ܷ‫݂݁݅ܮ݈ݑ݂݁ݏ‬ሺ‫ݏݎݕ‬ሻ‫ݐ݁ݏݏܣ݂݋‬

Scrap value is the residual value of an asset after its useful period. It is usually measured by how much it
can fetch if sold after its useful life.

Example 11

A machine needs to be depreciated from K25,000 to K5000 over a period of 5 years using straight line
method. Find:

a) Depreciation charge per annum


b) The net book value at the end of 3 years

Solution:
ை௥௚௜௡௔௟஻௢௢௞௏௔௟௨௘ିௌ௖௥௔௣௏௔௟௨௘
a) ‫ ݁݃ݎ݄ܽܥ݊݋݅ݐܽ݅ܿ݁ݎ݌݁ܦ‬ൌ
௎௦௘௙௨௟௅௜௙௘ሺ௬௥௦ሻ௢௙஺௦௦௘௧

௄ଶହǡ଴଴଴ି௄ହ଴଴଴
ൌ =K4000.00 per annum

b) Net book value at the end of year 3:

NBV=Original Book Value – (Depreciation charge p.a. x number of years of depreciation)

=25,000 – 4000 x 3
=25,000 – 12,000
=K13,000.00
278

BUSINESS MATHS & STATISTICS (TC3)


18.2.2 Sum of digits

The “sum of digits” technique of calculating depreciation uses the digits of the years (i.e. year 1, 2, 3, 4, etc)
of the useful life of the asset in reverse as weights to calculate depreciation charge for the years. If the asset
is to be depreciated over 3 years, the first year gets a weight of 3, the second 2 and the last gets 1. This way
depreciation is high in the early years of an asset.

Example 12

Based on example 11above calculate the depreciation charges for each yearand then calculate the NBV.

Solution: Digits: 1
2
3
4
5_
Sum of digits: 15

Depreciation charges (assign digits in reverse as the weights)

Table 18.2 calculation table – sum of digits depreciation


Year Weight DepChargefortheyear NBV
1 5 ('5/15)x20,000 = 6,666.67 25000Ͳ6666.67= 18,333.33
2 4 (4/15)x20,000 = 5,333.33 18,333.33Ͳ5,333.33= 13,000.00
3 3 (3/15)x20,000 = 4,000.00 9,000.00
4 2 (2/15)x20,000 = 2,666.67 6,333.33
5 1 (1/15)x20,000 = 1,333.33 5,000.00

18.2.3 Reducing balance method


Under reducing balance method depreciation for each year is a proportion of the net book value as at the
beginning of that year. The proportion or rate remains fixed unless deliberately changed

For an asset with an Original Book Value B, depreciating at a rate of r%, the depreciated value or NVB of
the asset at the end of the ݊௧௛ year is given by:

‫ ݎ‬௡
ܰ‫ ܸܤ‬ൌ ‫ ܤ‬ቀͳ െ ቁ
ͳͲͲ

௥ ௥ ௡ିଵ
And the depreciation charge D for the ݊௧௛ year is ‫ ܦ‬ൌ ‫ ܤ‬ൈ ൈ ቀͳ െ ቁ
ଵ଴଴ ଵ଴଴

Example 13

A main frame computer costing K220,000 will depreciate to a scrap value of K12000 in 5 years

a) If reducing balance method is used find the depreciation rate?

279

BUSINESS MATHS & STATISTICS (TC3)


b) What is the book value of the computer at the end of the 3rd year? What is the depreciation charge
for the year?
c) How much more would the book value be at the end of the 3rd year if we use straight line method?

Solution:
௥ ௡
a) ܰ‫ ܸܤ‬ൌ ‫ ܤ‬ቀͳ െ ቁ
ଵ଴଴

‫ ݎ‬ହ
‫ʹͳܭ‬ǡͲͲͲ ൌ ʹʹͲǡͲͲͲ ቀͳ െ ቁ 
ͳͲͲ

ఱ ଵଶ଴଴଴
‫ ݎ‬ൌ ൬ͳ െ ට ൰ ൈ ͳͲͲ = 44.11
ଶଶ଴ǡ଴଴

Discounting rate=44.11%

b) At the end of the third year:

ସସǤଵଵ ଷ
ܰ‫ ܸܤ‬ൌ ‫Ͳʹʹܭ‬ǡͲͲͲ ቀͳ െ ቁ = K38,408.29
ଵ଴଴

c) If straight line had been used


௄ଶଶ଴ǡ଴଴଴ି௄ଵଶǡ଴଴଴
d) ‫ ݁݃ݎ݄ܽܥ݊݋݅ݐܽ݅ܿ݁ݎ݌݁ܦ‬ൌ ൌ ‫ܭ‬Ͷͳǡ͸ͲͲ

At the end of the third year ܰ‫ ܸܤ‬ൌ ‫Ͳʹʹܭ‬ǡͲͲͲ െ ͵ ൈ ‫ܭ‬Ͷͳǡ͸ͲͲ =K95,200

The NBV would have been ‫ͻܭ‬ͷǡʹͲͲ െ ‫͵ܭ‬ͺǡͶͲͺǤʹͻ ൌ ͷ͸͹ͻͳǤ͹ͳ more if straight line method had been
used.

18.3 DISCOUNTING

Discounting is connected to the concept of time value of money. Time value of money refers to the fact that
the same amount of money occurring at different time points will have different values or will purchase
different quantities of similar goods due to among many factors, inflation. Specifically, Money looses value
with time. Theidea that money available at the present time is worth more than the same amount in the
future due to its potential earning capacity makes it difficult to compare sums of money realised at different
times. This concept is referred to as time value of money. Discounting helps to determine this time value of
money.

Definition

Discounting is the process of measuring how much a future sum of money is worth now. In other words,
discounting is the process for finding the present value of money.

Note that discounting is the reverse of compounding.

280

BUSINESS MATHS & STATISTICS (TC3)


Given a future sum A, discounted a rate r% for n periods, its Present Value is given by:
‫ܣ‬
ܸܲ ൌ
௥ ௡
ቀͳ ൅ ቁ
ଵ଴଴

Example 14
Mrs Kalima expects to earn K500,000 from her maize seed project this time next year. Find the present
value of the earnings if the rate of interest on the market is 10%

Solution


Present Value: ܸܲ ൌ ೝ ೙
ቀଵା ቁ
భబబ

௄ହ଴଴ǡ଴଴଴
ൌ భబ భ
ൌ ‫ܭ‬ͶͷͶǡ ͷͶͷǤͶͷ
ቀଵା ቁ
భబబ

Example 15
Mr. Kalima intends to sell his pickup 3 years from now for about K700,000. Advise him as to how much
that sum is worth now if the rate of interest is 8%

‫ܭ‬͹ͲͲǡͲͲͲ
ܸܲ ൌ ଷ

ቀͳ ൅ ቁ
ଵ଴଴
ൌ ‫ܭ‬ͷͷͷǡ ͸ͺʹǤͷ͹

Example 16
A departmental store advertises goods at K70,000.00 deposit and 3 further equal annual paymentsof
K50,000.00. If the discount rate is 8%, calculate the present value of the goods.

Solution
Year 0 1 2 3
| | | |
Cash flow 70,000 50,000 50,000 50,000
(payments)

௄ହ଴ǡ଴଴଴ ௄ହ଴ǡ଴଴଴ ௄ହ଴ǡ଴଴଴


ܸܲ ൌ ‫ܭ‬͹ͲǡͲͲͲ൅ൌ భబ భ
൅ൌ భబ మ
൅ൌ భబ య
=K198, 854.85
ቀଵା ቁ ቀଵା ቁ ቀଵା ቁ
భబబ భబబ భబబ

Discounting Factors
A discounting factor the is the present value of one unit currency (One Kwacha in our case)

i.e. ‫ ݎ݋ݐ݂ܿܽ݃݊݅ݐ݊ݑ݋ܿݏ݅ܦ‬ൌ ೝ ೙
ቀଵାభబబቁ

281

BUSINESS MATHS & STATISTICS (TC3)


ቀ ቁ
భబబ


ൌ‫ܣ‬ ೝ ೙
ቀଵା ቁ
భబబ

Thus ܸܲ ൌAൈ ‫ݎ݋ݐ݂ܿܽ݃݊݅ݐ݊ݑ݋ܿݏ݅ܦ‬

Example 17
Use discounting factors to calculate the Present values of the cash flow in example 30 above

Solution

Table 18.3 Calculation table – Present values

Year Net Cash flow Discounting factor Present


(K) (DF 8%) Value (K)
0 ͺ ଴
70,000 ൬ͳ ൅ ൰ ൌ ͳǤͲͲͲͲ 70,000
ͳͲͲ
1 ͺ ଵ
50,000 ൬ͳ ൅ ൰ ൌ ͲǤͻʹͷͻ 46,296.30
ͳͲͲ
2 ͺ ଶ
50,000 ൬ͳ ൅ ൰ ൌ ͲǤͺͷ͹͵ 42,866.94
ͳͲͲ
3 ͺ ଷ
50,000 ൬ͳ ൅ ൰ ൌ ͲǤ͹ͻ͵ͺ 39,691.61
ͳͲͲ

While discounting factors can be calculated on a calculator using the formula presented above, tables of
already calculated discounting factors are available as shown below. Note that the factors are calculated to
three decimal places only.

282

BUSINESS MATHS & STATISTICS (TC3)


Table 18.4 Discounting factors
RATE ValueofK1inyearn
1 2 3 4 5 6 7 8 9 10 11
4.0% 0.962 0.925 0.889 0.855 0.822 0.790 0.760 0.731 0.703 0.676 0.650
5.0% 0.952 0.907 0.864 0.823 0.784 0.746 0.711 0.677 0.645 0.614 0.585
6.0% 0.943 0.890 0.840 0.792 0.747 0.705 0.665 0.627 0.592 0.558 0.527
7.0% 0.935 0.873 0.816 0.763 0.713 0.666 0.623 0.582 0.544 0.508 0.475
8.0% 0.926 0.857 0.794 0.735 0.681 0.630 0.583 0.540 0.500 0.463 0.429
9.0% 0.917 0.842 0.772 0.708 0.650 0.596 0.547 0.502 0.460 0.422 0.388
10.0% 0.909 0.826 0.751 0.683 0.621 0.564 0.513 0.467 0.424 0.386 0.350
11.0% 0.901 0.812 0.731 0.659 0.593 0.535 0.482 0.434 0.391 0.352 0.317
12.0% 0.893 0.797 0.712 0.636 0.567 0.507 0.452 0.404 0.361 0.322 0.287
13.0% 0.885 0.783 0.693 0.613 0.543 0.480 0.425 0.376 0.333 0.295 0.261
14.0% 0.877 0.769 0.675 0.592 0.519 0.456 0.400 0.351 0.308 0.270 0.237
15.0% 0.870 0.756 0.658 0.572 0.497 0.432 0.376 0.327 0.284 0.247 0.215
16.0% 0.862 0.743 0.641 0.552 0.476 0.410 0.354 0.305 0.263 0.227 0.195
17.0% 0.855 0.731 0.624 0.534 0.456 0.390 0.333 0.285 0.243 0.208 0.178
18.0% 0.847 0.718 0.609 0.516 0.437 0.370 0.314 0.266 0.225 0.191 0.162
19.0% 0.840 0.706 0.593 0.499 0.419 0.352 0.296 0.249 0.209 0.176 0.148
20.0% 0.833 0.694 0.579 0.482 0.402 0.335 0.279 0.233 0.194 0.162 0.135
21.0% 0.826 0.683 0.564 0.467 0.386 0.319 0.263 0.218 0.180 0.149 0.123
22.0% 0.820 0.672 0.551 0.451 0.370 0.303 0.249 0.204 0.167 0.137 0.112
23.0% 0.813 0.661 0.537 0.437 0.355 0.289 0.235 0.191 0.155 0.126 0.103
24.0% 0.806 0.650 0.524 0.423 0.341 0.275 0.222 0.179 0.144 0.116 0.094
25.0% 0.800 0.640 0.512 0.410 0.328 0.262 0.210 0.168 0.134 0.107 0.086
26.0% 0.794 0.630 0.500 0.397 0.315 0.250 0.198 0.157 0.125 0.099 0.079
27.0% 0.787 0.620 0.488 0.384 0.303 0.238 0.188 0.148 0.116 0.092 0.072
28.0% 0.781 0.610 0.477 0.373 0.291 0.227 0.178 0.139 0.108 0.085 0.066
29.0% 0.775 0.601 0.466 0.361 0.280 0.217 0.168 0.130 0.101 0.078 0.061
30.0% 0.769 0.592 0.455 0.350 0.269 0.207 0.159 0.123 0.094 0.073 0.056
31.0% 0.763 0.583 0.445 0.340 0.259 0.198 0.151 0.115 0.088 0.067 0.051
32.0% 0.758 0.574 0.435 0.329 0.250 0.189 0.143 0.108 0.082 0.062 0.047
33.0% 0.752 0.565 0.425 0.320 0.240 0.181 0.136 0.102 0.077 0.058 0.043
34.0% 0.746 0.557 0.416 0.310 0.231 0.173 0.129 0.096 0.072 0.054 0.040
35.0% 0.741 0.549 0.406 0.301 0.223 0.165 0.122 0.091 0.067 0.050 0.037
36.0% 0.735 0.541 0.398 0.292 0.215 0.158 0.116 0.085 0.063 0.046 0.034
37 0% 0 730 0 533 0 389 0 284 0 207 0 151 0 110 0 081 0 059 0 043 0 031
BUSINESS MATHS & STATISTICS (TC3)
18.4 CAPITAL (INVESTMENT) APPRAISAL

Capital appraisal (or investment appraisal) is process used to determine whether long term investments
such as new machinery, replacement of machinery, new plants, new products, and research development
projects are worth pursuing.

Many formal methods are used in capital appraisal and the common ones include:

x The payback period


x The Net present value (NPV)
x Internal rate of return
x Accounting rate of return
x Return on capital employed

This text will only consider the first three.

18.4.1 The pay back method

Payback period refers to the period of time required for the return on an investment to "repay" the sum of
the original investment. For example, a K200,000 investment which returned K50000 per year would have a
four year payback period. Payback period intuitively measures how long something takes to "pay for itself."
In essence, shorter payback periods are preferable to longer payback periods.

Example 18

A business project is being considered which requires K240,000 initial capital outlay. Net revenues over the
following 4 years of K80,000.00, K70,000 , K50,000.00 and K65000.00 respectively.

Calculate the payback period state whether the project is worthwhile if the expected payback period is three
years or less.

Solution

Table 18.5 Calculation table - payback


Year Cash flow (K) Balance (K)
0 (240,000.00) (240,000.00)
1 80,000.00 (160,000.00)
2 70,000.00 (90,000.00)
3 50,000.00 (40,000.00)
4 65,000.00 25,000.00
At the end of 3 years the project has K40,000.00 to be “repaid”

Hence the project will pay back the initial capital in

‫ܭ‬ͶͲǡͲͲͲ ൈ ͳʹ݉‫ݏ݄ݐ݊݋‬
‫ ݏݎܽ݁ݕ‬൅ ൌ ͵‫݀݊ܽݏݎܽ݁ݕ‬͹ǤͶ݉‫ݏ݄ݐ݊݋‬
‫ܭ‬͸ͷǡͲͲͲ

The project is not viable as its payback period exceeds the desired one (3 years)
284

BUSINESS MATHS & STATISTICS (TC3)


Advantages of payback period

Payback period has the following advantages:

i) It is simple to calculate
ii) It is easy to understand
iii) It is practical (It uses actual cash flows) the payback period itself can be set to reflect reality.

Disadvantage of payback period

The technique has one major disadvantage and that is it ignores the time value of money.

18.4.2 The net present value of cash flows


A project is expected to have cash outflows and cash inflows. A net present value (NPV) of a project is the
sum of the present values of all cash flows associated with the project.

A project would be considered worthwhile if the net present value is positive.

Example 19
Consider the example 33 and decide whether or not the project is worthwhile using the NPV technique
given the cost of capital on the market is 13%

Solution

Table 18.6 Calculation table – present values


Year Cash flow df Present Value
0 (240,000.00) 1.000 (240,000.00)
1 80,000.00 0.885 70,796.46
2 70,000.00 0.783 54,820.27
3 50,000.00 0.693 34,652.51
4 65,000.00 0.613 39,865.72
NPV (39,865.05)

The project’s NPV is negative hence it is not a viable investment.

Example 20
Chikondi is buying a machine costing K120,000 for a small scale business. In order to run it he will spend
K85,000 in the first year, K30,000 in the second, k15,000 in the third year and a further K15,000 in the
fourth year. Revenues expected from the business are K80,000, K120,000, K100,000 and K65,000 in the 1 st,
2nd, 3rd, 4th years respectively. If the cost of capital is at 15% per annum, advise Chikondi as to whether the
business is viable.

285

BUSINESS MATHS & STATISTICS (TC3)


Solution
Table 18.7 calculation table NPV
Year Cash inflows Cash outflow Net cash flows DF (15%) Present value
0 120,000.00 (120,000.00) 1.000 (120,000.00)
1 80,000.00 85,000.00 (5,000.00) 0.870 (4,347.83)
2 120,000.00 30,000.00 90,000.00 0.756 68,052.93

3 100,000.00 15,000.00 85,000.00 0.658 55,888.88


4 65,000.00 15,000.00 50,000.00 0.572 28,587.66

NPV 28,587.66
NPV is positive hence the business is viable.

18.4.3 The Internal Rate of Return (IRR)


The internal rate of return is a discounting rate which would give an NPV of Zero.
When calculated the decision on whether to proceed with the project or not is determined by comparing the
IRR with the known desired rate of interest. If this rate is higher than the cost of capital or desired rate, the
project is considered viable.

Techniques for calculating the IRR


The internal rate of return can be calculated using either algebraic or the graphical techniques

The algebraic method

Example 21
A person invests K500 000 in a project. If the project gives a net inflow of K700 000 for 1 year only, find
the IRR. Using the IRR decide if the project is worthwhile given a cost of capital of 12% on the market.

Solution
The IRR is a value r% which gives an NPV of zero

PV of inflows – PV of outflows =0 or NPV = 0

‫ܭ‬͹ͲͲǡͲͲͲ
ଵ െ ‫ܭ‬ͷͲͲǡͲͲͲ ൌ Ͳ

ቀͳ ൅ ቁ
ଵ଴଴

଻଴଴ǡ଴଴଴ ௥
ൌͳ൅
ହ଴଴ǡ଴଴଴ ଵ଴଴
‫ݎ‬
ൌ ͳǤͶ ൌ ͳ ൅
ͳͲͲ
‫ ݎ‬ൌ ͶͲ

IRR=40%.

Since the cost of capital is the IRR is greater than the cost of capital(12%) then the project is worthwhile.

Therefore IRR =40%. The rate on the market is 12% project worthwhile.

286

BUSINESS MATHS & STATISTICS (TC3)


Example 22
A project requires an initial investment of MK500,000 and yields MK300,000 and MK250,000 in the 1st and
2nd years respectively. Calculate the IRR and comment on the project considering the rate of interest on the
market is 20%.

Solution:

If the IRR is r%
Therefore NPV (at r%)=0

‫ͲͲ͵ܭ‬ǡͲͲͲ ‫ʹܭ‬ͷͲǡͲͲͲ
െͷͲͲǡͲͲͲ ൅ ௥
൅ ଶ ൌͲ

ቀͳ ൅ ቁ ቀͳ ൅ ቁ
ଵ଴଴ ଵ଴଴

‫ ݎ‬ଶ ‫ݎ‬
െͳͲ ቀͳ ൅ ቁ ൅ ͸ ቀͳ ൅ ቁ൅ͷൌͲ
ͳͲͲ ͳͲͲ

If we let݅ ൌ
ଵ଴଴

Then െͳͲሺͳ ൅ ݅ሻଶ ൅ ͸ሺͳ ൅ ݅ሻ ൅ ͷ ൌ Ͳ

െͳͲ െ ʹͲ݅ െ ͳͲ݅ ଶ ൅ ͸ ൅ ͸݅ ൅ ͷ ൌ Ͳ

ͳͲ݅ ଶ െ ͳͶ݅ െ ͳ ൌ Ͳ

ିଵସേඥଵସ మ ିሺସൈଵ଴ൈିଵሻ
` ݅ൌ
ଶൈଵ଴

݅ ൌ ͲǤͲ͸ͺ‫ ݎ݋‬െ ͳǤͶ͸ͺ


‫ݎ‬
ൌ ͲǤͲ͸ͺ
ͳͲͲ

Hence r=6.8

IRR= 6.8% which is less than the rate of interest on the market hence the project is not viable.

The algebraic method works better for projects that do not go beyond 2 years. For projects that go beyond 2
years it is recommended to use the graph or formula to work out the IRR.

The graphical technique

Procedure:

x Choose any two different discount rates different (neither of them should equal the desired discounting rate)
x Draw a pair of axes (NPV against discounting rate). Plot two sets of points (rate and corresponding NPV)
on a graph.

287

BUSINESS MATHS & STATISTICS (TC3)


x Draw a line through the two points and note the point where the graph cuts the discount rate axis. The value
of the discount rate coordinate for this point is the IRR

Example 23

Calculate the IRR for the project in example 36 above.

Solution:

Table 18.8 Calculation table – IRR workings


Year Net cash flows DF(6%) PV DF(7%) PV
0 (500,000.00) 1  (500,000.00) 1 (500,000.00)
1 300,000.00 0.94339623 283018.8679 0.93457944 280,373.83
2 250,000.00 0.88999644 222499.11 0.87343873 218,359.68
NPV 5517.977928 Ͳ1266.48616

Figure 18.1 IRR


NPVK'000

2
IRR=6.8%
1

0
6 7 Rate(%)
Ͳ1

Ͳ2

Using formula
The internal rate of return can be calculated as follows

Let a = first interest rate of interest


A = the NPV corresponds to a
b = the second rate
B = the NPV corresponding to b

஺ሺ௕ି௔ሻ
Then ‫ ܴܴܫ‬ൌ ܽ ൅
ሺ஺ି஻ሻ

288

BUSINESS MATHS & STATISTICS (TC3)


Example 24
Calculate the IRR for problem in example 36 using the formula.

Solution

From the solution of example 37, a = 6%; A = 5517.98


b = 7%; B = -1266.49

18.5 ANNUITIES

An annuity is a sequences of fixed amounts of at regular intervals.

18.5.1 Classification of annuities

Based on time of payment/receipt


- Ordinary annuity-an annuity in which payments are made at the end of time interval
- Due annuity-an annuity in which payments are made at the beginning of the time interval
Based on term of the annuity
- Certain annuity- dates of beginning and ending of the annuity are fixed
- Contingent annuity- annuity whose end date depends on some uncertain event to occur
Perpetuity an annuity which goes on indefinetely

Example 25
George is buying a plasma screen worth K460,000 and has agreed to pay a deposit of K100,000 and the
balance in 12 equal monthly instalments. Identify the annuity part.

Solution.
Balance after the deposit = K460,000 – 100,000 = 360,000
Instalment: = 360,000/12
= 30,000 per month

The annuity is the sequence of the K30,000s payable every month.

18.5.2 Areas of application


A lot of payments or receipts occur in form of annuities. Good examples are pension payments over time,
insurance premium payments, settlement of consumer goods bills inform of instalments for a number of
months.

Of interest are the future value to which these sums accumulate and of course given a number of payments,
their net present value. Further analyses also deal with the problems of determining how much payments
should made be to accumulate to a desired value.
oThe basic tools in dealing with annuities include the geometric progression, compounding and present
289

BUSINESS MATHS & STATISTICS (TC3)


The basic tools in dealing with annuities include the geometric progression, compounding and present
values.

18.5.3 Future (maturity) value of an annuity


There are basically two methods for calculating the future value of an annuity.

a) Creating an annuity schedule which shows the value of the fund period by period taking into
account new payments at each period and all accumulating interests.
b) Considering accrued value of each payment as a term of a geometric progression (GP) and using
the formula of sum of terms a GP to find the total maturity value of the payments

Example 26
Mr. Yona is investing a fixed amount of K50,000 into a fund every year in advance for 5 years. If the fund
earns an interest of 6.5% per annum:

a) Construct a schedule to show the value of the fund at the end of each year.
b) Use an appropriate formula to calculate the value of the fund at the end the five years.

Solution

a) Table 18.9 Calculation table Annuity


Year Payment into Amount at start Interest Amount in
the fund of year in the the fund (end
fund of year)
1 50,000.00 50,000.00 3,250.00 53,250.00
2 50,000.00 103,250.00 6,711.25 109,961.25
3 50,000.00 159,961.25 10,397.48 170,358.73
4 50,000.00 220,358.73 14,323.32 234,682.05
5 50,000.00 284,682.05 18,504.33 303,186.38

The amount at the start of the year = The payment for that year plus the cumulated amount at the end of
the previous year.

The amount at the end of the year =The amount at the start of the year plus interest earned in the year.

The schedule shows the value of the fund at the end of each year.

b) The future value of the annuity is in form of a sum of terms of geometric progression. Each fixed
payment made into the fund will earn interest for the period it is invested.

If A denotes the fixed payments, i = interest rate:

Future value
=‫ܣ‬ሺͳ ൅ ݅ሻହ ൅ ‫ܣ‬ሺͳ ൅ ݅ሻସ ൅ ‫ܣ‬ሺͳ ൅ ݅ሻଷ ൅ ‫ܣ‬ሺͳ ൅ ݅ሻଶ ൅ ‫ܣ‬ሺͳ ൅ ݅ሻଵ

Rearranged we have: ‫ܣ‬ሺͳ ൅ ݅ሻଵ ൅ ‫ܣ‬ሺͳ ൅ ݅ሻସ ൅ ‫ܣ‬ሺͳ ൅ ݅ሻଷ ൅ ‫ܣ‬ሺͳ ൅ ݅ሻଶ ൅ ‫ܣ‬ሺͳ ൅ ݅ሻଵ

290

BUSINESS MATHS & STATISTICS (TC3)


This is a sum of a geometric progression:

௔ሺ௥ ೙ ିଵሻ
The sum of the first n terms of a GP ൌ
௥ିଵ
1st term ܽ ൌ ‫ܣ‬ሺͳ ൅ ݅ሻ
Common ratio ‫ ݎ‬ൌ ͳ ൅ ݅
A = 50,000, therefore ƒ ൌ ͷͲǡͲͲͲšሺͳǤͲ͸ͷሻ
ൌ ͷ͵ʹͷͲ

i = 0.065 r = 1 + 0.065 = 1.065

ହଷǡଶହ଴ൈଵǤ଴଺ହఱ ିଵ
The value after 5 yearsൌ
ଵǤ଴଺ହିଵ

=K303,186.38

18.5.4 Present value of an annuity


Apart from the future value of an annuity another important concept is that of the present value of the
annuity. The calculation can involve discounting factors or another formula derived from the geometric
progression formula.

Calculation (discounting factors)

Assuming the same stream of fixed amounts A set aside for n years, the present value (actually the net
present value) of these amounts can be shown to be

‫ܣ‬ ‫ܣ‬ ‫ܣ‬ ‫ܣ‬


ܲൌ ൅ ଶ
൅ ଷ
൅ ‫ڮ‬൅
ሺͳ ൅ ݅ሻ ሺͳ ൅ ݅ሻ ሺͳ ൅ ݅ሻ ሺͳ ൅ ݅ሻ௡

Factoring out A we have


ͳ ͳ ͳ ͳ
ܲ ൌ ‫ܣ‬൤ ൅ ଶ
൅ ଷ
൅ ‫ڮ‬൅ ൨
ሺͳ ൅ ݅ሻ ሺͳ ൅ ݅ሻ ሺͳ ൅ ݅ሻ ሺͳ ൅ ݅ሻ௡

The part in brackets is simply cumulative discounting factor which can be obtained for a calculator or
standard tables.

Example 27

A man will earn K200,000 in rentals at the beginning of each year for five years. Find the present value of
the rentals if the cost of capital is 8%.

ଵ ଵ ଵ ଵ
Solution ܲ ൌ ‫ܣ‬ቂ ൅ ൅ ൅ ‫ڮ‬൅ ቃ
ሺଵା௜ሻ ሺଵା௜ሻమ ሺଵା௜ሻయ ሺଵା௜ሻ೙

A=K200,000; i=K0.08

let the present value be P 


ͳ ͳ ͳ ͳ ͳ
ܲ ൌ ʹͲͲǡͲͲͲ ൤ ଴
൅ ଵ
൅ ଶ
൅ ଷ
൅ ൨
ሺͳǤͲͺሻ ሺͳǤͲͺሻ ሺͳǤͲͺሻ ሺͳǤͲͺሻ ሺͳǤͲͺሻସ

291

BUSINESS MATHS & STATISTICS (TC3)


ൌ ʹͲͲǡͲͲͲ ൈ ͶǤ͵ͳʹ
=K862,400

Not that the rentals are at the start of the year and therefore the first figure has discounting factor = 1
because it is at Present or “now”

Using geometric progression

If viewed a geometric progression point of view:

‫ܣ‬ ‫ܣ‬ ‫ܣ‬ ‫ܣ‬


ܲൌ ൅ ൅ ൅ ‫ڮ‬ ൅
ሺͳ ൅ ݅ሻ ሺͳ ൅ ݅ሻଶ ሺͳ ൅ ݅ሻଷ ሺͳ ൅ ݅ሻ௡

஺ ଵ
Then ܽൌ ‫ݎ‬ൌ
ሺଵା௜ሻ ሺଵା௜ሻ

Example 28
A lady is promised a fixed monthly salary of K15,000 for the next 5 years following her injury which
incapacitated her. Find the present value of the monthly salaries if the rate of interest on the market is 6%
per annum

Solution
This is a case for a geometric progression particularly because of the many terms involved (60) and very
few tables can give that many discounting factors). Salaries are paid in arrears.

͸
‫ ݁ݐܽݎݕ݈݄ݐ݊݋ܯ‬ൌ Ψ
ͳʹ

=0.5%
The first term
ͳͷǡͲͲͲ
ܽൌ
ሺͳǤͲͲͷሻ
ൌ ‫ͳܭ‬ͶͻʹͷǤ͵͹


The common ratio ‫ ݎ‬ൌ  ൌ ͲǤͻͻͷ
ଵǤ଴଴ହ

The number of terms n = 5 x 12 = 60

௔ሺଵି௥ ೙ ሻ
Present value ܲൌ
ଵି௥

‫ͳܭ‬ͶͻʹͷǤ͵͹ ൈ ሺͳ െ ͲǤͻͻͷ଺଴ ሻ

ͳ െ ͲǤͻͻͷ

ൌ ‫ܭ‬͹͹ͷ͵ͶͲǤʹ͸

18.5.5 The present value of a perpetuity.


As stated above a perpetuity is an annuity which goes on indefinitely.

292

BUSINESS MATHS & STATISTICS (TC3)


For such an annuity the geometric formula with n = ’ can be used

Example 29
A mineral prospecting company discovers rare earth mineral whose belt runs into the home of Mr.
Chimbuzi,s family land. To be able to extract the mineral they must negotiate a payment in order to have the
Chimbuzi’s family relocate. They agree on a payment of K100,000 to the family at the beginning of each
year for as long as they mine the mineral. Experts estimate that the mineral deposits are in abundance that
they cannot estimate the years it will last. Estimate the present value of the payments to the Chimbuzi’s if
interest rate is 7%.

Solution
It can be assumed that the payment will run for an indefinite period.
Therefore n = ’.

The formula for a sum of GP into perpetuity is
ଵି௥

ܽ ൌ ‫ͲͲͳܭ‬ǡͲͲͲ
ͳ
‫ݎ‬ൌ ൌ ͲǤͻ͵Ͷ͸
ͳǤͲ͹
௄ଵ଴଴ǡ଴଴଴
ܲ‫ ݁ݑ݈ܸܽݐ݊݁ݏ݁ݎ‬ൌ
ଵି଴Ǥଽଷସ଺

ൌ ‫ͳܭ‬ǡͷʹͻǡͲͷͳǤͻͻ

18.5.6 Amortization and sinking fund.


Amortisation is the term used to mean reducing or wiping out a debt or a sum of money over time through
an annuity. When a debt has been wiped out by amortisation it is said to be amortised.

The amount A necessary to amortise a debt P over n years at an interest rate i=r% is given by.

ܲ݅
‫ܣ‬ൌ
ͳ െ ሺͳ ൅ ݅ሻି௡

Example 29
A company borrows K5, 000, 000 with interest at 5% compounded 6-monthly is amortised by equal semi-
annual payments over the next three years. Find the value of each semi-annual payment.

Solution
Let the semi-annual payment be A

ܲ݅
‫ܣ‬ൌ
ͳ െ ሺͳ ൅ ݅ሻି௡

‫ܭ‬ͷǡͲͲͲǡͲͲͲ ൈ ͲǤͲʹͷ
ൌ
ͳ െ ሺͳǤͲʹͷሻି଺

= K907,749.86
293

BUSINESS MATHS & STATISTICS (TC3)


A sinking fund is really just an annuity set aside to help repay a debt or buy an asset. Typically, one is
required to make periodic payments into the fund and normally these payments would earn an interest. The
payments would be made in such away as that future value would be sufficient to pay the debt or for the
asset.

Example 30
Chiko wants to buy a car estimated to cost MK850,000 car in 18 months time. The plausible way to save
money for it is to set aside a certain amountof money on a monthly basis and invest it at an interest bearing
account, The account possible is one at a bank paying an interest rate of 12% per annum
compoundedmonthly.

a) How much should Chiko be setting aside each month,

b) Prepare a sinking fund schedule for the first six months.

Solution a) Interest =12% pa; therefore monthly rate =12% ÷ 12 = 1%

Let the amount to be set aside be A

Assuming the amounts are invested at the start of each month, the future value,F, of
the streams of money is

‫ ܨ‬ൌ ‫ܣ‬ሺͳǤͲͳሻ ൅ ‫ܣ‬ሺͳǤͲͳሻଶ ൅ ‫ ڮ‬൅ ‫ܣ‬ሺͳǤͲͳሻଵ଼

This is a geometric a geometric progression and the future value is the sum of the n terms (number of
payments or instalments).

ܽሺ‫ ݎ‬௡ െ ͳሻ
‫ܨ‬ൌ
‫ݎ‬െͳ

where a = A(1.01), r = 1.01

but F = 850,000
ଵǤ଴ଵ஺ሺଵǤ଴ଵభ ିଵሻ
‫ܭ‬ͺͷͲǡͲͲͲ ൌ
ଵǤ଴ଵିଵ

ͺͷͲǡͲͲͲ ൈ ͲǤͲͳ
‫ܣ‬ൌ
ͳǤͲͳ ൈ ሺͳǤͲͳଵ଼ െ ͳሻ

A = 850,000 x (1.01 – 1)
1.01 x (1.0118 – 1)

= K42,905.68

(b) The sinking fund schedule is as follows

Table 18.9 Sinking fund

294

BUSINESS MATHS & STATISTICS (TC3)


0 42,905.68 Ͳ 42,905.68 42,905.68
1 42,905.68 429.06 43,334.74 86,240.42
2 42,905.68 862.40 43,768.09 130,008.51
3 42,905.68 1,300.09 44,205.77 174,214.28
4 42,905.68 1,742.14 44,647.83 218,862.11
5 42,905.68 2,188.62 45,094.30 263,956.41
6 42,905.68 2,639.56 45,545.25 309,501.66
7 42,905.68 3,095.02 46,000.70 355,502.36
8 42,905.68 3,555.02 46,460.71 401,963.07
9 42,905.68 4,019.63 46,925.31 448,888.38
10 42,905.68 4,488.88 47,394.57 496,282.95
11 42,905.68 4,962.83 47,868.51 544,151.46
12 42,905.68 5,441.51 48,347.20 592,498.66
13 42,905.68 5,924.99 48,830.67 641,329.33
14 42,905.68 6,413.29 49,318.98 690,648.31
15 42,905.68 6,906.48 49,812.17 740,460.48
16 42,905.68 7,404.60 50,310.29 790,770.77
17 42,905.68 7,907.71 50,813.39 841,584.16
18 8,415.84 8,415.84 850,000.00

CHAPTER SUMMARY

In this chapter we have looked at the following:

o Interest: Simple and Compound:


ƒ Definition of interest.
ƒ Calculation of simple interest.
ƒ Calculation of compound interest.
ƒ Effective rate of interest.
o Depreciation:
ƒ Definition of depreciation.
ƒ Depreciating assets using straight line, sum of digits and reducing balance methods of
depreciation.
o Discounting:
ƒ Definitions of a present value and discounting factor.
ƒ Calculation of present values of a future sum.
o Capital (investment) appraisal using:
ƒ Payback period.
ƒ Net Present Values (NPV).
ƒ Internal Rate of Return (IRR).
o Annuities including amortisation and sinking fund
ƒ Definition of an annuity.
ƒ Types of annuities.
ƒ Calculation of the maturity value of an annuity using a schedule.
ƒ Calculation of the maturity value of an annuity as a sum of a geometric progression.
ƒ Debt repayment using amortisation.
Sinking f nd as a method for debt repa ment and replacement of depreciated assets

BUSINESS MATHS & STATISTICS (TC3)


END OF CHAPTER EXERCISES
1. An employee, who received fixed annual increments had a final salary of K90,000 after 10 years. If
her total salary was K650,000 over the 10 years what was her initial salary?

2. A firm buys a power press for 32,500 which is expected to last for 20 years and to have a scrap value
of K7,500. If depreciation is on a straight line method how much is the deprecation charge per year.

3. Insert 5 numbers between 8 and 26 such that the resulting series is an AP

4. Which term in the GP 2, 4, 8, 16 is 1024

5. A local bank is advertising that you can double your money in eight years if you invest with them.
Suppose you have K2000 to invest. What interest (simple) rate is the bank offering

6. Wezi has K50,000 to invest, and two funds that she can use are available: one at Last Discount house
offering 14% simple interest and the other at Fist Discount House which pays 6% interest. Wezi only
needs to earn K4,500 in interest this year and she really wants to try both banks How much should
she put in each fund
7. A company sets up a sinking fund and investsK10,000.00 each year for 5 years a 9%. Compound
interest what will the fund be worth after 5 years?

8. Calculate the sum of the terms in the following sequence: 1, ½, ¼, 1/8, 1/16......

9. Mr. Banda’s company is considering an investment costing K5.6 million which would earn K1.6
million cash per annum for five years. The Company expects to make a return of at least 11% per
annum.

 Assess whether or not the project is viable.    

10. At the beginning of each year a company sets aside K2 million out of its profits to form a reserve
fund. This is invested at 10% p.a. compound interest. What will be the value of the fund after four
years?

11. On 1 January 2003, K0.2 million was invested. It remained invested and on 1 January of each
successive year, K0.1 million was added to it.What sum would have accumulated by 31
December 2007 if interest was compounded each year at 10% p.a.?

12. K4 million is borrowed from a building society, repayable over 20 years at 14% p.a. compound
interest.How much must be repaid each year?

296

BUSINESS MATHS & STATISTICS (TC3)


13. A firm has bought an asset worth K500,000 which has a life of 10 years where its scrap value will be
zero. As per its practice the firm will sell the asset at the end of the 5th year and it wishes to select a
depreciation technique, which will result in a lowest NBV at the time of selling in order not to
overvalue the asset. Calculate the NBV at the end of 5 years using the straight line, sum of digits and
reducing balance techniques and suggest the suitable technique, according to the firms wish.

14. Find the PV of a debt K2500.00 taken out over 4 years (with no intermediate payments) where the
borrowing rate is 12% and the worth of money (Discounting rate) is 9.5%. How much is the cost of
this debt?

15. Find the present value of 150,000 to be received in 5 years time. Rate of interest is 12% (use the
discounting factor method)

297

BUSINESS MATHS & STATISTICS (TC3)


CHAPTER 19 CALCULUS
OBJECTIVES

By the end of this chapter students should be able to

i. Differentiate functions up to the second derivative.


ii. Evaluate indefinite and definite integrals.
iii. Find minimum and maximum values of a given function.
iv. Apply calculus on revenue, cost and profit functions with the aim of finding optimum points.

19.0 INTRODUCTION

Calculus is a branch of mathematics which is the study of rate of change of functions.

Using calculus, one can study the rates of change of functions and interpret the results to determine when
functions are at their minimum or maximum. For example, we use differentiation to obtain the marginal
profit given the cost and revenue functions. The marginal profit function is then used to determine the level
of production that will maximize profit.

Calculus is generally divided into two parts: differential and integral calculus.

19.1 DIFFERENTIATION

19.1.1 The process of Differentiation


Given a function y = f(x) the rate of change of f(x) with respect to x is called the derivative of y. The
derivative of y = f(x) with respect to x is denoted by

ௗ௬
‫ ݕ‬ᇱ ǡ ݂ ᇱ ሺ‫ݔ‬ሻ or
ௗ௫

read as y prime, f prime of x and dee y by dee x respectively. The process of finding ‫ ݕ‬ᇱ is called
differentiation.

19.1.2 Rules for calculating derivatives


There are some rules which enable us to find derivatives quickly and easily.
1. Power Rule: If ‫ ݕ‬ൌ ܽ‫ ݔ‬௡ , then ݀‫ݕ‬Ȁ݀‫ ݔ‬ൌ ݊ܽ‫ ݔ‬௡ିଵ . Note that the original index “n” multiplies
into the function and the new index worked out as “n-1”.

2. Derivative of a constant: If ‫ ݕ‬ൌ ܿ, where c is a constant then ‫ ݕ‬ᇱ =0. That is, the derivative of a
constant is zero.

3. Addition rule: If ‫ ݕ‬ൌ ݂ሺ‫ݔ‬ሻ ൅ ݃ሺ‫ݔ‬ሻǡ then ‫ ݕ‬ᇱ = ݂ ᇱ ሺ‫ݔ‬ሻ ൅ ݃ᇱ ሺ‫ݔ‬ሻǤ That is, the derivative of a sum
is the sum of the derivatives of the individual terms.

Example 1 (General concept of differentiation)

Differentiate the following with respect to x


298

BUSINESS MATHS & STATISTICS (TC3)


a) ‫ ݕ‬ൌ  ‫ ݔ‬ଷ b) ‫ ݕ‬ൌ  ‫଺ ݔ‬ c) ‫ ݕ‬ൌ ‫ݔ‬

d) ‫ ݕ‬ൌ ͹‫ݔ‬ e) ‫ ݕ‬ൌ Ͷ‫ ݔ‬ଶ f) ‫ ݕ‬ൌ ͵‫ ݔ‬଴Ǥହ

Solution
a) ݀‫ݕ‬Ȁ݀‫ ݔ‬ൌ ͵‫ ݔ‬ଶ b) ݀‫ݕ‬Ȁ݀‫ ݔ‬ൌ ͸‫ ݔ‬ହ
c) ݀‫ݕ‬Ȁ݀‫ ݔ‬ൌ ͳ d) ݀‫ݕ‬Ȁ݀‫ ݔ‬ൌ ͹
e) ݀‫ݕ‬Ȁ݀‫ ݔ‬ൌ ͺ‫ݔ‬ f) ݀‫ݕ‬Ȁ݀‫ ݔ‬ൌ ͳǤͷ‫ି ݔ‬଴Ǥହ

Note that for f) the index is a fraction.

Example 2 Differentiate ‫ ݕ‬ൌ ͳʹ.

Solution ݀‫ݕ‬Ȁ݀‫ ݔ‬ൌ Ͳ.

Example 3 Differentiate with respect to x


a) ‫ ݕ‬ൌ  ‫ି ݔ‬ଶ b) ‫ ݕ‬ൌ ͶȀ‫ ݔ‬ଷ

Solution
a) ݀‫ݕ‬Ȁ݀‫ ݔ‬ൌ  െʹ‫ି ݔ‬ଷ
b) ‫ ݕ‬ൌ ͶȀ‫ ݔ‬ଷ  ൌ Ͷ‫ି ݔ‬ଷ , so ݀‫ݕ‬Ȁ݀‫ ݔ‬ൌ  െͳʹ‫ି ݔ‬ସ  ൌ  െͳʹȀ‫ ݔ‬ସ 

Example 4 Differentiate the following with respect to x

a) ‫ ݕ‬ൌ  ‫ ݔ‬ଷ  െ Ͷ‫ ݔ‬ଶ  ൅ ͳ͵‫ ݔ‬൅ ͳͲ b) ‫ ݕ‬ൌ  ‫ ݔ‬ସ Ȃ‫ି ݔ‬ଷ  ൅ ʹ‫ݔ‬

Solution a) ݀‫ݕ‬Ȁ݀‫ ݔ‬ൌ ͵‫ ݔ‬ଶ Ȃ ͺ‫ ݔ‬൅ ͳ͵


ௗ௬
b) ൌ Ͷ‫ ݔ‬ଷ െ ሺെ͵ሻ‫ି ݔ‬ସ  ൅ ʹ ൌ Ͷ‫ ݔ‬ଷ  ൅ ͵‫ି ݔ‬ସ  ൅ ʹ
ௗ௫

19.1.3 First and second derivatives

When an expression is differentiated for the first time, the result is known as the first derivative. If the result
is different from zero, it can be differentiated again to give the second derivative.

The 1st and 2nd derivative are denoted by the following symbols:

First derivative Second derivative


Usual symbol ݀‫ݕ‬Ȁ݀‫ݔ‬ ݀ ଶ ‫ݕ‬Ȁ݀‫ ݔ‬ଶ
Alternative ‫ݕ‬ᇱ ‫ ݕ‬ᇱᇱ

Example 5. Given ‫ ݕ‬ൌ ʹ‫ ݔ‬ଷ  ൅ ‫ ݔ‬ଶ  ൅ Ͷ‫ ݔ‬െ ͸,

݀‫ݕ‬Ȁ݀‫ ݔ‬ൌ ͸‫ ݔ‬ଶ  ൅ ʹ‫ ݔ‬൅ Ͷ is the first derivative of y,
ௗమ௬
and =‫ ݕ‬ᇱᇱ = 12x + 2 is the second derivative of y.
ௗ௫ మ

299

BUSINESS MATHS & STATISTICS (TC3)


19.1.4 Interpretation of differentiation
As stated in the introduction, calculus is concerned with the rate of change of a function.

By differentiating a function one actually finds the slope or gradient of that function. Recall that the slope of
a function ‫ ݕ‬ൌ ݂ሺ‫ݔ‬ሻ is the rate at which y is changing with respect to x.

Example 6

Differentiate the following functions and interpret the results

i) ‫ ݕ‬ൌ Ͷ‫ ݔ‬൅ ͵ ii) ‫ ݕ‬ൌ  ‫ ݔ‬ଶ  െ ‫ ݔ‬൅ Ͷ

Solution
i) ݀‫ݕ‬Ȁ݀‫ ݔ‬ൌ Ͷ This is the slope of the equation (confirm by considering the general linear equation y
= mx + c) where c is the slope.

ii) ݀‫ݕ‬Ȁ݀‫ ݔ‬ൌ ʹ‫ݔ‬Ȃ ͳ Because the original equation is not linear, the slope is not a constant but
depends on the value of x.

For example, when x = 3 the slope at that point will be 5 (substitution.)

19.1.5 Maximum and Minimum points


It is possible to investigate whether or not a function has a maximum or minimum point or both.

The process is as follows

Step 1 Find the first derivative‫ ݕ‬ᇱ or ݂ ᇱ ሺ‫ݔ‬ሻǤ

Step 2 Set the first derivative to zero and solve the resulting equation (‫ ݕ‬ᇱ ൌ Ͳ).
The roots of ‫ ݕ‬ᇱ ൌ Ͳ are called critical points.

Step 3 Find the second derivative of the original equation

Step 4 Substitute the values (critical points) found in step 2 into the second derivative.

Step 5 Interpreting the results:If the substitution result in step 4 is negative then the original
equation is maximum at the value found in step 2. If the substitution result is positive then the original
equation is at its minimum.

Note: We can use the mnemonic NEMAto help us remember that when the second derivative is NEgative at
a critical point then the function is MAximum at that point.

Example 7

Given the function ‫ ݕ‬ൌ ʹ‫ ݔ‬ଷ Ȃ ͵‫ ݔ‬ଶ  െ ʹͲ‫ ݔ‬൅ ʹͲ. Find the maximum and minimum points.

Solution

Following the steps outlined above the process of finding minimum and maximum points is as follows
300

BUSINESS MATHS & STATISTICS (TC3)


Step 1 Find the first derivative of the equation

dy/dx = 6x2 – 6x – 20

Step 2 Set the 1st derivative to zero and solve for x:

͸‫ ݔ‬ଶ  െ ͸‫ݔ‬Ȃ ʹͲ ൌ Ͳ. Dividing throughout by 3 gives ͵‫ ݔ‬ଶ െ ‫ݔ‬Ȃ ͳͲ ൌ ͲǤ This is a quadratic
equation and we solve using the formula:
ଷേξଽାଵଶ଴
‫ݔ‬ൌ and ‫ ݔ‬ൌ ʹǤ͵ͻor‫ ݔ‬ൌ െͳǤ͵ͻ

ௗమ௬
Step 3 Find the second derivative: ൌ ͳʹ‫ݔ‬Ȃ ͸.
ௗ௫ మ

Step 4 Find the values of the 2nd derivative at the critical points.

At‫ ݔ‬ൌ ʹǤ͵ͻǡ ‫ ݕ‬ᇱ ൌ ͳʹ‫ݔ‬Ȃ ͸ ൌ ͳʹሺʹǤ͵ͻሻ െ ͸ ൌ ʹʹǤ͸ͺ which is positive.


At‫ ݔ‬ൌ െͳǤ͵ͻ, ‫ ݕ‬ᇱ ൌ ͳʹ‫ݔ‬Ȃ ͸ ൌ ͳʹሺെͳǤ͵ͻሻ െ ͸ ൌ െʹʹǤ͸ͺ. This is negative.

Step 5 Interpreting and concluding

‫ ݕ‬ൌ ʹ‫ ݔ‬ଷ Ȃ ͵‫ ݔ‬ଶ  െ ʹͲ‫ ݔ‬൅ ʹͲ is at it’s minimum point when x =2.39and at its maximum
when x = -1.39

The actual minimum and maximum values of y are calculated by substituting x into the
original equation.

At ‫ ݔ‬ൌ ʹǤ͵ͻ, y = ʹ ൈ ʹǤ͵ͻଷ Ȃ ͵ ൈ ʹǤ͵ͻଶ െ ʹͲ ൈ ʹǤ͵ͻ ൅ ʹͲ=-17.63, so the minimum value


is െͳ͹Ǥ͸͵ and the maximum value is ͵͸Ǥ͸͵ which occurs when ‫ ݔ‬ൌ  െͳǤ͵ͻ.

19.1.6 Sketching Graphs


Take a look at fig. 19.1 given below:

301

BUSINESS MATHS & STATISTICS (TC3)


This graph is defined for values of x ranging from a to e and has turning points at b, c and d. The turning
points take either an ‫ ځ‬െ shape (read as ‘an n shape’) that is at x=b and x=d or a ‫ڂ‬Ȃshape (‘u shape’) for
example at x=c. We can use differentiation to tell where the graph of a function will either of the two
shapes.

Generally, a function will have a maximum value at a critical point x=a and hence an ‫ځ‬Ȃ shape if the
second derivative at that point is negative. Also, if the second derivative of a function at a given point x = c
is positive, then the function will have a minimum value at that point and the graph will assume the
‫ڂ‬Ȃshape.

Here is a list of steps of how the graph of y = f(x) can be sketched.

Step 1 Find the x intercept. This is done by solving the equation f(x)=0.

Step 2 Find the y interceptǤ This is done by finding f(0).

Step 2 Find the critical pointsǤ

Step 2 Use the second derivative to determine which critical points will give rise to a ‫ڂ‬Ȃshape or
‫ځ‬Ȃ shape.

Step 3 Plot the points and sketch the graph.

Example 8 Sketch the graph of f(x) = 2x3 + 3x2 - 36x.

Solution

Step 1 We solve 2x3 + 3x2 - 36x = 0 to obtain, x = 0, x = 3.56 and x = -5.06

Step 2 When x = 0, y = 0. This tells us that the graph will pass through the origin.

Step 2 f(x) =2x3 + 3x2 - 36x so ݂ ᇱ ሺ‫ݔ‬ሻ ൌ ͸š ଶ  ൅ ͸š െ ͵͸ from which the critical points are x=-3 and
x=2.

Step 2 ݂ ᇱᇱ ሺ‫ݔ‬ሻ ൌ ͳʹš ൅ ͸ and ݂ ᇱᇱ ሺെ͵ሻ ൌ ͳʹሺെ͵ሻ ൅ ͸ ൌ െ͵Ͳ ൏ Ͳ. So x=-3 is a maximum therefore the
graph has a ‫ځ‬Ȃ shape at x=-3.
At x=2, ݂ ᇱᇱ ሺʹሻ ൌ ͳʹሺʹሻ ൅ ͸ ൌ ͵Ͳ ൐ Ͳǡso x=2 is a minimum and so the graph has a ‫ڂ‬Ȃshape at x=2.

302

BUSINESS MATHS & STATISTICS (TC3)


Step 3 Plotting the points gives the graph below.

Figure 19.2 Minimum and maximum points

19.2 INTEGRATION

19.2.1 Introduction
If we are given a function y=f(x) we know how to find its derivative. In this section we are now going to turn things
around. We now want to ask the question, ‘what function did we differentiate in order to get a given function?’. For
example, we know that if ‫ ݕ‬ൌ ‫ ݔ‬ଶ then ‫ݕ‬Ԣ ൌ ʹ‫ݔ‬ǤThe question we would like to address is, can we find
a function whose derivative 2x? This question will be answered by carrying out an operation which will
undo the effects of differentiation. The reverse of differentiation is called integration.

Let’s go back to the question, can we find a function whose derivative 2x? It is easy to see that ‫ ݕ‬ൌ ‫ ݔ‬ଶ is
one such function. Also ‫ ݕ‬ൌ ‫ ݔ‬ଶ ൅ ʹ has its derivative equal to ʹ‫ݔ‬. In fact any function ‫ ݕ‬ൌ ‫ ݔ‬ଶ ൅ ܿ where c
is a constant has a derivative equal to 2x. We can see therefore that we need more information if we are to
determine a specific value of c.

Definition 19.2.1 Given a function, › ൌ ˆሺšሻ , an anti-derivative of ˆሺšሻ is any function ሺšሻ whose
derivative is ˆሺšሻ. That is, ᇱ ሺšሻ ൌ ˆሺšሻ.

If ሺšሻ is any anti-derivative of ˆሺšሻ, then the most general anti-derivative of ˆሺšሻ is called an indefinite
integral and is denoted,

‫ˆ ׬‬ሺšሻ†š ൌ ሺšሻ ൅ …, … is any constant.

In this definition the ‫׬‬Ȃsymbol is called the integral symbol, ˆሺšሻis called the integrand, x is called the
integration variable and the “c” is called the constant of integration. The †š part specifies the variable of
integration.

19.2.2 Rules of Integration


௫ ೙శభ
1. ‫ ݔ ׬‬௡ ݀‫ ݔ‬ൌ ௡ାଵ ൅ ܿǡ ݊ ് െͳ and c is a constant.
2. ‫ ݔ݀݇ ׬‬ൌ ݇‫ ݔ‬൅ ܿ, k and c are constants.

303

BUSINESS MATHS & STATISTICS (TC3)


3. ‫݂ ׬‬ሺ‫ݔ‬ሻ ൅ ݃ሺ‫ݔ‬ሻ݀‫ ݔ‬ൌ ‫݂ ׬‬ሺ‫ݔ‬ሻ݀‫ ݔ‬൅ ‫݃ ׬‬ሺ‫ݔ‬ሻ݀‫ݔ‬, the integral of a sum is the sum of the integrals.

Example 9: Integrate y = x3 with respect to x

Solution œx dx
3
= ( x4)/4 + c

Example 10: Integrate y = 8x3

Solution œ (8x ) dx
3
= 8x4/4 + c = 2x4 + c

Example 11: Find the integral of y = 5 (w.r.t. x)

Solution œ5dx = 5x + c

Example 12: Simplify (2/x3)dx œ


Solution œ2/x dx 3
= œ(2x-3
)dx ( 2x-2)/-2 + c = -1/x2 + c

Example 13: Integrate y = 6x4 + x3 - 2/x4 + 2

Solution œ œ
ydx = (6x4 + x3 - 2/x4 + 2)dx œ (6x 4
+ x3 - 2x-4 + 2
= (6x5)/5 + (x4)/4 + (2/3x-3) + 2x + c

19.2.3 Definite Integrals

Definition 19.2.3A definite integral is an integral which has limits within which the integral is to be

evaluated.The limits are indicated on the integral sign as follows‫׬‬௔ ǡ where a, b are the limits.

௕ ௕
A definite integral ‫׬‬௔ ݂ሺ‫ݔ‬ሻ݀‫ݔ‬ǡ is evaluated as ‫׬‬௔ ݂ሺ‫ݔ‬ሻ݀‫ ݔ‬ൌ ‫ܨ‬ሺܾሻ െ ‫ܨ‬ሺܽሻ where ‫ܨ‬ሺ‫ݔ‬ሻ is an antiderivative of

݂ሺ‫ݔ‬ሻ. That is, ‫׬‬௔ ݂ሺ‫ݔ‬ሻ݀‫ ݔ‬is equal to the antiderivative of ݂ሺ‫ݔ‬ሻ evaluated at the upper limit ܾ minus the
antiderivative of ݂ሺ‫ݔ‬ሻ evaluated at the lower limit ܽ.

Example 14 Evaluate‫׬‬ଵ ሺʹ‫ ݔ‬ଶ ൅ ‫ ݔ‬െ ʹሻ݀‫ݔ‬

ଶ௫ య ௫మ
Solution The anti-derivative is ‫ܨ‬ሺ‫ݔ‬ሻ ൌ ൅ െ ʹ‫ݔ‬.
ଷ ଶ
ଶൈଷయ ଷమ ଶൈଵయ ଵమ
Now, ‫ܨ‬ሺ͵ሻ ൌ ቀ ൅ െ ʹ ൈ ͵ቁ and ‫ܨ‬ሺͳሻ ൌ ቀ ൅ െ ʹ ൈ ͳቁǤ
ଷ ଶ ଷ ଶ
‫ܨ‬ሺ͵ሻ െ ‫ܨ‬ሺͳሻ ൌ ሺͳͺ ൅ ͶǤͷ െ ͸ሻ െ  ሺͲǤ͸͹ ൅ ͲǤͷ െ ʹሻ ൌ ͳ͹Ǥ͵͵.

This means ‫׬‬ଵ ሺʹ‫ ݔ‬ଶ ൅ ‫ ݔ‬െ ʹሻ݀‫ ݔ‬ൌ ͳ͹Ǥ͵͵.

Example 15 Evaluate the integral of y = 2x3 + 4 between x =0 and x = 2.


304

BUSINESS MATHS & STATISTICS (TC3)



Solution Here the problem is ‫׬‬଴ ሺʹ‫ ݔ‬ଷ ൅ Ͷሻ݀‫ݔ‬Ǥ

ଶ௫ ర ଶൈଶర
Here the anti-derivative is ‫ܨ‬ሺ‫ݔ‬ሻ ൌ ൅ Ͷ‫ݔ‬. Now, ‫ܨ‬ሺʹሻ ൌ ൅ Ͷ ൈ ʹ ൌ ͳ͸, and
ସ ସ
ଶൈ଴ర ଶ
‫ܨ‬ሺͲሻ ൌ

൅ Ͷ ൈ Ͳ ൌ ͲǤ So ‫׬‬଴ ሺʹ‫ ݔ‬ଷ ൅ Ͷሻ݀‫ ݔ‬ൌ ‫ܨ‬ሺʹሻ െ ‫ܨ‬ሺͲሻ ൌ ͳ͸ െ Ͳ ൌ ͳ͸Ǥ

19.3 APPLICATIONS OF SOME CONCEPTS OF CALCULUS

19.3.1 Finding minimum or maximum points in general

Differentiation can be used to find the price that maximizesprofit, the dimensions that minimize the cost to
construct a box, and the productionlevel that minimizes costs. This will be done in the same manner we
approached maximum and minimum values. We will therefore use the secondderivative test on a critical
value to check whether a critical point maximizes or minimizes a given function.

Example 16 The profit for selling x hundred units of a product can be approximated
by ܲሺ‫ݔ‬ሻ ൌ െ‫ ݔ‬ଷ ൅ Ͷͷ‫ ݔ‬ଶ ൅ ͳʹͲͲ‫ ݔ‬൅ ͺͲͲͲͲ(up to x = 50). What level ofsales maximizes the profit?

Solution We begin by finding the derivative:ܲᇱ ሺ‫ݔ‬ሻ ൌ െ͵‫ ݔ‬ଶ ൅ ͻͲ‫ ݔ‬൅ ͳʹͲͲ. Now we will set
ᇱ ሺ‫ݔ‬ሻ
ܲ equal to zero and solve for x.

Doing that gives െ͵‫ ݔ‬ଶ ൅ ͻͲ‫ ݔ‬൅ ͳʹͲͲ ൌ Ͳǡ from which, after dividing throughout by -3, we obtain ‫ ݔ‬ଶ െ
͵Ͳ‫ ݔ‬െ ͶͲͲ ൌ ሺ‫ ݔ‬െ ͶͲሻሺ‫ ݔ‬൅ ͳͲሻ ൌ Ͳ. Hence ‫ ݔ‬ൌ ͶͲ and ‫ ݔ‬ൌ െͳͲ. We disregard the x=-10 as we cannot
sell x units. What remains now is to check if x=40 maximises the profit.

Now to do this, we need to plug x=40 into the second derivative and check if the result is negative. Let’s do
that: ܲᇱᇱ ሺ‫ݔ‬ሻ ൌ െ͸‫ ݔ‬൅ ͻͲ and

ܲᇱᇱ ሺͶͲሻ ൌ െ͸ ൈ ͶͲ ൅ ͻͲ ൌ െͳͷͲ. The conclusion is that x=40 maximises since the second derivative is
negative (NEMA!).

Example 17 A farmer wishes to make a rectangular chicken run (wire “khola”) using an existing wall as
one side. He has 16 meters of wire netting. Find the dimensions of the run which will give the maximum
area. What is this area?

Solution Let x = length of pen opposite existing wall and A = area

Then the sum of the lengths of the other sides = 16-x


One such side has length =(16 - x)/2 = 8 - x/2

305

BUSINESS MATHS & STATISTICS (TC3)


Figure 19.3 Maximum area of enclosure
existingwall

8Ͳx/2

x
Area A = length * width
= x(8 – x/2) = 8x – x2/2

dA/dx = 8 - x

Set to zero: 8–x =0


x =8

2nd derivative d2A/dx2 = -1 which is negative

Max Area A = 8 * (8 – 8/2)


= 32 m2

19.3.2 Marginal and Total Functions


Given Total Revenue, Total Cost or Profit functions it is possible to find marginal functions using
differentiation. Note that marginal revenue, marginal cost or marginal profit are the rates of change of their
total functions. Therefore finding the first derivatives of Total Revenue, Cost and Profit functions gives the
marginal revenue, marginal cost or marginal profit functions respectively.

Example 18 The following functions relate to Total cost (TC), Profit (ʌ) and Total revenue (TR) where q
is quantity of sales or output.

a) TC = 4q3 + 7q + 12
b) Ʌ = q2 + 12q + 60
c) TR = 10q – q2

Required:

Determine the
i) The marginal and the average functions for each of the total functions.
ii) Evaluate each one of the expressions in i) where q = 3.

Solution
ௗ்஼
a) i) MC = = = 12q2 + 7
ௗ௤
ሺସ௤య ା଻௤ାଵଶሻ ଵଶ
AC = = Ͷ‫ ݍ‬ଶ  ൅ ͹ ൅ 
௤ ௤

ii) When q = 3 MC = 12 x 32 + 7 = 115


AC = 4 x 32 + 7 + 12/3 = 47

306

BUSINESS MATHS & STATISTICS (TC3)


b) i) MɅ = d(Ʌ)/dq = 2q + 12
AɅ = (q2 + 12q + 60)/q = q + 12 + 60/q

ii) When q = 3 MɅ = 2 x 3 + 12 = 18
AɅ = 3 + 12 + 60/3 = 35

c) i) MR = d(R)/dq = 10 – 2q
AC = (10q – q2)/q = 10 – q

ii) When q = 3 MR = 10 – 32= 1


AR = 10 – 3 = 7

Note: Average functions are not derived through calculus.

Example 19 A firm has the marginal cost = K2.5 with fixed cost of K50,000. The marginal revenue varies
with sales and it is given as MR = 45 - x. Find the Total cost and Total revenue functions.

Solution Total cost function is the integral of the marginal cost

œ
TC = 2.5 dx = 25x + c
When x = 0 TC = 50,000 Therefore c = 50000 and TC = 2.5x + 50000

œ
TR = (45 - x)dx = 45x - x2/2 + c

When x = 0, Revenue is 0 (no sales, no revenue)


Therefore c = 0

TR = 45x - x2/2

19.3.3 Revenue and profit maximisation and cost minimisation

Calculus can be used in revenue and profit maximisation as well as cost minimisation. The only requirement
is that these should be in form of non linear functions.

Example 20 Given the Cost and Revenue functions found in Example 2 above, under 19.4.2. Find:

a) The output that maximises revenue, and profit


b) Find the maximum revenue and maximum profit

Solution a) TR = 45x - x2/2

d(TR)/dx = 45 - x
Set to zero: 45 - x = 0 x = 45

d2(TR)/dx2 = -1 Therefore maximum

307

BUSINESS MATHS & STATISTICS (TC3)


Revenue is max at x = 45

Profit ʌ = TR - TC = (45x - x2/2) – (2.5x + 50)


= 45x - x2/2 – 2.5x - 50
= 42.5x - x2/2 - 50

d ʌ/dx = 42.5 – x set equal to zero 42.5 – x = 0;


x = 42.5,
d2 ʌ/dx2 = –1 ʌ is maximum at x = 42.5

Example 21 Toleza farm are making brickets used as fuel-wood for the community around as part of their
social responsibility. The total cost (K’000) of producing x brickets is defined by the function C(x)= 20 +
3x. The farm discovers that the selling price is given by the function:P = 32 – 2x. The manager wants you to
help him determine:

a) Output which maximises revenue and the maximum revenue


b) Output which maximises profit and the maximum profit.
c) Break even output
d) Price at break even

Solution

a) Let R = Revenue R = Px = x(32 – 2x) = 32x – 2x2

dR/dx = 32 – 4x

Set to 0: 32 – 4x = 0, so x = 8.

d2R/dx2 = –4 This is negative, which means Revenue is maximum at x = 8

The maximum revenue R = 32 x 8 - 2 x 82 = 128 i.e. K 128, 000

b) Let ɥ = profit, and C = Cost

ɥ =R - C (R is defined in a) above

ɥ = (32x – 2x2) - (20 + 3x)


= 29x – 2x2 - 20
dɥ/dx = 29 – 4x

Set to 0: 29 – 4x = 0 x = 7.25
d ɥ/dx =
2 2
–4 This is negative, which means ɥ is maximum at x = 7.25

Example 22 If additional cost for the brickets above are given by x2. So that the cost function is

C(x) = 20 + 3x - x2

308

BUSINESS MATHS & STATISTICS (TC3)


Find the point at which costs are minimum.

Solution dC/dx =3 – 2x
Setting dC/dx equal to 0 gives 3 – 2x = 0, and solving for x,
we obtain x = 1.5.
d2ɥ/dx2 = – 4 This is negative, which means ɥ is maximum at x = 7.25

Example 23 The marginal revenue from the sale of units of a product is 12-0.0004x . If the revenue from
the sale of the first 1000 units is K124,000, find the revenue from the sale of the first 5000 units.

Solution Here we have ܴᇱ ሺ‫ݔ‬ሻ ൌ ͳʹ െ ͲǤͲͲͲͶ‫ ݔ‬and that ܴሺͳͲͲͲሻ ൌ ͳʹͶͲͲͲ. We are required to find
ܴሺͷͲͲͲሻ. Now, we know that marginal revenue is the derivative of the total revenue so to find the total
revenue function, we are going to integrate the marginal revenue. Doing that gives:

‫ܴ ׬‬ᇱ ሺ‫ݔ‬ሻ݀‫ ݔ‬ൌ ܴሺ‫ݔ‬ሻ ൌ ‫׬‬ሺͳʹ െ ͲǤͲͲͲͶ‫ݔ‬ሻ݀‫ ݔ‬ൌ ͳʹ‫ ݔ‬െ ͲǤͲͲͲʹ‫ ݔ‬ଶ ൅ ܿ . Now, since ܴሺͳͲͲͲሻ ൌ ͳʹͶͲͲͲ we
have that ͳʹሺͳͲͲͲሻ െ ͲǤͲͲͲʹሺͳͲͲͲሻଶ ൅ ܿ ൌ ͳʹͶͲͲͲ so ܿ ൌ ͳͳʹʹͲͲ. Putting this into the revenue
function we obtain ܴሺ‫ݔ‬ሻ ൌ ͳʹ‫ ݔ‬െ ͲǤͲͲͲʹ‫ ݔ‬ଶ ൅ ͳͳʹʹͲͲǤ We are now ready to calculate ܴሺͷͲͲͲሻ as we just
have to plug ‫ ݔ‬ൌ ͷͲͲͲ into ܴሺ‫ݔ‬ሻǤ ܴሺͷͲͲͲሻ ൌ ͳʹሺͷͲͲͲሻ െ ͲǤͲͲͲʹ ൈ ͷͲͲͲଶ ൅ ͳͳʹʹͲͲ ൌ ‫ͳܭ‬͸͹ǡʹͲͲ.

19.4 SUMMARY OF APPLICATIONS OF CALCULUS TO REVENUE COST AND


PROFITFUNCTIONS

Table 19.1 Summary of revenue, cost and profit functions


FUNCTION BASIC DERIVATION
1 Total Cost C(x)

2 Marginal Cost C'(x)

3 Average Cost

4 p(x)
Price Function
5 Revenue Function R(x) = xp(x)

6 Marginal Revenue R'(x)

7 Profit Function P(x) = R(x) - C(x)

8 Marginal Profit P'(x) = R'(x) - C'(x)

CHAPTER SUMMARY

In this chapter we have looked at the following:

x How to differentiate functions using different rules.


x How the first and second derivatives can be used to determine the optimum values of a function.
309

BUSINESS MATHS & STATISTICS (TC3)


x How to use critical points and their classifications as local maximum or local minimum to sketch the
graph of a given function.
x How to find the integral of a function.
x How the methods of calculus can be used in revenue and profit maximizationand cost minimization.

END OF CHAPTER EXERCISES

1. Find the derivatives of the following:

a) ‫ ݕ‬ൌ ʹ‫ ݔ‬ଶ (wrt x)


b) ‫ ݌‬ൌ ͸Ͳ െ ͵‫ݍ‬ (wrt q)
c) ‫ݕ‬ൌͶ (wrt x)
d) ‫ݕ‬ൌ‫ݔ‬ (wrt x)
e) ‫ ݕ‬ൌ ʹ‫ ݔ‬ଷ ൅ ͷ‫ି ݔ‬ଶ ൅ ͵‫ ݔ‬൅ ͹ (wrt x)

2. Evaluate
a) ‫ ׬‬ሺʹ‫ ݔ‬െ  ‫ ݔ‬ଶ ሻ݀‫ݔ‬

b) ‫׬‬ଶ ሺ‫ ݔ‬ଷ  െ ʹ‫ ݔ‬ଶ  ൅ ͷሻ݀‫ݔ‬

3. Suppose the demand curve is: ‫ ݌‬ൌ ͳͺ െ ͲǤͷ‫ݍ‬


a. Find the Total Revenue function (TR).
b. Find Average Revenue and Marginal Revenue.
c. Evaluate Marginal Revenue at ‫ ݍ‬ൌ ͳͲ. If MC = 7 should the firm increase production?

4. Suppose a firm has the following profit function.

ߨ ൌ െ‫ ݍ‬ଶ ൅ ͳͳ“ െ ʹͶ

Determine the amount of output this firm should produce to maximize its profits.

5. Suppose a firm has the following total revenue and cost functions:

ܴܶ ൌ ͳͶͲͲ‫ ݍ‬െ ͸‫ݍ‬ଶ and ܶ‫ ܥ‬ൌ ͳͷͲͲ ൅ ͺͲ‫ݍ‬

Determine the amount of output this firm should produce to maximize its profits.

6. The manufacturing process of a particular item at a company has a total cost function given by ‫ ܥ‬ൌ
ଵ ଵ
ሺʹͲ‫ ݔ‬൅ ͸ሻ and a revenue function given by ܴ ൌ ሺͻͲ‫ ݔ‬െ ͳͲ‫ ݔ‬ଶ െ ͷͶሻ where x is the number of
ଷ ଷ
items produced (in hundreds), and C and R are both units of K1,000.00

Required
a) Express in its simplest form the total profit P for the process,
where P=R-C
b) Plot a graph of P against x for values of x from 0 to 7 at intervals of 1.
c) Use the graph to determine the values of x which give:
i. The maximum profit, giving the value of P.
ii. The break even points for the process (i.e. where profit P=0).

310

BUSINESS MATHS & STATISTICS (TC3)


7. An economist of XYZ limited producing special tucks reckons that for this product weekly revenue
and cost functions are

Marginal revenue = ͳͶ െ ʹ‫ݔ‬


Marginal cost =͵‫ ݔ‬ଶ െ ͳͷ‫ݔ‬

where x is the number of tucks in hundreds produced each week. The company has fixed cost of
K65.00

(a) Find the equations for Total cost and Total revenue.
(b) Find the total profit function.
(c) For values of ‫ ݔ‬ൌ Ͳ to ‫ ݔ‬ൌ ͺ, draw a graph of the profit function.
(d) Using your graph write down the estimate of maximum profit.
(e) Usethe methods of calculus to find the profit maximizing output and state the maximum
profit.

8. Suppose a firm faces the following demand curve



‫ ݌‬ൌ ͵Ͳ െ  ‫ݍ‬

and has the following total cost curve:

ܶ‫ ܥ‬ൌ ͳͷͲ ൅ ͳͲ‫ݍ‬


Required

a) Determine the amount of output this firm should produce to maximize its profits.
b) Use the quantity you found in part (a) (called the “optimal quantity”) to determine the price
the good is sold at.
c) Using the optimal quantity, determine the firm’s profits.

9. A manufacturer of a newpatented product has found that he can sell 70 units a week direct to the
customer if the price is K48. In error, the price was recently advertised at K78 and, as a result, only
40 units were sold in a week. The manufacturers fixed costs of production are K1,710 a week and
variable costs are K9 per unit. You are required:

a) To show the equation of the demand function linking price (P) to quantity demanded
ሺ‫ݔ‬ሻ, assuming it to be a straight line, isܲ ൌ ͳͳͺȂ ‫ݔ‬.
b) To find where the manufacturer breaks even.
c) To recommend a unit price which would maximize profit, and to find the quantity demanded
and profit generated at the price.  

311

BUSINESS MATHS & STATISTICS (TC3)


EXAMINATION QUESTIONS EXAMPLES

Examination No.________________________

THE INSTITUTE OF CHARTERED ACCOUNTANTS IN MALAWI

ACCOUNTING TECHNICIAN PROGRAMME

PAPER TC 3: BUSINESS MATHEMATICS

INSTRUCTIONS

Number of questions on paper – 9.

The paper is divided intoSections A and B. ALL questions to be answered in Section A andANY
TWOfromSection B.

The maximum number of marks for each answer is indicated against each question.

Mathematical Tables, Formulae Sheets and Graph Paper are provided.

Use of non-programmable calculators is allowed.

Show all your workings in order to gain full marks. Method marks will be awarded throughout.

Begin each answer on a fresh page.

8. DO NOT OPEN THIS PAPER UNTIL YOU ARE INSTRUCTED BY THE INVIGILATOR.

312

BUSINESS MATHS & STATISTICS (TC3)


SECTION A

ANSWER ALL THE QUESTIONS IN THIS SECTION

1. (a) Add 6y – 3x – 2y to 6x – y – 2xy 2 Marks

(b) Two types of tea costing K650 per kilogram and K720 per kilogram respectively are blended
in the ratio 7:3.

Required:

Find the cost of 1 kilogram of the blended tea. 5 Marks


(TOTAL : 7 MARKS)

2. A clerk employed in a certain company, starts with a salary of K120,000 per annum. At the
beginning of each year he gets an increment of n Kwacha per annum so that his salary for the second
year is (120,000 + n) Kwacha.

Required:

Find:

(a) the salary that he gets in the third, fourth, fifth and sixth years of his
employment. 4 Marks

(b) (i) the total sum of money that he gets for the first five years, in its simplest form.

(ii) the sum if n = 60 4 Marks


(TOTAL : 8 MARKS)
3 Evaluate :

‫׬‬଴ ሾͳ ൅ ͵‫ݔ‬ሺͳ ൅ ‫ ݔ‬ଶ ሻሿ݀‫ݔ‬ 4 Marks

4. (a) Solve the following inequality: x2 > 25 8 Marks

(b) The following data give the quantities and costs of materials for the four divisions of a
company for two years.

Quantity (tonnes) Cost (K)


DIVISION Year 1 Year 2 Year 1 Year 2
A 175 201 1540 1830
B 32 46 1270 1490
C 48 43 2760 2490
D 65 66 2160 2070

Taking year 1 as the base, calculate:

313

BUSINESS MATHS & STATISTICS (TC3)


(i) the Paasche price (cost) index for year 2

(ii) the Paasche quantity index for year 2. 7 Marks


(TOTAL: 15 MARKS)

5. (a) A woman goes to the French Cultural Centre every week to watch drama, each time paying
K450 for her ticket. She decides that she would pay less if she bought a video cassette
recorder for K25,615 and hired one film each week at a cost of K200.

Required:

How many complete weeks will it take to make the new method cheaper
than the weekly visits to the French Cultural Centre? 7 Marks

(b) Two shops are offering the same model of TV set for sale which had an original price of
K36,000.

Shop A is offering a discount of 8% followed by a special offer of 3% off that discounted


price.

Shop B is offering a single discount of 11%.

Required:

What saving would be made by buying the set at the cheaper price? 7 Marks
(TOTAL: 14 MARKS)

6. The following data show the periods (in minutes) that a sample of employees needed to complete a
particular task.

76 59 93 87 38 50 56 113
102 34 54 85 85 50 45 67
51 40 82 92 79 38 44 33
29 107 63 46 68 49 86 34
61 72 79 45 70 40 99 62

Required:
(a) Construct a frequency distribution using the class limits 29 – 43, 44 –58,…
(b) Use your distribution to estimate
(i) the arithmetic mean
(ii) the standard deviation.

(c) Draw a histogram for these data.


(d) Explain whether you would expect the median to be greater than, less than, or about the same
as the mean. (NB : You do not need to calculate).
13 Marks
(TOTAL : 13 MARKS)

314

BUSINESS MATHS & STATISTICS (TC3)


SECTION B

ANSWER TWO QUESTIONS ONLY FROM THIS SECTION

7. (a) A man wishes to invest K120,000 over a period of 8 years. The “Kulemera Trust” offers a
compound interest rate of 8.25%, payable yearly, and “Kusauka Kwatha Investments” offers
a rate of 4.5%, payable every 6 months.

Required:
(i) Calculate the accrued amount obtained from investing in each of the two investment
companies. 4 Marks

(ii) Which investment company gives the better return, and by how much? 2 Marks

(iii) If the man decides to invest in “Kusauka Kwatha Investments”, how much principal
sum will have to be invested to accrue K245,000 over the 8 year period? 5 Marks
TOTAL 11 Marks

(b) The Southern Manufacturing Company completed a feasibility study concerning the
development of a new product. The study includes the following estimates.

K’000
Initial cost outlay 1,500
Further outlay at the end of 3 years 800
Residual value after 5 years 500
Net returns at the end of each year for 5 years 550

Required:

(i) Find the net present value. Use a discounting factor of 15%.
(ii) State whether or not the venture is profitable. 9 Marks
(TOTAL : 20 MARKS)

8. (a) State whether the following statements are true or false and give a brief explanation.

(i) A correlation coefficient of –1 means that the appropriate regression line is downward
sloping.
(ii) The mode of a data set is always greater than the median.
(iii) A quota sample is more accurate than a random sample. 6 Marks

(b) (i) What causes non-response in a survey?


(ii) Explain how non-response arises in postal/mail surveys conducted randomly.
(iii) You have been asked to conduct a postal survey of people’s holiday plans. Your boss
is particularly interested in the popularity of different destinations and types of
accommodation. You send out questionnaires during the Christmas break.

Required:
In what ways would you expect non-response to affect your estimates? 11 Marks

315

BUSINESS MATHS & STATISTICS (TC3)


(c) You toss a coin twice. What is the probability that you get:
(i) one head and one tail overall?
(ii) A tail on the first throw and a head on the second throw? 3 Marks
(TOTAL : 20 MARKS)

9. A survey was conducted in 9 areas of Blantyre to investigate the relationship between Divorce rate
(y) and Residential mobility (x). Divorce rate is the annual number of divorces per 1000 in the
population and the Residential mobility is measured by the percentage of the population who have
moved house in the last 5 years.

The results of this survey are shown in the table below.

Residential Mobility Divorce Rate


40 3.9
38 3.4
46 5.2
49 4.8
47 5.6
43 5.8
51 6.6
57 7.6
55 5.8
Required:

(i) Draw and label the appropriate scatter diagram. 5 Marks


(ii) Compute the linear regression line of y on x for these data. 11 Marks
(iii) Use this regression equation to estimate the Divorce Rate for an area that has a residential
mobility of:

1. 39
2. 60

(iv) Which of these estimates is likely to be more accurate? Give a reason for your answer.
4 Marks
(TOTAL : 20 MARKS)

END

316

BUSINESS MATHS & STATISTICS (TC3)


THE INSTITUTE OF CHARTERED ACCOUNTANTS IN MALAWI

ACCOUNTING TECHNICIAN PROGRAMME

PAPER TC 3: BUSINESS MATHEMATICS

INSTRUCTIONS

Number of questions on paper – 9.

The paper is divided intoSections A and B. ALL questions to be


answered in Section A andANY TWOfromSection B.

The maximum number of marks for each answer is indicated against each question.

Mathematical Tables, Formulae Sheets and Graph Paper are provided.

Use of non-programmable calculators is allowed.

Show all your workings in order to gain full marks. Method marks will be awarded throughout.

Begin each answer on a fresh page.

8. DO NOT OPEN THIS PAPER UNTIL YOU ARE INSTRUCTED BY THE INVIGILATOR.

317

BUSINESS MATHS & STATISTICS (TC3)


SECTION A

ANSWER ALL THE QUESTIONS IN THIS SECTION


ଵଷ ଵଵ ଵ
1. (a) Simplify: ͳ  ൈ ʹ ൊ ͵ 2 Marks
ଶ଻ ଵଶ ଽ

(b) K37,000 is to be shared between John, Mary and Jane, whose ages are 10, 12 and 15 years
respectively, in the same ratio as their ages.

Required:
How much does each receive? 5 Marks
(TOTAL : 7 MARKS)

2. (a) Solve the following equation for x:


2x + 3 - 2(x – 4) = 7 5/6
2 3 5 Marks

(b) A salesman has to decide between two jobs. The first job offers a salary of K15,000 per
week plus a commission of 2% of sales and the second offers a salary of K7,500 per week
plus 5% of sales.

Required:
If he estimates that he is likely to sell K12,500,000 worth of goods in a year, which job will give him
the greater return for the year and by how much? 8 Marks
(TOTAL : 13 MARKS)

3. (a) Find the equation of the line passing through (4, 12) and (8, -2). 3 Marks

(b) 20 shirts and 6 ties cost a shopkeeper K5,300 and 18 shirts and 20 ties cost him K5,500.

Required:

Find the cost of one shirt and of one tie. 8 Marks


(TOTAL : 11 MARKS)

4 A mower is marked for sale at K52,000. A man buys this mower after obtaining a K12,000 trade-in
on his old mower. He agrees to pay off the mower in 18 equal monthly payments. The shop
calculates that he will pay K9,000 interest.

Required:

Calculate the following:

(a) The total amount to be paid. 3 Marks


(b) The amount of each repayment. 2 Marks
(c) The rate of interest being charged. 3 Marks
(TOTAL : 8 MARKS)

318

BUSINESS MATHS & STATISTICS (TC3)


5. Kesale Estate Agents released the following daily sales figures for the past 14 weeks.

Houses sold Number of days on which


sales were made
0 2
1 17
2 30
3 26
4 15
5 6
6 or more 2

Required:
(a) How many sales were made over the 14 week period? 2 Marks
(b) Find the mode, median and mean number of houses sold. 5 Marks
(c) If the average selling price was K1,160,000 and the agents’ commission was 10%, how much
commission was earned over the period? 3 Marks
(TOTAL : 10 MARKS)

6. A company is faced with the following marginal cost and marginal revenue functions:

MC = 16 – 2Q
MR = 40 – 16Q
Where Q is the production.

It is also known that fixed costs are 8 when production is zero.

Required:

Find:

(a) The total cost function. 4 Marks


(b) The total revenue function. 4 Marks
(c) The output to give the maximum sales revenue. 3 Marks
(TOTAL : 11 MARKS)

319

BUSINESS MATHS & STATISTICS (TC3)


SECTION B

ANSWER TWO QUESTIONS ONLY FROM THIS SECTION

7. Linda Furniture Ltd released the following table of goods against stock-on-hand for the financial
year 2005/06.

Month Average stock Average storage


on hand costs (MK’000)
July 34 73
August 12 28
September 17 40
October 7 20
November 8 23
December 3 12
January 11 27
February 17 42
March 14 30
April 26 54
May 20 48
June 28 60

The following statistics can be used: ¦xy = 9333 : ¦x2 = 4177; ¦y2 = 20999

Required:

(a) Draw a scatter diagram for the data. 4 Marks

(b) (i) Calculate the Product Moment Correlation Coefficient (r) for thedata.
(ii) Interpret the value of r. 7 Marks

(c) (i) Find the least squares regression line of storage costs on stock-on-hand.
(ii) Plot your regression line on the scatter diagram.
(iii) Predict the storage cost if the average stock-on-hand was 30 units.
9 Marks
(TOTAL : 20 MARKS)

8. (a) A company owns a fleet of 100 cars, each of which has either manual or automatic
transmission and either 2 or 4 doors. There are 25 cars which are 2 door models and of these
10 have automatic transmission. There are 35 cars with manual transmission.

Required:

If a car is picked at random from the fleet, calculate the probability that it is:

(i) Automatic;
(ii) 2 – door;
(iii) Automatic or 4 – door;
320

BUSINESS MATHS & STATISTICS (TC3)


(iv) Automatic and 4 – door;
(v) Automatic, given it is 2 – door;
(vi) 2 – door, given it is automatic;
(Give your answers to 2 decimal places). 7 Marks

(b) A dealer buys a computer, costing him K90,000 and sells it to a company, making a profit of
45%.

Required:

(i) At what price does the dealer sell the computer? 2 Marks
(ii) The company estimates that the computer will have a useful life of 4 years,
depreciating at a rate of 25% per annum.
Required:

What is the residual value of the computer after 4 years? (Give your answer to the nearest
Kwacha). 3 Marks

(c) (i) Chiyembekezo has taken out a mortgage of K600,000 to be paid over 25 years.
Interest is to be charged at 12% p.a.

Required:
Calculate the monthly repayment. 3 Marks

(ii) After nine years, the interest rate changes to 10% p.a. What is the new monthly
repayment? 5 Marks
(TOTAL : 20 MARKS)

10. The table below gives the life of a particular brand of a car tyre (in kilometers travelled) obtained
from a road service survey.

Kilometers travelled Number of tyres


0 - 3,999 1
4,000 - 7,999 3
8,000 - 11,999 8
12,000 - 15,999 14
16,000 - 19,999 20
20,000 - 23,999 15
24,000 - 27,999 10
28,000 - 31,999 6
32,000 - 35,999 2
36,000 - 39,999 1

Required:

321

BUSINESS MATHS & STATISTICS (TC3)


(a) Draw a cumulative frequency distribution curve to represent these data. 8 Marks

(b) From the curve, determine the median and the upper and lower quartiles for these data.
3 Marks

(c) Determine the interquartile range for these data and explain what it shows.State why this
measure is preferred to other measures of variation. 5 Marks

(d) What percentage of the tyres have a life

(i) above 25,000 km;


(ii) below 15,000 km? 4 Marks
(TOTAL : 20 MARKS)

END

322

BUSINESS MATHS & STATISTICS (TC3)


THE INSTITUTE OF CHARTERED ACCOUNTANTS IN MALAWI

ACCOUNTING TECHNICIAN PROGRAMME

PAPER TC 3: BUSINESS MATHEMATICS & STATISTICS

INSTRUCTIONS

1. Number of questions on paper - 9.

2. The paper is divided into Sections A and B. ALL questions to be answered


in Section A and ANY TWO from Section B.

3. The maximum number of marks for each answer is indicated against each question.

4. Mathematical Tables, Formulae Sheets and Graph Paper are provided.

5. Use of non-programmable calculators is allowed.

6. Show all your workings in order to gain full marks. Method marks will be
awarded throughout.

7. Begin each answer on a fresh page.

8. DO NOT OPEN THIS PAPER UNTIL YOU ARE INSTRUCTED BY THE INVIGILATOR.

323

BUSINESS MATHS & STATISTICS (TC3)


SECTION A

ANSWER ALL QUESTIONS IN THIS SECTION

1. (a) Simplify:
͵‫ ݔ‬ଶ  ൅ ʹ
 െ ͳ
͵‫ ݔ‬൅ ʹ
3 Marks

(b) The daily wage rates of three women, Mary, Maria and Margret, are in the ratios 24:16:35.
Required:
Calculate:

(i) The total value of the wages earned, assuming that Mary earned K1,200; 4 Marks

(ii) Maria’s and Margaret’s wages. 4 Marks


(TOTAL: 11 MARKS)

2. (a) Solve the following simultaneous equations:

x + y= 1;
3
x + 2y = 5. 5 Marks

(b) A company has bought an asset which has a life span of four years. At the end of the four
years, a replacement asset will cost K120,000 and the company has decided to provide for
this future commitment by setting up a sinking fund into which equal annual investments will
be made, starting at year 1 (one year from now). The fund will earn interest at 12%.

Required:

Calculate the annual repayments. 3 Marks


(TOTAL: 8 MARKS)

3. (a) Given that matrix

ͺ ͻ
A = ቂ ቃ
ͳʹ ͹
ͳ͵ Ͷ
B = ቂ ቃ
ʹ ͸

Required:

Find:

(i) matrix C such that C = A.B; 3 Marks


(ii) the inverse of A. 5 Marks
324

BUSINESS MATHS & STATISTICS (TC3)


(b) A firm keeps details of component parts used in the make up of each product (Matrix K) and
products made on each day of the week (Matrix M) as follows:

Product
1 2 Parts
A B C
Mon 0 1
Tue 2 2 1 3 2 1
Wed 3 2 = K Products =M
Thu 1 1 2 1 4 2
Fri 1 0

Required:

Using matrix multiplication, find a matrix that describes the number of component parts used on
each day of the week. 5 Marks
(TOTAL: 13 MARKS)

4. (a) A machine costing K256,500 depreciates to a scrap value of K5,000 in ten years.

Required:

Calculate:

(i) The annual percentage rate of depreciation using the reducing balance method of
depreciation. 5 Marks

(ii) The book value at the end of the sixth year using the reducing balance method of
depreciation. 2 Marks

(b) It is estimated that a mine will yield an annual net return (i.e after all operating costs) of
K500,000 for the next 15 years. At the end of this time the property will be valueless.

Required:

Calculate the purchase price of the mine if the purchase price is equivalent to the present value of a
K500,000 annuity over 15 years at 12% discount rate. . 4
Marks
(TOTAL: 11 MARKS)

5. Sampling methods are widely used for the collection of statistical data in industry and business.

Required:

Illustrating your answers with practical examples, explain the following terms:

325

BUSINESS MATHS & STATISTICS (TC3)


BUSINESS MATHS & STATISTICS (TC3)
BUSINESS MATHS & STATISTICS (TC3)
TECHNICIAN DIPLOMA IN ACCOUNTING

THE INSTITUTE OF
CHARTERED ACCOUNTANTS
IN MALAWI

Institute of Chartered Accountants in Malawi


Stansfield House
Haile Selassie Road
P.O. Box 1
Blantyre

Tel: 01 820 301/318/423 Fax: 01 822 354


Email: icam@icam.mw Website: www.icam.mw
BUSINESS MATHS & STATISTICS (TC3)

You might also like